COMBANK QUESTIONS

Pataasin ang iyong marka sa homework at exams ngayon gamit ang Quizwiz!

what does a person have who presents with painful raised lesions with sharply demarcated borders?

Erysipelas caused primarily by group A streptococci. Any surgery or procedure where trauma to the skin occurs predisposes the patient to superficial skin infections. Erysipelas is often accompanied by fever, chills shakes, and general illness as well as lymphedema and red streaking caused by infected lymph vessels inthe superficial layer of the dermis. pcn remains drug of choice to treat erysipelas.

A child presents with bed wetting what can be used?

Desmopressin and then imipramine. Desmopressin is preferred over imipramine due to a more favorable side effect profile.

Pseudomembrane colitis

Clindamycin or ampicillin

Agranulocytosis

Clozapine, carabmazepine, colchicine and dapsone

behavioral and/or psychological changes plus symptoms of sympathetic nervous system activation. patients may experience perceptual disturbances, commonly manifested as tactile hallucination what overdose is this?

Cocaine.

does combined oral contraceptive pills increase the risk of cancer?

Combined oral contraceptive pills increase the risk of breast cancer, decrease the risk of endometrial cancer and decrease the risk of ovarian cancer. and a slight small increase in risk of cervical cancer.

combined oral contraceptive is contraindicated in what patient?

Combined oral contraceptives increase blood pressure and are contraindicated in patients with untreated htn. An alternate method of contraception such as intrauterine device (IUD) or progesterone only should be employed.

when you hear bilateral hilar lymphadenopathy?

Sarcoidosis

what is a side effect of clomiphene

Multiple enlarged follicles clomiphene is a partial estrogen agonist used in the induction of ovulation in anovulatory patients that have PCOS. Patients taking this medication will commonly experience enlarged and cramping ovaries, hot flashes, vasomotor flushing, multiple gestations and visual disturbances. they will have multiple enlarged ovarian follicles secondary to they hyperstimulation from increased gondotropin levels.

2nd and 3rd intercostal space chapman's point

Myocardium and upper lung

helminths-cestodes

Tapeworms

where would you expect to see tissue texture changes of the SA node?

Tissue texture changes of the SA node would be expected to be seen on t1-t5 ON THE RIGHT wHILE MYOCARDIAL AND av NODE PARAVERTEBRAL VISCEROSOMATIC REFLEXES WOULD BE EXPECTED AT T1-T5 ON THE LEFT

torsional strain occurs with what axis?

Torsional strain patterns at the SBS occur when the sphenoid and occiput rotate in opposite directions around one AP axis.

positive symptoms include the perception of additional phenomena while negative symptoms involve the restriction of normal functions? true or false?

True

dementia of rapid onset, visual hallucinations and lewy bodies on autopsy denotes?

lewy body dementia-key is visual hallucinations.

if you see notched P-waves on limb leads and/or biphasic P-wave in V1 what should you suspect? why does HOCM (hypertrophic cardiomopathy) increase with valsalva?

Mitral stenosis and those findings are due to left atrial enlargement murmur of mitral stenosis begins in diastole with the opening snap of the mitral valve followed by low-pitched rumbling. best heard at apex. remember s of stenosis is like s of snap Increases with valsalva because of decreased left ventricular filling because valsalva decreases preload. Giant T-wave inversions in the precordial leads are electrocardiogram (ECG) Findings classically associated with apical HOCM. other findings of HOCM=left atrial hypertrophy, left ventricular hypertrophy, and deep Q-wave in the precordial leads.

The patient blood type is B+, per preoperative screening, the most appropriate replacement blood product for this patient is who started losing blood during surgery?

Packed red blood cells, type O- This patient is quickly losing blood and the most appropriate replacement product available is matched packed red blood cells. Since B+ RBCs are not offered as an answer, O- is the most appropriate and is the universal donor type. blood type is named by the antigens that are present upon the red blood cells. both B+ and B- RBCs would be acceptable choices if they were available.

An increase in lordosis of the lumbar spine or space between the posterior knee and the table indicate spasm of what muscle?

PSOAS/Iliopsoas muscles

tHE MECHANISM BEHIND BREAST Dimpling is that the cancer invades the suspensory ligaments of cooper in the breast. Peu d' orange, on the other hand is associated with erythema and edema and is commonly associated with inflammatory breast cancer.

true

true or false, hospice is intended for individuals with a life expectancy of six months or less. patients do not have to be DNR to receive hospice, and they can be re-hospitalized if needed.

true

if a patient at a teaching hospital refuses to be seen by a trainee, the attending physician should always be notified so that she/he may address the situation.

true

read question well In menopause-when estrogen levels are low what happens?

DECREASED ESTROGEN LEVELS ALLOWS FOR AN INCREASE IN OSTEOCLASTIC ACTIVITY.

sleep study finding on a person with narcolepsy will show?

Decreased REM Latency

what does REM stand for?

Rapid Eye movement

Prinzmetal angina associations

Raynaud's phenomenon, Migraine headache, history of cocaine drug abuse.

anti-dna topoisomerase 1

Sclerosis (scleroderma)

supination of the foot involves?

Adduction, inversion and plantar flexion do it on your own foot. supination is as if you are going more medial.

name some of risk factors for breast cancer?

Age, nulligravida, early menarche and obesity. Obesity increases risk because such adipose tissue generates estrogen

what are some food types that may contain tyramine?

Aged cheese, red wine, beer, soy products and cured meats. cheesrs athet are save are ricotta cottage, cream and Neufchatel cheeses), sour cream, and yogurt; so milk and ricotta cheese are both safe with MAOIs.

Drug of choice for enterobius vermicularis

Albendazole

What can alcoholism lead to?

Alcoholism can cause a dilated cardiomyopathy, which may lead to congestive heart failure due to systolic dysfunction. --This alcoholic patient is presenting with congestive heart failure (signs include peripheral edema, dyspneam distended neck veins, and fullness of the posterior axillary folds) and ventricular enlargement. the most likley diagnosis is a dilated cardiomyopathy from thiamine deficiency, which is commonly seen in alcoholics. dilated ventricles have fibrotic non-contractile tissue replacing their myocardium. this leads to weak contractions.

What is alkaptonuria caused by?

Alkaptonuria is caused by the absence of homogentisate oxidase, and thus homogentisate accumulates in cartilage.

How does Baclofen work?

Baclfen is a spasmolytic commonly used for cerebral palsy. iT BINDS GABA-B receptors AND REDUCES the release of excitatory neurotransmitters.

upper quadrant abdominal pain, high temp, skin appears yellow. ultrasound shows dilation os the common bile duct. her condition is most likely due to?

Bacterial infection of the biliary tract secondary to obstruction you know there is an infection because of the charcoats triad seen in ascending cholangitis.

95% of acromegaly cases are caused by?

Pituitary adenoma that secretes excess amounts of GH

The most common adverse effect of electroconvulsive therapy is?

Headache and it should be expected to occur after each treatment.

what do you expect to see on light and electron microscopy of membranoproliferative glomerulonephritis

-Mesangial proliferation on light microscopy and -spliting of basement membrane on electron microscopy.

using HVLA to treat T6-where should the thenar eminence be placed?

1/2 SEGMENT superior to the spinous process of T6. BASED ON THE RULES OF THREES THE TRANSVERSE PROCESS WILL BE LOCATED 1/2 A SEGMENT SUPERIO TO THE SPINOUS PROCESS OF t6.

For normal sleepers, what is a normal sleep cycle?

100-110 minutes long beginning with non-rapid eye movement (NREM) sleep and transitioning to REM sleep after 80-100 minutes

Chapman's reflex points represent the somatic manifestation of a visceral dysfunction. they are smooth, firm, discretely palpable nodules located within the deep fascia. where is the chapman point for the adrenal gland?

2'' superior and 1'' lateral to the umbilicus.

describes the axis of the torsional strain at the SBS?

A torsional strain at the SBS occurs when the sphenoid and occipital bones rotate in opposite directions around an anterior posterior axis. this motion occurs in the coronal plane.

what counterstrain tenderpoint is this: inner ala of the ilium i n the mid axillary line bilaterally?

AT12

It passes from the anterior margin of the fibular malleolus, forward and medially, to the lateral articular facet of the talus. SUSCEPTIBLE TO INVERSION SPRAINS. COMPLETE DISRUPTION -> positive anterior drawer test of the ankle.

ATFL "ATF"-Always torn first-this is the most commonly injured ligament in ankle sprains.

vagus nerve to the gastric parietal cells comes from ACT ON WHAT RECEPTOR??

Activation of the phospholipase C-it acts on m3 REMEMBER THE RECEPTOR IS QIQ.

What kind of acidosis do you see in COPD exacerbations?

Acute COPD exacerbations results in respiratory acidosis (worsening hypercapnia), hypoxia, changes in mental status, and marked increase in dyspnea

The patient self-determination act, passed by congress in 1990 in response to the cruzan decision, requires that, at the time of admission, health care institutions that participate in medicare and medicaid programs provide patients with information on ?

Advanced directives 1-Ask patients whether they have an advance directive 2-Inform patients of their rights to accept or refuse 3-Incorporate advance directives into patient's medical records.

The afferent limb of the cornel reflex is? what about the efferent limb?

Afferent limb is the cranial nerve V1(ophathalmic division of trigeminal) and the efferent limb is cranial nerve VII (facial). the fascial nerve also innervates the stapedius muscle.

where is the chapman point of the prostate?

Along the lateral and posterior margin of the IT band.

how do you treat tensor fascia lata and iliotibial band tender points?

BOTH ARE TREATED WITH FLEXION AND ABDUCTION.

what nerve root is tested in DTR for biceps tendon?

C5

what receptor does the ciliary muscle of the eyer have

Ciliary muscle of the eye has only M3 receptors so any medication that affects muscarinic receptors will cause the patient's near or far vision to change.

eyelid-you will see helioptrope rash for

DERMATOMYOSITIS

what about dermocentor species -what diseases are they a vector for?

DERMOCENTOR species are the vector for rocky mountain spotted fever (RMSF), CAUSED BY ricketssiae rickettsia.

EEG waveform associated with non-REM sleep=stage N3?

Delta

name the shock--normal/low preload, high cardiac output, decreased vascular resistance

Distributive shock

24 yo male has widespread molluscum contagiosum how do you screen?

ELIZA The screening test for HIV is done using ELISA, then results are confirmed with a Western blot. Consider HIV/AIDS in patients with widespread molluscum contagiosum.

you ask the pt to lie down and place the pads of your index fingers bilaterally on her forehead and apply a medial to lateral stroking towards the chin. what is this technique called?

Effleurage-a soft tissue technique which enhances the lymphatic drainage in the face. it's performed by placing the pads of the index fingers bilaterally on the forehead and applying a medial to lateral stroking towards the chin.

found in males less than 15 years of age, composed of small anaplastic blue cells with hyperchromatic nuclei and scant cytoplasm and appear like onion-skin on radiographs in the diaphysis of long bones?

Ewing sarcoma.

prevents formation of DHT which is the hormone that stimulates hyperplasia of the prostate?

Finasteride

What is the treatment for Ankylosing spondylitis?

First line is physical therapy and NSAIDS like indomethacin (or other types of NSAIDS) if NSAID fails-second line agents include sulfasalazine and TNF inhibitors Ankylosing spondylitis is a chronic inflammatory disease of the spine and pelvis that leads to fusion. it is a seronegative spondyloarthropathy with a strong association with HLA-b7 (90% of patients). clinical findings include low back pain and stiffness that begins in adolescence or young adulthood and is three times more common in males.

Hyperhidrosis, coarse facial features, macroglossia, swelling of the extremities

GROWTH HORMONE

Macrognathia (abnormal overgrowth of the jaw); enlargement of the facial bone structure, enlargement of the heads and feet, and visceral overgrowth (including macroglossia=enlargement of the tongue), enlargement of the heart muscle, thyroid, liver, and kidney)

Growth hormone

left anterior innominate rotation. using muscle energy how would you treat it which muscle would perform the isometric contraction?

Hip extensors muscle energy treatment to an anterior innominate dysfunction involves flexion of the hip and contraction of the extensors to draw the bone back into place.

What is the disease-modifying anti-rheumatic drug that causes retinal deposits?

Hydroxychloroquine this drug has an unknown mechanism of action but is thought to act through antigens responsibel for hypersensitivity reactions.

when do you want to do a ligamentous articular strain technique

IF THERE IS AN INTERSEGMENTAL RESTRICTIONS

factor v leiden aka activated protein C resistance is?

INABILITY OF activated protein C to bind factor Va; Factor V Leiden is the most common hereditary cause of thrombosis.

increased asymmetry of the sacral base with sphinx test/backward bending indicates what?

Indicates a backwar torsion or unilateral extension.

cause thyroid and kidney dsfunction

Lithium

Tumors of what type goes to the brain?

Lung, breast, skin or GI tract

what are the two main receptors for the pupil?

M3 and alpha 1

drug of choice for microbial infections are routinely tested on level 1. what is the drug of choice for Mycoplasma pneumoniae?

Macrolide-acithromycin or erythromycin.

what is the general theme in the location where mannitol will act on?

Mannitol will act on at sites in the nephron permeable o water such as: proximal convoluted tubule, thin descending limb of the loop of henle and the collecting ducts.

affects weighr bearing joints-knees, hips, cervical, lumbosacral spine and feet.

Osteoarthritis

what is the moa of phenytoin

Phenytoin not only blocks sodiun channels to increase refractory period of neurons, but they drug also inhibits glutamate release from excitatory presynaptic neurons.

used to treat seizure, can cause hirsutism, gingival hyperplasia, macrocytic anemia and osteomalacia?

Phenytoin.

helminths-nematodes

Roundworms (n is close to an R so that can help you remember that nematodes are roundworms)

Name non physiological strains?

SBS compression, vertical strains, and lateral strains.

negative predictive value indicates how likely a pt will be healthy after a test comes back negative

True.

given case of acute pyelonephritis what is the definite diagnosis?

Urine analysis

inferior division of middle cerebral artery deficit will be in

Wernicke's area

Know how L5 behaves with sacral somatic dysfunctions What happens to L5 when the sacrum is rotated right?

When L5 is rotated, the sacrum rotates in the opposite direction so sacrum will be rotated left. A sacral oblique axis is engaged on the same side as L5's sidebending

where is the anterior chapman point for the uterus?

inferior pubic rami

what is pyelonephritis?

inflammation of the kidney parenchyma. most commonly caused by ascending urinary tract infection.

With an inversion sprain what nerve can be damage and what is the mechanism of injury

inversion sprain-the common fibular nerve can be damaged and this is because the fibular head moves posterior and this will cause a foot drop. How to remember that inversion sprain has to deal with foot drop is that you think-inversion-inferior foot so it is foot drop

Why is there a really high pH with proteus mirabilis?

it breaks down urea.

know viscerosomatics well, t10-l1 corresponds to what viscerosomatics point?

kidneys and upper ureters

Crohn's disease most often affects the terminal ileum, the chapman point for the terminal ileum and ileocecal area is 10th intercostal space on the right.

know this

muscles of mastication include masseter, temporalis, and medial and lateral pterygoid muscles which aid in movement of the jaw. which of these muscles does something different?

lateral pterygoid muscle differs from the other muscles of this group due to its ability to depress the mandible allowing for opening of the mouth.

what is the chapman point for the prostate?

lying at the posterior edge of the proximal IT band near the tfl muscle. You can also see posterior chapman points overlying the sacral base bilaterally.

what is a normal cri and how does compression often occur? what does cri REPRESENT? compression often occurs from trauma to the back of the head-so a posterior head trauma. in this case where the a person has a ccmpression resulting in a decreased CRI. often, trauma to the back of the head will result in a compression and slowing of the CRI.

normal cri is 10-14 CRI=cranial rhythmic impulse

diagnosis influenced heavily by clinical picture but diagnosis can be supported if radiographs show joint space narrowing, subchondral sclerosis, or osteophyte formation. crp and esr(erythrocyte sedimentation rate)

osteoarthritis

pain in weight bearing joints after use improving with rest, morning stiffness for less than thirty minutes and age greater than 50.

osteoarthritis

what type of joint movement allows one to reach the anatomic barrier?

passive joint movement. and any movement pass this anatomic barrier can cause injury. Any movement beyond the anatomic barrier will cause anatomic (ligament, tendon, or muscle) injury.

what does stimulation of the M3 receptor in the eye produce?

stimulation of the M3 receptor produces miosis and blocking the receptor causes mydriasis

Swarming motility-positive nitrite test?

proteus mirabilis

what kind of shunt causes cyanosis and which one causes pulmonary hypertension?

right-to-left shunts cause cyanosis left to right shunts cause pulmonary hypertension

Name the paried bones?

temporal, parietal, maxillae, zygoma

what class of drugs does Isocarboxazid, phenelzine, selegiline, tranycypromine belong to?

they belong to Monoamine oxidase

A posterior tibia on talus, or anterior talus, can be daignosed over the anterior ankle over the talus. what happens with a posterior tibia on the talus

with a posterior tibia on the talus, the ankle has a decreased dorsiflexion due to the interference of the talus as it protrudes anterior to the tibia. of note, the ankle is more stable in dorsiflexion ans therefore trauma is more likely to occur while the ankle is plantarflex.

what is the pathogenesis of the snapping feeling frequently reported with thomas test from?

"SNAPPING" feeling is also frequently reported, which is caused by the iliopsoas tendon catching on the pelvic bone when the hip is flexed.

1. what is the normal Physiologic V/Q ratio in the base of the lung? 2. What is the normal physiologic V/Q at the apex of the lung?

1. 0.6 2. 3.

1. what does the umbilical artery become? 2. What is the ligamentum teres a remnant of? 3. What is the median umbilical ligament a remnant of? 4. What is the ligamentum arteriosum a remnant of? 5. What is the ligamentum venosum a remnant of?

1. medial umbilical ligament, not if a fetus developed only one umbilical artery, he or she would have a single medial umbilical ligament. 2. Ligamentum teres is a remnant of the umbilical vein 3. Median umbilical ligament is a remnant of the urachus 4. Ligamentum arteriosum is a remnant of the ductus arteriosus 5. LIgamentum venosum is a remnant of the ductus venosus

you suspect aortic dissection as the cause of chest pain in a patient with marfanoid habitus; i.e, tall stature, long extremities, spindly fingers, and hypermobile koints. marfan syndrome is caused by a deect of fibrillin due to point mutation of the FBN1 gene on what chromosome?

15

1. One or more significant somatic symptoms that are distressing or disrupt daily life. Patients have excessive thoughts or behaviors related to concern about one or more somatic dysfunction? 2. have preoccupation with having or acquiring a serious illness. have minimal (or very mild) somatic disorder. high level of anxiety about their health and may perform excessive health related behaviors A-Illness anxiety disorder B-Somatic Symptom disorder

1= B-Somatic symptom disorder 2= Illness anxiety disorder

hypotension, hyperkalemia, and increased renin activity as well as masculinization and female pseudohermaphroditism what problem are you thinking of?

21-hydroxylase deficiency.

when is screening for gestational diabeted performed?

24-28 weeks the cutt off for an abnormal glucose tolerance tests is 130-140mg/dl with gestational diabetes generally diagnosed with a value over 200mg/dl. An abnormal 1-hour test is followed by a 3 hour glucose tolerance test. the 3 hour glucose tolerance test provides a 100g load of glucose and includes a fasting (95 or 105 mg/dl cutoff), 1 hour (180 or 190 mg/dl) 2 hour (155 or 165 mg/dl) 3 hour (140 or 145 mg/dl) two or more values equal to or greater than the cutoff values indicate gestational diabetes.

When palpating a patient's cranium using the vault hold, where is the 4th digit placed? where does the other digits go?

4th digit is placed on the mastoid process of the temporal bone. The index finger (2nd digit) is located on the greater wing of the sphenoid The middle finger (3rd digit) is located on the preauricular portion of the temporal bone. the ring finer (4th digit) is located on the mastoid process. The little finger (5th digit) is located on the squamous portion of the occiput.

physical examination reveals an overweight female with arrest in inspiration upon deep palpation of the right upper quadrant. an anterior chapman point is expected to be found at? what is this organ? what other organ has this chapman point?

6th Intercostal space, midclavicular Gallbladder-pt has cholelithiasis. -four F's-female, fat, fertile and forty-upper quadrant pain radiates to the shoulder following a fatty meal The liver. the liver also corresponds to the 5th right ICS.

29 yo pregnant female, at 32 weeks gestation presents to emergency department with severe RUG abdominal pain and a temp of 37. acute tenderness int he right upper quadrant of the abdomen, hepatomegaly and an enlarged abdomen that is dull to percussion and positive for a fluid wave. murphy's negative. liver function tests shows elevated Alkaline phosphatase but normal AST AND ALT. abdominal ultrasounf demonstrates a spider web of collateral veins in the liver. which of the following pathological findings would you expect to see in this patient's liver?

A centrilobular congestion and necrosis Budd-chiari syndrome is a liver disease caused by occlusion of hepatic venous outflow, which leads to hepatic congestion and subsequent microvascular ischemia. patients commonly have a condition that places them in a hypercoagulable state and present with right upper quadrant pain, hepatomegaly, and ascites. obstruction of the intrahepatic veins leads to centrilobular congestion, and eventually necrosis, as a result of blood flowing into the liver but not being able to flow out. Acute thrombosis presents with the classic triad of right upper quadrant abdominal pain, hepatomegaly, and ascites. pts will have slightly elevated liver enzymes (AST, ALT, alkaline phosphatase), but laboratory values are otherwise typically normal. In both acute and chronic cases, abdominal ultrasounf will show collateral vessels in a spider web pattern and decreased hepatic venous blood flow. mortality is high-initial medical treatment is thrombolysis followed by anticoagulation. treatments that are more definitive include the TIPS (transjugular intrahepatic portosystemic shunt) procedure or hepatic transplantation.

what does a deficiency in CD40L called?

A deficiency in 40L is associated with an inability to class switch, resulting in hyper-IgM syndrome.

What is rumination?

A disorder characterized by voluntary regurgitation and re-chewing of partially digested food, which either is swallowed or expelled. this rare condition has been reported mostly in children and adults with mental retardation.

Which antibiotic has a disulfiram type reaction?

A disulfiram-like reaction is characterized by nausea, vomigint, headaches, and hypotension after consuming alcohol. This is a potential side effect of metronidazole.

The posterior limb of the internal capsule contains the descending corticospinal and corticobulbar motor tracts. A lacunar infarction of this area will result in what?

A lacunar infarction will result in purely motor symptoms of the contralateral hemiparesis and hemiplegia.

A lesion to what part of the brain will present with lower extremity weakess?

A lesion to the medial part of the brain.

35 yo woman, general malaise and a fever of two days duration. she has been congested with facial pain and mucus in the back of her throat. history reveals that she smokes a pack of cigarettes per day, has not drunk alcohol in five years and is married. her blood pressure is 130/80 mmhg, AND HER respiratory rate is 20/min. Osteopathic manipulative technique were performed, beginning with decreasing lymphatic congestion in the thoracic outlet--the most likely etiologic agent is? A-gram negative, lactose-negative, oxidase-positive rod B-Gram-negative, oxidase positive cocci C-Gram-positive, catalase-negative cocci D-Gram-positive, catalase-positive coagulase positive COcci E-Gram-positive , oxidase-negative rod

A is the answer. Strep pneumo is a gram positive, catalase-negative organism that is alpha-hemolytic and it is the msot common bacterial cause of sinusitis. --Remember MOPS. (MENINGTITIS, OTITIS MEDIA, PNEUMONIA AND SINUSITIS). strep pneumoniae is a gram positive, catalase-negative organism that is alpha-hemolytic and it is the most common bacterial cause of sinusitis. patients with sinusitis present with fever, post nasal drip, pain int he maxillary sinus region, and congestion. This is commonly caused by infection. smoking or deviated septum. pneumonia can often follow sinusitis if left untreated, and this can result in rusty-colored sputum with a lobar pneumonia. by decreasing lymphatic congestion in the thoracic outlet, the physician allows for adequate lymphatic drainage through their contents into the superior vena cava. This may be accomlished via direct or indirect techniques. B-gram-negative, lactose-negative oxidase positive rod is pseudomonas aeruginosa=MOST common cause of infection in cystic fibrosis patients. signs and symptoms of cystic fibrosis includes nasal polyps, recurrent respiratory infections, and malabsorption, and P.aeruginosa can cause a green-colored sputum in cystic fibrosis patients. C-gram negative, oxidase positive cocci=nesseria meningitidis. it wil ferment glucose and maltose. it is a common cause of meningitis but does not usually cause sinusitis D-Staphlococcus aureus is a gram-positive, catalase-positive coagulase positive coccus. while S. aureus causes many different clinical diseases, in the respiratory tract, it is found in patients with nosocomial pneumonia and can cause pneumonia following influenza A infections. S. aureus causes salmon-colored sputum D-Clostridium botulinum is a gram-positive, oxidase-negative anaerobic rod. it releases a toxin which causes inhibition at the neuromuscular junction. In babies, ingestion of honey where this commonly grows, results in "floppy baby" syndrome.

At the OA JOINT What type of motion occurs there?

At the OA joint, rotation and sidebending occur in opposite directions, although the OA joint does not follow fryette mechanics.

WHAT IS THE inheritance patten of MEN 1?

Autosomal dominant.

poor suckling in an infant is due to compression of what and what cranial nerve is affected? HOW DO YOU TREAT?

COMPRESSION OF THE OCCIPITAL CONDYLE AND CRANIAL NERVE 12. CONDYLAR DECOMPRESSION. KNOW where nerves exit the skull. this will be tested in cranial omm questions

Anti-centromere

CREST

Physical examination of the cranium reveals a slow, irregular cranial rhythmic impulse with very little amplitude. The most likely diagnosis at the sphenobasilar synchondrosis is?

Compression. may be palpated as a bowling ball that has very little motion.

What is a conversion disorder?

Conversion disorder is a psychological condition involving deficits of voluntary motor or sensory function that cannot be explained by a medical condition. Scheduled, regular follow-up i s the best form of treatment for these patients. mOST CASES will resolve on their own; however, it is still necessary to rule out any other medical conditions or underlying mood disorders in these individuals.

The diabetic drugs most likely to cause hypoglycemia are the sulfonylureas,

Correct

excess bleeding and widespread thrombosis. fibrinogen is decreased while fibrin split products are elevated? what is the most likely diagnosis?

DIC=Widespread thrombosis leads to ischemia and infarctions

A 44 yo male is admitted to the hospital with seizures and is noted to have a brain mass on imaging. he refuses to take anti-epileptics and does not want a biopsy of the mass. he continues to have seizures. which of the following is the first step towards treating this patient's seizures over his objection on a medical error?

Documentation of lack of capacity Documentation of a lack of capacity is necessary in order to treat a patient over objection. Treatment over objection is one of the most challenging ethical dilemma a physician will face. of note, treatment over objection on an inpatient psychiatry unit is a separate topic. typically to treat a pt over objection on an inpatient psychiatry unit the treating physician must go to court and petition a judge for an order to treat the patient. In this case the patient is on a medical floor (not an inpatient psych unit) and is refusing treatment for seizures that he continues to have as a result of the brain tumor. he could suffer morbidity and /or mortality from the untreated seizures. if the patient has capacity, i,e. can demonstrate an appreciation of the risks and benefits of the decision he is making, it is ethically appropriate to allow him to refuse medications. this would be respecting his autonomy. if he lacks capacity, however, and does not understand the consequence of his decision, then it could be ethically appropriate to treat him over his objection, to ensure his safety and reduce the risks of recuurent seizures and their sequelae. A capacity assessement is the first step in figuring out how to handle this complicated case.

Russel sign

Dorsal hand calluses from induced vomiting in bulimia nervosa

Which of the following arteries divides around the scapular attachment and descends to the inferior angle of the scapula? what nerve innervates the levator scapula muscle?

Dorsal scapular artery Dorsal scapular artery divides around the levator scapular muscle at the superior angle of the scapular and descends along the medial aspect of the scapula. dorsal scapular artery anastamoses with infraspinatus branch of the suprascapular artery and the circumflex sacpular artery to complete the anastamosis of the scapular. Levator scapular muscle is innervated by the nerve root of c3-c5 dorsal scapular nerve from c5 innervates rhomboids a branch of the dorsal scapular nerve (nerve to the levator scapulae) -also innervates it derived from c5.

Long axis kneading will be performed. which of the following is a description of this technique?

Engaging deep cervical muscular and utilizing rhythmic lateral bow stringing. soft tissue can be performed in various ways, but typically long axis kneading type soft tissue technique involves lateral bow stringing of the deep paraspinal musculature in a rhythmic fashion until softening of the deep paraspuanl musculature in a rhythmic fashion until softening, increased warmth, and reduced tenderness in the tissue is achieved.

Name Pinworm that presents with intense anal prutitus (itching) Pinworms are nematodes and dioecious (distinct male and female organisms)

Enterobius Vermicularis obligate parasites of humans

convex shaped head lesion on Ct think?

Epidural hematoma-middle meningeal artery=lucid state with progressively worsening headache confusion with depressed consciousness with headache often presenting sign and symptoms of a subdural hematoma chronic headache, seizures and papilledema are some of the symptoms associated with a tumor. tumor location highly depends on the brain location

Tinel's test is positive, how would you position the patient to provide the greates sympathetic relieve?

Extend the wrist and apply transverse traction to the flexor retinaculum. MFR aims to stretch and release muscle fascia restrictions.

physical examination of the cranium reveals external rotation if the parietal and temporal bones with flexion of the occipital and sphenoid bones-during this phase of craniosacral motion, the frontal bone moves through

External rotation The frontal bone acts as paired bones do, rotating externally under the influence of sphenoid during flexion of the midline bones. Despite the frontal bone been in the midline of the body embryologically it was formed from two bones that net together at the metopic suture.

what is fenfluramine and dexfenfluramine used for and what is the adverse effect?

Fenfluramine or dexfenfluramine are appetite suppressants and if used for more than four months-person has an increased risk for valvular defects

how to tell what T-scores mean Greater than -1? btwn -1 and -2.5 less than -2.5

Greater than -1=normal btwn -1 and -2.5=osteopenic less than -2.5=osteoporotic people who smoke or have DVT. calcitonin is another alternative

when does hPL increase in pregnancy?

Human Placental lactogen is elevated during the third trimester. It decreases the utilization of glucose, which may lead to glucose intolerance or gestational diabetes. Normal pregnancy is characterized by hyperplasia of the pancreatic beta cells, increased insulin secretion, and an early increase in insulin sensitivity followed by progressive insulin resistance. human placental lactogen (hPL), also called human chorionic somatomammotropin (hCS) or chorionic growth hormone, causes the increase in pancreatic beta cells in pregnancy. hPL has lactogenic and growth hormone actions and is most elevated during the third trimester. hPL increases maternal lipolysis, enhances ketogenesis, and decreases the utilization og glucose, which may lead to glucose intolerance or gestational diabetes. maternal insulin resistance results from increased placental secretion of growth hormone, CRH (increased ACTH and cortisol), hPL/hCS, and progesterone.

Deficiency of a-L-iduronidase think Hurler and has corneal clouding deficiency of Iduronate-2-sulfatase think hunters and no corneal clouding. which one is x-linlked recessive?

Hunter is X-linked recessive both of them have an accumulation of derman sulfate and heparan sulfate. remember Hurler enzyme as having a L in its name.

a child brought in by her mother. for third degree burns on her rightt hand after touching a hot stove. second time she has burned herself. she has become increasingly clumsy at times, dropping cups and pencils. family hx is only significant for multiple sclerosis. on the image you see an image in her spinal cord.this condition is associated with?

Hydrocephalus-in the mri you see collection of fluid within the central canal of the spinal cord. this leads to damage of the crossed fibers. (anterior white commissure) of the spinothalamic tract. syringomyelia occurs due to chiari malformation type 1 which is a caudal extension of the medulla and cerebellar vermis through the foramen magnum--majority of symptoms of chiari malformation are those related to obstructive hydrocephalus, abnormal eye movements, and cerebellar deficits, like the propensity to drop objects as in this case. caudal extension of the medulla and the vermis through the foramen magnim compress the spinal cord often leading to obstructive hydrocephalus--symptoms of hydrocephalus can be non-specific but headache is the prominent symptom-caused by distortion of the meninges and blood vessels. Can also be associated with nausea and vomiting indicating an increase in intracranial pressure. This increase in intracranial pressure can lead to a number of physical findings including bradycardia, hypertension, altered respiratory rate, cranial palsies, and motor dysfunction.

difference btwn insulin dependent and non-insulin dependent diabetes

If a person is insulin-dependent, a beta blocker may compromise the body's effort to break down glucose to use-this will lead to crisis (i.e shock( if the person is not given appropriate amounts of glucose to feed the brain and other organs. Metoprolol is a cardiospecific beta-adrenergic blocker, meaning that it works as an antagonist on beta-1 receptors. the functions of a beta =-blocker are negative chronotropic and inotropic effects. they are indicated in myocardial infarction and thereafter for rate and blood pressure control in order to decrease cardiac workload and prevent harmful remodeling of cardiac tissue. in diabetic patients, it may precipitate hypoglycemia by inhibiting glycogenolysis, or otherwise mask the early clues of low glucose, such as tachycardia.

IF YOU RAISE THE CUTT OFF OF A TEST WHAT IS LIKELY TO OCCUR?

If you raise the cut off of a test-this will increase the specificity and decrease the sensitivity.

An abnormal pap smear has been seen what is the next step in management?

If you see an abnormal papsmear. the next step in management would be to perform colposcopy and cervical biopsy.

what is the pathologic barrier?

In a joint with somatic dysfunction, this is the barrier that is met.

what is aprosodia?

Inability to change voice inflection, tone, pitch or rhythm. it also results in the inability to understand the emotional implications of the same in another's voice. It is caused by damage to the language centers in the non-dominant hemisphere of the brain. --neurological deficit where a person cannot portray or understand emotional context of language.

What type of Q angle do you expect to see in patients with patellar subluxation?

Increased Q angle

would inducing or inhibiting CYP 450 raise or decrease toxicity from acetaminophen?

Inducing P450 enzymes (phase 1 reactions) increases the risk of liver failure because it increases the amount of acetaminophen metabolites.

what type of breast cancer is this? Grows in lactiferous ducts of the breast, typically located beneath the areola. it often presents as unilateral nipple discharge consisting of bloody or serous material.

Intraductal papilloma

bowel telescoping on itself most commonly at the ileocecal junction. happens to infants 6-36 months of age.

Intussuception

Abdominal pain tends to be colicky and severe, vomiting can be non-bilious at first and then becomes bilious as obstruction occurs, and lethargy is another common presentation. patients are usually awakened from sleep with severe abdominal pain, which causes them to be irritable and draw up their legs-they vomit and may pass a stool and improve temporarily before the cycle repeats.

Intussusception

Cheyne-stokes respiration

Is a pattern of cyclic bradypnea, followed by apnea and then hyperventilation. it is indicative of central sleep apnea which differs from obstructive sleep apnea in that it is not a structural airway abnormality but rather a dysfunctional respiratory drive resulting from a lesion in the CNS respiratory CENTERS. Commonly associated with stokes, traumatic brain injuries, brain tumors or other CNS insults. It is commonly seen in pts who are close to death.

A 23 yo female presents to her gynecologist for her annual check-up. the first day of her last period was 20 days ago. the hormone which is most elevated in her blood right now?

Is progesterone which causes cervical mucus to become thick. progesterone casues the cervical mucus to become thick and provides negative feedback for LH secretion, it is the most elevated hormone at day 20 of the menstrual cycle. This patient is in the secretory phase of her menstrual cycle (day 20) which means that the most dominant hormone is progesterone. progesterone causes the cervical mucus to become thick in an effort to prevent sperm and other foreign material from entering the uterus. this is appropriate after ovulation and fertilization have taken place. estrogen causes the cervical mucus to be thin and watery to aid sperm in reaching the ovum.

A type of eccentric contraction used to release adhesions is called?

Isolytic.

What is the the moa of sirolimus?

It binds mTOR, inhibiting the proliferation of T-cells in response to IL2. THE PRimary mecahnism of action of sirolimus is that it creates a complex with intracellular immunophilins (FKBP) AND BINDS TO mTor inhibiting T-cell PROLIferation in response to IL-2. this is normally given in conjunction with cyclosporin and corticosteroids after a kidney transplant to suppress organ rejection. this drug can cause hyperlipidemia, thrombocytopenia, and leukopenia.

What will acute alcohol intoxication lead to?

It can lead to hypoglycemia. signs slurred speech, ataxia, and impaired consciousness. acute alcohol intoxication can lead to hypoglycemia due to reduction of the metabolites involved in gluconeogenesis.

What is the pronator teres syndrome?

It is a less common cause of median neuropathy that is easily confused with carpal tunnel syndrome. Pronator teres syndrome is a median neuropathy that is caused by compression of the median nerve between the two heads of the pronator teres. It is due to overuse of the pronator teres muscle and seen in patients who do repeated forearm pronation activities such as using a screwdriver. Pronator teres syndrome is a less common cause of median neuropathy than carpal tunnel syndrome. It can be differentiated from carpal tunnel syndrome by the presence of forearm pain and a negative Tinel's sign in the carpal tunnel, and a negative phalen sign. carpal tunnel syndrome symptoms are worse at night, unlike pronator teres syndrome symptoms, which are worse during the day.

A pharmaceutical company wants to give you money and a stethoscope, which one should you accept?

It is okay to accept medically related gifts from pharmaceutical companies, but it is not okay to accept money or gifts unrelated to your education.

27 year old female presents to the office complaining of left knee pain. history reveals the pain starting gradually over the last three weeks while training for a marathon. the pain is located over the left lateral hip and lateral knee which of the following is true regarding the affected structure in terms of where it originates? and what would be the most appropriate orthopedic test to perform to confirm diagnosis?

It originates proximally from the tensor fascia latae muscle ITB (Iliotibial band) syndrome is the most common cause of knee pain in athletes, especially long distance runners. the iliotibial band arises from iliac crest, tensor fasciae latae and gluteus minimus and medius muscles proximally, travels along the lateral side of thigh and inserts on Gerdy's tubercle on lateral aspect of tibia. Ober's test

adhesive capsulitis

Joint capsule contracture adhesive capsulitis is a condition of decreased shoulder range of motion affecting primarily middle-aged and elderly patients. diabetes is a common comorbid condition. mostly caused by immobilization. Pay close attn to abduction. remember shoulder abduction can be achieved at both the glenohumeral joint and the scapulothoracic joint. ADHESIVE CAPSULITIS involves thickening and contracture of the glenohumeral joint capsule but does not directly affect the scapula to glie the thorax. this means that there should still be some smooth abduction and elevation of the shoulder, but the astute physician will note that the glenohumeral joint remains at the same angle while the scapula glides upward.

what is a possible complication of measles?

Koplik spots are diagnostic of measles. delayed, fatal encephalitis is a possible complication of measles infection.

when palpating her lumbar spine, you note asymmetry and tissue texture abnormalities. the pt is diagnosed with L2ERS(R). USING MUSCLE ENERGY TECHNIQUE, what is the correct initial treatment position for this pt?

L2FRS(L) read well and make sure when you pick you pick the correct answer choice. don't make dump mistake.

What are some risk factors for squamous cell carcinoma of the penis?

Lack of circumcision, phimosis, HPV 16 and 18, tobacco use and carcinoma insitu. induration on skin of penis, uncircumcised male, prepuce is malodorous and necrotic. biopy shows keratin pearls-risk factor is hpv. Note-peyronnie's disease i a benign condition of the penis that does not advance to carcinoma. it clinically presents as a bent penis due to fibrous tissue formation.

The action of a screwdriver is more like that of a tennis racket (grasping with elbow twist) than of keyboarding (wrist flexors). what kind of injury do you expect with using a screwdriver? with several tenerpoints just distal to the radial head?

Lateral epiconylitis associated with using screwdriver, racket sports. these activities results in excessive extension and supination of the forearm. tenderpoints near the lateral epicondyle results from microtears in the extensor muscle tendons that attach at the lateral epicpndyle, especially the extensor carpi radialis brevis. patients often complain of lateral elbow pain which can be reproduced with resisted wrist extension (cozen's sign). Lateral epicondylitis (tennis elbow) is associated with activities that involve grasping at the hand and twisting at the elbow. Treat with conservative measures such as rest, activity modification, oral NSAID's, corticosteroid injections, or OMT (usually with counterstrain or myofascial release techniques). if these measures fail surgical debridement may be useful.

you see dominant S wave in V1 (that looks like a W); N=WITH NOTCHED (M-shaped) r wave in V6 what do you suspect?

Left bundle branch block

A person was uppercut on the left jaw-how would you expect the changes to be like when doing the vault hold?

Left index fingers would be superior to the right. When fingers are in vault hold position palpating for a left torsion strain, the physician will note that the left index finger moves superiorly in relation to the right.

what branch of the mandibular nerve provides sensation to the anterior tongue?

Lingual nerve. mental nerve provides sensation for the chin.

where does the lower extremity receive sympathetic innervation from?

Lower extremity receives sympathetic innervation from T10-L2.

where is the vagina, prostate, an broad ligament chapman point?

MEDIAL POINT OF PSIS

65 yo old patient int he intensive care unit (ICU) on a respirator develops a fever. lab eval reveals leukocytosis. pHYSICAL exam reveals decreased breath sounds. Chest x-ray shows pulmonary infiltrates. which of the following is the most likely diagnosis?

Pneumonia caused by pseudomonas aeruginosa. pseudomonas aeruginosa is the most common pathogen to cause nosocomial pneumonia due to use of a respirator. why is it not legionella?--Legionelle pneumophila is a less likely cause of respirator pneumonia than p. AERUGINOsa, ALTHOUGH IT IS A POSSIBILITY. l. pneumophila is acquired via contaminated aerosol droplets, commonly from air conditioning units, showers, and mist sprayers. In addition, L. penumophila typically presents as an atypical pneumonia with other symptoms, such as hyponatremia, confusion, and diarrhea.

What kind of tears can be detected with hawkins-Kennedy test, apprehension test, or near impingement test?

Rotator cuff tears. The Hawkins-Kennedy impingement test is performed by forward flexing the shoulder and elbow to 90 degrees while passively (by the physician) internally rotating the shoulder. Pain indicates a posiive test. other tests likely to be positive include a drop arm test, neer impingement test, and apprehension test, as well as a poor range of motion and muscle strength.

most common cause of bacterial meningitidis in the elderly?

S. pneumonia

WHEN IS STEADY state reached?

Steady state is reached within 4-5 half lives. regardless of the patient's weight, type of drug, or rate of infusion, the time to reach steady state can be quickly calculated by multiplying the half life of the drug by 4 and 5. the steady state will be reached within this range.

What is Staghorn Caliculi?

Stones that involve the renal pelvis and extend into at least two calyces.

what provides sympathetic innervation to the lower extremity?

T10-L2

what bones makes up the pterion

Temporal, parietal, sphenoid and frontal bones

what type of headache is this? mild headache associated with muscular stress and equally distributed pressure across affected areas

Tension headache.

physical examination reveals ribs 4-6 are restricted during inhalation. what best described the patient's somatic dysfunction a-anterior aspect of rib 4 moves caudally b-the posterior 4th rib angle is depressed ?

The anterior aspect of rib 4 moves caudally

With sacral torsions, the axis will be on what side of the positive seated flexion test?

The axis will be on the opposite side

The fibular has see-saw motion btwn the fibular head and the lateral malleolus. the lateral malleolus moves anterior as the fibular head moves posterior and vice versa. As the lateral malleolus moves anterior what happens to the foot?

The foot inverts and plantarflexes. note posterior fibular head dysfunctions may result from inversion ankle sprains due to the mechanics btwn the ankle and fibular head.

what does lesions in the right lateral medulla which involves the trigeminal nerve also possibly associated with"

The lesion in the right medulla, probably involving the spinal tract of the trigeminal nerve and surrounding structures, including the spinothalamic tract, horner's syndrome, which is always ipsilateral, may result from damage to the descending hypothalamic fibers, which course with the spinothalamic tract in the lateral medulla. --the spinothalamic tract carries pain and temperature sensation from the body to the primary sensory cortex of the parietal lobe. Horner syndrome is the triad of miosis, ptosis, and anhidrosis.

what is the null hypothesis in a cohort study stated as?

The null hypothesis in a cohort study is that the risk of developing disease is the same in the exposed and unexposed group.

The use of anti-arrhythmic medication procainamide can result in a drug-induced lupus syndrome. Anti-histone antibodies will be present in the serum. explain some of the drug-induced lupus condition this patient is experiencing?

The pt has a new onset chest pain that is pleuritic and positional in nature, suggestive of acute pericarditis. pts with drug induced lupus can develop a variety of systemic symptoms, including fever, myalgias, rash, arthrlagias, arthritis, pericarditis, pleurititis, and serositis. Anti-histone antibodies are present in drug induced lupus. The anti-histone antiboides are formed against a complex of the histone dimer H2A-H2B and DNA. in contrast to that seen in systemic lupus erythematous, anti-double stranded DNA antibodies are absent in drug-induced lupus. the gold standard diagnosis is spontaneous resolution of the disease after offending drug has been discontinued.

Direction of the rotation is dependent on the results of the lumbosacral spring test In patients with a negative lumbosacral spring test (a forward sacral torsion) the sacrum will rotate in what direction os the axis?

The sacrum will rotate in the same direction as the axis (i.e. left on left) this concept is high yield for step

Direction of the rotation is dependent on the results of the lumbosacral spring test In patients with a positive lumbosacral spring test (a backward torsion), the sacrum rotates in what direction?

The sacrum will rotate opposite of the axis (right on left). this concept is high yield for step

Vitamin B12 deficiency can result in subacute combined degeneration, commonly manifested by upper motor neuron signs and proprioceptive changes. The schilling test can be utilized to determine the etiology. explain what the schilling test is used to confirm?

The schilling test can be utilized to confirm the suspicion that a lack of IF (intrinsic factor) is to blame for the cobalamin deficiency.

what adverse effects can digoxin, calcium channel blockers and beta blockers cause?

They can cause first degree heart block PR interval >200ms. pts are usually asymptomatic and the diagnosis is usually an incidental finding on ECG. it may be idiopathic or it may be related to other underlying cardiac diseases (eg, coronary artery disease or valvular disease). it is also seen as a normal variant in highly trained atheletes. Iatrogenic causes include digoxin, centrally acting calcium channel blockers, or beta blockers. WHILE MOST PATIENTS can just be monitored, in cases where the patient is symptomatic or thee is concern for progression to higher degree AV block, the first step in treatment is to discontinue the offending drug.

whAT STAGE OF SLEEP does night terror occur in?

This occurs during non-REM stages of deep sleep (stage 3/4). these are the deepest stages of sleep, and patients cannot recall events that occur during these stages. generally treatment with medication is not necessary if sleep terrors are infrequent and do not interfere with daily functioning. In severe cases of sleep (night) terrors as described int he case, benzodiazepines such as clonazepam or diazepam can be effective.

65 yo female presents to the emergency room with left sided chest pain of three months duration, she describes the pain as burning, constant, non-radiating, and not associated with exertion or shortness of breath. she states that there was a painful bumpy rash in that area for which she had taken some pills prescribed by her doctor and it resolved a month ago. what do you think this patient has?

This patient has post-herpetic neuralgia, which is a condition that is defined as pain for more than one month duration after the disappearance of a herpetic zoster rash, otherwise known as shingles.

A pt with atrial fibrillation was given a medication to convert her sinus rhythm. she complains of two months history of dyspnea. on exam you notice decreased chest wall motion globally, hypertonic scalenes and trapzius muscles bilaterally and paravertebral hypertonicity from T2-T6 bilaterally. what medication did this patient take and what does this patient have?

This patient took Amiodarone And this patient has pulmonary fibrosis which amiodarone can cause Pulmonary fibrosis has literally hundreds of etiologies. patients present differently depending on the underlying cause. Drugs, radiation exposure, and autoimmune processes are some common culprits. Patients can present with dyspnea, digital clubbing, and crackles on auscultation. examination may reveal increased use of accessory respiratory muscles, viscerosomatic changes from T2-T6, poor chest wall motion secondary to the restrictive nature of this lung disease. Typically, pulmonary function testing will reveal a restrictive pattern and a low carbon monoxide diffusion capacity (DLCO). Amiodarone, in particular, is a medication known to cause pulmonary fibrosis with longstanding use. The patient in this clinical scenario has likely been taking amiodarone for the last 20 years and now presents with dyspnea on exertion likely secondary to pulmonary fibrosis.

this was a third order question-you know someone has strep throat and you know it can lead to rheumatic fever but the question asks what condition is this person likey to suffer if not treated with pcn which ic supposed to help prevent rheumatic fever?

This person is likely to suffer from one of the JONES criteria that rheumatic fever can cause such as arthritis.

A 12 year old female presents with weight loss, abdominal pain, and proglottids in her feces what Interleukin IL4 OR IL5 do you think this is associated with?

This will be associated with Interleukin 5 Interleukin 5 stimulates the production of eosinophils, and it is integral in immunity against helminthic infections. IL4 is involved in humoral immunity. It promotes B lymphocyte isotype switching to IgE. It plays a role in asthma and atopy. it does not play a role in helminthic infections.

what orthopedic test do you use to test for psoas syndrome="snapping" sensation with flexion of the hip? and a tenderpoint medial to the asis on one side?

Thomas test

True or false, the hypoxia caused by obstructive sleep apnea will increase poduction of erythropoietin from the kidneys, which will increase production of red blood cells. true or false?

True

True or false- DO NOT resuscitate in the case of cardiac arrest. It does not mean that the patient should not be intubated in the case of respiratory distress. iF THE PATIENT WISHED TO NOT BE INTUBATED AS WELL, THEN HE/SHE WOULD HAVE TO BE DESIGNATED AS dnr/dni (DO NOT RESUSCITATE/Do not intubate). True or false?

True

The foramen ovale allows oxygenated blod to bypass the pulmonary circulation.

True note only 50% of the fetal blood bypasses the portal circulation via the ductus venosus==true

Lithium is a common augmenting agent in patients with resistant depression and can also be used as maintenance therapy to prevent recurrence of depression true or false?

True.

where does a wide QRS come from and a narrow QRS?

Wide QRS comes from a ventricular focus while a narrow QRS comes from a supraventricular focus for a wide QRS-ventricular tachyardia-immediate cardioversion via defibrillation is warranted.

A clinical scenario is presented that is vague but it is likley transmitted by a flea-the most likely cause then is therefore?

Yersinia pestis out of the other choices. yersinia pestis is a bacterium, gram-negative, coccobacillus that is nonsporulatin. it can be spread by fleas which are commonly found on rodents, especially rats worldwide (particular southeast asia) and in the prairie dog in the southwest U.S. DISEASE appears after 2-8 days of incubation and is characterized by the presence of exquisitely tender, palpable lymph nodes called buboes. the skin ulcers seen in Y.pestis can be painful. in addition Y.pestis can be spread via aerosol droplets causing a pneumonic plaque which is a hemorrhagic, highly fatal pneumonia. this form of the disease does not require fleas for transmission but can be spread person-person. Aminoglycosides, such as streptomycin, is the drug of choice to treat bubonic plaque. post-exposure prophylaxis include a seven day course of oral doxycylcine and ciprofloxacin.

Do those with sarcoidosis have an elevated angiotensin converting enzyme (ACE) level and hypercalciuria?

Yes

does EMTALA EMERGENCY medical treatment and Active labor act provide treatment to employees injured at work?

Yes

Can Alendronate increase bone density in women with post-menopausal osteoporosis?

Yes Osteoporosis is defined as a T score of -2.5 or less. T score is an indicator of bone density of a patient compared with 30-year old patient of the same-sex. Normal T score is -1.0 or greater Osteopenia is defined as a T score of -1.0 to -2.5 Alendronate is a bisphosphate that inhibits bone resorption (osteoclast activity) and has been found to increase bone density in women with post-menopausal osteoporosis. it must be taken with water to prevent esophageal perforation. Osteonecrosis is another complication that is caused by suppression of bone turnover.

does endometriosis cause an increase risk of infertility?

Yes, ENDOMETRIOSIS IS DUE TO functioning glands and stroma outside of the uterus and causes an increased risk of infertility. diagnosis of endometriosis is by direct visualization. treatments includes NSAIDS, oral contraceptives or GnRH analogues,

Korsafoff is caused by chronic thiamine deficiency often due to alcoholism. is cofabulation associated with korsakoff?

Yes, Korsakoff syndrome is characterized by anterograde and retrograde amnesia, and patients confabulate to fill in memory gaps. korsakoff is caused by chronic thiamine deficiency often due to alcoholism. The mamillary bodies are the area of the brain affected by thiamine deficiency, not the frontal lobe.

can you see a positive babinski sign in multiple sclerosis?

Yes. Multiple sclerosis is an autoimmune neurological disorder that may present with injury to any location in the CNS, INCLUDING the upper motor neuron. Upper motor neuron lesion can present with hyperreflexia, positive babinski sign and spasticity.

In practice anterior sacral torsions are most commonly physiologic dysfunction which occur while the patient is walking or stumbling forward, while posterior sacral torsions are more commonly caused by forces pushing the patient backward. sacral torsion equation for diagniosis is?

axis over rotation

What is bipolar type 1 and type II?

bIPOLAR type II is characterized by at least one hypomanc episode and at least one major depressive episode. depressed episode are usually more frequent than hypomanic episodes. bipolar 1 is defined by the presence of at least one manic episode. pts symptoms currently do not meet criteria for a manic episode because they have lasted only for four days and have not caused social or occupations impairment.

what fly transmits onchocerciasis? AND WHAT IS THE TREATMENT?

bLACK FLY AND TREATMENT IS ivermectin.

WHERE IS THE CHAPMAN POINT FOR THE APPENDIX LOCTED?

chapman point for appendix is located at the tip of the 12th rib and halfway between the spinous and transverse process of T11

where is the chapman point for the bladder located?

chapman point for the bladder is located in the periumbilical area.

PT IS 87, HAS RHEUMATOID ARTHRITIS AND AND HAS ACUTE ON CHRONIC NECK PAIN. INVOLVING MYOFASCIAL TISSUE. BEST TREATMENT TO DO?

counterstrain.

63 yo female, chest pain exacerbated by breathing began three hours ago. hx reveals hip replacement surgery 3 wks ago and prefers to stay in bed bc of pain. PE reveals mild erythema and swelling of the left lower extremity. An electrocardiogram shows sinus tachycardia. computes tomography of the chest is obtained. most likely additional finding would be?

elevated fibrin degradation product risk factors for PE include immobility and vascular injury (such as surgery). Fibrin degradation products will be elevated, indicating a hypercoagulable state this is consistent with a PE. pe of swelling and redness in the lower extremity can indicate DVT.

pulmonary-renal syndrome suggest?

goodpasteure syndrome.

SLE

hydralazine and procainamide

Hypertensive strokes form lacunar infarcts in what part of the brain?

hypertensive strokes form lacunar infarcts in the basal ganglia.

what are the risk factors to obstructive sleep apne?

hypothyroidism, large neck circumference, obesity, male sex, a narrowed airway, a family hx and use of alcohol or sedatives.

Anti-U1 RNP

mIXED CONNECTIVE TISSUE DISEASE

A pregnant woman has chlamydia trachomatis D-K what is the treatment of choice?

macrolide such as erythromycin, azithromycin or amoxicilin.

note even though you may not know what the murmur is cardiac valvular disease should make you think of--mitral stenosis.

mitral stenosis --most common by rjeumatic fever.

ampicillin and clindamycin can lead to?

pseudomonas colitis

presence of fever with dysuria, urinary frequency, and flank pain suggests?

pyelonephritis secondary to a UTI.

what muscle protects the brachial plexus when the clavicle fractures?

sUBCLAVIUS SUBCLAVIUS will pull the lateral portion medial and anteriorly away from the brachial plexus

two interventions that have been shown to improve mortality in persons with COPD?

smoking cessation and supplemental oxygen administration if the pt qualifies for it.

most common type of ankle sprain involves inversion of the foot. which stretches what ligament?

stretches the lateral ligaments of the ankle-most commonly sprained ligament is the anterior talofibular ligament.

what do you use to treat post-herpetic neuralgia?

tricyclic antidepressant like amytriptyline, gabapentin and pregabalin

Peptic ulcer disease is very common and is usually caused by an infection with H.pylori. It has been shown that peptic ulcer disease is more prevalent in patients with the type O blood group t/F?

true how to remember this is Peptic Ulcer think Olcer peptic ulcer are found more in the first part of duodenum.

True or false--type 2 diabetes mellitus is a result of insulin sensitivity in ther peripheral tissues and an inability to autoregulate hepatic gluconeogenesis resulting in elevated blood glucose levels. These mechanisms result in an excessive amount of endogenous insulin production until B-islet cells eventually burn out and pt becomes insulin dependent on exogenous forms.

true type 2 diabetes contain beta islet cells, but chronically elevated blood glucose levels can lead to a decreasd B-islet cell function and decreased endogenous levels of insulin. remember two separate fasting glucose levels >126 is indicative of type 2 dm.

With impaired kidneys-25-hydrocycholecalciferol (calcidiol) will accumulate in the blood

true 25-hydroxycholecalciferol is also known as calcidiol pay attn --don't go and pick what you know to be the correct answer-make sure you check to see if that is actually answering the question. kidney is not functioning so the end product will not be produced.

Adults with chronic exposure to lead may develop a peripheral neuropathy which can manifest as a foot or wrist drop, delirium, hallucinations or hypersomnolence. lead toxicity results in the characteristic basophilic stippling of the red blood cells seen on a peripheral blood smear. what is the most likely mechanism of the peripheral blood smear finding?

true. Denaturation of the enzyme ribonuclease-thus ribosomes persist in the red blood cells and leads to basophilic stippling. these ribosomal precipitates are seen as blue granules of various sizes that are dispersed throughout the cytoplasms of the red blood cell. it is most often seen in beta thalassemia, alcohol abus, iron overload and lead poisoning.

Unstable angina is characterized by progressively worsening chest pain and can often occur at rest. It is a harbinger of impending plaque rupture that could lead to myocardial infarction. it is impt ot distinguish between the various presentations of coronary artery disease. pain that begins when sitting and watching TV is classic for?

unStable angina due to disruption and thrombosis of an atherosclerotic plaque

what would a right torsion feel like?

A superior right index finger.

what medication slow the progression of diabetic nephropathy and are therefore, the initial drugs of choice for diabetic with hypertension?

ACE inhibitors. Must know things about ACE inhibitors 1-cough and angioedema are possible side effects 2-they cause hyperkalemia 3-they increase survival in congestive heart failure 4-they are contraindicated in bilateral renal stenosis

AIDS dementia complex

AIDS-dementia complex presents in end-stage disease and is associated with subacute combined degeneration of the spinal cord. multi-nucleated giant cells are diagnostic. tHIS PATIENT has AIDs-dementia complex, which affects people with end-stage disease. physical examination showed subacute combined degeneration (corticospinal and dorsal column tracts degenerate), which occurs when human immunodeficiency virus affects the spinal cord. multi-nucleated giant cells are diagnostic of AIDS dementia.

34 yo male with no significant past medical hx present to his physician's office for a pre-employment physical exam. his review of systems is negative and his physical exam is unremarkable. at the end of the visit he presents the doctor with a form that must be completed, the form requests the results of a number of genetic tests that the company requires for all potential salaried employees. the most appropriate course of action is for the physician t?

Advise the patient that is could be a violation of GINA (Genetic information nONDISCRIMINATION Act) of 2008--made it unlawful employment practice for an employer to discriminate based on genetic information or to request genetic information. prior to GINA there was no federal protection for employment and potential employees. while many individual states had their own genetic non-discrimination laws, they varied significantly and did not adequatley protect individuals. COMpanies cannot require genetic testing as a pre-requisite of employment. the genetic information nondiscrimination act of 2008 was designed to prevent discrimination in health coverage and employment based on genetic information.

?describes an episode of retinal ischemia leading to transient monocular loss of vision. it is suggestive with a apid and painless onset in which the loss of vision is comparable to a curtain or shade falling down over the eye.

Amaurosis fugax

43 yo female left sided chest pain. had a cold last week but otherwise healthy. has pain with resisted adduction of the arm while the shoulder is in a flexed position, and tenderness to palpation over the upper left anterolateral rib cage. You localize a point of tenderness and decide to treat osteopathic manipulation. given the above hx, and physical exam findings you decide to trat with?

Anterior rib counterstrain technique physical exam findings only suggest tenderness without a specific segmental finding that would be required for proper setup in order to treat with ME or HVLA. as there is no mention of restricted motion, asymmetry of rib motion, or any specific segmental somatic dysfunction, there is no clear indication for muscle energy or HVLA technique.

where is the chapman reflex for the prostate located?

Anteriorly along the posterior margin of the iliotibial band.

Asbestos is one of several pneumoconioses and is commonly linked to occupational exposures in old naval shipyards, roofing material, ceiling tiles, and the demolition of old buildings. pts most commonly present with dyspnea on exertion and dry cough which can progress to hemoptysis. --Additionally, pulmonary function testing in these patients often reveals a restrictive pattern with a decreased FEV1, decreased FVC, and a normal FEV1/FVC ratio. Asbestos is a family of crystalline hydrated silicates that exists in two different geometric forms, serpentine and amphibole fibers. what are asbestos bodies?

Asbestos bodies are golden-brown, fusiform or beaded rods that consist of these asbestos fibers which are coated with an iron-containing proteinaceous material.

undecided on whether to do dialysis on a pt and the pt's wife will inherit money. pt is uremic and can't decide what to do what should you as the physician do before starting dialysis??

Ask the patient's wife whether or not to initiate dialysis We trust that surrogates are making decisions in the best interest of the patient unless there is evidence to the contrary.

what food should those with celiac disease avoid?

Avoid wheat as gluten is found in wheat. when someone has one autoimmune condition another autoimmune condition is also likely

after 7 days what else is a viable method of emergency contraception?

Copper IUD may be used as a method of emergency contraception and is effective for up to seven days after an unprotected sexual encounter. copper exert direct inhibition of fertilization and creating an intrauterine environment that is not suitable for implantation.

Respiratory syncytial virus is the most common cause of bronchiolitis, especially in the winter months

Correct rHINOVIRUS IS THE SECOND MOST COMMON CAUSE OF THE COMMON COLD. RHINOVIRUS IS ASSOCIATED WITH BRONCHIOLITIS IN THE SPRING AND FALL.

what allows microbes that infect the kidney to ascend up the urinary tract?

FLAGELLA

True or false chapman reflex points are seen only in chronic disease states?

False Chapman reflex points correlate with increased sympathetic tone to a particular body area. they can be acute or chronic, and there are about fifty discrete points throughout the body.

true or false? The sympathetic nervous system arises from the craniosacral region, while the parasympathetic nervous system arises from the thoracolumbar region?

False. read question well-they were asking what is true or false. highlight things in question. you do not want to miss easy points.

A person once immunized against one vaccine, is also immunizes against another one, which one is this?

Hepatitis D virus is a delta virus that requires a prior infection with hepatitis B in order for the virus to cause clinical disease.

10yo male is brought to the emergency department by a neighbor after suffering a laceration to the forehead while playing outside. both parents are notified and are on their way to the hospital. physical examination reveals a 2-cm laceration. it is only oozing blood but will require sutures. the most appropriate next step is to? should you go ahead and suture?

Hold pressure dressing and delay sutures until consent is obtained from one parent-reason is because parental consent is necessary for the treatment of minors in a non-emergent setting.

Pregnant woman at 35 weeks of gestation is involved in an accident. which of the following findings on ultrasound would be most specific for her current condition?

Hyperechoic hemorrhage in comparison to the placenta. Placental abruption is an emergency. The key difference between previa and abruption is the presence of pain. aBRUPTION IS A PAINFUL bleed. Placental abruption is premature separation of the placenta from the uterine wall, often leading to delivery of the placenta before delivery of the fetus. it is generally associated with painful bleeding. if the fetus is not delivered immediately there is significant morbidity and mortality to both mother and fetus. Acutely a hemorrhafe is generally hyperechoic in comparison to the placenta and retroperitoneal clot can be noted. However, not all abruptions are detectable on ultrasound. ultrasound is used mainly to rule out other causes of third trimester bleeding in order to narrow the diagnosis down to placental abruption.

what is the treatment for crush injury that can result from hyperkalemia?

Hyperkalemia is treated with insulin, B-agonists, and sodium polystyrene sulfonate to lower circulating potassium Calcium gluconate can be used for cardioprotection when arrhythmias are present. Calcium gluconate is often the first medication administered as a cardioprotective agent. Other medications that would improve hyperkalemia include Beta-adrenergic agonists (such as terbutaline or albuterol), insulin, and sodium polystyrene sulfonate (Kayexalate). Note Glucagon helps transfer potassium out of cells. it would therefore, worsen hyperkalemia.

A patient comes in with pain, an acute condition, what treatment would you want to use?

Indirect techniques are typically used for acute somatic dysfunctions, as they prevent further strain that may occur in a direct technique. Examples of indirect techniques are myofascial release, counterstrain, and balanced ligamentous tension. using direct technique on an acute patient may result in increased muscle spasm and/or pain

when palpating the sacrum and the index fingers move superiorly and the fifth fingers move inferiorly. the most likely diagnosis at the sphobasilar synchrondrosis is?

Inferior vertical strain.

While palpating the cranium in the vault hold, you notice that the greater wings of the sphenoid seem to shift to the right, while base of the sphenoid deviates left in relation to occiput. you also note that the CRI is approximately 11 cycles per minute. which strain pattern is consistent with your finding?

Left lateral strain. The case is consistent with lateral strain of the SBS. lateral strain are palpable when you feel the base of the sphenoid shift in relation to the occiput. The lateral strain is named for the direction the base of the sphenoid deviates. If the base of the sphenoid deviates to the left in relation to the occiput it is termed a left lateral strain. This will result in the greater wings of the sphenoid shifting to the right, creating a parallelogram shape to the cranium.

poor rib motion on the left. ribs 2-5 on left do not fall well into exhalation while the remaining ribs all move symmetrically into inhalation and exhalation. you to treat him with BLT of the ribs. what it the correct diagnosis and which rib will you use to treat the dysfunction?

Left rib 2-5 ISD, key rib 5, USE RIB 5 FOR blt. key rib is with the bite mnemonic

physical exam reveals the greater wing of the sphenoid is more superior on the left and the occiput seems to be rotated in the opposite direction. the most likely diagnosis is ?

Left torsion.

A woman inquires when she is ovulating what do you tell her?

Ovulation can be approximated using basal body temperature----Higher levels of progesterone released by the corpus luteum after ovulation raises basal body temperatures. This increase is often seen the day following ovulation

An acute asthma attack should be treated with?

Oxygen, short, acting beta-agonists and IV steroids. B-blockers are contraindicated in asthma

Porphyria cutanea tarda results from either acquired or inherited disorder in the enzyme uroporphyrinogen decarboxylase. clinical manifestations of the disease are triggered by anything that increases body iron levels (IRON SUPPLEMENTS). What is a classic presentation?

Painful bliters on her hands and forearms and darkened urine. physical examination reveals hyperpigmentation and hair growth on the face.

you find a tenderpoint 2 inches medial to the ASIS deep in the iliac fossa. how should you position the patient to treat the iliacus somatic dysfunction using counterstrain?

Pateint is supine with hips and knees flexed and ankles crossed with external rotation of the thigh=frog position

SOMEONE HAS DYSMENORRHEA WHERE ARE YOU LIKELY TO FIND THE CHAPMAN POINTS?

Pateints suffering from dysmenorrhea are likely to have chapman points at L5 and medial to the PSIS.

what pathologic effects leads to this-insulin antagonism, nitrogen retentionm increased risk of colon polyps/tumors and acral (affecting limb or apex) overgrowth

Pathologucal effects of growth hormone

on sacroiliac exam, you find positive seated flexion test on the right, deep sacral sulcus on the left, posterior/inferior ILA on the right, and a positive spring test. what is the correct treatment ?

Patient is in left lateral recumbent position with back toward the table and right hip flexed and knee slightly in front of the left leg. The patient is then instructed to raise the right leg towards the ceilling against the physician's resistance. for treatment of all sacral torsions the patient lies in lateral recumbent on the side of the axis. the front of the patient;s torson is on the table in forward torsions and the back of a patient's torso is on the table in backward torsions. both legs are used for forward torsions and only the top leg is used for backward torsions.

on physical exam, you note tenderness at the proximal insertion of the plantar fascia, restricted dorsiflexion and a tender point on the proximal portion of her posterior calf. How do you treat the posterior calf tenderpoint with counterstrain?

Patient prone, knee flexed, plantar flex the ankle, fine tune movements until tenderness is alleviated by at least 70%.

drugs that are eliminated have a constant rate of elimination regardless of plasma drug concentration -also known as zero order are?

Phenytoin, Salicylates and ethanol. mnemonic is PEA-a PEA is shaped like zero just like zero order elimination of drug, the mnemonic is Phenytoin, ethanolc, and aspirin.

What is the difference between physiological strains and non physiological strains?

Physiological strains do not interfere with the flexion/extension components of the primary respiratory mechanism while non-physiological strains does impair flexion/extension.

how do you find the piriformis tenderpoint? How do you treat the piriformis tenderpoint?

Piriformis tenderpoint is half the distance btwn the greater trochanter of the femur and inferior lateral angle of the sacrum you want to externally rotate and abduct the side of the piriformis that is tender.

how to treat right posteriorly rotated innominate?

Position the patient supine so the right SI joint is off the table. stand at the right side of the pt and extend the pt's right hip until a restrictive barrier is felt. hold the position until a fascial unwind is palpated, then er engage the new restrictive barrier by further extending the right hip.

where is the poterior L4 tenderpoint located?

PostERIOR l4 TENDERPOINT IS LOCATED ON THE LATERAL aspect of the L4 spinous process. posterior lumbar tenderpoint are typically located on either side of the spinous process or on the transverse process of the named vertebrae.

Pay attention to the question. what is the posterior chapman point for the appendix

Posterior chapman point for the appendix is on the transverse process of T11. --If you see two answer choices that could be the answer-make sure you reread the question again.

physical exam reveals tenderness over the right radial hed and decreased active and passive supination of the right forearm. the most likely diagnosis is?

Posterior radial head.

WHAT muscle would you use to treat exhalation somatic dysfunction of rib 2?

Posterior scalene muscle.

Patient has recurrent episodes of anginal chest pain, commonly at rest, and is found to have transient ST-segment elevation on an ECG.

Prinzmetal angina

primary sclerosing cholangitis is associated ulcerative cholangitis. what is the skin lesion associated with ulcerative cholangitis called? what is erythema nodosum?

Pyoderma gangrenosum-this skin lesion is ulcerated, with a purulent base and surrounding erythema. PSC is an idiopathic disorder characterized by inflammation, fibrosis, and strictures of bile ducts and usually presents in young men with ulcerative cholitis. Pyoderma gangrenosum is a chronic slow healing or non-healing ulcer, most commonly seen on the lower legs. it is a rare, poorly understood, noninfectious skin disease that occurs in a variety of inflammatory conditions including: ulcerative colitis, rheumatoid arthritis, chronic active hepatitis, IgA monoclonal gammopathy, and hematologic and lymphoreticular malignancies. PSC HAS INVOLVEMENT OF THE EXTRA AND INTRAHEPATIC BILE DUCTS.--IMAGING studies with contrast will show classic "beads on a string" appearance of the biliary ducts. This is due to intermittent sclerosis of the bile ducts causing a stricture and subsequent dilation. erythema nodosum is not purulent. it is usually described as red, node-like swellings over the shins and , as a rule, both legs are affected. erythema nodosum is usually seen in conditions such as drug reactions, thyroid disease, post-streptococcal infection, and sarcoidosis. erythema nodosum occurs in 4% of people with ulcerative colitis.

what does pyuria mean what does an esterase positive urine analysis mean?

Pyuria is the presence of white blood cells within the urine. An esterase positive urine analysis is referring to the presence of leukocyte esterase. leukocyte esterase is found in the presence of pyuria.

On evaluation, you note she is tender along rib 6 on the left side, which of the following most accurately describes the biomechanics of this rib?

RIB 6 WILL move in a bucket handle motion. rib 1-5 move in a pump handle motion rib 6-10 move in a bucket handle motion ribs 11-12 move in a caliper motion

A pt was placed in a prone position, his right leg was dropped off the table, then flexed at the hip and knee. In that position, he was instructed to extend his hip by pushing his right foot against his therapist's counterforce for several seconds before relaxing and being further flexed into the position. this process was repeated for several rounds. what is his diagnosis?

RIGHT ANterior innominate rotation The treatment description is the proper set up and treatment of an anterior innominate rotation with muscle energy in a prone position. In a prone treatment set up, the patient was instructed to swing his right leg off at both the hip and knee. tHIS ROTATES the hip posterior and right innominate posteriorly into the restrictive barrier. The patient is then instructed to then push his foot into the operator, while the operator resists this movement resulting in an isometric contraction for several seconds. after the patient relaxes for several seconds the operator moves the patient to the new restrictive barrier.

Although this was used in an HIV case but if you ever have a client who you refer to a consultant and the consultant refuses to treat, it is impt to

Refer the pt to another consultant for care. you cannot force physicians to treat pts against their will.

imaging studies show periarticular swelling, edema, marginal erosions, joint subluxations and ankylosis as the disease progresses

Rheumatoid arthritis

what parasite that after exposure to contaminated water presents with severe itching? what is the drug of choice?

Schistosomiasis This patient most likely has the trematode (fluke) disease of schistosomiasis. It is usually acquired by exposure to contaminated water, with Schistosoma mansoni and S hAEMATOBIUM commonly in Africa. Thus patient had the classic swimmer's itch dermatitis that occurs with initial skin penetrating by free-swimming cercariae. S. mansoni adult worms infecting the portal vein of the liver and casuing diarrhea while S. Haemotobium ADULT WORMS infect the venous plexus of the bladder casuing hematuria and in chronic cases, bladder cancer. These schistosomes are often diagnosed by the presence of eggs in the stool or urine, respectively. S. mansoni eggs have a subterminal spine (S for mansoni=S for side); and S. haemotobium has a terminal spine (tfor terminal t for hematobium). Praziquantel is the drug of choice for schistosomiasis. Prazquantal causes calcium influx that depolarizes the tough outer tegument of the worm, leading to paralysiss and death.

how does surfactant work>

Surfactant works to decrease alveolar surface tension and increase compliance.

what is the most commonly affected sinus in adults with sinusitis? What is the most commonly affected sinus in children?

The maxillary sinus Cough and rhinorrhea are commonly reported due to post nasal drip. The ethmoid sinus is the most commonly affected sinus in children with sinusitis.

what are factors that will increae/decrease the CRI what does infectious (septic) meningitis present as?

(Normal 8-14 ) factors that will increase CRI include: fever, strenous exercise and OMT. factors that will decrease CRI include: stress (emotional or physical), depression, chronic fatigue, and chronic infections. Infectious (septic meningitis) presents with fever, headache and nuchal rigidity. (additional things-malaise and headache, migraines and a recent collision injury during a team practice, high temp, bld pressure of 98/68 mmhg), hrt rate of 102/min and respirations rate of 15/min-structural exam shows OA to be extended, rotated left, sidebent right with decreased range of motion and pain upon passive flexion). CRI WAS ELEVATED. PT WAS 18 yo. pt is likely suffering from infectious septic meningitis which is presenting as a febrile headache and nuchal rigidity (the cardinal signs of meningitis). due to the pt's age is likely neisseria meningitis-gram negative coccus that contain a polysaccharide capsule, ferments maltose, and oxidizes glucose. bacterial meningitis is a serious infection that must be treated promptly due to its high mortality rate. there is currently a vaccine available to prevent meningococcal meningitis; therefore, the incidence rate of this disease is on the decline. this particular infection usually starts with a systemic meningococcal septicemia that spreads to the meninges. These patients can sometimes present with circulatory collapse and disseminated intravascular coagulation. The treatment of choice for meningococcal meningitis is penicillin (if susceptible) or ceftriaxone. Prophylactic rifampin can be given to anyone who was in close contact with the patient prior to his or her illness. what would be found on the analysis of the cerebrospinal fluid?=glucose of 32mg/dl lab findings will show positive culture glucose <40mg/dl protein>50mg/dl wbc-1,000-10,000 (neutrophils >90%)

main primary motion of the cervical spine

-Occipital-atlantal segment is the segment that allows us to nod our head (flexion and extension) -the atlantal-axial segment, along with c2-c4 segments primarily allow us to rotate our head on a transverse plane. The C5-C7 segments do the majority of the sidebending in the cervical spine.

different type of cranial nerve palsies that is likely to happen?

-Oculomotor nerve presents with eye in the down and out position along with ptosis and nonreactive pupil. -Trochlear nerve palsy results in a vertical diplopia. ---abducens nerve palsy results in horizontal diplopia and inward deviation of the eye.

A V-spread technique performed at the occipital-mastoid suture can be performed to reduce restriction around the vagus nerve true or false? what bones come together to form the occipitomastoid suture?

-True -The temporal and occipital bone where cranial nerves 9, 10 and 11 exit.

-painful blistering due to splitting of the epidermis at the stratum granulosum

-bullous impetigo caused by staphlococcus aureus exfoliative toxin other skin infection involving the exfoliative toxin is staphlococcus scalded-skin syndrome, where blisters forms that do not contain any bacteria (the symptoms are a result of the toxin being present) followed by desquamation of the epidermal layer over the entire body, this most commonly occur on neonates but childrne can have it as well.

Ricin is a potent natural toxin that can cause lethal pulmonary edema an destruction. the best initial treatment is supportive with airway support and adequate ventilation. true or false?

True

tumor markers for 1. choriocarcinoma 2. yolk sac tumor 3.testicular sex cord and stromal tumors such as leydig cell tumors

1. Beta-Hcg 2. AFP 3. INHIBIN.

1. Surfactant decreases alveolar surface tension, preventing alveolar collapse with expiration or inspiration?

1. Expiration remember boyle's low PV=K. alveoli do not typically collapse during inspiration due to the inherent negative thoracic pressures involved with filling air spaces. -Alveoli would only collapse during inspiration if there was an obstruction or air leak leading to compression from surrounding air spaces. lack of surfactant will result in alveolar collapse st the end of expiration, not inspiration. 2. Surfactant is a lung product produced by type II pneumocytes which decreases alveolar surface tension and increases compliance (C=V/P). Surfactant deficiency leads to atelectasis and V/Q mismatch which leads to hypoxemia and hypercarbia. this leads to respiratory and metabolic acidosis, which leads to pulmonary vasoconstriction leading to right to left shunting at the foramen ovale and ductus arteriosus. Type 1 pneumocytes are primarily responsible for gas exchange in the alveoli, and they account for >95% of the surface area of the lung. Type II pneumocytes produce surfactant in order to decrease the alveolar surface tension. In such a preterm deliverly, the type II pneumocytes are not producing sufficient surfactant. surfactant deficiency will decrease lung compliance (less lubrication) and decrease functional residual capacity (as less alveoli are open) resulting in a large V/Q mismatch and concomittant respiratory acidosis.

1. what is this germinal centers and have anti-peroxidase antibodies? 2.anti-thyroid stimulating hormone receptor

1. Hashimoto thyroiditis 2. Graves disease

WHAT ARE SOME OF THE 9 physiologic principles used in muscle energy?

1. Joint mobilization with muscle force 2. Respiratory assistance 3. Oculocephalogyric reflex 4. Reciprocal inhibition 5. Crossed extensor reflex 6. isokinetic strengthening 7. Isolytic lengthening 8. postisometric relaxation 9. Using muscle force in one region to achieve movement of another region

TRUE OR FALSE: when palpated, chapman reflex points are 2-3 mm and are smooth and firm

True

1. Valvular stenosis can be thought of in terms of problem with opening and valvular regurgitation can be thought of in terms of as in problems with filling true or false? 2. A holosystolic murmur loudest at the left fourth intercostal space is descriptive of? 3. what are the two murmurs heard in the tricupsid area that that does not follow the norm and how would you describe it?

1. True 2.VSD=Ventricular septal defect. 3. Ventricular septal defect=this is a holosystolic murmur/Atrial septal defect this is a diastolic murmur

the proper positioning of a right posterior innominate would engage the restrictive barrier whether the patient is prone or supine by passively extending the right hip. true or false?

True

1. Alternating collections of neutrophils sandwiched between layers of parakeratotic stratum corneum? 2. hyperkeratosis, acanthosis, accentuated rete ridges, focal spongiosis and parakeratosis?

1. psoriasis psoriasis which manifests is a chronic, relapsing, inflammatory skin disorder with a strong genetic basis. the disease usually can be diagnosed by observing the characteristic lesions which most commonly occur on extensor surfaces of adults. THE LESIONS TEND TO BE RAISED AND EASILY PALPABLE DUE TO THE THICKENED EPIDERMIS, EXPANDED DERMAL VASCULAR COMPARTMENT, AS WELL AS INFILTRATE OF NEUTROPHILS AND LYMPHOCYTES. THEY TEND TO BE WELL-DEFINED; WITH SHARPLY DEMARCATED BOUNDARIES WITH A VERY DISTINCTIVE RICH, FULL RED COLOR. HISTOLOGY-MITOTIC ACTIVITY OF BASAL keratinocytes is increased almost 50-fold, with keratinocytes migrating from the basal to the cornified layers in only 3-5 days rather than the normal 28-30 days. The stratum corneum is also thickened, and the retention of cell nuclei in this layer is referred to as parakeratosis. Neutrophils and lymphocytes can be observed migrating upwards from the dermis into the acanthotic epidermis. --neutrophils may form localized collections known as munro microabscesses. the presence of alternating collections of neutrophils sandwiched between layers of parakeratosis stratum corneum is virtually pathognomonic for psoriasis. 2. seborrheic dermatitis lesions are commonly localized on the oily and hair bearing areas of the head and the neck.

Action of sternocleidomastoid action can be remembered on test day by a three step proces

1. remember that it is named for its attachements: the sternum, clavicle, and mastoid. 2. Remember that muscle attachments (typically) approximate when the muscle is freely contracted. 3. in your seat, bring the mastoid portion of your temporal bone (behind your ear) toward the ipsilateral clavicle. nOW you can feel the ipsilateral sidebending and contralteral rotation. damage to sternocleidomastoid will result in retraction of the scapula, ipsilateral sidebending of the clavicle spine and contralateral rotation of the cervical spine.

1.Starling's equation states that the exchange of fluid between the plasma and the interstitium is determined by the hydraulic and oncotic pressures in each compartment. in patients with nephrotic syndrome, damage to the glomeruli results in increased renal permeability and excretion of certain plasma proteins (especially albumin) that are essential to maintaining plasma colloid oncotic pressure. this results in the fluid shift from the capillaries into the interstitium, causing generalized edema. True or false? 2. what is the cause of edema in septic shock patients?

1.True 2. Increased capillary permeability (Kf) allows proteins in the capillaries to more freely leave and enter the interstitum. this is the mechanism that causes edema in septic shock, burn patients, and those exposed to certain toxins. it is not the cause of edema in nephrotic syndrome.

which muscle do you utilize when doing muscle energy on the 1st rib 2nd rib 3-5th rib 6-8th rib 9-10th rib 11-12th rib

1st rib= anterior and middle scalenes 2nd rib=posterior scalene ribs 3-5=pectoralis minor rib 6-8= serratus anterior ribs 9 &10=latissimus dorsi ribs 11 &12=quadratus lumborum-treated indirectly

The confidence interval is a range of values in which you could expect the mean of a data set to fall at a given probability. if when evaluating odds ratio, the confidence interval contains the value 1, the null hypothesis is not rejected. true or false?

True

what is the most common location for meckel diverticulum? A pediatric pt with noninfectious bloody stool most likely has what?

2 feet from the ileocecal valve in the right lower quadrant. Pt most likely has meckel diverticulum. --the diverticulum most likely contains ectopic gastric mucosa, which binds to the radiotracer--the acid produced from the ectopic gastric mucosa is what causes rectal bleeding. --A finding of diffuse gas within the intestines is a non-specific finding but can be seen in patients with ileus, which may occur in patients with meckel diverticulum. complications of meckel diverticulum include intussusception, volvulus, or obstruction near the terminal ileum.

Guillain-Barre syndrome is an immune attack on peripheral nerves that commonly presents as ascending paralysis with a preceding gastrointestina or respiratory infection that incites an autoimmune response against myelin. evaluation of the CSF will demonstrate an elevated total protein and normal white blood cell count. when does Guillain bare occur?

2-4 weeks after the inciting infection has resolved. evaluation of the cerebro-spinal fluid (CSF) will reveal an elevated protein level with normal white blood cell count, known as albuminocytologic dissociation. --protein present is due to the break down of myelin. --there is no elevated WBC in the fluid as there is no active infection.

The most common etiology of a posteriorly rotated innominate is a tight hamstrings. remember the hamstrings attach to the ischial tuberosity: consequently, contraction of the hamstrings result in posterior rotation of the innominate.

True

A new test for the detection of prostate cancer is being studied. it is positive in 90% of the men with prostate cancer and negative in 70% of the men without prostate cancer. The prevalence of prostate cancer in the population is 300 out of 1000. of the 1000 Men who took part in the study, what is the number of false positive test result?

210 A false positive occurs when a person without the disease tests positive. In this case, 70% of the men without cancer tested negative, which means that 30% of the men without prostate cancer tested positive. 700 of the men in the study do not have prostate cancer. therefore, 30% of 700, or 210, men without cancer tested positive.

Tibial stress fracture can be diagnosed using a bone scan within 72 hours of injury. they are more common in female runners and people with bilateral pes planus. true or false

True

what is decreased the most in an obstructive shock?

Cardiac output is decreased the most obstructive shocj is classified as reduced preload, low cardiac output, and high systemic vascular resistance. Another cause of obstructive shock is large pulmonary embolisms, which also put strain on the right ventricle and cause it to fail.

name the shock--high preload, low cardiac output, adequate intravascular volume, what type of shock is this?

Cardiogenic shock

what does approximately 40% of dizzy patients have?

40% of dizzy patients have peripheral vestibular dysfunction with the elderly having a higher incidence of central vestibular causes. Lack of spinning sensation cannot be used to exclude vestibular disease given the difficulty many patients have in putting their dizzy experience into words. The vestibule contains the utricle, saccule, and semicircular canals which participate in balance, equilibrium, and position sense of the body in space. The semicircular canals sense changes in movement of the body. Head positioning is sensed by the hair cells contained within the utricle and saccule. this patient is experiencing oscillopsia due to loss of the vestibulo-ocular reflex. This results in a blurring or rapid movement of the visual field with head movement. This also leads to unsteadiness impaired proprioception and sensory input is diminished such as in walking in a dark room or on uneven ground. The gait in vestibular dysfunction causes a deviation of walking to the side of the affected ear. The gait varies from an occasional stumble to frank veering. The legs are slightly wide-based and stride length is reduced.

To decrease the false negative-lower the serum marker threshold for a positive result

True

someone with carcinoid syndrome is at risk of a deficiency in?

NIACIN BECAUSE tryptophan is the precursor for both serotonin and niacin. Thus carcinoid syndrome can be associated with niacin deficiency.

True or false, Asthmatic patients with acute exacerbation will present with shortness of breath, frequent cough, and wheezing when exposed to different "triggers". The elevation of serum eosinophils in these patents may predict exacerbations and responsiveness to inhaled corticosteroids.

True

sit with support, started to jabber and make sounds and she has started to feed self appropriate finger foods. asuming develoment is normal age?

6 months.

normal urine pH

6.5 to 8.0 depending on the time of day. In the morning-it is 6.5-8.0 and becomes more basic throughout the day.

True or false: if an anterior chapman reflex point is present, then the posterior reflex point will be present as well?

True

Reciprocal inhibition is a physiological principle of muscle energy that can be used to lengthen a shortened muscle

9 physiologic principle used in muscle energy 1. joint mobilization 2. respiratory assistance 3. oculocephalpgyric reflex 4. reciprocal inhibition 5. crossed extensor reflex-used to treat severly injured areas 6. isokinetic strengthening 7. isolytic lengthening-although this treatment is used to lengthen a shortened muscle, the pt is instructed to contract a muscle against a greater force where the operator force exceeds the pts contracting force causing vibration, mechanical and circulatory effects resulting a lengthening of the targeted muscles. 8. postisometric relaxation 9. using muscle energy in one region to achieve movement of another region

Lacunar infarcts result from the effects of chronic hypertension on the walls of the arterioles. it results in lipohyalinosis, which appears as an eosinophilic deposits in the connective tissue of the vessel wall. The vessels responsible for supplying the posterior limb of the internal capule are lenticulostriate branches that branch off the anterior and middle cerebral arteries. there are five classic lacunar syndromes that can develop according to the affected area of the brain, and these include pure motor hemiparesis, pure sensory stroke, sensorimotor stroke, ataxic HEMIPARESIS and clumsy hand-dysarthria syndrome what about an infarct of the corona radiata and anteiror limb?

A lacunar infarct of the corona radiata and anterior limb of the internal capsule will result in ataxic hemiparesis. Patients will develop ipsilateral weakness and limb ataxia, with gait deviation towards the affected side. weakness occurs in both the upper and lower extremities on the same side but is more profound in the lower limb, especially the ankle and toes. There is also an associated dysmetria of the arm and leg on the same side. Physical examination will reveal hemiparesis, hyperreflexia, and babinski sign along with ataxia on the same side of the body. Ataxic hemiparesis results from disruption of the connection between the pyramidal system and adjoining fronto ponto cerebellar system, and this occurs within the corona radiata and the anterior limb of the internal capsule. tHE LACUNAR INFARCT specifically occurs in the thalamoperforate brnaches off the posterior cerebral artery.

what does a large volume of distribution mean

A large Vd=dose/CO (INITIAL PLASMA CONCENTRATION BEFORE ELIMINATION) INDICATES THE DRUG IS DISTRIBUTED WIDELY throughout the body. thus a smaller percent remains in the plasma. the minimum Vd is the plasma volume.

obtunded 36 yo male presents to the emergency department with an empty bottle of aspirin in his hand. An arterial blood gas is obtained and reveals the following pH-7.4; CO2=21, pO2= 80; hco3=14; SaO2=97%. This is indicative of?

A mixed metabolic acidosis and respiratory alkalosis mixed disorders with opposing processes can have a neutral pH. Since HCO3 is <24 mEq/L and PCO2 is <33 mmhg, this particular mxed disorder is a primary metabolic acidosis and respiratory alkalosis which is commonly seen in aspirin overdose or septic shock. Mixed disorders with two primary opposing processes can result in neutral pH. salicylate overdose and septic shock can produce mixed disorders. Cardiorespiratory arrest can also cause a mixed disorder with primary metabolic acidosis and primary respiratory acidosis. This will result in a markedly lowered pH.

A 34 YO MALE presents to your office with a 2 weeks of increasingly worse right calf pain that first occured while he was training for a local triathlon. He describes the pain as dull and achy. he has tried ice, stretching, and NSAIDs with no relief of symptoms. He previously has a smilar injury after runing a marathon and remembers his symptoms were effectively treated when you performed counterstrain. which of the following exam findings is consistent with Jones' counterstrain tenderpoint of the right calf muscle?

A one centimeter area of tenderness in the muscle belly located at the lower popliteal margin. Counterstrain is a passive indirect technique; the first step in performing counterstrain requires you to locate a significant tenderpoint. tenderpoints are typically found where the patient complains of pain, as in the above question. Gastrocnemius muscle tenderpoint is a common lower extremity point to be tested upon: this tenderpoint is located within the two bellies of the gastrocnemius muscle at the lower popliteal margin. it is treated by placing the ankle in plantat flexion until the pain resolved at least by 70%. Remember: Jones counterstrain tenderpoints are small, tender areas approximately the size of your fingertip. They are typically found near the bony attachments of tendons or in ligaments or muscle bellies. They do not radiate pain when pressed, which differentiates them from trigger points.

pericarditis is a potential complication of uremia in chronic renal failure patients, and is best treated with dialysis. There is a mnemonic used to remember the indication for dialysis what are they?

AEIOU A=Acidosis E=Electroylte disturbances I=intoxication O=overload of fluid U=uremia A complication for any patient with chronic renal failure is uremic pericarditis.

Autoimmune hepatitis presents with symptoms of acute or chronic hepatitis or cirrhosis. A liver biopsy will show increased lymphocytes without destruction of the bile ducts, and blood tests reveal a positive antibody of what?

A positive anti-smooth muscle antibody. Antismooth muscle antibodies are antibodies directed against smooth muscle, typically actin, the presentation of autoimmune hepatitis varies widely. patients can have acute hepatitis, chronic hepatitis or established cirrhosis. patients with cirrhosis can present with symptoms related to portal hypertension including esophageal varices, ascites, and hepatic encephalopathy.

what type of fibular head is a common cause of entrapment of the common peroneal (fibular) nerve?

A posterior fibular head is a common cause of entrapment of the common peroneal (fibular) nerve.

A short leg on the left would cause sidebending of the spine to what position and what type of scoliosis would result?

A short leg on the left would cause sidebending of the spine to the right and subsequently levoscoliosis.

Make sure you are able to use process of elimination to remove answers that may not be correct. Methotrexate slows the progress of joint destruction in rheumatoid arthritis, but it causes hepatoxicity? what is another side effect of methotrexate and how do you reverse it?

Another side effect of methotrexate is myelosuppression and you can reverse it by given folinic acid

THORACIC OUTLET SYNDROME IS DUE TO COMPRESSION FROM WHAT MUSCLES?

Anterior and middle scalene. so treatment of this muscle can help relieve the symptoms. the neurovascular bundle passes between the anterior and middle scalene, between the clavicle and first rib and under the pectoralis minor.

What is the anterior chapman point for the terminal ileum?

Anterior chapman point for the terminal ileum and ileocecal area is the right proximal femur.

where does the anterior choroidal artery arise from? what does anterior choroidal artery supplY

Anterior choroidal artery arises near the terminal bifurcation of the internal carotid artery just distal to the posterior communication artery. Anterior choroidal artery supplies the optic tract, lentiform nucleus and internal capsule. occluding anterior choroidal artery will cause contralateral homonymous hemianopsia-which is blindness in that eye. Also because the internal capsule is supplied by the choroidal artery, an occlusion will lead to hemiparesis

What are the ABCDE of sarcoidosis?

ACE (elevated ACE levels) Bilateral hilar lymphadenopathy Calcium (elevated calcium levels) Vitamin D (elevated vitamin D levels Erythema nodosum PRESENCE OF restrictive lung disease and bilateral hilar lymphadenopathy is pathognomic. when cranial nerves involved, 7th cranial palsy (Bell's palsy) is most common. -it can present with unilateral drooping of the eyelid and mouth, is the most common cranial nerve lesion of sarcoidosis. lofgren syndrome may also be present (bilateral hilar lymphadenopathy, erythema nodosum, and arthritis) may also be present. 60% of sarcoid patients will have elevated ACE levels. This is due to the increased production of ACE by the non-caseating granulomatous cells found in the lung tissue. many patients will not require treatment and symptoms will remit but other patients may require glucocorticoid and immunosuppressive agents.

what are reportable diseases?

AIDS, Chickenpox, gonorrhea, hepatitis A, B, and , measles, mumps, rubella, salmonella, shigella, syphilis, and tuberculosis. The physician is required to report this cases to the state health department with or without the patients consent. When a patient has a trnsmissible infectious disease, such as HIV, a physician should do all in his/her power to convince the patient to tell their love ones at risk of contracting the illness. sUPPORT SYSTEMS such as disease intervention specialists, HIV partner counseling, and referral services should be made available to the patient. Some states have partner-notification laws that require a physician to notify all third parites at risk of contracting HIV if a patient refuses to do so themselves. however, all measures to assits the patient in disclosing the information themselves should be taken before to maintain a trusting doctor-patient relationship.

The proper positioning for the AL1 tenderpoint is to have the patient supone, knees and hip flexed side bend towards and rotation towards. where is AL1 counterstrain point located?

AL 1 counterstrain point is located just medial to the ASIS. The AL1 point is where the ilioinguinal nerve pierces the transversus and internal oblique muscles. treatment will include flexion, rotation away (pelvic rotation toward), and sidebend towards the tenderpoint.

damage to the anterior spinal artery is likely to present with?

ANTERIOR SPINAL ARTERY SUPPLIES THE ANTERIOR 2/3 OF THE SPINAL CORD, including the anterior horns, corticospinal and spinothalamic tracts. the artery of Adamkiewicks usually arises from a left posterior intercostal artery and enters the spinal column to feeds into the anterior spinal artery around T8 and below. lower extremities will be affected with loss of motor strength and loss of pain and temperature sensation. Anterior spinal cord lesions above C7 will result in tetraplegia with involvement of the upper and lower extremities. Those with injuries from T1-T6 have full upper extremity use, although abdominal and chest muscles may be affected with diminished respiratory excursion. The region of T6 is the thoracic watershed zone; lesions below this level result in loss of bowel, bladder, and, sexual fuctions. This patient has corticospinal tract (weakness and reflex changes) and spinothalamic tract (bilateral pain and temperature) involvement below T8. tactile, position, and vibratory sensation will be normal, but the patient will additionally have urinary and bowel incontinence.

A 12 yo male is rushed to the emergency department with facial swelling. he had recently bit into a freshly picked apple and within minutes he experienced tingling around the mouth. within ten minutes, his lips begin to noticeably swell and his nose is nearly occluded by his large lips. there are no other symptoms besides facial swelling. the reaction of this patient is best described as?

ANgioedema Angioedma or Quincke's edema, is a sudden onset of subcutaneous non-inflammatory edema. it often occurs around the nose, eyes, and moth. Rarely, it may lead to airway obstruction. It can be the result of a hereditary disorder of bradykinin production or breakdown, an allergy, or a drug response.

3 yo, difficulty ambulating, abnormal head movements, pmhx recurrent sinus infections and pneumonia. PE- blood shot appearing eyes with associated involuntary lateral movements of her eyes. likely diagnosis?

ATAXIA telangiectasia. autosomal recessive present at approx-2 years of age. central nervous system is affected which leads to cerebellar atrophy due to loss of purkinje cells. --this leads to the gait disturbances. abnormal blood vessel formation known as telangiectasias are commonly seen int he eyes, bridge of the nose, ears, and extremities. The skin also loses elasticity. pts often have difficulty coordinating head and eye movements due to periodic nystagmus of the eyes. Patients with ataxia-telangiectasia develop progressive cerebellar ataxia, abnormal eye movements, telangiectasias, and immune deficiency with results in frequent sinopulmonary infections. gene ATM. Three A's ATAXIA, spider ANGIOMATA (TELENGIECTASIA), IGA deficiency leading to the pulmonary infections.

What is the mechanism of action that leads to damage to the ATFL and the Tibiofibular ligament?

ATFL (anterior talofibular ligament), runs between the talus bone of the foot and the fibula, with the anterior tibiofibular ligament which holds the tibia and fibular together near the ankle joint. Anterior talofibular ligament is most commonly injured with lateral ankle sprains and the anterior tibiofibular ligament is most commonly injured with high ankle sprains.

Dysfunction of the frontal lobe can lead to what?

Abolia=(An absence of willpower or an inability ti act decisively as a symptom of medical illness. akinesia (loss or impairment of the power of voluntary movement) mutism, slowness and lack of spontaneity, and urinary incontinence. frontal release signs such as the rooting REFLEX (turn head toward simulus to suck) AND THE SUCKING REFLEX may be present A hematoma from a subdural hemorrhage, as occurs in elerly patients due to tearing of the bridging VEINs, can result in a subfalcine herniation. The elevated intracranial pressure fromt he hematoma results in papilledema, morning headache, vomiting or brain herniation.

acute otitis media-where do you suspect the chapman point to be found?

Above the proximal third of the clavicle

questions on comlex will present irrelevant things to you so be on the lookout for things that are just there to distract you 4 year old male presents via his mother for generalized weakness, she describes her son as having increased difficulties running, jumopin, and climbing up their stairs at home. he has a waddling, toe walking gait and lower extremity hyporeflexia. osteopathic examination reveals a ropy texture with decreased bulk to both glutus maximus muscles. what does this person have?

Absence of dystrophin gene=duchenne muscular dystrophy presents around 2-3 years of age with proximal muscle weakness patients presents with wadling gait, and a positive Gower's sign, in which the child uses his hands to walk up the legs from a seated position. Osteopathic examination reveals chronic tissue texture changes in the gluteus maximus muscles, which demonstrates decreased ability to perform hip extension. This leads to posture compensation via an exaggerated lumber lordosis, a waddling gait, and Gower's sign.

A minor says she is forced to have sexual intercourse what should be physician do?

According to AMA, In cases when the physician believes that without parental involvement and guidance, the minor will face a serious health threat, or physical threat, and there is reason to believe that the parents will be helpful and understanding, disclosing the problem to the parents is ethically justified. When the physician does breach confidentiality to the parents, he or she must discuss the reasons for the breach with the minor prior to the disclosure. in this case, the rationale for breaching the patient's confidence is that she is being pressured to engage in sexual relationship (rape) that is placing her health at risk. it should be noted that, "when the physician does breach confidentiality to the parents, he or she must discuss the reasons for the breach with the minor prior to the disclosure. "while informing the parents, also, the physician is obligated to inform the police also.

what does acetazolamide allow for?

Acetazolamide allows faster acclimatization to high altitudes. it ameliorates the symptoms of acute mountain sickness.

What NT is REM sleep controlled by? WHat is REM CONSIDERED?

Acetylcholine aCTIVE SLEEP SLEEP STAGE, WHERE PEOPLE EXPERIENCE BURSTS OF RAPID EYE MOVEMENTS ALONG WITH SHARPLY HEIGHTENED BRAIN ACTIVITY AND TEMPORARY PARALYSIS OF THE MUSCLES OF THE BRAIN THAT CONTROL POSTURE AND BODY MOVEMENT.

what is the neurotransmitter and receptor to which it binds that is responsible for the direct release of sweat from the sweat glands?

Acetylcholine; muscarinic receptor. Sweat glands are a notable exception in which sympathetic stimulation on tissue itself uses acetylcholine. In sweat glands, acetylcholine acts through the M3 receptor to increase phospholipase C and intracellular calcium levels of stimulate secretions. This also applies to the adrenal medulla. so keep this in mind-adrenal medulla and sweat glands are part of the sympathetic nervous system but are innervated by cholinergic fibers.

direct coombs test utility

Acquired alloantibodies such as anti-Rh or anti-Jka are occasionally implicated, but this most commonly occurs due to a clerical error when patients receive the wrong blood type (e.g. A or B products given to an O patient). this results in rapid destruction of donor erythrocytes by preformed recipient antiboidies. patients may present with a triad of fever, flank pain, and reddis-brown urine due to hemoglobinuria. leads to DIC, Shock, and acute tubular necrosis. lab studies will reveal signs of hemolysis (e.g. anemia, jaundice, and hyperbilirubinemia) with a positive direct coombs test.

Excess in growth hormone when the epiphyseal growth plates are closed?

Acromegaly

G2P2 female presents to office with signs of abdominal pain and heavy periods for the past two months. her temperature is 37.1 (98.8), heart rate is 76/min, and blood pressure is 115/68 mm hg. pelvic examination reveals a diffusely enlarged, tender uterus without palpable adnexal masses nor cervical motion tenderness. b-hcg levels, cervical pao smear andan endometrial biopsy are obtained and are unremarkable. the most likely cause of the pt's symptoms involve?

Adenomyosis Endometrial glandular tissue located within the myometrium. Adenomyosis is a condition that is characterized by normally-functioning endometrial glandular tissue inside the myometrium. symptoms include dysmenorrhea, menorrhagia, and lower pelvi-abdominal pain associated with menstruation. physical examination will reveal a globular, tender, symmetrically-enlarged uterus with no adnexal masses. Adenomyosis id more common in multiparous women, women over the age of 35, and those with a history of uterine surgeries such as c-section.

A drug to diagnose and abolish supraventricular tachycardia

Adenosine. supraventricular tachycardia (SVT) is a general term used to describe a tachyarrhythmia that requires atrial and/or atrioventricular (AV) nodal tissue for its initiation and maintenance. this includes sinus tachycardia, atrial fibrillation, atrial flutter, multifocal atrial tachycardia, AV nodal reentry tachycardia and paroxysmal atrial tachycardia. EKG findings for most SVT's include a narrow QRS cmplex tachycardia with a regular, rapid rhythm (exceptions include atrial fibrillation (AF) and multifocal atrial tachycardia (MAT). Adenosine is the drug of choice to abolish supraventricular tachycardias. its mechanism of action involves increasing potassium shift out of atrial myocardial and AV nodal cells (shortens phase 3 on a pacemaker action potential curve) and leads to hyperpolarization of pacemaker cells. At high doses adenosine decreases conduction velocity, prolongs the refractory period, and decreases automaticity in the AV node (causes prolongation of PR interval on ekg). The risk of toxicity is rare due to its extremely short duration of action (approximatley 15 seconds). however it may cause flushing, chest pain and hypotension.

mainstay of treatment for acute aortic regurgitation

Afterload reduction such as hydralazine and captopril(ace inhibitor). reducing afterload will create less myocardial wall stress and optimize left ventricular function. The administration of vasodilators results in improved systolic function and decreased afterload

TAMSULOSIN MOA? COMMONLY USED IN Adverse effects?

Alpha-1 receptor antagonism-it will block smooth muscle contraction of the prostatic urethra, making urination more normal and improving the quality of life for patients. BPH is what it is used in Orthostatic htn and dizziness is its adverse effect

What does the anterior interossesous nerve innervate?

Anterior interosseous nerve (AIN), though considered a branch of the median nerve, provides isolated motor innervation to the deep muscles of the hand, flexor pollicis longus (FPL), the lateral half of the flexor digitorum profundus (FDP) and pronator quadratus (PQ). on physical examination, the test of choice for these muscles is to make an A-OK sign. inability to make this sign shows weakness in the flexors of the hallux and distal interphalangeal joint of the 2nd digit. Of note, when examining the patient's ability to make an A-Ok sign, the examiner will see flattening of the 1st and 2nd digit rather than a true circle when there is compromise of the AIN. This occurs because flexor digitorum brevis and adductor brevis are considered synergists of FPL and FDP, and are not innervated by the AIN. The AIN traverses the forearm along the volar aspect of the interosseous membrane along with the interosseous artery. Due to its location with a significant distal forearm injury, such as galeazzi fracture, disruption of the forearm complex can lead to direct compression of inflammation of this nerve. The quickest test for AIN is to make an A-OK sign. Think of this injury as similar to carpal tunnel syndrome based on location, but without sensory deficits. the most common mechanism of injury is through trauma to the mid-shaft and distal forearm complex.

what type of hypothermia is this associated with a core temperature of 28 to 32 degree celsius (82 to 90 degree F), absence of shivering, and an altered mental status. The warning signs that a pt has entered into the moderate range of hypothermia are altered mental status, reduced heart rate, hypoventilation, central nervous system depression, absence of shivering and body temperature btwn 28 to 32 degree C. --The normal point for core temperature is 37 +/- 0.5 C. cold environments provide quite a challenge to the human body as we are limited in our autonomic response to maintain heat when in dangerous environmental conditions. The human body utilizes the hypothalamus to mount a response to the change in temperature by shivering, changes in thyroid hormone, adrenal activity, and catecholamaines. pts in mild hypothermis typically are shivering and are mentally intact. their core body temp is routinely 32 to 35 (90 to 95 F). a core temerature above 32 is vital to improve your likelihood of survival when exposed to a cold weather environment, symptoms most likely seen in severe hypothermia (HTIII) are loss of consciousness and a core temp of 24 to 28 (75 to 82 F). this is more likely to casue areflexia, oliguria, pulmonary edema, coma, hypotension, asystole, and ventricular arrhythmias. someone with altered mental status, palpable kidneys, bibasilar crackles and an S3 gallop. LAB REVEALS hypercalcemia and proteinuria. most likely cause of these findings is?

Amyloidosis -due to multiple myeloma. Amyloidosis id a clinical disorder caused by deposition of insoluble amyloid fibrils. the precursor protein is a clonal immunoglobulin light chain or light chain fragment which is found in monoclonal plasma cell disorders such as multiple myeloma.

what is myxedema coma?

An extreme case of hypothyroidism and is associated with high mortality rate even with treatment. This rare clinical state presents as a lige-threatening decompensated state taht may include altered mental status, hypothermia, bradycardia, hypoventilation, and cardiovascular collapse. Myxedema coma/crisis occurs most commonly in older women with long standing, undiagnosed or undertreated hypothyroidism who experience an additional significant stress, such as infection, a systemic disease, certain medications (eg, narcotics, hypnotics, ansthetic agents, sedatives and exposure to a cold environment. The recent history of infection or the use of hydrocodone in this hypothyroid patient (as evidenced by her lab values) could have triggered the crisis to occur. management includes supportive therapy with mechanical ventilation, passive rewarming and blood pressure control. Immediate IV levothyroxine and hydrocortisone are mandatory with careful monitoring of the patient's hemodynamic state.

Which cell is Androstenedione converted to estrone? thecal lutein cell or granulosa lutein cell?

Androstenedione is converted to estrone by aromatase within the granulosa lutein cells. The granulosa cells and the theca cells-these cells participate in the production of steroids; together they synthesize estradiol and progesterone. as the main products. both cells are able to produce progesterone independently of each other; however, the production of estradiol involves both cellular levels. Androstenedione is converted to estrone by aromatase, and estrone is converted to estradiol by 17-Beta hydroxysteroid dehydrogenase. Androstenedione is synthesized in the theca cells through conversion of pregnenolone by 17 alpha hydroxylase. Androstenedione then diffuses across the basement membrane into the granulosa cells and is converted to estradiol with aromatase.

what should you suspect in a patient with a chronic infectious, inflammatory, or malignant condition who presents with a normocytic, normochromic anemia?

Anemia of chronic disease. first line therapy is directed at the underlying cause of the anemia of chronic disease. Anemia of chronic disease is due to a chronic inflammatory, infectious or malignant disease. examples include rheumatoid arthritis (as in this patient), systemic lupus erythematosus, cancer, heart disease, and diabetes mellitus. Anemia of chronic disease is primarily regulated by the protein hepcidin. hepcidin is secreted by the liver regulates the intestinal absorption of dietary iron, the release of iron from macrophages, and the transfer of iron stored in hepatocytes. An increase in hepcidin levels occurs in chronic inflammatory diseases. HEPCIDIN causes iron to be trapped inside macrophages. this produces decreased plasma iron levels, along with an increased ferritin level, and the anemia is most likely normocytic and normochromic. It also presents with normal to low serum transferrin (total iron binding capacity), normal to increased serum ferritin, and elevated erythrocyte sedimentation rate and C-reactive protein. you will see normal MCV, normal Fe2+, low TIBC. swan neck deformity=hyperextension of the proximal interphalangeal joint with distal interphalangeal joint flexion.

What are Angelman syndrome and prader-willi syndrome due to?

Angelman syndrome can be due to a maternally derived deletion on chromosome 15, OR BY PATERNAL uniparental disomy. presents with inappropriate LAUGHTER AND hyperactivity Prader-willi syndrome can be due to a paternally derived deletion on chromosome 15, or by maternal uniparental disomy. Disomy refers to the condition of having a chromosome represented twice in a chromosomal complement. maternal disomy means both chromosomes came from the mother and paternal disomy means bothe chromosomes came from the father.

weakness in ankle dorsiflexion (deep involvement), great toe extension (deep involvment) and ankle eversion (superficial involvement)

Ankle inversion can lead to this. so with inversion think of extension and eversion. sensory loss may be found over dorsum of the foot (superficial branch) and/or in first web space (deep branch). sensory symptoms may be absent in inversion injury. so things that start with aN E issue and dorsiflexion because think of leg dorsiflexion resembles extension.

A woman presents with fever and malaise. vital signs reveal a temp of 41 C (105.8 F). A subsequent brain MRI reveals the presence of an intraparenchymal bleed in the area of the hypothalamus. the most likely affected area of the hypothalamus is the?

Anterior nucleus. anterior hypothalamic nucleus is responsible for control of body temperature. this can be remembered by anterior hypothalamuc nucleus =A/C-anterior cooling. hypothalamus is located below the third ventricle just above the optic chiasm and pituitary gland. hypothalamic nerve terminals are located in the median eminence, where hormones can be released directly into the primary capillary venous plexus to travel down the pituitary stalk into the anterior pituitary gland. the function to conteol body temperature and thermoregulation is under parasympathetic control and is responsible for cooling the body through stimulation of panting or sweating. --thus a lesion oint he anterior hypothalamus will result in an inability to lower the core body temperature

A 37 yo fatigue, constant sleepiness, sleeps with three or four blankets while wearing socks and house slippers at all times. occasionally he wears gloves to keep his hands warm. pt say he has gained weight because he does not exercise as he used to. a physical exam is notable for periorbital edema.. lab studies show increased serum creatine linase. the antibody associated with this patient's presentation is?

Anti-thyroid peroxidase (anti-TPO) ANTIBODIES also known as antimicrosomal antibodies are associated with this condition due to the body producing antibodies to thyroid cell microsomes when there has been damage to thyroid cells.. antimicrosomal and antithyroglobulin antibodies are associated with Hashimoto thyroiditis. HYPOTHYROID myopathy typically manifests as polymyositis-like myopathy with proximal muscle weakness and an increased creatinine kinase level. however, it sometimes manifests as muscle enlargement (pseudohypertrophy); in adults, this condition is called hOFFMAN Syndrome. Several case reports describe rhabdpmyolysis associated with hypothyroidism. iN THESE CASES hypothyroidism is thought to gave predisposed individuals to rhabdomyolysis.

what kind of OMT can be performed for a thoracic outlet syndrome?

Any form of OMT can be performed for thoracic outlet syndrome but treatment should focus on the location of compression. key areas to focus on will be the cervical spine, first rib, clavicle, and the muscles and fascia of the thoracic inlet.

crescendo-decrescendo systolic ejection murmur followed by an ejection click-radiates tot he carotids and is associated with a diminished and delayed carotid upstroke (pulsus parvus et tardus). parodoxical splitting is also present. sustained grip for 20-30 seconds will increase systemic vascular resistance, arterial pressure, cardiac output, and left ventricular volume and filling pressures. A handgrip is most useful in differentiating between two murmurs?

Aortic stenosis and mitral insufficency. The intensity of aortic stenosis decreases while mitral regurgitation will increase in intensity. Abrupt standing will also diminsh the intensity of aortic stenosis.

The patient quickly becomes pale and responds only to forceful tactile and verbal stimuli. Her respirations are shallow and rapid. pulse oximetry is 75% on 6L oxygen via nasal cannula and is declining. the most appropriate next step in management is to? AND her code status is DNR/DNI

Apply noninvasive positive-pressure ventilation. In CASES of respiratory distress with DNR status, seek aggressive but noninvasive support such as a mask supporting positive-pressure ventilation. Do not intubate or perform cricothyrotomy. The patient is crashing and requires a more powerful intervention than nasal cannula oxygen, which is unable to deliver large amounts of oxygen or maintain airway patency. Because her code status is DNR/DNI, procedures such as intubation and cricothyroidectomy are prohibited. Noninvasive positive-pressure ventilation uses a mask to deliver high levels of oxygen while maintaining an open airway. it is typically used for acute exacerbation of copd OR ASTHMA BUT HAS BECOME INCREASINGLY popular as a noninvasive option for respiratory distress.

44 yo woman comes into the your clinic complaining of terrible shoulder pain and difficulty raising her arms above her head. you assess the motion at the sternoclavicular and acromioclavicular joints and the clavicle. you note poor motion at the SC and AC joints and a right clavicle which is superior, anterior and internally rotated while she is seated you stand in front of her with your thumbs contacting the inferior surface 1/3 lateral from the sternal end and 1/3 medial to the AC joint. How would you complete this balanced ligamentous tension technique?

Apply slow, steady cephalad force and have her turn her contralateral shoulder away from you. In order to perform BLT you must move the segment in the direction of the lesion and when the tension is properly balanced the respiratory or muscular cooperation of the pt is employed to overcome resistance. in this case since the clavicle is superior and anterior you would apply a cepahlad force and have her turn her left shoulder away from you.

24 yo male medical student presents to your office complaining of back and L groin pain upon standing after prolonged studying while sitting at his desk. As he walks into the exam room you notice that he is having difficulty standing upright from the pain. based on this, one would expect to palpate a tender counterstrain point in conjunction with which of the following locations?

Approximately 7cm medial to the ASIS=TENDERpoint for the iliacus.. This pt is likely experiencing spasm of the iliopsoas complex on the left side, leading to back and anterior groin pain after prolonged sitting. The tenderpoint for the iliacus is approximately 7cm medial to the ASIS.

working in shipyards-fabrication of ships think

Asbestos-stain positive with prussian blue because asbestoes fibers are coated with protein and iron=ferruginous bodies.

55 YO male physician with a family hx of early-onset heart disease is admitted to the hospital, where he practices, with chest pain. His long term cardiologist, who is also his colleague at the hospital, is on-call and serves as the admitting physician. the best course of action is for the admitting physician to?

Ask the patient how to respond to respond to colleagues' inquiries about his care. Caring for peers is an honor and a responsibility and can lead to some ethical challenges that should be recognized. according to the AMA, physiciand providing care to a professional colleague have an obligation to respect information and physical privacy of physisicn-patients as they would for any patient. Treating physicians should consider, and possibly discuss with the physician-patient, how to respond appropriately to the inquiries about the physician patient's medical care from other physicians or medical staff. the physician in this case will likely receive inquiries from colleagues about the patient. he should ask the patient or, if the patient lacks capacity his surrogate , how to respond.

What is MOA of tropicamide?

Atropine is the prototype. Tropicamide is a muscarinc blocker. it will cause pupillary dilation by blocking the muscarinic receptors, and will cause a change in the shape of the lens by binding to muscarinic receptors on the ciliary muscle. this will impair a patient's near vision.

what is the inheritance pattern of autosomal polycystic kidney disease?

Autosomal dominant the ADult form=Autosomal Dominant is the most common form and the Autosomal recessive form occurs in children and is usually fatal

what does avulsion fracture of the fibular head suggest? what nerve innervates the anterior and lateral compartment of the leg?

Avulsion fracture of the fibular head suggests a lateral collateral ligament (LCL) tear. and one would find an abnormal varus stress test of the lower leg Anterior compartment of the lower leg is innervated by the deep peroneal nerve. the lateral compartment is innervated by the superficial peroneal nerve and the posterior compartment is innervated by the posterior tibial nerve. extensor hallucis longus muscle is part of the anterior compartment. things part of the posterior compartment includes flexor digitorum longus, gastrocnemius, popliteus and tibialis posterior

what major apolipoprotein helps with chylomicron remnant recognition by the LDL receptors?

B-48 (MEDIATES CHYLOMICRON SECRETION) AND Apo E

what is the dug Cilostazol used for?

Cilostazol is a phosphodiesterase inhibitor used for intermittent claudication. Cilostazol is a phosphodiesterase III inhibitor, and thus increases cAMP in platelets to inhibit platelet aggregation. this results in vasodilation as well as for intermittent claudication and coronary vasodilation.

WHAT IS an example of isokinetic training?

Bicycling at a constant speed with varied resistance Isokinetic, sometimes referred to as "isovelocity," is a muscle contraction where the velocity remains constant while the force of the contraction is allowed to vary. these types of contractions are very rare in the body and only occur when using specialized equipment to control the speed of contraction within the range of motion. It provides muscular overload at a constant speed while the muscle mobilizes its force through the full range of motion. THE FOLLOWING TYPES OF muscle contraction are imperative to know for the comlex exam such as ISOTONIC ISOMETRIC CONCENTRIC ECCENTRIC PLYOMETRIC

15 yo female presents to her gynecologist with a history of recurrent nosebleeds and heavy bleeding during menses. famhx shows an aunt who had excessive bleeding during a recent routine tooth extraction. the girl's coagulation profile would most likely prolong what?

Bleeding time and PTT. Von willebrand disease is the most common autosomal bleeding disorder.

What is the mechanism of action of clopidogrel?

Block ADP receptors P2Y12. clopidogrel inhibits platelet aggregation by irreversibly blocking the ADP receptor P2Y12. this medication is useful for prevention of stent thrombosis with coronary artery stents, acute coronary syndrome, or in the prophylaxis of strokes.

primary site of erythropoiesis in a 2 day old male is?

Bone marrow Young Liver Synthesize Blood Y=Yolk sac-3-8 weeks L=Liver-6 weeks =until birth S=spleen-10-28 weeks B=bone marrow-18 weeks through adulthood

patients with scoliotic curves greater than 20 degrees should be treated with what?

Bracing goal of treatment is to prevent progression of the curves as patients with curves greater than 45 degrees may be at risk for organ compromise.

4 yo presents with extensive brusing, deformity of the distal femur, painless hepatosplenomegaly and yellow fatty deposits on the sclera. Enzymatic assay showed low glucocerebrosidase activity in fibroblasts. what enzyme is deficient here?

Beta-glucosidase Gaucher disease in children presents as extensive bruising, painless hepatosplenomegaly, and fatty yellow deposits on the sclera and other organs. Definitive diagnosis is by enzymatic assay to show decreased glucocerebrosidase activity in fibroblasts and lymphocytes.

Broca versus wernicke's aphasia?

Occlusion of the left Middle cerebral artery manifests as aphasia. broca's aphasia has non-fluent speech, but good understanding, and involves the left frontal lobe. Wernicke's aphasia has fluent speech, but lack of understanding, and involves the left superior temporal lobe.

BLT for ribs?

BLT is perforemd through three main steps: disengagement, exaggeration and balance. for the ribs, the physician provides lateral traction while holding the ribs anteriorly and posterioly. the enhancement that is used is the pateint holding his/her breath in inhalation.

Frequency and urgency of urination and generalized prostatic enlargment, without nodules, suggests what diagnosis?

BPH (Benign prostatic hyperplasia)

superior division of middle cerebral artery deficits will be in

BROca's area

facet orientation

BUM, BUL, BM cervical, thoracic and lumbar respectively.

FACET orientation of cervical , thoracic and lumbar

BUM=Backward, upward and medial for cervical BUL=backward, upward and lateral for thoracic BM=backward and medial for lumbar

what is Behcet's syndrome?

Behcet syndrome is an autoimmune vasculitis that can involve immune complex mediated inflammation and classically manifests in patients with recurrent oral aphthous ulcers, genital ulcers, and uveitis. it can also present with erythema nodosum. this is an immune complex mediated vasculitis.

This person has agoraphobia

Benzodiazepines (enhance GABA) are short acting medications that can be used "as needed" (unlike serotonin=daily) to alleviate severe anxiety and panic symptoms that occur when the pt with agoraphobia are exposed to situations such as using public transportation, being in open or closed spaces, standing inlines, or other crowded spaces or being outside of the home alone. Benzodiazepines enhance the binding of GABA neurotransmitters to their receptor sites, which facilitates opening of chloride channels and magnification of the inhibitory effect of GABA in the central nervous system.

drug of choice to treat cocaine induced pscychomotor agitation

Benzodiazepines.

someone presents with gross hematuria after an upper respiratory tract infection and less often following gastitis? what type of disease do you think they have

Berger disease, also known as IgA nephropathy Berger disease, also known as IgA nephropathy, is classified as a nephritic syndrome indicating an inflammatory process in the kidneys. IgA nephropathy is characterized by predominant IgA deposition in the glomerular mesangium, and it is the most common cause of glomerulonephritis in the world. Pts WITH A HX OF RECENT upper RESPIRATORY TRACT INFECTION AND NEW ONSET HEMATURIA SHOULD BE EVALUATED FOR THIS PARTICULAR DISEASE. tHE PATHOGENESIS OF BERGER DISEASE REMAINS INCOMPLETELY UNDERSTOOD, HOWEVER THE CHARACTERISTIC PATHOLOGIC findings by light and immunofluorescencee microscopy of granular deposits of IgA and complement 3 (c3) IN THE GLOMERULAR MESANGIUM SUGGESTS THAT THIS DISEASE IS THE REUSLT OF THE DEPOSITION OF CIRCULATING IMMUNE COMPLEXES LEADING to the activation of the complement cascade. Up to 80% of patients presenting with recurrent gross hematuria have a concomittant upper respiratory infection, mainly acute pharyngotonsilitis. Hematuria usually appears simultaneously or within the first 48-72 hours after the infection begins. It continues less than 3 days and is sometimes accompanied with loin pain throught to be due to renal capsular swelling. berger disease usually follows a benign clinical course. in most pts; however, some patients are at risk for slowly progressing to end stage renal disease, which is estimated to occur in20-40% of pts afgter 20 years of the disease. poststreptococcal glomerulonephritis --sudden development of hematuria, proteinuria, red blood cell casts in the urine, edema, and hypertension with or without oliguria. there is always a latent period btwn the initial streptococcal infection and the development of the glomerulonephritis-usually 1-2 weeks after a throat infection and 3-6 weeks after a skin infection. light microscopy reveals enlarged enlarged and hypercellular glomerulu, neutrophils, and lumpy-bumpy appearance. ----Although immunofluorescence studies g AND c3 ALONG THE glomerullary wall and mesangium, this would take a minimum of a week to develop, which is outside of the range of this patients with the hx of an upper resp infect, the latent period does not match the time course of this particular disease and there was no Physical Findings or edema

A patient with sickle cell disease and also has osteomyelitis, how would you treat them?

Ciprofloxacin and vancomycin osteomyelitis most commonly occur in the metaphysis of long bones. because most common cause is staph aureus, but because in sickle cell patients the most common cause is salmonella-you want to cover both and give them ciprofloxacin and vancomycin.

what is breach of duty?

Breach of duty is this-breach of duty refers to negligence of a physician to an established patient, which results in direct harm to the patient. under the category of negligence in malpractice law and states that a physician's direct action, or lack thereof, directly causes harm to the patient. it is important to remember, though, for negligence to occur there must b harm to the patient. The correct action would have been to call for ambulance in order to have the patient brought to the emergency department. Doing this would have removed the physician from any liability and ensured that the patient became medically stable to drive home.

The patient follows your commands, but cannot communicate with you and is obviously frustrated. the most likely affected region of the brain is the?

Broca's area is located within the inferior frontal gyrus. patients have a nonfluent aphasia with intact comprehension.

Remember the spinothalamic tract (pain and temperature) is an ascending tract that crosses the cord (decussates) within 1-2 levels after entering the spinal column. The dorsal columns (touch, vibration, and proprioception) ascend ipsilateral and cross the spinal column at the medulla. this is why you get the characteristic pain pattern associated with brown-sequard lesions. what does brown sequard show?

Brown-sequard displays ipsilateral loss of touch vibration, a proprioception below the lesion, contralateral loss of pain and temperature below the lesion, and ipsilateral spastic paralysis below the lesion.

Brown sequard syndrome-describe the findings likely to be found?

Brown-sequard syndrome is a hemisection of the spinal cord, most commonly resulting from penetrating trauma to the spine. It presents with a characteristic pattern of symptoms, including ipsilateral loss of motor function and proprioception beginning at the level of injury, and contralateral pain and temperature sensation loss beginning two to three levels below the injury.

Patient states that her and her friend shopped, watched a play and spent the afternoon in a hot tub. A sputum sample is collected and gram stain reveals a weakly staining gram negative organism. --which of the following types of selective growth media would be used to culture the most causative organism?

Buffered charcoal yeast extract agar. legionella pneumophila is a weakly staining gram-positive organism that is transmitted via contaminated water sources and grows on bufferred charcoal yeast extract agar.

Drug of choice for treating Carcinoid syndrome?

Octreotide=somatostatin analog-acts by suppressing secretion of vasoactive substances like serotonin.

Btwn C an A fibers which one does the following -SLOW AND POORLY LOCALIZED -FAST AND WELL LOCALIZED

C fibers=slow and poorly localized. C= sissy (weak) A fibers- fast and well localized (AWESOME A)

What tract or pathway does C fibers transmit its information through?

C would transmit its information through the first division of the lateral Spinothalamic tract

Osteoporosis, osteomyelitis, Fractures in the area of thrust, bone metastasis, severe rheumatoid arthritis and Down syndrome. all of this are all contraindication to what treatment technique?

HVLA

true or false-Achalasia is caused by a loss of ganglion cells in the myenteric plexus which results in less nitric oxide and inability of LES relaxation. Botulinum toxin injected into this area can relax the LES and relieve signs and symptoms of achalasia for a short period of time.

True

true or false-chapman reflex points are small points of tenderness and sensitivity found in the deep fascial layers?

True

what tumor marker is elevated in ovarian cancer?

CA-125 IN 80% of women. don't confuse with CEA which is found in a variety of cancers, including colon, pancreas, stomach, lung, and breast, as well as benign conditions like cirrhosis. IBD, CHRONIC lung disease and pancreatitis.

causes of sideroblastic anemia and the first line treatment?

CASUES of sideroblastic anemia include alcohol, isoniazid toxicity, lead toxicity, vitamin B6 deficiency, iron overload, copper deficiency, and zinc toxicity. the first line in treatement for all suspected sideroblastic anemia is a trial of vitamin B6.

What is carcinoid syndrome characterized by?

Carcinoid syndrome is characterized by asthmatic wheezing, diarrhea, flushing, and right sided heart valve dysfunction. right heart disfunction-this will be heard on the right side of the heart and this would be a systolic murmur of the left lower sternal border-where the tricupsid valve is located.

triceps reflexes are weak bilaterally. what is the spinal nerve involved?

C7=Spinal level for the triceps reflex.

Ejection fraction, end diastolic volume, and heart rate can be used to calculate? what else can this be used to calculate?

Cardiac output there are four calculations that can be utilized to determine cardiac output: 1) Q=SV*HR 2) (EDV-ESV)*HR (EDV-ESV=Ejection fraction) 3)MAP/TPR 4)Rate of O2 consumption/{arterial O2-Venous O2)

cardiac output can be calculated using fick's principle which is?

Cardiac output =oxygen consumption/(arterial o2-venous o2) -make sure to look at the units even when you carry a number over to make sure it matches up.

Fever, night sweats, and weight loss are typical "B" SYMPTOMS associated with Hodgkin's lymphoma. aNOTHER CLUE IS THE PROGRESSIVE COUGH and dysphagia for solids due to an obstruction as opposed to the dyphasgia of solid and liquids due to peristalsis problem. obstruction of solids and liquids is due to the location of the Hogkin's lymphoma in the anterior mediastinum, which compresses the trachea and esophagus. leading to dysphagia and cough. Hodgkin lymphoma arises from germinal B cells and the disease is most common in young adults around 20 years and the elderly in the sixth decade of life. most common presentation is non tender lymphadenopathy in the neck or mediastinum. What are the CD findings and histological findings of REED Sternberg cell?

CD 15 AND 30 CD15 IS found on neutrophils and is responsible fir the phagocytosis and chemotaxis of neutrophils. CD30 is a tumor necrosis factor receptor protein that leads to activation of NF-kappaB. NF-kappaB prevents apoptosis and promotes cell proliferation. Hodgkin's lymphoma does not express the Pan-B cell antigens CD 19 or CD20 or the pAN-T cell antigens CD3 and CD7.

what drugs are associated with disulfiram-like reaction?

CEFOPERAZONE AND CEFAMANDOLE which are third generation and second generation cephalosporins as well as with procarbazine=first generation sulfonylureas, and metronidazole.

EEG findings with periodic or pseudoperiodic paroxysms of sharp waves or spikes on a slow background. in a person who is experiencing progressive dementia?

CJD=CReutzfeldt-Jakob disease

what i the clearance equation

CL=0.7*Vd/t1/2 vd=volume of distribution cl=clearance t1/2=half life

What cranial nerves course through the cavernous sinus?

CN III, IV, VI, V2 and VI.

placental abruption, IUGR, and increased risk of pre-term labor and delivery what drug abuse is this with?

COCAINE

GET GAP on the METRO-what does this mnemonic mean?

CONDITIONS THAT METRONIDAZOLE CAN BE USED TO TREAT Giardia, Entamoeba, Trichomonas, Gardnerella, Anaerobes (clostridium, bacteroides, Actinomyces) and h. Pyelori.

pneumothorax will present with dyspnea, unilateral chest pain, decreased breath sounds, increased resonance on percussion, and decreased tactile fremitus (palpable chest vibrations during vocalization, which usually increases with lung consolidation). what may have caused this?

COPD is a disease process closely linked to pneumothorax most common findings of tension pneumothorax, include mediastinal shift, rib cage expansion, and diaphragmatic depression. due to the air trapping occuring with tension pneumothorax you will note a descended or flattened pneumothorax. management includes supplemental oxygen, chest tube insertion, or video assisted thorascopy. the thing that gave it away in the image is the diaphragm depression.

Physical exam reveals a cranial rhythm impulse of 10 cycles per minute with decreased amplitude. the most appropriate next step in managment includes?

CV4 CV4 can be utilized to help enhance the cranial rhythm impule rate and/or amplitude, which is commonly decreased in patients with depression, increased stress, or chronic infections. note normal CRI is 10-14 CV4 is generally used to stimulate the body's inherent therapeutic forces to deal with whatever dysfunction is present. It is often used to enhance the amplitude of the CRI and is performed by first resisting the flexion phase and encourage the extension phase of cranial movement until a "still point" is reached, then allowing restoration of normal flexion and extension which often approximates the same cycle as thoracic respirations.

left flank pain radiates into her groin, headaches behind her eyes and left finger pain over the past month. bp 190/129 mmhg, plain film radiographs of her abdomen and fingers are taken and show presence of a calcium oxalate stone, and subperiosteal bone resorption on the radial aspect of her middle phalanges (osteitis fibrosa), respectively. lab tests reveal an elevated calcium level. which serum marker is likely elevayed in this pt?

Calcitonin. pt has MEN2A Signs and symptoms.

Nephrolithiasism aka kidney stones is common in makes. they present with acute colicky flank pain and no fever. the most common composition of the stone is?

Calcium oxalate although the patient has two increased risk factors for uric acid stones such as use of loop diuretics and alcohol. calcium oxalate is still the most common type of stone.

what is the most common cause of death in women aged 35-64?

Cancer pror to 35 most common cause of death is unintentional injuruy. in men unintentional injury is the most common cause of death in age 1-44. heart disease is the most common cause of death in women 65+. stroke overall is the third mot common leading casue of death in womne.

A woman was treated with trimetoprim-sulfamethoxazole. and afterwards physical exam reveals erythema and an irritated appearance of the mucosa. which of the following is the most likely cause of her symptoms?

Candida albicans Candidiasis, caused by the fungus candida albicans, presents with vaginal itching and can occur after recent antibiotic use. Candidiasis is the second most common cause of vaginitis after bacterial vaginosis. Diabetes, pregnancy and recent treatment with antibiotics are risk factors. While antibiotics are often effective in the treatment of urinary tract infections, they can also kill off the normal flora of the vagina, opening the door for fungal infection.

The physician is in the process of starting up his own practice. He signs a contract with a HMO in which he is to be reimbursed a flat fee per patient, regardless of the services that are rendered to his patients. this concept is referred to as? what is bundled payment?

Capitation.--Capitation is a contract between a physician and a HMO in which a flat fee per patient is paid each month regardless of the number of services performed. Bundled payment is the reimbursement of health care providers on the basis of the expected cost of the course of treatment for that episode. it has been proposed as a new health care reform under president Obama. Bundled payments would link payments for the multiple services that patients receive during an episode of care. for example, if a patient underwent a surgical procedure, the entire team is compensated with a bundled payment instead of the separate payments to each provider that is currently being practiced. it is designed to provide more efficient care to patients.

62 YO FEMALE presents to the ER with urinary incontinence and paresthesias of he medial thigh. She reports that she is unable to feel toilet paper on her perineum when wiping after urination. the pt reports mild back pain with the initial onset of symptoms. osteopathic examination reveals a step-off of the L3 spinous process. the most likely diagnosis is?

Cauda equina syndrome---Cauda equine syndrome can be distinguished by the following: isolated LMN signs, perineal paresthesia (saddle anesthesia), and bowel and bladder incontinence. On osteopathic examination, the step off of the L3 spinous process demonstrates a probable spondylolisthesis with anterior slippage of the L3 on L4 vertebral bodies-this occurs from bilateral fatigue fractures of the pars interarticularis. Spondylolisthesis is a common cause of CES. Emergent surgical consultation is needed because permanent paralysis and irreversible neurological damage can occur if this diagnosis is not addressed immediately.

infant doing well and all of a sudden when stopped breast feeding and introduce table food. there is darrhea, microcytic anemia because of lack of absorption of iron due to malabsorption think of ?

Celiac sprue IgA antigliaden antibody intestinal and extraintestinal symptoms. affects duoenum and jejunim. T cell lymphoma risk.

Which of the following has had the most impact on lowering the incidence rate of cervical carcinoma?

Cervical cytology aka papsmear. papsmear is the screening test used to detect cervical dysplasia and has had the biggest impact on lowering the incidence rate of cervical cancer. papsmear is performed by obtaining a smaple of cervical epithelial cells and studying their cytology. the presence of koilocytes is indicative of HPV infection. I thought it was the gardasil vaccine but it is said it is not widely used as compared to the papsmear. so cervical cytology aka papsmear.

cervicothoracic diaphragm is rotate left, abdominal diaphragm is rotated left, pelvic diaphragm is rotated right. the most appropriate initial treatment is?

Cervico-thoracic diaphragm. so remember OA is to the left and after that keep alternating. so this scenario is a non-compensatory pattern because the cervicothoracic junction does not follow the pattern that the rest of the diaphragms follow. Since the cervicothoracic junction doe snot follow the same pattern of alternating motion, it should be treated first.

discharge of chlamydia versus gonorrhea

Chlamydia has a mucopurulent discharge and most common cause of STI. GONORRHEA has a copious creamy purulent discharge note for chlamydia -note it is the most common-so if you get confused -it may be best to pick it MUCUPURULENT fluid in the vaginal VAULT, gram stain reveals many neutrophiuls but negative for organisms-this is because it doe not stain well-another hint it could be chlamydia we are dealing with.

what can help you distinguish chlamydia from other sexually transmitted diseases?

Chlamydia is a non-gram staining bacteria, with elementary and reticulate bodies. that is the number one cause of bacterial sexually transmitted infections in the united states. Symptoms inclue vaginal discharge and fever. --The thing to keep in mind is that it has a vaginal discharge and the fact that it is the most common-so pick it.

turbid synovial fluid with elevated neutrophils (purulent, turbid synovial fluid characterizes septic arthritis, which can occur from an untreated gonorrhea infection). this condition typically only affects one joint and presents more acutely with pain and fever. what differentiates this from Chlamydia trachomatis>

Chlamydia trachomatic can cause reiter's syndrome which is an acute onset of arthritis, urethritis, and conjunctivitis due to the presence of an infection, most commonly chlamydia, seen in young, sexually active men.

Name reportable diseases for the CDC

Chlamydia trachomatis, gonorrhea, hepatitis A acute, hepatitis B acute, Hepatitis B chronic, Hepatitis B perinatal infection, Hepatitis C acute, HIV infection, Influenza associated pediatric mortality, measles, mumps, rubella

what drug causes aplastic anemia?

Chloramphenicol and propylthiouracil

A 34 yo female presents with episodic cramping abdominal pain. she states that she has had two episodes per week for the last month, and all have resolved spontaneoulsy. physical exam reveals a non-tender abdomen without guarding or inspiratory arrest on deep palpation of her upper quadrant. Osteopathic examination confirms the presence of mild, dull tissue texture changes throughout the mid-thoracic, at the level of t6-t9. the most likely pathology of this patient's chief complaint is due to?

Cholelithiasis. when a pt has routine gall bladder attacks, the associated palpatory findings will be either absent, or mild; think of them as echoes of repeat attacks this patient is having twice a week. if the pt has an acute attack, the palpatory findings would be much more definitive. This patient is presenting with intermittent biliary colic an a negative abdominal physical exam which is seen in cholelithiasis. cholelithiasis is the presence of gallstones within the gallbladder that migrates into the opening of the cystic duct and block the outflow of bile during gallbladder contraction. this results in episodes of cramping abdominal pain due to an increase in the gallbladder wall tension from contraction. biliary colic usually resolves after an hour as the gallbladder relaxes and may radiate to the right scapula. The pain begins postprandiallly, usually following a faty meal. gallstones form whrn bile becomes supersaturated with cholesterol, and induces the precipitation into crystals. physical examination will reveal a benign abdominal exam without rebound tenderness, guarding or fever since the gall-bladder is not inflamed. risk factors include female sex, native american ancestry, obesity, crohn disease, cystic fibrosis, estrogen use and pregnancy. first step in management is an ultrasound, cholelithiasis may develop into cholecystitis. Cholelithiasis results in episodic biliary colic that occurs postprandially following a fatty meal. Cholecyctitis results in fever, persistent right upper quadrant pain with guarding, and a positive Murphy's sign. Choledocholithiasis results in obstructive jaundice without fever. Ascending cholangitis results in fever, right upper quadrant pain, and obstructive jaundice.

organophosphates poisoning is?

Cholinesterase-inhibitor poisoning when you have a cholinesterase inhibitor poisoning-you inhibit the enzyme that degrades cholinesterase-and thus you have excess acetylcholine activity. Reverse is with atropine followed by pralidoxime Pralidoxime reactivates chilinesrterase that has been inactivated by phosphorylation due to exposure to organophosphate pesticides and cholinesterase inhibiting nerve gases (E.g sarin); it does this by displacing the enzyme from its receptor sites and removing removing the phosphoryl group from the active site of the inactivated enzyme

huntington disease is caused by inherited mutation in what chromosome? in which trinucleotide repeats result in decreased and ineffective transcription.

Chromosome 4 autosomal dominant inheritance CAG REPEATS -WHICH RESULTS IN A DECREASED TRSNCRIPTION of a brain-derived neurotrophic factor.

What does chylothorax contain?

Chylothorax is caused by a leakage of lymphatic fluid from the thoracic duct and contains increased triglycerides on fluid analysis. This is most commonly due to lymphoma and trauma caused by thoracic surgery.

biopsy of pseudomembrane show: inflammatory exudate composed of mucinous debris, fibrin, necrotic epithelial cells and polymorphonucear cells , the underlying crypts show disruption by mucous and inflammatory debris what bacteria infection is this describing?

Clostridium difficle The pathology of clostridium difficile colitis is a pseudomembranous colitis with the formation of an inflammatory fibromuscinous exudate that covers the colonic wall. two toxins toxin A is an enterotoxn Toxin B is a cytotoxin both toxins participate in the pathogenesis of pseudomembranous colitis, although toxin A is primarily responsible for c. DIFFICILE diarrhea. colonoscopy is not routinely recommended for the diagnosis or management of C. difficle infection due to the high risk of perforating the intestinal wall, this procedure may demonstrate the presence of raised, yellowish-white , 2 to 10 mm plaques overlying an erythematous mucosa. These plaques are termed pseudomembranes. biopsy of pseudomembrane show: inflammatory exudate composed of mucinous debris, fibrin, necrotic epithelial cells and polymorphonucear cells , the underlying crypts show disruption by mucous and inflammatory debris

when are the two times to remember not to give a beta blocker before an alpha blocker?

Cocaine toxicity and pheochromocytoma. the beta-blocker will prevent the beta-2 mediated vasodilation, thus, worsening hypertension. Also, think of pheochromocytoma every time you see high urinary VMA.

unilateral foot pain, has peptic ulcer disease with an active gastric ulcer. tender MTP on right hallux. appropriate medicine?

Colchicine Aspirin and indomethacin are both classified as non-steroidal anti-inflammatory drugs (nsaids) which are usually first line treatments for gout, with the exception of aspirin because it can alter uric acid levels and potentially prolong and intensify an acute attack. Indomethacin is the classic NSAID that is prescribed to treat acute gout, however, it is contraindicated in this patient due to the hx of peptic ulcer disease.

describe colorado tick fever

Colorado tick fever is a viral infection transmitted by a wood tick, dermacenter andersoni. Fifty percent of the patients present with a "saddleback fever" coloarado tick fever is a viral infection transmitted by the bite of a wood tick dermocentor andersoni. The infectious agent is a double -stranded RNA virsu of the genus colitivirus in the family of Reoviridae. Colorado tick fever present as a nonspecific febrile illness; however, up to 50% of the patients infected present with saddleback fever which consists of cycles of fever 2-3 days followed by absence of fever 2-3 days. Common symptoms include headache, myalgias, arthralgias, and fatigue. A small percentage of patients may develop a maculopapular or petechial rash. However, this is not present in the majority of patients with colorado tick fever. The virus causing colorado tick fever. The virus causing colorado tick fever is the second most common arbovirus in the US. seconf to west nile virus. the virus is found in the midwestern and western united states. because the clinical features of colorado tick fever are nonspecific, the diagnosis must be established in the proper epidemiological context. confirmation is based on serologic test results of the presence of antibodies to the virus. In addition, reverse transcriptase polymerase chain reaction techniques are available that may help diagnose he disease in the first five days of illness.

you ask pt to contract her biceps while you provide resistance.. while doing this the angle of her arm decreases. this is an example of what kind of contraction? what if the tension of the muscle was the same throughout the entire contraction what type of contraction would this be?

Concentric contraction. when there is approximation of the muscles's origin and insertion, it is referred to as a concentric contraction. isotonic contraction.

WHAT TYPE OF PERICARDITIS IS THIS-ASSOCIATED WITH FIBROTIC calcifications of the pericardium ,positive kussmaul's sign, and pulsus paradoxus?

Constrictive pericarditis pt will present as fatigue, weight loss, orthopnea, and lower extremity edema. move to US at age 10 and returns to visit his parents in mexico. afebrile, with a blood pressure of 138/80 mmhg, heart rate of 72/min, and respiration rate of 15/min. while examining the pt's neck, physician notes jugular venous distention upon inspiration. a the likely clinical presentation here is constrictive pericarditis This patient is most likely suffering from constrictive pericarditis secondary to mycobacterium tuberculosis. The CT scan shows fibrotic calcifications around the prricardium. TB is considered the number one cause of constrictive pericarditis in developing countries. PE is impt he has a positive Kussmaul's sign, which is jugular venous distention upon inspiration, and if palpated, he could also demonstrate pulsus paradoxus. other clinical manifestations include: fever, weight loss, night sweats, cough/dyspnea, chest pain, pleurisy, orthopnea, tachycardia, pleural dullness, hepatomegaly, ascites, and peripheral edema. four pathological stages of TB pericarditis include: fibrinous exudative, lymphocytic effusion, absorption of effusion with granulomatous caseation, and constrictive scarring. right atrial pressure tracing will show marked y descent-which is when the right atrium is emptying. this Y descent is absent in cardiac tamponade.

You suspect a patient has Kallman syndrome, what would be the most appropriate in this patient at this time?

Contrast -enhanced MRI of the brain patients with Kallman syndrome can develop abnormalities of several organ systems, and screening tests for these are indicated. however, the diagnosis must be first confirmed using mRI, which rules out other structural lesions and shows abnormalities of the olfactory system.

What disorder is this: motor or sensory dysfunction (blindness, paralysis, seizure, etc) that is incompatible with any known medical diagnosis. neurological symptoms are often transient and may manifest with different neurological symptoms over time.

Conversion disorder or functional neurological symptoms disorder this symptoms may have bee n triggered by maybe a car accident.

? deficiency leads to microcytic anemia, neutropenia, fragile silvery hair, skeletal abnormalities, and skin depigmentation. there is also ataxia and seizzures, physical exam hypotonia, hepatosplenomegaly, and the presence of fragile silver-colored hair. a complete bld count reveals a microcytic hypochromic anemia along with a neutropenia.

Copper. This pt is presenting with Menke kinky hair syndrome due to a deficiency in copper. this presents as a fine fragile silvery hair, depigmentation of the skin, muscle weakness, neurological abnormalities, and hepatosplenomegaly. A deficiency in copper results in osteoporosis due to defective collage. it is also associated with skin depigmentation, due to dysfunctional melanin synthesis from the copper-dependent tyrosinase enzyme. a copper deficiency also results in a microcytic , hypochromic anemia and neutropenia.

In the setting of upper gastrointestinal bleed, there can be an elevation of the blood urea nitrogen: creatinine ratio

Correct

painful vaginal bleeding in the second or third trimester. htn, smking and use of cocaine are all risk factors for?

Placental abruption. the bleeding is the result of a premature detachment of the palcenta fromt he uterine wall. tHE FETUS IS NOT Viable. the risk for placental abruption increases in women who abuse toabcco, cocaine, and those with htn. the treatment for placental abruption is the immediate delivery of the fetus.

what is dumping syndrome

Dumping syndrome is a condition that is precipitated with large meals that draw flui into the intestinal lumen to produce symptoms such as watery diarrhea, disphoresis, tachycardia, and weaknnes. symptoms often seen in patients who underwent a gastric bypass surgery. The meal's hyperosmolar content combined with decreased transit time, acts to draw fluid into the intestinal wall and cause diarrhea. patients should b instructed to eat smaller meals spread throughout the day.

true or false-two third of the cardiac cycle is spent in diastole. As the heart rate increaes, the time spent in diastole falls more rapidly than systole?

True

type II penumocytes and respiratory tree development occur significantly at week 24 true or false?

True

WAGR syndrome

Wilms tumor, Aniridia (absence of iris), Gneitourinary malformation, and mental Retardation) is another highly tested syndrome.

Beckwith-Wiedemann syndrome

Wilms tumor, macroglossia, organomegaly, neonatal hypoglycemia (due to excess insulin production) hemihyperplasia (WT2) mutation

Thrombocytopenia leading to increased bleeding time and petechiae, eczema, and recurrent infections because of T and B-cells. TRIAD=Thrombocytopenia, eczema, and infections.

Wiskott-Aldrich Syndrome X-linked recessive leads to a decrease in T and B-cells resulting in a decrease in IgM.

with an inversion ankle sprain what happens to the distal fibula (lateral malleolus)?

With an inversion ankle sprain the distal fibula-lateral malleolus goes anterior.

Vagus nerve is responsible for elevating the palate. A lesion of the vagus nerve may present as a contralateral deviation of the uvula

true

Vitiligo is a skin condition associated with autoimmune disease, genetic predisposition, and is wood's lamp positive. the hypopigmented lesions are smooth and distinct.

true

ehlers danlos-berry aneurysm in the circle of willis.

true

left sided hemineglect is caused by a deficit in the contralateral parietal lobe

true

most efficacious medications for treating acute mania are olanzapine risperidone and halperidol

true

What is primary lateral sclerosis?

Primary lateral sclerosis presents with upper motor neuron signs without associated lower motor neuron signs, which are seen in amyotrophic lateral sclerosis. treatment is focused on symptomatic management. oral antispasticity medications should be tried prior to consideration of more invasive treatment options, such as intrathecal baclofen pump or botulinum toxin injections.

Episodes of angina pectoris at rest, symptoms usually occur at night btwn midnight and early morning. episodes of chest pain lasts between 5 to 15 minutes.

Prinzmetal angina

caused by transient coronary artery vasospasm?

Prinzmetal angina

myofascial release, lymphatic treatment, articulatory techniques and muscle energy may be okay to perform in patients with pain symptoms who also may have rheumatoid arthritis. however, HVLA is absolutely contraindicated,

true

which of the following established that patients have the right to refuse life-sustaining treatments, including artificial nutrition and hydration?

Cruzan V. director In Cruzan V Director, missouri department of health 1990, involved in the case of Nancy Cruzan, who was in a persistent vegetative state after a car accident. Her parents went to court to have a feeding tube and artificial nutrition and hydration discontinued. Through her case, the United States supreme court determined that patients have the right to refuse life-sustaining treatments and confirmed that a feeding tube was a medical treatment that did not have unique status. lIKE OTHER LIFE-SUSTAINING treatments artificial nutrition and hydration could, legally and ethically, be discontinued.

someone has carpal tunnel what would you do to relieve the pain?

Work on the flexor retinaculum

A neonate develops sudden onset of tachypnea, nasal flaring, expiratory grunting, and intercostal retractions. the most likely physiological finding in this neonate is?

DECREASED COMPLIANCE. WHAT ARE THE EFFECTS OF SURFACTANT?-Surfactant works to decrease surface tension, to increase compliance and to decrease elastic recoil. In the premature lung with inadequate surfactant, there is a resultant higher surface tension that leads to instability of the lung during expiration. C=V/P

knee pain, after being struck in the side of the knee during a soccer game. structural exam reveals that the left fibular head moves posteriorly with ease but is restricted in anterior motion. Which of the following additional finding is most likely?

DIstal fibula resists posterior springing. when the fibular head moves posterior, the distal fibular head moves anterior and resists posterior springing. anterior distal fibula facilitates plantar flexion and inversion of the foot. the fibula has see-saw motion with an axis in the midshaft of the fibula. fibular head dysfunctions can be treated with muscle energy by internally or externally rotating the foot to meet the restrictive barrier.

transmits sensation of vibration, proprioception, and fine touch discrimination. its axons ascend in an ipsilateral fashion before eventually decussating in the medulla? what column is this?

DORSAL column. dorsal column consists of the fasciculus gracilis and the fasciculus cuneatus. They are organized anatomically such that the cervical section lies most laterally within the fasciculus cuneatus, while the sacral segments lie medially within the fasciculus gracilis. these pathways will cross at the decussation of the medulla. given their posterior location, the dorsal columns would be most susceptible to stab injury. Furthermore, a stab injury with ipsilateral vibratory deficit in the lower extremity localizes the spinal cord lesion to the dorsal columns.

physician recommends ibuprofen for pain relief. What is a potential biochemical result from taking this medication?

Decrease prostacyclin production. Note-Ibuprofen is a medication that inhibits the COX enzyme and as a result there is a potential for decreased amounts of product downstream. Since prostaglandin is produced downstream of COX pathway inhibiting this enzyme will result in decreased amounts of prostaglandin, not increased amounts. Non-steroidal anti-inflammatory drugs (NSAIDs) inhibit cyclooxygenase (COX), thus preventing the synthesis of prostaglandins, prostacyclin, and thromboxanes.

What is the first sign of magnesium toxicity?

Decreased deep tendon reflexes deep tendon reflexes decreases because high magnesium levels prevent the release of acetylcholine at the presynaptic terminal and competitively inhibt calcium entry through voltage-gated calcium channels. muscle weakness may also occu later. deep tendon reflexes should be monitored throughout magnesium treatment because signs of magnesium toxicity occur in a stepwise fashion. first, decreased deep tendon reflexes are seen, followed by hypotension, respiratory paralysis, and cardiac arrest if magnesium levels continue to rise.

66 yo male presents with severe abdominal pain, PE shows hepatomegaly and mild jaundice. lab analysis reveals hematocrit of 64%, platelet count of 525,000/mm^3 and a white blood cell count of 14,000/mm^3 a peripheral blood smear reveals a normal morphology of all cell lines, the most likely lab finding in this pt would be

Decreased erythropoietin polycythemia vera presents with elevated serum red blood cell mass, leukocytosis, and thrombocytosis. due to negative feedback, the serum erythropoietin levels is decreased. there is often associated splenomegaly, generalized pruritus, increased thrombotic events, and erythromelalgia. pruritus occurs following a warm bath or shower due to mast cell degranulation and release of histamine. Erythromelalgia presents as a burning pain and associated erythema of the hands or feet due to the microvascular thrombotic events that occur. increased red blood cell mass and platelets results in an increased viscosity of the blood, which results in an increased incidence of venous and arterial thrombosis. venous thrombosis is most common in the superior sagittal sinus, mesenteric vein, and hepatic vein. bone marrow biopsy shows hypercellularity with prominent erythroid , granulocytic, and megakaryocytic proliferation. pts with polycythemia vera have low serum erythropoietin concentration due to the negative feedback effect from the elevated red blood cell mass.

cHNAGES IN ARTERIAL pressue are detected by baroreceptors in the carotid sinus which , via the CNS, ALTER cardiac activity to compensate. the valsalva maneuver decreases cardiac output and thus decreases the firing rate of these baroreceptors. woman presents with dizziness and fatigue everytime she has a bowel movement. vitals 90/60 mmhg, a heart rate of 72/min, and a respiratory rate of 18/min. doctor asks her to expire against a closed glottis. the primary and most sensitive mechanis regulating her increase in heart rate soon afterwards is?

Decreased firing rate of baroreceptors located in the carotid sinus. Valslava maneuver is generated agaisnt a closed glottis, this increases intrathoracic pressure and limits venous return of blood back to the heart. this decrease in venous reTURN causes consequent decrease in stroke volume (starling's mechanism) and arterial pressure. --If baroreceptor reflex is intact, the baroreceptors of the carotid sinus, located in the intenral carotid artery, near the bifurcation of the common carotid arteries, will sense this as a change in blood volume and decrease the rate of signaling to the vasomotor centers of the brain stem (receptors within the aortic arch have a higher threshold pressure and are less sensitive than the carotid sinus receptors and are, thus, not the primary mechanism of response). maximal carotid sinus sensitivity occurs near the normal mean arterial pressure. therefore, very small changes in arterial pressure around this set point dramatically alters receptor firing so that automomic control can be reset in such a way that the arterial pressure remains very near to the set point. This point changes during exercise, hypertension, and heart failure and explains how arterial pressure can remain elevated dueing exercise or chronic htn. This leads to an increase in sympathetic outflow (the SNS is less inhibited) to the heart and blood vessles. in the test an increase in heart rate would be noted. When the person stops the maneuver, there is a rebound increase in venous return and cardiac output. the increase in arterial pressure is sensed by the baroreceptors, which direct a decrease in heart rate back to its set point, which is usually near 100 mmhg.

Deficiency of which adrenal enzyme will lead to decreased production of mineralcorticoids, glucocorticoids, and sex steroids?

Deficiency of 3-Beta-hydroxysteroid dehydrogenase which is responsible for the initial step in the production of each of the adrenal steroids. decreased mineralcorticoids (aldosterone) result in salt-wasting, leading to emesis, dehydration, hypotension, and possibly shock and circulatory collapse. it can also cause hyponatremia and hyperkalemia. decreased cortisol causes hypoglycemia and prevents negative feedback on the anterior pituitary leading to elevated adrenocorticotropic hormone (ACTH) levels. This can cause enlargement of both adrenal glands (hence the name bilateral adrenal hyperplasia). decreased sex hormone production presents differently depending on the patients sex. male patients may be incompletely masculinized with a small penis and varying degrees of hypospadias. Testicles are usually palpable but may be smaller. In males with a milder defect, sexual symptoms may not appear until adolescents with ambiguous genitalia, poor virilization, and gynecomastia. Female infants with 3BNSD deficiency usually appear normal at birth or may have varying degrees of clitoromegaly and labial fusion. treatment requires replacement of all adrenal steroids with oral hydrocortisone and fludrocotisone acetate beginning after duagnosis and sex hormone replacement (testosterone for males, cyclic estrogen-progesterone for females) beginning at puberty. congenital adrenal hyperplasia is a group of autosomal recessive disease. All forms of congenital adrenal hyperplasia result in elevated ACTH levels and enlargement of the adrenal galnds.

3 yo male presents via his mother for complaints of increased clumsiness. the mother reports a family hx of cognitive impairment occuring in males. PHYSICAL EXAMINATION OF THE CHILD REVEALS A hip waddling gait with lumbar lordosis. his lower extremity muscle strength is decreased bilaterally. OSTEOPATHIC exam confirms a generalized hypotonicity of the gluteus musculature, positive hip-drop test, and hyper-tonicity of the bilateral achilles tendons. the most likely cause of his symptoms is?

Duchenne muscular dystrophy the affected child has difficulty running, jumping, and walking up steps. An unusual waddling gait and lumbar lordosis are usually observed. cognitive impairment has been noted in literature a direct neurological component has yet to be established. there is also hyporeflexia.

65 yo male brought into the ER department after fainting a half hour ago while he was putting away his dishes. hx reveals he has chronic htn, and DMII. PE reveals Right sided hemiparesis. computes tomography of the head is negative for an acute bleed. the mechanism of action of the therapy required in this pt most likely involves?

Degradation of fibrin through conversion of plasminogen to plasmin this pt is likely suffering from an embolic stroke, which if left untreated could cause cerebral ischemia and irreversible brain injury and possibly death. the pt experienced this symptoms half an hour ago and is exhibiting neurological deficits. also, hemorrhagic stroke has been ruled out by CT scan. the best therapy of choice is thrombolytic therapy aimed at breaking the clot. Tissue plasminogen activator (t-PA) is involved in breaking down clots. These proteins catalyze the conversion of plasminogen to plasmin, which is the major enzyme responsible for clot breakdown. plasmin breaks down the fibrin clots that form inside blood vessels in a process known as fibrinolysis. Aspirin inhibits platelet aggregation -for many pts aspirin may help prevent strokes or further strokes from occuring. however, its role in treating an existing embolic stroke is limited and a thrombolytic would be the best therapy to use in this scenario.

sleep walking and terrors occur in stage n3 sleep. what will EEG show?

Delta waves.

conotruncal cardia anomalies (truncal arteriosus, interrupted aortic arch, tetralogy of fallot, vascular ring, and ASD/VSD), hypoplastic thymus, and hypocalcemia due to parathyroid hypoplasia are all associated with what syndrome?

DiGeorge syndrome.

Classic Duputyren's contracture involves the fourth and fifth digits and is commonly associated with what?

Diabetes mellitus Duputyren disease is a fibrosing disorder that results in slowly progressive thickening and shortening of the palmar fascia and leads to debilitating digital contractures, particularly of the metacarpophalangeal (MCP) joints or the interophalangeal (PIP) joints. the fourth and the fifth digit are unable to extend in the classic Dupuytren;s contracture. Dupuytren's contracture of palmar fascia beneath the skin. One of both hands may be affected. The fourth digit is affected the most often, followed by the fifth, then the third. The index fingers may be involved. typically, a small painless nodule develops in the connective tissue below the skin on the palmar aspect of the hand. Over time, it thickens into a cord-like band and leads to an inability to extend or straighten the fingers. Dupuytren's contracture can be associated with diabetes mellitus and may also be genetic. it is usually seen in caucasian males over 50 years of age. Treatment varies depending on severity and is geared towards improving the motion and flexibility of the fingers. iF THE PALM is painful, glucocorticoid injection can be used for analgesia. moderate to severe cases require fasciectomy.

what patients are likely to experience gastroparesis?

Diabetic patients Gastroparesis is a common chronic problem of uncontrolled diabetes mellitus. It commonly presents with early satiety, heartburn, and vomiting of undigested food within a couple of hours of eating.

Anti-ribosomal P antibodies are highly specific for the diagnosis of SLE though they lack specificity. so if a person has this and you are asked what nephitic/nephrotic syndrome they have what would you say?

Diffuse proliferative glomerulonephritis. Wire looping can be seen on light microscopy-so it looks like a loop, it appears as though there is a wire surrounding the glomerulus. wire looping represents capillary loop thickening and is indicative of diffuse proliferative glomerulonephritis. light microscopy will reveal hypercellularity of endothelial and mesangial cells. Immunofluorescent microscopy reveals a granular deposition of immunoglobulins, complement, and fibrin along the glomerular basement membrane, tubular basement membrane, and peritubular capillaries. DPG evolves into chronic renal failure in most cases and is a common cause of death in SLE.

Cilastin inhibits dehydropeptidase 1 in order to decrease he renal excretion of imipenem. what does dihydrobiopterin reductase do?

Dihydrobiopterin reductase is associated with the biochemical pathway of phenylalanine, the enzyme takes dihydrobiopterin and converts it using NADPH to tetrahydropbiopterin.

on exam, you find a positive counterstrain point associated with this injury. where is the most likley location of this counterstrain point?

Distal to the medial malleolus Although counterstrain points are typically tested as they related to the area of injury, don't forget the relationship of agonist/antagonist muscle as you may find counterstrain points opposite the area of injury. counterstrain will be tested a lot on this test so know it well. In an inversion ankle sprain, you likely injure the lateral soft tissue structures of the ankle. the concept of counterstrain describes a reflexive change in the tissue (counterstrain) in response to this initial injury (strain). THIS CREATES A PATTERN Of pain AND TENDERNESS THAT OCCURS at the site of injury as well as on the opposite neck injury can result in anterior counterstrain point. This means that although there is a lateral ankle injury 9LIKELY atfl), pain and a counterstrain point will often also be located on the opposite side (medially in this example) from the injury. Since the lateral points provided in the answer choices do not typically present with inversion injuries, you then need to know the classic description of the medial ankle counterstrain point which is located in an arc distal to the medial malleolus.

terminally ill pt with metastatic ovarian cancer has intractable pain and nausea. she is dying and has a do not resuscitate and does not want to be intubated . her physician administers a morphine bolus and starts morphine drip, with the plan to titrate it as needed to control her symptoms until she dies. pt becomes sedated and her respiratory rate decreases. which of the following justifies the continued administration of opioids for pain control, in spite of the sedation and respiratory depression?

Double effect. The principle of double effect justifies the continued administration of opioids for pain control, in spite of sedation and respiratory depression. The ethical principle of double effect maintains that an action that has both negative and positive consequences may be ethically justified if the intention is to achieve the positive effect.

Anti-histone

Drug induced lups

which of the following is not a tissue texture change associated with acute somatic dysfunction?

Dry SOMATIC dysfunction can be acute or chronic. on physical exam, acute somatic dysfucntion presents as boggy, warm, moisturized, tender, and edematous tissue.

A person has wrist drop what nerve is affected?

Wrist drop is associated eith weaknesss of the wrist extensor muscles leading to unopposed wrist flexion. This injury, though rare, occurs most commonly secondary to improper crutch use.

A 10 yo female presents with pruritic rash. physical examination reveals an erythematous scaly plaque with central clearing located over her left buttock. A KOH preparation of skin scraping reveals many hyphal forms. which of the following is the most likely cause? A-candidiasis B-Langerhan cell histiocytosis C-Psoriasis D-Seborrhic dermatitis E-Tinea corporis

E-tinea corporis Tinea corporis presents as a pruritic, scaling annular shaped plaque with central clearing, and is caused by an infection with a dermatophyte fungus such as trichophyton rubrum The lesions becomes annular shaped with slightly raised borders. It is more common in preadolescents and in hot, humid climates. tINEA CORPORIS is most often due to the fungus trichophyton rubrum, althogh numerous dermatiphytes, including T. tonsurans and Microsporum canis, can cause the disease, diagnosis is via a KOH preparation that shows segmented hyphae and spores. treatment of tinea corporis is with the topical antifungals miconazole, clotrimazole, or ketoconazole. Versus Candidiasis infection is due to fungus candida albican, and typically involves the skin creases of the pelvic region. iT PRESENTS AS a fiery erythematous rash with papules and plaques. beefy red plaques, satelite papulesm and superficial pustules can also be present. Diagnosis is confirmed with KOH preparation showing budding yeasts with pseudohyphae. Treatment is with the topical antifungal medications such as ketoconazole or miconazole

While performing a decompression of the fourth ventricle, the physician will?

Encourage extension while discouraging flexion. think of e and e you encourage the e. CV4 is done to increase the amplitude of the cranial rhythmic impulse. it is performed with the physicians' hand cupped over the area of the fourth ventricle at the occiput. the physician gently follows the motion of extension, while resisting the motion of flexion. these movements are followed until a still point is reached, at which point the motion of flexion and extension will stop, and then restart.

postmenopausal woman with bleeding appropriate next step?

Endometriall biopsy is the preferred initial diagnostic test for postmenopausal bleeding in women who can tolerate sampling and have a normal physical exam. most common cause of bleeding in post menopausal women are atrophy of the vagina mucosaor endometrium, but carcinoma must be ruled out.

A person has an STI with burning and purulent discharge. culture and gram staon of the discharge reveal no bacteria but many neutrophils. which of the following does the most likley causative organism use to infect cells?

Elementary body. so you had to know they are talking about chlamydia even though it appears to be that it is gonorrhea. what makes it not gonorrhea is that gonorrhea-gonorrhea would stain as gram-negative, kidney bean shaped dipplococci. also s2-s4 corresponds to pelvic diaphragm and ischiorectal fossa technique is useful in this treatment.

how do you treat mild GERD

FOR mild GERD-treatment begins wih lifestyle modification +/- a proton pump inhibitor. A viscerosomatic reflex modification +/1 a proton pump inhibitor. A viscerosomatic reflex associated with GERD would likleu be found in the mid to lower thoracic spine and ribs. T5-T9 or ribs 5-9.

chills and sharp pain on side for past three days. vomiting present, urge to pee-dating a new boyfriend, high temp, increased WBC count and WBC casts in urine, organisms is urease negative. what is the organism causing this

E.coli. this is pyelonephritis and E.coli is the most common cause.

45 yo african american woman has a hx of uterine fibroids. she complains of heavy bleeding during and between her menstrual cycles. today she feels weak and lightheaded. her hb is 8g/dl. lab analysis id most likely to reveal?

Elevated total iron binding capacity iron deficiency anemia is associated with an elevated total iron binding capacity. this woman has iron deficiency anemia due to chronic blood loss from her uterine fibroids. lab findings for iron deficiency anemia include decreased iron, decreased ferritin, and increased TIBC. this is because the liver responds by increaing transferrin (the protein that carries iron in the blood); thus the capacity to bind iron increases.

which antipsychotic medication can be used to treat the hallucinations, delusions or schizophrenia-like syndromes that are seen in huntington chorea?

Olanzapine. Tetrabenazine is used as first-line treatment for chorea seen in huntington disease. Second generation atypical antipsychotics such as olanzapine has been shown to improve the chorea-like symptoms in these patients. oLANZAPINE IS A SEROTONIN-dopamine antagonist. side effects include increased prolactin, orthostatic hypotension, anticholinergic effects, weight gain, and somnolence.

cONGESTIVE HEART FAILURE CAN cause a transudative pleural effusion, the best diagnostic modality to confirm congestive heart failure is?

ECHOCARDIOGRAM note--a pt with pleural effusion the first thing to do after a thorough history and physical exam is to perform a thoracentesis in order to analyze the fluid and employ light criteria. this will assist in determining whether a transudative or exudative effusion is present. From there you can create an adequate differential diagnosis to determine the etiology of the patient's effusion. light criteria 1. effusion protein/serum protein>0.5 2. effusion LDH/serum LDH>0.6 3. EFFUSION LDH >2/3 Upper limit of normal serum LDH if one of them above is positive then it is considered exudative fluid. -if none is true then it is considered transudative. differential diagnosis for exudative pleural effusion are: infection, malignancy, pulmonary embolism and collagen vascular disease. differential diagnosis for a transudative pleural effusion: congestive heart failure, cirrhosis, and nephrosis. this pt presented clinically in with congestive heart failure-progressive shortness of breath of two weeks duration, decreased breath sounds in lung bases, and pitting pretibual edema. was provided with the creatinine and PT/INR levels which were normal, making renal failure and cirrhosis unlikely in this setting. To confirm this pt has heart failure and echocardiogram is the next best test to quantify the ejection fraction, and rule out any valvular or anatomical abnormalities that could be contributing to the heart failure. note if you suspect pulmonary embolism or malignancy then a CT of chest with intravenous contrast would be useful.

Enteroinvasive e.coli cause a bloody diarrhea with white blood cells in the stool and fever due ti the immune mediated inflammatory reaction. EIEC condition is self-limited, so treatment consists of fluid replacement. note EIEC IS THE ONLY ONE THAT causes fever and bliidy diarrhea. what about EHEC

EHEC DO NOT INVADE THE THE GASTRIC mucosa so neither fever nor wbc in a stool sample are present with ehec. since EHEC DOES NOT CAUSE fever and does not cause white bld cells to be present in the stool. and does not ferment sorbitol.

thoracic counterstrain point in the back use?

ESARA t5 if rotated right how would you treat? -Patient's head is rotated left. the left shoulder is pulled posterior and caudad. Thoracic counterstrain tenderpoints are classically treated by positioning the pt in a way that extends, rotates away and side bends away from the side of the tender point. In this case, since it is the T5 tenderpoint on the right, the correct treatment position would be extension, rotate left (away) and sidebend left (away). rotation is achieved by rotating the head as wel as pulling the shoulder posteriorly. The caudad force on the shoulder will induce side bending to the left in order to achieve the proper treatment position.

what site should not be injected with lidocaine?

Ears and fingers, as the epinephrine will cause vasoconstriction which may lead to ischemia and possible necrosis. ---Microvascular areas of the body should not be further compromised with a vasoconstrictor when they are injured. --end organs with small capillaries are most at risk for ischemia and necrosis with vasoconstriction caused by epinephrine, a catecholamine. tHE DIGITS, THE EARS, NOSE TIP, and penis are areas to avoid injecting with epinephrine--read the answer choices well--it will help explain things much better and using your knowledge of what the drugs do can help a lot.

weight was heavy that it pulled his elbow into a fully extended position just prior to him dropping the weight. what type of contraction was occuring during the described motion?

Eccentric contraction Eccentric contraction involve the lengthening of a muscle during contraction due to an external force. muscle injuries commonly occur due to a concentric rather than eccentric contractions

when an external force overcomes the force of contraction what is this called?

Eccentric contraction.

A tapeworm transmitted from eggs found in dog feces and is responsible for large, locular cysts foind in the liver, lungs, and brain. it is most commonly sylvatic disease of sheep and wolves or sheepdogs. what is this?

Echinococcus granulosus is associated with contamination of food by dog fecrs and presence of large, locular cysts in the brain, liver, and lungs. pretreat with alcohol or antihitamine-medical treatment of choice is surgery with adminstration of albendazole.

Amblyomma americanum better known as the lone star tick is a common vector for?

Ehrlichiosis (ehrlichia chaffeensis), tularemia (francisella tularensis), and southern tick associated rash illness (STARI, which is presumed to be borrelia lonestari)

you find the greatest restriction at T5 where would you a counterstrain tender point associated with this dysfunction be found anteriorly?

On the midline of the sternum at the level of rib 5 Anterior thoracic counterstrain points associated with T2-T6 are located on the midline of the sternum at the level of the corresponding rib and are treated with marked flexion.

77 yo widow with mild dementia is admitted due to malignant htn. this is the third admission in the past six months for elevated blood pressure. she has no living will or durable power of attorney for healthcare. after she i stabilized the son tells the doctor he does not want his mother receiving any medications for htn. as the medication will surely kill her. as the patient herself lacks capacity, the most appropriate course of action is for the physician to?

Ensure the surrogate makes decisions in the patient's best interest. if a surrogate has no knowledge of a patient's prior wishes ad no basis for substitutes judgement, then decisions should be based on the best interests standard.

when is enuresis diagnosed? what is the chapman point posterior and anterior for the bladder?

Enuresis can be diagnosed once symptoms progress past the age of 5. it is most common in male children and the first of treatment is alarm therapy which is a form of cognitive behavioral therapy. nocturanl enuresis will typically resolve in time as the child continues to grow. Chapaman point for the urinary baldder is anteriorly about the umbilicus and posteriroly at the transverse process of L2.

5 yo female presents to the clinic with her parents for fever and abdominal pain. Her mother states that they have been going to the dog park daily with their golden retriever. physical examination reveals right upper quadrant tenderness and hives distributed symmetrically on her extremities and trunk. you are concerned she might be suffering from a zoonotic infection. the most likely finding on laboratory studies is?

Eosinophilia--most common laboratory finding in a pt with toxocariasis is eosinophilia. this pt has toxocariasis, an infection caused by the dog woundworm toxocara canis. Although the main clinical manifestations vary depending on the organs infected, the most common characteristic is eosinophila. A secondary or reactive increase in blood eosinophils, tissue eosinophils, or both is associated with a wide variety of infections especially helminthic parasites which include the flatworms cestodes and trematodes as well as nematodes (roundworms). The soil of parks and playgrounds often harbors the eggs of T. canis, and infection may cause a variety of human disease that involves the liver, heart, lung, muscle, eye, and brain. The eggs are deposited in the soil through dog feces. The eggs are typically ingested in humans through oral contact with contaminated hands. Once the eggs are ingested, they decorticate in the intestine and the larvae penetrate the bowel wall and migrate through vessels of the human body, often collecting in the muscles, liver, lungs, and sometimes the eyes or brain. The destination of the larvae will localize symptoms. there are three recognized symptoms caused by this roundworm: covert toxocariasis, visceral larvae migrans, and ocular larva migrans. Covert toxocariasis commonly presents in children and includes a mild, febrile illness accompanied by cough or abdominal pain, headaches and behavioral problems. Pruritic urticaria-like cutaneous lesions, also known as hives, are also a common manifestation. In this patient, the larvae traveled to the liver and caused the abdominal pain and right upper quadrant tenderness.

in what type of patients should Etomidate not be used in?

Etomidate is a sedative that reduces steroid synthesis. Etomidate should not be used for elective surgery in patients with addison's disease A patient presents with symptoms of lethargy and fatigue coupled with orthostatic hypotension and unusual pattern of hyperpigmentation of the axillae and buccal mucosa are suggestive of addisons's disease, or insufficiency of the adrenal cortex, leads to low levels of glucocorticoids, mineralcorticoids, and adrenal androgens. The adrenal cortex function may be reduced by infection, malignancy or autoimmunity. In response to low production of adrenal cortex hormones, the pituitary increases production of adrenocorticotropic hormone and melanocyte-stimulating hormone leading to increased pigment production. Although etomidate could be used to induce general anesthesia before the patient's elective surgery, this medication has the undesirable side effect of inhibiting steroidogenesis, which would worsen her adrenal insufficiency. Etomidate is an extremely short acting nonbarbiturate hypnoic medication that can be used for anesthesia. it has minimal cardiac effects, making it ideal choice for patients with cardiac disease.

What kind of depression would you expect to see with a stable angina?

Expect to see ST depression. ST depressions are associated with myocardial ischemia that is not transmural, such as in stable angina. this patient is experiencing stable angina defined as chest pain triggered by exertion and relieved by rest and nitrates. A clinical diagnosis of angina pectoris has a 90% predictive value of myocardial ischemia that is not transmural (has not transversed the entire cardiac wall). ST elevations indicate acute transmural ischemia, which is found in prinzmetal's angina (also known as variant angina) and myocardial infarctions.

A pt is frightened and requests her family not be informed regarding her condition or prognosis. the patient's husband soon arrived at the emergency epartment inquiring anxioiusly about the situation. the most appropriate course of action is to?

Explain that the patient requested privacy until further notice and ask him to wait patiently in the lobby. remember that your responsibility is to your patient, but you also have an obligation to be professional and courteous. it is reasonable to explain that the patient requested privacy rather than simply dismissing those who come asking; especially considering the patient;s husband is likely aware the individual is present at this particular emergency departement and will specifically ask about the patient's condition.

FACE mnemonic for the diagnosis of strep throat?

FACE mnemonic to remember the centor criteria for diagnosing of strep throat Fever, (lymph) Adenopathy, Cough absent, and Exudates. first line treatment for group A strep (strep pyogenes) is penicillin. classic symptoms of strep throat=temperature, sore throat, strawberry tongue, exudates and tender lymph nodes. In order to prevent further complications like rheumatic fever and glomerulonephritis in this pt, it is necessary to treat with pcn immediately. Macrolides are second line agents for treatment of group A strep infections particularly in pcn allergic pts.

FISH VERSUS KARYOTYPING

FISH INVOLves treating a chromosomal sample with a fluorescent DNA probe and directly visualizing the sample under the microscope. In the case of williams syndrome, a probe against the microdeletion region could be used and diagnosis confirmed by the lack of staining. Karyotyping allows for direct gross visualization of chromosome and can visualize chromosomal duplication or deletion, but not microdeletion

True or false the goal of treatment of counterstrain is to reduce tenderness of the tender point?

False, the patient should be completely passive for the treatment to work. During counter-strain technique, the patient should be completely passive while being brought into position and should remain passive until returned to a neutral position. A goal of at least 70% reduction in tenderness should be maintained, however 100% reduction in tenderness is preferred. anterior points generally require a position of flexion and posterior points require a position of extension. the pulse appears at approximately 90 seconds and often correlates with myofascial tissue relaxation, not further tissue injury. The goal of treatment of counterstrain is to reduce muscle spasm and somatic dysfunction. the way to assess proper positioning is through tenderness of tender point.

True or false-viscerosomatic changes are acute changes they are not chronic changes?

False. note this statement is true-the degree of somatic dysfunction is directly related to the degree of visceral pathology

12 YO male presents with abdominal pain, pancreatitis and skin lesions. physical exam indicated gross hepatomegaly. fmhx includes pancreaititis secondary hypertriglyceridemia in the patient's grandfather. lab values reveal hypertriglyceridemia with low LDL and HDL levels. most likely diagnosis is?

Familial lipoprotein lipase deficiency aka type 1 familial chylomicronemia=autosomal recessive. familial lipoprotein lipase deficiency is an inherited hyperlipidemia due to a deficiency of lipoprotein lipase, an abnormality in lipoprotein lipase, or a defect in apolipoprotein CII. The result is elevated chylomicrons, VLDL, and triglycerides within the circulation, placing the patient at increased risk for pancreatitis. CII is a lipoprotein lipase cofactor that catalyzes cleavage which is necessary for the action of lipoprotein lipase. lipoprotein lipase is an enzyme located on the endothelium of blood vessels within adipose and muscle tissue. it is responsible for the removal of triglycerides and fatty acids from chylomicrons and VLDL for transport into extra-hepatic tissues. In this disease state, this is not possible; therefore, there is a build up of chylomicrons, VLDL, and triglycerides within the circulation. LDL and HDL levels will be low because VLDL cannot be acted upon for transformation into LDL, and HDL does not have a lipoprotein to exchanage its cholesterol ester with. Since LDL levels are not elevated, there is low risk for coronary artery disease and atherosclerosis. However, there is a high risk for pancreatitis due to hypertriglyceridemia. In these patients, there will be hepatosplenomegaly and risk for lipemia retinalis, a white appearance of the retina secondary to lipid deposition.

what is farmer's lung?

Farmer's lung is a hypersensitivity pneumonitis associated with exposure to thermophilic actinomyces in moldy hay.

WHAT is fatal hydantoin syndrome and what medication causes it?

Fatal hydantoin syndrome is caused by the anti-epileptic medication phenytoin. congenital defects include microcephaly, dysmorphic craniofacial features, hypoplastic nails, and distal phalanges, cardiac defects, intrauterine growth retardation, and intellectual disabilities later on in life.

which of the following is the best initial colon cancer screening test?

Fecal occult blood testing-the best initial test is usually the safest, quickest, and cheapest but be sure to understand the screening. Fecal occult blood testing is the initial inexpensive test used to screen but (Not diagnose colorectal cancer). Flexible sigmoidoscopy every five years or colonoscopy every ten years is recommended for all adults beginning at age 50, or ten years earlier than the diagnosis of colorectal cancer in any first-degree relative.

A 42 yo male presents with a bulge in the upper thigh next to his groin. he is otherwise asymptomatic; however, he notes that he has occasional abdominal pain, nausea and vomiting. what type of hernia do you think this patient has?

Femoral hernia A femoral hernia is the type most commonly associated with bowel incarceration. It occurs through the femoral canal and is more commonly found in women--it protrudes through the femoral ring. femoral hernias are acquired, the least common type of hernia, and occur more common in women. up to 40% present as emergencies with incarceration or strangulation due to strangulation within the femoral ring.

you see midfacial underdevelopment, microcephaly and delayed CNS development. what do you think this person has?

Fetal alcohol syndrome

what syndrome does phenytoin cause?

Fetal hydantoin syndrome-characterized by cleft lip or palate, can also cause nail hypoplasia, growth retardation, mental deficiency, cardiac defects. phenytoin can reduce absorption of folic acid by inhibiting folate conjugase in the jejunum (leading to neural tube defects); providing folic acid can help diminish the risk of neural tube defects

Constellation of things associated with an inversion ankle sprain?

Fibular head moves posteriorly; the tibia externally rotates, the femur internally rotates, ipsilateral pelvis rotates posteriorly and the sacrum goes into an anterior torsion facing the injury (the left inversion sprain has a left on left sacral torsion) HOW TO remember this-actually practice doing it on yourself. ankle inversion- -fibular head goes posterior -tibia externally rotates -femur internally rotates -pelvis goes posterior -sacrum anterior torsion. -if left inversion strain then it would be a left on left sacral torsion.

what is the fick's principle?

Fick's principle can be used to calculate cardiac output. the stroke volume can be calculated using more simplified equation for cardiac output: Q/min=HR *SV Initially knowing the concentration of oxygen (O2) of fully oxygenated blood in the pulmonary vein and the O2 of deoxygenated blood in the pulmonary artery, the oxygen utilization, per liter of blood (O2ml/L), CAN BE CALCULATED as the difference of the two such that: O2ml/L=(O2mL/dl pulmonary vein)-(O2ml/dl pulmonary artery) =20ml/dl-15ml/dl =5ml/dl (necessitates conversion to ml/l as 0.1L=1dL =50ml/L Since we know resting oxygen consumption is 200ml/min, we can calculate the cardiac output (Q/min): Q/min=(200ml/min)/(50ml/l) =4l/min or 4000 ml/min cardiac output using this calculated cardiac output (Q/min), the SV (stroke volume) can be solved using the equation Q/min=HR*SV 4000ml/min=70*SV, rearranging to SV=57mL/beat

what do you do when you localize a chapman point?

Once you locate a chapman point, firm pressure is applied to the point. Pressure is applied in a circular pattern with the attempt to flatten the mass, and is continued until the lesion is resolved or the patient can no longer tolerate treatment.

Patient comes in to the clinic after she has a fall during her roller derby competition. she is uncomfortable and sits slightly bent forward. physical exam findings shpow a negative lumbosacral spring test and a positive left seated flexion test. You also find a deep left sacral sulcus, a posterior and inferior left ILA. you diagnose her with a left unilateral sacral flexion and palce her prone while standing ont he side of dysfunction. which of these describes how you would treat this dysfunction with a BLT technique?

Flex the patient's knee, abduct and internally rotate the hip until you gap the sacroiliac joint. ASK the patient to take a couple of deep breaths and take the sacral sulcus forward in the direction it likes to go until you fell a release. A balanced ligamentous tension is a subtype of MFR hich treate somatic dysfunctions by moving the segment in the direction of ease and restoring ligamentous tensions. it is an indirect technique which uses thr patients's breathing or muscular cooperation to overcome resistance and release the lesion. remember that th respiratory motion of the sacrum occurs around the superior transverse axis which is at level S2. during inhalation the sacral base moves posteriorly. When the sacral base moves posteriorly this is also known counternutation. A good clinical peral is that the most common dysfunction of the sacrum in post-partum patients is bilateral sacral flexion.

On structural examination, hypertonicity is noted within the suboccipital muscles bilaterally with tenderness at C3. the C3 joint has restricted motion translating to the left while in the flexed position, which is improved in extension. what would be the best initial position to perform direct muscle energy technique to this somatic dysfunction?

Flex, sidebend right, rotate right lateral translation to the left will produce right side-bending. To diagnose somatic dysfunction in the cervical spine, you name the segment for the position of ease. in this case, there is ease with extension and sidebending to the left. In the inferior division of the cervical spine (C2-C7), It is generally accepted that sidebending and rotation occur to the same side. therefore, the diagnosis is ERLSL. For muscle energy, you place the patient at the restrictive barrier in all three planes and have the patient direct the force to the position of ease. muscle energy is one of the techniques used to treat somatic dysfunction in the cervical spine. it is a direct technique, which means it directly engages the restrictive barrier. Initial positioning by the physisician is AWAY FROM the position of ease. in this case, the initial position for muscle energy treatment is C3 flexed, sidebent right and rotated left right. The physician then instructs the patient to gently return his/her head to neutral, and this position is held for 3-5 seconds and then released. The physician awaits a few moments, and then introduces further motion into the new restrictive barrier for 3-5 seconds. The sequence is repeated three to five times in total. the patient is then returned to the neutral position and the physician will re-assess.

right sacral iliac pain. positive seated flexion test on the right, a shallow right sacral sulcus, and a positive lumbosacral spring test. the most likely intersegmental dysfunction present at l5 is?

Flexed, sidebent left, rotated left. The patient's complaint of sacroiliac pain is most likely from the recent fall on buttocks and is likely associated with an innominate, sacral and /or lower lumbar dysfunction. Torsions occur between l5 and the sacrum and the diagnosis at L5 will correlate to the diagnosis of the sacrum, and vice versa. The general rules for L5 in relation to sacral torsions are: 1) L5 rotates opposite of sacral rotation 2) L5 sidebends toward the side of the oblique axis of the sacrum forward torsions are associated with type 1 dysfunctions at l5 and bacward torsions are associated type II dysfunctions at L5. THE SACRAL DYSFUNCTION DESCRIBED IS A BACKWARD SACRAL TORSIOn because the lumbosacral spring test (patient prone, operator pushes with palm of hand at lumbosacral junction and release if motion/spring then positive) is positive. RECALL THE SEATED TEST will be positive on the opposite side to the oblique axis for sacral torsions (both forward and backward). Thus, the oblique axis is on the left here because the seated flexion test is positive on the right. Putting this together gives the diagnosis of right sacral rotation on a left oblique axis (right-on left sacral torsion). The fact that the right sacral sulcus is shallow is consistent with this diagnosis. The fact that the right sacral sulcus is shallow is consistent with this diagnosis. backward torsions are associated with fryette type II (NON-NEUTRAL) mechanics for L5. tHE SACRAL TORSION rules also state that L5 sidebends to the same side as the oblique axis (left in this case) and rotates opposite of the rotation of the sacrum ( left in this case); therefore, L5 is non-neutral (flexion is the only viable option here), sidebent left and rotated left.

female student presents with right anterior groin pain. worse after prolonged sitting, tenderpoint is elicited medially to her right anterior superior iliac spine, approximately 1/3 of the distance toward the midline. which of the following positions of the hip would most likely reduce the tenderness at the tenderpoint?

Flexion and external rotation Based on the above findings, this pt likely has a somatic dysfunction of her iliacus muscle. This is trreated by placing the patient's hip into bilateral flexion and external rotation, with her knees flexed as well. Of note, the psoas tender point is 2/3 of the way toward her midline and would be treated the same way, with the addition of sidebending toward the tenderpoint to the level affected.

how do you treat AT2 counterstrain point?

Flexion of head and neck with sidebending if necessary

A tenderpoint is palpated 7cm medial to the and slightly cephalad to the greater trochanter posteriorly. which of the following positions would most likley reduce tenderness at the tenderpoint?

Flexion, abduction, external rotation of the left hip.

Helminths-Trematodes

Flukes

An antiandrogen that competitively inhibits the action of androgens at the testosterone receptor. It is used as androgen deprivation treatment in patients with advanced prostate cancer. Anastrazole clomiphene danazol finasteride flutamide

Flutamide

decrease of bilirubin to normal levels after administration of phenobarbital. what syndrome is this true for?

Gilbert syndrome. Due to the mild nature of gilbert syndrome, treatment is not necessary, yet phenobarbital can be used for relief if medication interactions or other comorbid issues cause very elevated levels of indirect bilirubin >20mg/dl phenobarbital response will drop the bilirubin levels to within normal limits for glibert syndrome. phenobarbital functions to increase the enzymes within the liver thus increasing UDGPT. note to self-when you have done process of elimination on what it is probably not and there is somtething that you see and do not know what it is-go ahead and pick that as the answer.

government worker. after accidental breakage of vial containing a virulent pathogen--small puncture wound of his finger from the vial glass occured. within four days, he began to develop a temperature of 39.8 degree celsius (103.6 degree fahrenheit), chills and a new erythematous swelling of his right epitrochlear and axillary lymph nodes. closer exam shows a sharply demarcated ulceration of the third finger of the right hand with yellow exudate. serum titers were performed but results are still pending. initial cultures show a gram-negative coccobacillus growing in cysteine-glucose-blood agar. based on the clinical and laboratory findings, which of the following organisms would most likley be the infectious agent?

Francisella tularensis presentation of fevers, chills, and lymphadenopathy is consistent with ulceroglandular variant of tularemia, caused by francisella tularensis. Francisella tularensis is an aerobic, gram-negative bacillus which is extremely difficult to culture; best grown in cysteine-glucose blood agar. rabbits and rodents sere as the typical vectors for the francisella. ulceroglandular tularemia typically results from a tick or rodent bite, leading to an erythematous papule at the site. The papule will then grow, rupture and ulcerate over the next 2-3 days; the ulcer is well-circumscribed and will usually produce a yellow exudate before eventually turning black. ulcerative lesion is more present with francisella than yersinia

what is another name for adhesive capsulitis? who is at risk of adhesive capsulitis?

Frozen shoulder PATIENTS at risk of adhesive capsulitis include diabetics, females, and patients with hypothyroidism. Adhesive capsulitis often follows a trauma or joint immobilization. there is typically some type of insult to the joint resulting in inflammation. the local inflammation results in thickening and adhesion in the fascial layers of the joint capsule resulting in pain and decreased range of motion. you want to treat this adhesive capsulitis with spencer technique

what causes functional scoliotic curves to develop

Functional scoliotic curves develop as a compensatory change secondary to an underlying condition such as a short leg and psoas syndrome.

The clinician informs a woman that she would need to supplement her child's diet with vitamin D if she plans on breastfeeding because insufficient vitamin D may lead to?

Genu varum breastfeeding does not provide sufficient vitamin D levels to infants. this can result in rickets, which occurs because the bone cannot effectively mineralize without sufficient vitamin D. rickets manifests in children with craniotabes (thinning of the cranial bones), late closing fontanelles, rachitic rosary (thickening of costochondral joints), harrison groove (depression along the insertion of the diaphragm), pigeon breast, bowed legs (lateral bowing of the femur and tibia)=genu varum. and decreased height.

you are going home and see injured people on the road. what should you do before providing care?

Get an informed consent. while all ststes have some type of good samaritan provision, who is protected and the law is actually implemented varies by states. if possible, informed consent should be obtained prior to providing emergency care, providing that obtaining consent will not delay medical care or endanger anyone's life. Good samaritan laws are intended to protect individuals who help those that are injured or in danger, protecting the "good samaritan" who helps from liability if unintended consequence result from their assistance.

pt comes woth jaw claudication, scalp pain/ulceration, and polymyalgia rheumatica. treatment is high dose steroid. what is the diagnosis?

Gian cell arteritis Pts with this disease experience gradual occlusion of the temporal artery which can manifest in many different ways, such as a prominent and tender temporal artery, transient visual impairment, jaw claudication (leading to weight loss), scalp pain, and skin infarction in the distribution of the temporal artery (temporal and/or parietal scalp). Due to the serious potential effects of giant cell arteritis, high doese of prednisone are started immediately upon clinical suspicion. the patients can also suffer from polymyalgia rheumatica syndrome, which occurs in 50% of patients with giant cell arteritis. it is described as aching and pain in the muscles of the neck, shoulders, back and thighs. The patient's open mouth dysfunction is associated with temporalis muscle weakness, which does not allow his mandible to adeuately elevate and prevents optimal mastication.

women in first trimester of pregnancy what medication do you give them?

Give them PTU PTU in the Pirst (sound it out or say it as flipinos would). trimester of pregnancy and for women trying to become pregnant. methimazole is preferred in the 2nd and third trimester of pregnancy

differentiate the grades of sprains a person can have with an inversion ankle problem?

Grade 1-sprains will only have mild swelling and tenderness present and the patient will be able to bear weight and ambulate. Grade 2 sprains are associated with pain, swelling, tenderness, and ecchymosis with mild to moderate joint instability on examination. SUPINATION of the foot combines the movements of inversion, adduction and plantar flexion--you can remember it has plantar flexion by the fact that P is in suppination.

what glomerulonephritis type is this? ten pound weight loss, hematuria, recurrent respiratory infection and sinusitis over the past month. PE temp of 39 degree celcius, decreased breath sounds, with crackles on auscultation, urinalysis reveals sediment with more than seven rbc'S PER HIGH POWER (HPF) and numerous erythrocyte casts. Additional lab testing reveals a positive cytoplasmic antineutrophilic cytoplasmic antibodies (C-ANCA).

Granulomatosis with polyangitis.

What type of hypersensity is Graves disease and is it cytotoxic versus noncytotoxic?

Graves disease is a hyperhtyroid autoimmune disease noted to have antibodies against TSH receptors, resulting in a type II, noncytotoxic hypersensitivity reaction. it is type II non-cytotoxic hypersensitivity reaction since the autoantibodies interfere with the function of the cell but do not lead to a direct destruction of the affected cell (which would be type II, cytotoxic). type II cytotoxic hypersensitivity reactions results when circulating IgG antiboides bind to cells and target them for destruction by monocyte/macrophages (similar to opsonization). rheumatic heart disease is an example of a type II cytotoxic reaction. Additionally, the thyroid-specific T cells in gRAVES disease primarily act as a helper (CD4) rather than a suppressor for cytotoxic (CD8) CELLS.

hypoglycemia, decreased gluconeogenesis and loss of muscle mass-which hormone is deficient?

Growth hormone

the 5 H's and 5t'S OF acls for an unstable pt

H's= Hypothermia, hypovolemian, hypoxia, hydrogen ion(acidosis), hypo/hyperkalemia 5T's=Toxins, tamponade, tension pneumothorax, and Thrombosis hypothermia affects all organ system though perphasp none more significantly than the cardiovascular system. hypothermia causes decreased depolarization of cardiac pacemaker cells and results in bradycardia and eventually ventricular fibrillation or asystole. profound hypothermis is truly an emergency. as soon as possible, remove wet clothing and raie the core temperature above 30 degrees celsius. options for warming include; warmed, humified oxygen; heated intravenous saline, and warming balnkets and/or heat lamps. As the pt is warmed, continue resucitation protocols.

18 yo, severe shortness of breath, started yesterday, gotten worse over the last 24 hours, returned from an epic road trip to the grand canyon. P.E -interstitial infiltrates and auscultation reveals rales and wheezing. pts symptom worsen and he is put on a respiratory ventilator. what is the most likley cause of pt's symptoms?

HANTAVIRUS Hantavirus casues a severe, rapidly progressing pneumonia and is typically acquired in the southwestern United States. this pt is experiencing acute respiratory distress syndrome (ARDS) AS A result of a HANTAVIRUS infection. This virus is found in the southwestern unites states and this individual contracted it during his trip to the grand canyon. transmission is typically via inhalation of contaminated rodent fecesin the dry air. patients with hantavirus infections typically present with moderate to severe respiratory symptoms that quickly (within 24 hours) worsen into ards, WITH POSSIBLE HEMORRHAGE and renal failure. Severe disease is categorized by a cardiopulmonary syndrome where fluid accumulation in the lungs puts pressure on the heart that can lead to congestive heart failure. currently there is no effective treatment and this infection has a high mortality rate.

read the question well. A positive indicator of cardiovascular health will be?

HDL >62 MG/DL.

PPD-area of induration greater than 5 mm would be a positive test for who?

HIV-infected persons PPD test must be examined 48-72 hours after placement. in general population -induration greater than 15mm=positive test highest risk group with HIV-5mm or more is positive 5MM is positive also in person IN Contact recently in a person with TB disease, persons with fibrotic changes on chest radiographs consistent with prior TB, patients with organ transplants, persons who are immunosuppressed for other reasons (taking the equivalent of >15 mg/day of prednisone for 1 month or longer, taking TNF-a antagonist). 10MM or more=positive for immigrants <5 years from high prevlant countries, injection drug users, residents or employees of high risk congregate settings, childrens <4 years of age, infants, children and adolescents exposed to adult in high risk categories.

when treating HVLA to the thoracic spine do you trreat type 1 or type II dysfunctions firsT?

HVLA type II dysfunctions are treated first followed by type I dysfunctions. the setup for any technique is to enagage the barrier. In this case T6 has a type II dysfunction which would be treated directly by placing the thenar eminence on the posterior transverse process pointing cephalad. In this cASE t6 is rotated left, making the T6 left posterior transverse process the one being thrusted. The rest of the set follows by having the other hand's thenar eminence placed on the anterior transverse process pointed caudad. the thrust is anteriorly and cephalad. in summary-place the thenar eminence of the thrusting hand on the left transverse process of T6 with fingers pointed cephalad, with the other thenar eminence on the right transverse process with the fingers pointed caudad.

what is the Beck;s triad of pericardial tamponade

HYPOTENSION, JUGULAR VENOUS DISTENTION AND MUFFLED HEART SOUND. pulsus paradoxus (>10 mmhg decrease of systolic blood pressure during inspiration), increase in venous distention during inspiration, and electrical alternans (QRS compleexes changes in amplitude as the heart moves within the pericardial space) on ECG. and kussmaul sign (increase in venous distention during inspiration). in the acute setting treatment is always emergent pericardiocentesis. in young individuals it is caused by trauma to the chest. in an older person it is due to malignancy.

There is decreased sensation of the right hand when compared to the left. there is decreased sensation of the right hand, most notably on the palmar aspect of the thumn and first finger. which physical exam maneuver, is positive, would support your suspected diagnosis?

Have the pt hold her wrists in complete and forced flexion for about one minute. this is the phalan's test which is diagnostic for carpal tunnel syndrome-cts. manual therapy-or surgery for cases with evidence of nerve damage. carpal tunnel syndrome tend to be worst at night. the typical patient with CTS is a 45-60 yo caucasian female. comorbidities often associated with CTS include obseity, diabetes, hypothyroidism, recent menopause, and repetitive wrist activity. The initial workup of a pt with suspected CTS largely relies on the history and physical exam findings. EMG (electromyography) is useful in confirming the diagnosis. imagining plays a very limited role in diagnosing CTS, WITH THE exception of determining if a space-occupying lesion within the carpal tunnel is causing impairment.

A sexually active female with female with a frothy, foul-smelling greenish discharge should be suspcted of what?

Having trichomonas vaginalis infection. this organism is a motile protozoan. Trichomonas vaginalis is the most common sexually transmitted infection worldwide. Best diagnostic test for trichomonas vaginalis is nucleic acid amplification test. Trichomonas is an STD CAUSED by a motile protozoan. several diagnostic tests are available to aid in the detection of T vaginalis including nucleic acid amplification test (gold standard), culture, and direct microscopic visualization on a wet mount slide. wet mount is still used and a negative one should be followed by NAAT. THE AGENT USED TO TREAT TRICHOMONAS VAGINALIS IS METRONIDAZOLE and it forms free radicals that damage DNA.

what is the strongest predictor of a person's health ?

Health literacy it is greater than age, income, employment, status, education, level and race

What are heinz bodies

Heinz bodies are small inclusions of denatured hemoglobin, a metalloprotein, found inside erythrocytes as a result of oxidant damage. they are seen during episodes of hemolysis caused by underlying G6PD deficiency.

When should you suspect heparin-induced thrombocytopenia?

Heparin-induced thrombocytopenia should be suspected if labs reveal a drop of platelets 50% below baseline or if skin necrosis develops at the site of heparin injections. --classic presentation-she agrees to receive anticoagulation therapy to reduce risks of future strokes associated with this arrhythmia. Five days later you notice a small area of necrotic skin three inches lateral to her umbilicus accompanied by purpura at nearby sites. A complete blood count reveals platelets to be 21, 000/micoL. the most likely cause is= heparin.

At what location is hepatitis likely to cause tissue texture changes?

Hepatitis may cause tissue texture changes at T7 via a viscerosomatic reflex

ideal treatment for acute bacterial sinusitis?

High dose amoxicillin-clavulanate or amoxicilin.

Motion of the atlantoaxial joint primarily occurs in what plane>

Horizontal plane. -recall that a plane is the direction motion occurs in -Axis is the fixed structure around which the pane of motion occurs. the axes is vertical.

Sidebending axes of rotation are?

One anterior-posterior axis and two vertical axes. Sidebending rotation strains occur around three separate axes. the sphenoid and occiput rotate in the same direction around one anterior-posterior axis and in opposite directions around two vertical axes.

Prerenal acute renal failure is the most common type of acute renal failure and is due to a decrease in systemic blood volume or a decrease in renal perfusion. Urine chemistry will reveal a BUN: creatinine >20, urine osmolality >500 mOsm/kg h20, urine sodium <20 mEq/L and FeNa<1%. urinalysis will show what type of cast?

Hyaline cast hyaline casts are solidified Tamm-Horsfall Mucoprotein secreted from the tubular epithelial cells of individual nephrons. they are increased in the urine when there is low urine flow, concentrated urine, or an acidic renal environment. therefore, they are commonly increased in individuals with normal renal function who are severely dehydrated or after vigorous exercise. treatment involves fluid and electrolyte replacement as well as correcting the underlying cause.

Dilation and curettage, methotrexate and serial beta-HcG monitoring are the treatment for?

Hydatidiform molar pregnancy

28 yo female presents with attacks of nausea, vomiting, vertigo, and ringing in her ear. Physical exam reveals air conduction is greater than bone conduction when placed on the right mastoid bone. sound also lateralizes to her left ear when tuning fork is placed on the vertex of her skull. Osteopathic cranial and sinus drainage manipulative techniques were performed with the goal of decreasing congestion within the inner ear. the pathology of her condition involves?

Hydropic dilation of the cochlea. Meniere's disease results in episodic vertigo, sensorineural hearing loss and tinnitus. it leads to hydropic dilation of the cochlea due to excess endolymphatic fluid in the cochlea. The rinne test will reveal air conduction greater than bone conduction while the weber test will reveal lateralization to the opposite ear involved when there is the presence of sensorineural hearing loss. Tinnitus is low-pitched and described as an ocean-like roaring. hearing loss is sensorineural, fluctuating in nature, and typically associated with ear fullness or pressure. sensorineural hearing loss refers to dysfunction of the inner ear or vestibulocochlear nerve. this is contrast to conductive hearing loss which involved a dysfunction of the conductive structures of the ear, which include the middle ear, ossicular chain, or the tympanic membrane. OMM for this pt would best focus on cranial and lymphatic drainage techniques to decrease congestion within the right inner ear. This is accomplished via normalizing cranial strain patterns that could facilitate increased venous return to the heart from the head. Also, performing facial drainage/Galbreath techniques in which the frontal, nasal, maxillary, and zygomatic bones along with the tempoomandibular joint (TMJ) and lymphatic ducts are addressed could help reduce lymphatic congestion of the inner ear. episodic vertigo, tinnitus and hearing loss think of meniere's disease.

Nephrolithiasis may be secondary to hypercalcemia of malignancy. the most common cancers that cause hypercalcemia are breast cancer, lung cancer and multiple myeloma. .what is this called>

Hypercalcemia of malignancy. the level of T4 corresponds to the nipple. physical exam finding described Peau d' orange or a swollen pitted skin surface, which is caused by stromal infiltration and lymphatic obstruction with edema signifyig malignancy.

four or more day period with persistently elevated, expansive, or irritable mood, increased activity or energy, and three or more of the following symptoms (four is the mood is only irritable): grandiodiosity, decreased need for sleep, talkativeness, racing thought, distractibility, increased goal-directed activity, and increase in irresponsible behavior-what is the diagnosis?

Hypomaniac manic is greater than 4 days and cause marked impairment in social or occupational function manic episodes lasts about 1 week or any duration is hospitalization is necessary. diagnosis of manic episodeds requires the patient to have impaired occupation or social functioning.

80 yo female presents to the community health care clinic for recurrent presyncopal events. she states that she was recently started on a new medication because her blood pressure was too high. Lab studies reveal that her sodium is 129 mEq/L. blood pressure monitoring is performed in various positions and is Supine=120/80 seated=118/75 standing=97/72 Patient's physiologic state is best described as?

Hypovolemic hypotonic hyponatremia this indicates intravascular volume depletion and is often secondary to diuretic medications due to renal losses of sodium and subsequently water. hyponatremia is defined as a sodium level less than 135 mEq/L. Hypovolemic hypotonic hyponatremia indicates intravascular volume depletion with concomitant solute depletion. This patient has orthostatic hypotension, defined as a decrease in systolic blood pressure of 20 mmHg or a decreaswe in diastolic blood pressure of 10 mmgh within three minutes of standing when compared with blood pressure from sitting or supine position-this finding is a common clinical sign of volume depletion. other signs include decreased skin tugor, reduced jugular venus pressure, etc. decreased volume status leads to an increase in sympathetic tone to maintain an adequate systemic blood pressure. orthostatic hypotension may lead to presyncopal events which may manifests as dizziness, light-headedness, severe muscle weakness, or visual changes; there is no transient loss of consciousness or postural tone as in true syncope. one of the principle causes of hypovolemic hypotonic hyponatremia is renal losses secondary to diuretics, specifically thiazide diuretics. Thiazide diuretics inhibit sodium reabsorption in the distal convoluted tubule by blocking a sodium-chloride symporter, and this action leads to an increased excretion of sodium and water, as well as potassium and hydrogen ions. one of the adverse effects of this pharmacologic agent is hypotension secondary to this condition which may manifest as presyncopal events. additional causes of hypovolemic hypotonic hyponatremia include, gastrointestinal losses secondary to diarrhea or vomiting, 3rd space losses secondary to burns, pancreatitis, peritonitis, small bowel obstruction, and more.

HPV test reveals a person is postive for HPV type 16 what is the next step of management?

Hysterectomy Cervical adenocarcinoma in situ (AIS) should be managed with hysterectomy.

describe the sidebending rotation

IN SIDEBENDING rotation dysfunctions (physiologic dysfunction) the sidebending component results by rotation about two vertical axes. The sphenoid and the occiput rotate in opposite directions about these axes. The rotation part of this dysfunction ocurs about an anterior-posterior axis but here the sphenoid and the occiput are rotating in the same direction. it is named for the side of the convexity.

A study was conducted in order to determine the relationship between alcohol consumption and heart failure. Two matched groups were questioned about their histories of alcohol consumption. one group consisted of people diagnosed with congestive heart failure, while the second group had no history of heart disease. The study measured an odds ratio of 1.4 with a 95% confidence interval of 0.8 to 1.6. The results of this study can be interpreted as?

Insignificant because the confidence interval includes 1. If the 95% confidence interval of an odds ration includes 1.0, the study is statistically insignificant. Since the 95% confidence interval are justas likely to have consumed alcohol in the past as those without heart failure. This is the statement of the null hypothesis for this case-control study: therefore the study is statistically insignificant.

physicians caring for patients who are immediately dying should not initiate a request for organ donation. only trained representatives from an organ procurement organization should do this

true

idiopathic intracranial hypertension

Idiopathic intracranial htn is an idiopathic condition that presents with signs and symptoms of increased intracranial pressure such as headache, papilledema, and visual loss. It most common affects overweight women in the third decade of life but can be seen in patients of all ages. This pt has papilledema, which is optic disc swelling that occurs in pts with increased intracranial pressure.. it is commonly described as blurring of the optic disc margins. Pts with IIH frequently note diplopia due to dysfunction of cranial nerve VI. also known as the abducens nerve. ABDUCENS nerve palsy IS A DYSFIUNCTION OF THE LATERAL RECTUS MUSCLE which is innervated by CN VI. pts present with a lateral diplopia and an ipsilateral abduction deficit of the eye. the abducens nerve is most commonly affected due to its long intracranial course because the nucleus is located in the pons just ventral to the floor of the fourth ventricle. thus an increased intracranial pressure can casue enlargement of the fourth ventricle and put downward traction on the sixth cranial nerve. This causes stretching of the nerve at dorello's canal, which is the bony enclosure surrounding the nerve as it enters the cavernous sinus.

how to differentiate a left bundle branch block form a right bundle branch block

In a blind, the mnemonic WiLLiaM MaRRow may be helpful. if QRS>80 ms, then look at the precordial leads V1 and V6. if they make a W and M, then it is LBBB (WiLLiaM). If they make an M and W, then it is RBBB (MaRRoW). RBBB is recognized by QRS duration >120ms, rSR complex in V2, wide R wave in V1, and the QRS pattern with a wide S in V6. in LBBB there will be a domiant S wave V1 and an M wave in V6.--see M wave in V6 closer to the left side Note someone with a RBBB which can be determined by a QRS of greater than 120msec, rSR complex in V2, wide R wave in V1, and the QRS pattern with a wide S wave in V6. this condition is associated with COPD, valvular disease, or chronic coronary artery disease. It can also occur in normal individuals.

what is the axis of the lateral strains?

In lateral strains (non physiologic dysfunction) the sphenoid and occiput rotate in the same direction and around 2 vertical axes. Lateral strains are names for the position of the basisphenoid.

When do you use positive inotropes?

In the acute setting of cardiac Failure but would not be used in a patient without signs and symptoms of heart failure.

61 yo female seen for exacerbation of congestive heart failure, vital signs reveal a blood pressure of 160/90. heart rate of 66/min, respiratory rate of 22/min. she is afebrile and currently on mechanical ventilation. physical exam reveals 3+ pitting edema bilaterally with tissue texture changes at the thoracic inlet and rib6-10 bilaterally. the most appropriate initial lymphatic treatment is?

Indirect myofascial release to the thoracic inlet. treatment should be gentle and indirect in the ICU pt. pts on ventilation that is stable can still receive omm but it should be gentle.

where is the broca's area located?

Inferior frontal area Agraphia and motor aphasia occur in lesions to the frontal lobe motor aphasia is characterized by the inability to speak fluently. speech is slow and labored, and preposiions and pronouns are often omitted. language comprehension remains intact, but repetition is impaired. these symptoms result from disruption of broca's area. Agraphia is defined as the loss of the ability to write and spell. The superior frontal lobe contains components necessary for writing, such as coordination of hand motion, damage to the frontal lobe would lead to both agraphia and motor aphasia.

what region does the infraorbital nerve provide innervation to?

Infraorbital nerve is a branch of the maxillary division og the trigeminal nerve. it provides sensory innervations to the lower eyelid, side of the nose and upper lip.

upon prescribing her oral contraceptive pills containing both estrogen and progesterone, she wants to know how this will keep her from getting pregnant. which of the following mechanisms best explains the contraceptive effects of her medication?

Inhibition of the release of a mature ovum Oral contraceptive pills contain both estrogen and progestin which act to suppress FSH and LH, thus, preventing follicular maturation and ovulation, respectively. The primary goal and effect of these hormones is to suppress gonadotropins at the time of the mid-cycle LH surge, which inhibits ovulation or the release of mature ovum. The progestin component acts through a negative feedback mechanism to suppress the secretion of lH and, as a result prevent ovulation. The estrogen component enhances the negative feedback of follicle-stimulating hormone and prevents the follicles in the ovary from maturing. Estrogen also stabilizes the endometrium to prevent irregular menses. If no mature ovum is released, then fertilization cannot take place.

What is the initial pathophysiology finding that can be seen with Asthma?

Initial antigen stimulation acts on CD4 Th2 cells to release IL4 and IL5. These stimulates IgE production AND ACTIVATE eosinophils, respectively. In extrinsic asthma, initial allergen stimulation causes induction of CD4 Th2 CELLS TO RELEASE IL-4 and IL-5. IL4 STIMULATES ige PRODUCTION and IL5 activates local eosinophils. Upon secondary allergen exposure, antigens cross-link IgE antibodies on mast cells, which lead to the release of histamine. Most appropriate therapy for long term management will be Cromolyn sodium=stabilize mast cells and have prophylactic anti-inflammatory action. these drugs are only helpful if administered prior to an asthmatic episode. Anticholinergic agents such as ipratropium may be added to Beta-2 AGONIST therapy for acute exacerbations. Ipratropium has anti-secetory properties and, when applied locally, inhibits secretions from serous and seromucous glands lining the nasal mucosa. OMT for asthma treatment of ribs, thoracic spine, upper cervical complex, and muscles of respiration. in acute asthma attack, modulating sympathetic input to the lungs can be useful to encourage bronchodilation. This can be accomplished using rib raising or thoracic HVLA when appropriate.

physician offers his hands to the patient and notes that she takes an unusually long period of time to release her grip. this condition is caused by?

Instability during maternal meiosis--MEANINg IT WILL occur when inherited from the mother; overall maternal transmission for this disorder is much less common, possibly due to sperm selection factors. ---The CTG repeat alters RNA binding protein activity resulting in the abnormal function of a skeletal muscle chloride channel, insulin receptor, and cardiac troponin T. The skeletal muscle chloride channel dysfunction is responsible for the myotonia. This condition is Myotonic dystrophy which is recognized by its sustained muscular contractions or myotonia. myotonic dystrophy type 1 is a result of a trinucleotide repeat expansion which exhibits anticipation, which is an important feature of all trinucleotide repeat disorders. CTG REPEAT myotonic dystrophy type 2, which is both much less common and much less severe, and is almost exclusively an adult-onset disorder. it results from a tetranucleotide repeat (CCTG) rather than a trinucleotide repeat, though the number of repeats appear to have little correlation with disease onset or severity, with the sequence repeating 75 to more than 11,000

If a pt with gestational diabetes is unable to control their blood sugar levels with diet then what is the first line drug for treatment?

Insulin Diabetic diet is the first line treatment for gestational diabetes. pts should limit their calories and carbohydrates. blood sugar levels should be checked four times daily, a morning fasting glucose and two hours postprandial three times daily. if blood glucose cannot be controlled by diet alone, then medications is necessay. insulin is the treatment of choice for gestational diabetes mellitus because it does not cross the placenta. glyburide is another common medication utilized for gestational diabetes in those who refuse insulin injections.

What does insulin do in utero?

Insulin decreases the synthesis of surfactant. Mothers with gestational diabetes put their baby at risk for massive intrapulmonary shunting due to widespread lung collapse. A mother with gestational diabetes will expose the fetus to higher blood sugar levels, elevating fetal insulin production, which may inhibit the maturation of fetal lungs. this can lead to atelectasis in the baby just after birth, which leads to massive intrapulmonary shunting.

A 6 year old male with a history of seasonal allergies presents with pruritic, scaly plaques of his antecubital fossa. What Interleukin is this most closely associated with?

Interleukin 4 Interleukin 4 is involved in humoral immunity and is commonly elevated in patients with atopic disorders atopic dermatitis is a pruritic skin condition that typically begins in early childhood. patients develop rough flaky pataches of skin, particularly on the flexor surfaces such as the antecubital fossa. pruritus is a prominent feature and leads to excoriations and scaling of affected areas. This condition is most common in children that suffer from other atopic conditions, such as asthma and allergic rhinitis. Interleukin-4 is involved in humoral immunity and plays an important role in atopy. it promotes B lymphocyte isotype switching to igE.

Internal rotation of the temporal bone may result in what?

Internal rotation of the temporal bone may result in Bell's palsy, which can lead to the patient to complain of tinnitus, vertigo and a high pitched ringing. bell's palsy is due to facial nerve irritation. compression by internal rotation of the temporal bone, or inflammation and edema caused by lyme disease, herpes simplex 1, or autoimmune diseases are thought to be common causes of facial nerve lesions. facial nerve paralysis can be due to either a lower or upper motor neuron lesion. In lower motor neuron involvement the forehead is classically involved. This is in contrast to an upper motor neuron lesion of the facial nerve in which the forehead is spared due to bilateral innervation of the facial nerve nucleus by the upper motor neurons.

pt mentions the thought of remembering to take her pills is often bothersome. she has never had surgery, has no family hx of hypertension or cancer and does not smoke, drink or use any recreational drugs. At this time, what will be the most effective for the purposes of contraception?

Intrauterine device Contraception in adolescents should focus on methods that are long-acting and easily reversible, such as an intrauterine device (IUD). The patient should also be encouraged to use condoms as a method of preventing STDs. two types of IUD=Copper and hormonal Copper IUD-acts by releasing trace amounts of copper into the uterine cavity, which thickens the cervical mucus and prevents sperm from passage through the cervix; it can remain in place for ten years. Hormonal IUD systems release small amounts of levonorgestrel into the local environment, which causes decidualization of the endometrium and thickening of the cervical mucus; depending on the brand, it is indicated for use for up to three years (skyla) or five years (mirena). pregnancy rates with these devices are extremely low-as low as 0.6% per year for copper and 0.1% with levonorgestrel secreting systems, making them among the most effective methods of birth control available. Risks include a small incidence of uterine perforation during insertion, and the copper IUD in particular has been known to casue dysmenorrhea and menorrhagia during the first few cycles after insertion. If a patient with an IUD does become pregnant, the pregnancy is ectopic in nearly half of all cases; however, the absolute incidence is lower for both ectopic and intrauterine pregnancies. Finally, like most other forms of contraception, an IUD does not protect against sexually transmitted disease. this should be emphasized to the pt, and she should be encouraged to use condoms as well.

PAIN STARTED after starting to train for track and field. PE reveals hypertonicity and decreased range of motion of the hamstrings bilaterally. ME is used to treat the patient's restricted hamstrong. hip is flexed and the pt is asked to extend the hip against resistance with the knee extended. this type of muscle contraction that results in increased tension without an approximation of the origin and insertion is?

Isometric contraction.

when a contraction occurs that causes an increase in muscular tension but no change in length?

Isometric contration iso=same metric=length so length is the same.

The soft end of a cotton swab is lightly stroked along the soles of his feet and the patient with DM II denies any sensation. the type of sensory structure most closely associated to detection of this type of sensation are?

Meissner corpuscles the detection of light, discriminatory touch detected by the palms and digit of the hands and the soles of the feet are providing by small encapsulated nerve endings known as meissner corpuscles. This type of mechanoreceptor is found in abundance within the dermis of hairless skin.

Constant muscle tension as muscle changes length? During the flexion movement of the biceps curl, which of the following remains constant?

Isotonic Muscle tension. Biceps curl is an example of an isotonic contraction. an isotonic contraction is one that generates a force by changing the length of a muscle. In this type of contraction the muscle tension is constant as the muscle changes length. isotonic contractions can be concentric or eccentric. CONCENTRIC muscle contractions occur when a muscle shortens while generating force. in relation to the elbow, the upward motion when performing a bicep curl is considered a concentric muscle contraction as the bicep's origin and insertion approximate. ECCENTRIC muscle contractions occur when a muscle increases in length with contraction. the muscle elongates while under tension due to opposing forces being greater than the force generated by the muscle, such as the lowering motion of a bicep curl.

complaint of wrist pain, pain began after starting a new exercise regimen, including biceps curls. when lifting she is using appropriate controlled contraction, and when lowering the weight she is doing so in a controlled and proper fashion as well. the type of muscle contractions that take place during both phases of a properly performed biceps curl is an example of?

Isotonic contractions muscle contractions that result in the approximation of the muscle's origin and insertion without a change in tension are known as isotonic contractions. concentric contractions occur when the origin and insertion of the muscle are approximated. this occurs during the first phase of a biceps curl where the elbow is flexed. Eccentric contractions occur when the muscle lengthens while contracting. Eccentric contractions occur during the second phase of biceps curl where the elbow is extended.

where does the L4 dermatome run? where does the L5 dermatome run?

L4 dermatome runs from the L4 level of the spine, across the knee and into the medial surface of the foot. L5 dermatome runs from the band of skin that starts at L5 level before coursing laterally across the thigh and knee and into the anterior leg and dorsum of the foot. The L5 root supplies motor innervation to all three gluteus muscles, the tensor fascia lata, the tibialis anterior and posterior, extensor digitorum brevis ans extensor hallucis longus.

left anterior sacral base, with a negative lumbosacral spring test which of the following somatic dysfunctions is present at L5 given the above information?

L5NSRRL If the patient has a left sacral base anterior, the sacrum is rotating right With a negative lumbosacral spring test, they must have a right axis engaged which means there is a right on right sacral torsion present. Given this information, we can predict L5 mechanics knowing that a) when L5 is rotated, the sacrum rotates in the opposite direction; and b) A sacral oblique axis is engaged on the same side as the L5's sidebending. since this patient has sacral axis on the right, L5 must be sidebent right. we also know that the sacrum is rotating right: therefoe, L5, must be rotated in the opposite direction. the final piece of information we need to answer this question comes from Fryette's first principle: As L5 IS ROTATED AND SIDEBENT IN OPPOSITE DIRECTIONS , IT MUST BE IN A NEUTRAL PLANE.

An older gentleman is found minimally unresponsive in his home. pertinent past medical hx includes coronary artery disease, obstructive sleep apnea, and type II diabetes mellitus. initial lab results reveal bad kidney, high acid level. what medication for diabetes is the likely causer?

Lactic acidosis can occur as a result of metformin use, typically occuring in patients with underlying renal disease. use of this medication is contraindicated in patients with acute and chronic renal failure.

3 yo male, complaint of persistent non productive cough. fully vaccinated and has no other past medical hx. vitals reveal bp of 110/70mmhg, heart rate of 20/min, a temperature of 37.5 degree celsius. and an oxygen saturation of 99% on room air. hx shows a cough (ehh ehhk) the examination is remarkable for an erythematous oropharynx and the lungs are clear to auscultation. the most likely diagnosis is?

Laryngotracheobronchitis aka croup is a common cause of illness in children <4 yo and has a classic seal like barky cough. croup=common cause of partial upper airway obstruction in children. parainfluenza virus are mainly responsible for croup, although many other viruses can cause this syndrome. Due to the smal diameter of the trachea in childhood, inflammation/edema can pose a significant obstruction to the airway and create the classic seal-like, barky cough as heard ehh ehhk. An AP radiograph of the nack may show a "steeple"sign, which is a subglottic narrowing of the trachea, although this is rarely helpful as croup is a clinical diagnosis. Mild croup presents with cough and stridor only on exertion. moderate croup presents with cough and stridor at rest. Severe croup presents with stridor at rest with hypoxemia or respiratory failure. This patient is presenting with signs of mild croup.

what nerve is compressed in meralgia paresthetica?

Lateral femoral cutaneous nerve. a sensory nerve that can becomes compressed as it courses under the inguinal liagment and into the thigh. meralgia paresthetica consists of pain and/or paresthesia associated with compression of this nerve. it is cpmmpnly seen in patients with a large pannus from obesity. it can also be seen following surgery of the hip or pelvis, as well as with pelvis somatic dysfunction.

ATFL runs from ---to---?

Lateral malleolus to the neck of the talus.

subdural hematoma can present with headache and gradual dementia following a head trauma. it will display a large crescent-shaped hemorrhage area that crosses suture lines.

true

pt has difficulty performing HIS ACTIVITIES OF Daily living, gradual loss of dexterity, inability to brush his teeth, denies overt lower extremity weakness. physical exam reveals clonus to his biceps and triceps tendon reflexes-most likely affected region is?

Lateral precentral gyrus- upper motor neuron lesions result in the clinical findings of spasticity, increased deep tendon reflexes, and an extensor plantar response, lower motor neuron lesions result in atrophy, weakness, and fasciculations. An upper motor neuron involves the lateral corticospinal tract as it originates in the primary motor cortex of the precentra gyrus and passes through thr corona radiata and into the internal capsule before crossing over at the medullary decussation to synapse onto the naterior horn cells of the spinal cord. lateral corticospinal tract provides motor innervation to the skeletal muscles. upper motor neuron lesion can occur anywhere from the brain, brainstem, or spinal cord before they synapse onto the anterior horn cells of the spinal cord. upper motor neuron lesions also produce hyperreflexia, increased deep tendon reflexes, spasticity, slowness of movement, spastic gait, spontaneous leg flexor spasms, and incoordination. Also of note, THE POSTCENTRAL GYRUS IS THE PRIMARY SOMATOSENSORY CORTEX of the brain and procies sensory innervation to the body

15 yo male with right shoulder and back pain since his baseball game last week. for the first time he played as a pitcher. since then he felt soreness along his shoulder-blade and upper right portion of his back.pain is localized near the inferior angle of the scapula. before prescribing pain medication-you decide this patient may benefit from direct myofacial release. you place him in left lateral recumbent position and place your caudad hand under the patient's arm and place your cephalad hand over the patient's shoulder grasping the affecred tissues with your fingers. You then apply a lateral traction to the affected tissues. what muslce or muscle group are you treating in this scenarios?

Latissimus dorsi The latissimus dorsi muscle is the widest muscle in the human body. it aducts, extends and intenally rotates the arm. many spors activities such as tennis, gymnastics, and baseball can cause latissimus dorsi pain and patients commopnly feel pain inferior to the shoulder-blade. Myofascial release to this muscle can be done with the patient in lateral recumbent position with the affected side up. The physician reaches their caudad hand under the patients arm, and places their cephalad hand over the patient's shoulder. For treatment of the lower parascapular muscles, the patient's arm is draped over the physician's cephalad arm. traction is applied to the affected tissues by leaning away from the patient with the physician's elbow locked in extension.

Posterior inferior cerebellar artery (PICA)

Leads to lateral medullary (also known as wallenberg) syndrome. sensory deficits include a loss of sensation to the ipsilateral face from trigeminal involvement and contralateral loss of pain and temperature sensation to the body due to the involvement of the ascending spinothalamic tract. Motor deficits include ipsilateral cerebellar ataxia, bulbar weakness resulting in dysarthria and dysphagia., horner syndrome (ptosis, miosis and anhidrosis)

where is wernicke's area located?

Left posterior superior temporal lobe. wernicke's area is responsible for the comprehension of speech. strokes causing injury to this area prevent patients from producing menaingful speech because they cannot comprehend questions, albeit the words they use are properly formed.

Osteopathic structural exam reveals the sphenoid and occipital bones are inferior on the left and superior on the right. the left temporal bone and parietal bones appear to be in external rotation. the most likely diagnosis is ?

Left sidebending rotation. Sidebending rotation strains occur around three separate axes. the sphenoid and occiput rotate in the same direction around the anterior posterior axis and in opposite directions around two vertical axes. A sidebending rotation dysfunction is named for the side of convexity.

would you classify legionella pneumophilia as lacking a cell wall or as a facultative intracellular?

Legionella pneumophila is a facultative intracellular bacteria that infects alveolar macrophages. bacteria lacking a cell wall includes mycoplasma

65 YO MAN PRESENTS to the local emergency department with an acute onset of fever and chills that woke him from sleep. The patient now complains of shortness of breath and a productive cough. He has no past medical history, does not drink alcohol, and does not smoke cigarettes. oN PHYSICAL examination, the patient is febrile with a temperature of 39 degree celsius; a blood pressure of 155/94 mmHG, a heart rate of 72/min, and respirations at 15/min. a chest radiograph is obtained, as shown in the exhibit. The person is admitted to the hospital. what is the most appropriate initial monotherapY

Levofloxacin community acquired pneumonia is best treated with monotherapy, either with doxycycline or respiratory quinolones. community acquired pneumonia presents with fevers, chills, and a productive cough. the chest radiograph confirms the diagnosis by demonstrating a right upper lobe opacity. community acquired pneumonia (CAP) is one of the most common infectious diseases addressed by clinicians. CAP is an important cause of mortality and morbidity worldwide. A number of pathogens can give rise to cap. TYPICAL PATHOGENS THAT CAUSE the condition include streptococcus pneumoniae, haemophilus influenzae, and moraxella catarrhalis. These three pathogens account for approximatley 85% of CAP cases. Importantly, Staphlococcus aureus, klebsiella pneumoniae and pseudomonas aeruginosa are not typical causes of CAP in otherwise healthy hosts. S. aureus may cause CAP in individuals with influenza (human seasonal influenza and H1N1 {swine} influenza). K penumoniae primarily causes pneumonia in individuals with chronic alcoholism. P.aeruginosa is a cause of pneumonia in patients with bronchiectasis or cystic fibrosis. There is no optimal therapy for community acquired pneumonia (CAP); however, most experts feel that coverage should be divided against typical and atypical CAP pathogens. CAP may be treated with monotherapy or combination therapy. In the inpatient setting, effective monotherapy antibiotics include doxycycline or respiratory quinolones (levofloxacin). .Combination therapy usually consists of ceftriaxone plus doxycycline or azithromycin. Monotherapy coverage of typical and atypical pathogens in CAP is preferred over double-drug therapy. Monotherapy is less expensive than double-drug regimens are, while being just as effective. Clinicians should avoid empiric macrolide monotherapy, because approximately 25% of S.pneumoniae strains are naturally resistant to all macrolides. even though ceftriaxone does treat s. PNEUMONIAE YOU WILL not use it in a case of community acquired pneumoniae.

someone with liver disease-preferred treatment of choice who needs to undergo surgery but has a cirrhosis from hep c what is the best choice for sedation?

Lorazepam and oxazepam are the prefered benzodiazepined in patients with liver disease. they go directly to phase II metabolism, by-passing the phase 1 metabolism of the p450 ezyme system, and only create inactive metabolite ---In patients with liver disease, the ability to metabolize diazepam can be prolonged because of an ineffective p450 system oxazepam is a metabolite of diazepam.

A young female with bilateral conductive hearing loss and a positive family history most likely has otosclerosis. Loss of what reflex is a common finding?

Loss of stapedial reflex is a common finding. A young female with bilateral conductive hearing loss and a positive family history most likely has otosclerosis. In this disease, there is overgrowth of the middle ear bones, most commonly the stapes, which interferes with sound conduction. When the stapes is involved, it becomes fixed to the oval window and can no longer be pulled away by the stapedius muscle in response to loud sounds. This results in loss of the stapedial reflex. The Rinne test is performed with a tuning fork. it is first held 2.5 cm away from the ear (air conduction) and then is placed behind the ear on the mastoid process (bone conduction). In normal individuals, the patient hears the air conduction mote than the bone conduction, but this is reversed in conductive hearing loss, as seen in this patient.

59 yo female, gradual weakness, dizziness and hyperpigmented skin. lab studies show decreased serum levels of potassium and ACTH. which of the following likely explains this lab values

Lymphocytic destruction of the adrenal glands. primary adrenal insufficiency, also known as addison's disease is most commonly casued by tuberculosis worldwide. however, the most common casue int he western world is idiopathic and thought to result from autoimmune,lymphocytic destruction of the adrenal gland. patients commonly present with weight loss, weakness, hypotension, and skin hyperpigmentation. treatment involves adminstering exogenous adrenal steroids to maintain glucocorticoid and mineralcorticoid function. Addison's disease is treated with a daily oral glucocorticoid and fludrocortisone (mineralcorticoid) to replenish adrenal steroid levels.

3 yo male presents to your clinic with recurrent abscesses. had increased episodes of epistaxis. physical exam reveals a fair skinned, blue-eyed child with metallic blonde hair. his irises also have reduced pigmentation bilaterally. which of the following is most likely involved in the patient's disease?

Lysosomal emptying of phagocytic cells. this pt has Chediak-higashi syndrome Chediak-Higashi syndrome is an autosomal recessive disorder characterized by recurrent pyogenic infections, oculocutaneous albinism, progressive neurologic abnormalities, and coagulation defects. it is due to an abnormal lysosomal trafficking protein that results in defective phagolysosome formation. As a result, there is impaired lysosome degranulation, and phagocytized bacteria are not destroyed by the lysosomes. Patients present in infancy or early childhood with partial albinism and recurrent pyogenic infections. Most pts have fair skin with light blond or white hair that often has a metallic sheen to it. pts typically have light blue eyes, with a decreased pigmentation to the iris. pts suffer from recurrent pyogenic infections such as absecesses, ulcerations, and pyoderma gangrenosum. The diagnosis can be made by examination of a peripheral smear for the pathognomic giant cytoplasmic granules in leukocytes. Chediak-higashi syndrome presents in childhood with recurrent pyogenic infections and oculocutaneous albinism. The underlying defect is an abnormal fusion of phagosomes to lysosomes.

If a Patient's eye is dilated what kind of drug is the patient most likely administered? what if the patient is complaining of poor near vision?

M3 blockers or alpha 1 agonist. If the patient is also complaining of poor near vision, the only way this can happen is if the ciliary muscle has been affected. -Remember, the ciliary muscle is only innervatesd by muscarinic receptors. This indicates that the drug administered has an effect on muscarinic receptors otherwise the patients vision would be normal.

what hypoglycemic symptoms can beta blockers block?

MASK SYMPTOMS AS TREMOR, PALPITATIONS, PARESTHESIAS, AND BEHAVIORAL CHANGES WHEN PLAMA SUGAR LEVELS ARE LOW. BETA BLOCKERS ARE COMMONLY USED FOR RATE CONTROL OF SUPRAVENTRICULAR TACHYARRHYTHMIAS.

Androgens produced by the leydig cells in response to LH stimulation are responsible for the development of?

MESONEPHRIC DUCTS. which give rise to seminal vesicles, epididymis, ejaculatory duct, ductus deferens, and part of the prostate. mesonephric duct remnant can be remembered with the mneomic Gartner;s SEED Seminal vesicles, epididymis, ejactulatory duct and ductus deferens.

what is the primary rotation of the AA

MOTION at the AA is primarily rotations. The AA joint does not follow fryette's principle, for they only apply to typical thoracic and luymbar vertebrae.

What class of drug does Phenelzine belong to? What class of drug does Nortriptyline belong to? Which drug Phenelzine and Nortriptyline is likely to lead to severe htn if also taking and patient received wine and cheese?

Monoamine oxidase. Nortriptyline belongs to tricyclic antidepressant. Phenelzine-MAOIs, such as phenelzine and tranylcypromine, are associated with tyramine interactions, which can lead to severe htn and other symptoms of catecholamine overload. patients taking MAOIs are advised to avoid cheeses, wine, and aged meats. Tyramine induces the release of catecholamine and is particularly troublesome because it is also metabolized by monamine oxidase.

what reflex is this: abduction then adduction of the upper extremities when an infant's head is allowed to extend. it is present from birth to three to six months of age?

Moro-reflex

What is the drug of choice to both diagnose and treat supraventricular tachycardia when vagal maneuvers fail??

Make sure to review ACLS protocol for exam day as it is testable material. how to tell this was a supraventricular tachycardia. pt is experiencing shortness of breath and has an abnormal electrocardiogram. the rate is tachycardic in the 200bpm range and regular. In viweing the EKG, the axis is normal with upright QRS complexes in I, II, AVF. the R wave transition is correctly occuring between leads V2 and V3. The P wave is not immediatley distinguishable since it is hiding within the T wave. The QRS complex is narrow and normal in appearance. there are no significant Q waves, T wave flipping, or ST deviations. The tracing and patient presentation means the criteria for supraventricular tachycardia (SVT). Adenosine is used to chemically convert the electrical conduction of the heart. it causes transient AV nodal block. It is used to intervene in supraventricular tachycardia when vagal maneuvers fail. Drug of choice is Adenosine. Adenosine is used to chemically convert the electrical conduction of the heart. It causes transient AV nodal block. It is used to intervene in supraventricular tachycardia when vagal maneuvers fail.

Placenta previa

Make sure you avoid a digital exam here placenta previa ia an abnormal implantation of the placenta near the cervical os. how it may present---29 weeks gestation-vagianal bleeding, started and stopped without need for intervention. prior c-section, painless bright res bleeding that ceases within 1-2 hours with or without uterine contractions. there is usually no fetal distress with placenta previa, unless it is not diagnosed until labor progresses.

32 yo female just arrived in the post-operative care unit, recovering from a surgical repair of a torn anterior cruciate ligament. vital signs are blood pressure 150/90; heart rate=100/min; temperature is 38.9 degree celsius, respirations are 20/min. blood tests reveal an elevated creatinine phoshokinase and a low blood pH. the most likley diagnosis is?

Malignant hyperthermia. Malignant hyperthermia can be precipitated postoperatively in susceptible individuals due to the administration of a volatile gas anesthetic or the depolarizing muscle relaxant succinylcholine. Clinical presentation includes tachycardia, tachypnea, hyperthermia, acidosis, muscle rigidity, and can potentiate rhabdomyolysis. the treatment of choice is dantrolene.

since pinworms like enterobius vermicularis is transmitted via fomites-how would you treat family members?

Mebendazole or Pyrantel Pamoate

what is the mechanism of injury for a posterior and anterior radial head?

Mechanism of injury for a posterior radial head is fall forward on an outstretched hand. Mechanism of injury of anterior radial head is fall backward on an outstretched hand.

structural examination reveals L1 to be neutral, rotated right, sidebent left and an anterior L1 tenderpoint. which of the following best describes the location of the tenderpoint?

Medial to the anterior superior iliac spine. There is a cluster of anterior lumbar tenderpoints (l2, 3, 4) located around the anterior inferior iliac spine (AIIS), while there is only one anterior tenderpoint (L1) located medial to the ASIS.

what is Res ipsa loquitur? what is gross negligence?

Medial-legal term meaning, "the thing speaks for itself" and refers to situations in which persons suffer injury caused by negligent action of another party that would not have otherwise occured and that requires no further proof of negligence or injury suffered. The only requirement for res ipsa loquitur is showing that an end result happened; in this case, follow-up X-RAYS IDENTIFIED the scapel, requiring no further explanation to show negligence, as there is no excuse for this action. in general, negligence, as there is no excuse for this action. in general, negligence is based on the assumption that a physician is assumed to treat patients at a standard of care that would be expected to treat patients at a standard of care that would be expected by his/her colleagues. it essentially refers to a deaprture from the ordinary. for example, a physician who fails to order an ECG in a pt suffering from myocardial infarction could be sued for negligence if damages are suffered. Gross negligence implies a careless or reckless individual who seemingly intends to cause harm to another. in this case, leaving the scapel in the patient was an accident and therefore not gross negligence.

for inflare somatic dysfunction, the pt is positioned to the restrictive barrier and asked to isometrically push?

Medially. restrictive barrier will be putting pt in FABERE Muscle energy is a direct technique , therefore the pt is always positoned to their barrier. using post-isometric relaxation muscle energy, the pt is asked to contract toward their ease while the physician applies an equal counterforcr. after the patient relaxes the pt is passively toward their new restrictive barrier. given that this pt has an inflare, the pt's ease is adduction and internal rotation. with the patient's hip positioned at their barrier of external rotation and abduction, the patient is asked to push their knee medially.

15 yo female presents with six month history of calorie restriction. she has an intense fear of gaining weight weighs less than 85% of what is expected for her age. blood pressure is 80/50, and heart rate is 44/min. exaggerated kyphosis is seen on physical exam as well. lab results are notable for hypokalemia and hyponatremia. which treatment is most appropriate at this time?

Medical hospitalization. This pt has significant weight loss and is less than 85% her expected weight. she is also showing signs of malnutrition, such as hypotension and electrolyte disturbance, which can be dangerous medical complications from severe eating disorders. this patient should be hospitalized and closely monitored for re-feeding syndrome, which is a potentially life threatening condition that results from rapid shifts in fluids and electrolytes when nutrition is replaced too rapidly. once stabilized, therapy (family-based for teenagers) and nutritional rehabilitation are the primary recommendation treatments for patients with anorexia nervosa.

what is the most common testicular neoplasm in infants and what is usually elevated?

Most common testicular neoplasm is yolk sac tumor and AFP is elevated.

What does melena indicate?

Melena indicates a UGIB and should be investigated for localization and possibly treatment immediately with EGD. Upper gastrointestinal bleeding *UGIB) is defined as bleeding from a source proximal to the ligament of Treitz. Commonly seen symptoms include hematemesis, melena, syncope, presyncope, dyspepsia, epigastric pain, weight loss, and Jaundice. it occurs twice as often in males than in females. UGIB can present with hematochezia if the bleeding is brisk, but most often hematochezia is associated with a lower gastrointestinal bleed. Other history commonly seen with UGIB is the use of non-steroidal anti-inflammatory drugs (NSAIDS) or aspirin, history of peptic ulcers, heavy alcohol use and hepatitis. Esophagogastroduodenpscopy (EGD) is the first choice evaluation because the likelihood of a UGIB is high for a patient with melena. EGD can offer important information regarding the source and site of the bleeding. it is a diagnostic test but can also be therapeutic. EGD can help differentiate etiologies such as duodenal ulcer, gastric ulcer, gastritis, esophageal varices, mallory-weiss tear, esophagitis, ans neoplasm among others.

1) A proliferation of mesangial and endothelial cells and expansion of the mesangial matrix 2)thickening of the peripheral capillary walls by subendothelial immune deposits and/or intramembranous dense deposits 3)Mesangial interposition into the capillary wall that gives rise to a double contour (tram-track) appearance on light microscopy what disorder is this?

Membranoproliferative glomerulonephritis an example of a case-14 yo girl complains of fatigue, recent weight gain, swelling of her ankles and feet, and dark urine and foamy urine. urinalysis shows hematuria and proteinuria. light microscopy shows proliferation of mesangial and endothelial cells of the glomeruli and expansion of the mesangial matrix along with immune deposits.

port wine cutaneous stain, near or following the distribution of the trigeminal nerve, is the hallmark feature of sturge-weber syndrome, a neurological disorder present at birth. what other symptoms would you see?

Mental retardation, seizure disorder, neurological findings, such as hemiplagia. mental retardation is more prominent than choroidal hemagioma

Electron microscopy of a kidney biospy is ordered and shows extension of a mesangial cell splitting the basement membrane. The most likely additional finding is?

Mesangial proliferation on light microscopy. Electron microscopy showing glomerular basement membrane splitting due to mesangial cells is also referred to as "tram track" appearance and light miscroscpy showing mesangial proliferation can be seen in type 1 membranoproliferative glomerulonephritis. think of proliferative as the mesangial cell proliferating.

Microglia can be visualized under the microscope with special silver stain. They have a central irregularly shaped nucleus in their cell body with several branching spindles extending to the periphery. mICROGLIAS ARE the only cells of the nervous system derived from what embryological layer?

Mesoderm microglia-these cells have spindle processes emanating from the central cell body. on routine H & E stain, all one would be able to see is a small irregularly shaped nucleus and relatively little cytoplasm because these cells do not stain readily with most nissl stains. other glia cells are derived from neuroectoderm. microglia are mesodermal similar to other immunological cells. they arise from monocytes and invad the developing nervous system in week three along with the developing blood vessels. Main function of the microglia is phagocytosis of cellular debris. microglias are a type of glial cell that are resident macrophages of the brain and spinal cord, and thus act as the first and main form of active immune defense in the central nervous system (CNS). Microglias are constantly scavenging the CNS for plaques, damaged neurons, and infectious agents. they act via phagocytosis and as antigen-present cells to activate T-cell response.

although metformin decreases gastrointestinal absorption of glucose and decreases hepatic gluconeogenesis which one is the single best answer as to how it works-questions can present you with things that can have two possible answers/

Metformin is commonly used as a first line treatment for type II diabetes mellitus. it decreases hepatic gluconeogenesis, sensitizes tissues to insulin, and does not usually cause hypoglycemia. metformin primarily works by decreasing fasting glucose levels. it also works through insulin sensitization and decreasing intestinal glucose absorption but these are not the primary mechanisms of action of this drug.

which of the following is a suture of the frontal bone that allows for its motion as a paired bone?

Metopic suture The metopic suture allows the frontal bone to behave as a paired cranial bone; therfore, it moves into external rotation during craniosacral flexion and internal rotation during craniosacral extension.

soreness of her left buttock and a shooting pain down the back of her left leg. you perform Screening exam on sacrum and note a positive seated flexion test on the left. right sacral base is posterior and the left ILA is posterior and inferior. you palpate the lateral of the sacrum and notice a sausage like mass on the left. in order to treat the most likely diagnosis with strain/counterstrain you would most likley find a tender point in which location?

Midway between ILA and the greater Trochanter. This woman is experiencing piriformis syndrome as a result of piriformis muscle spasm causing irritation to the sciatic nerve. When the muscle is in spasm, the sacrum will rotate toward the ipsilateral side on a contralateral oblique axis, resulting in a sacral torsion. A common palpatory finding with piriformis syndrome is a "sausage like" mass lateral to the sacrum. the piriformis muscle tenderpoint is approximatley 7cm medial to and slightly cephalad to the greater trochanter and the ILA of the sacrum. it is normally treated with the patient int he prone position with the hip and knee flexed. the thigh is then abducted and externally rotated.

psychostimulant that is FDA approved for the treatment of narcolepsy, shift work sleep disorder and excessive daytime sleepiness associated with obstructibe sleep apnea. increases amount of monoamines, specifically norepinephrine, serotonin, and histamine in the brain.

Modafinil

physical examination reveals tenderness and warmth in paraspinal tissues of L2-L5. The most likely additional finding is? would you classify moist skin as chronic or acute?

Moist skin Acute somatic dysfunctions may present with swelling, heat, moisture, fullness, bogginess, hypertonic muscles, edema, tension, erythema, an acute tenderness. --think of sudden of sudden increase of sympathetic tone on blood vessels and musculature. note prolonged blanching is found in chronic somatic dysfunction due to localized vessel constriction secondary to localized hypersympathetic tone.

what class of drug is phenelzine in?

Monoamine oxidase inhibitor other MAO inhibitors include-Tranylcypromine, phenelzine, isocarboxazid and selegiline.

A 1 day old male born to a primiparous mother via vaginal forceps deliverly remains in the newborn nursery after a complicated delivery. the child's birth weight WAS 9 Ibs. 7 oz and labor lasted approximately 13 hours. Patient continues to feed well but the nursery staff tells you the child has been fussy anytime they try to turn his head and neck to the left. on your examination, you notice that the left upper extremity remains by the child's side, is rotated medially, the forearm is fully extended and pronated. patient's sensory examination is grossly intact in the right upper extremity, but the patient does not wothdraw or cry with pinprick sensation upon the lateral aspect of the left forearm. what nerve is damage?

Musculocutaneous nerve Erb Duchenne's palsy secondary to a traumatic delivery presents commonly seen as "waiter's tip," position (arm held in medial rotation, extension and pronation. . Birth trauma is a common cause of injury to the upper nerve root (s) of the brachial plexus. this patient's C5-C6 trunk has most likley been stretched leading to injury to the musculocutaneous nerve, which provides innervation to the biceps muscle, along with the brachialis and coracobrachialis muscles.

85 year old male presents to your clinic for a rash. Physical examination reveals numerous, painless, skin ulcerations on the patient's hands as well as a reddish patch of kin on his elbows with diminished sensation. He has recentrly immigrated to the united states from haiti where he was a fisherman. laboratory findings reveal acid fast bacilli. which of the following does he have A-Mycobacterium marinum B-Mycobacterium leprae

Mycobacterium Leprae Leprosy is a chronic disease characterized by painless skin lesion and neuropathies leading to joint deformities, is caused by an acid fast, rod-shaped bacillus. infection of leprae is definitively established when at least one of the characteristic physical findings, including hypopigmented or reddish patch on skin with sensory findings, including hypopigmented or reddish patch on skin with sensory deficits, is present and a skin biopsy obtained from the leadingedge of the skin lesion conforms the presence of acid fast bacilli in a cutaneous nerve. Most cases of leprosy found in the united states are among immigrants or people with extensive travel history. Mycobacterium marinum is an atypical mycobacterium species found in cold or warm, fresh or salted water. M. marinum infection occurs following skin and soft tissue injuries that are exposed to an aquatic environment or marine animals. The infection presents as a localized granuloma (usually called fish tank granuloma) that begins as a painful papule or nodule at the site of trauma, slowly enlarges, and then suppurates or ulcerates. Although M. Marinum causes ulcerated skin lesions, this patient had a decreased sensation of the skin lesions suggestion another etiology.

patient presenting symptoms consistent with damage to the nuclei of the vagus and glossopharyngeal cranial nerves, which are located in the lateral medulla deviation of the uvula suggests damage to the lateral medulla. the medulla develops from what vesicle of the brain?

Myelencephalon gives rise to the medulla and lower part of fourth ventricle A way to remember this is that there are five vesicles and they all occur in the fifth week. telencephalon is at the top and the rest are alphabetical. telencephalon gives rise to the cerebral hemispheres and basal ganglia. ---gives gives to the lateral ventricles Diencephalon gives rise to all structures that contain thalamus namely the thalamus, hypothalamus, epithalamus and subthalamus. ---gives rise to third ventricle mesencephalon gives rise to the midbrain.--Aqueduct Metencephalon gives rise to the cerebellum and pons- (upper part of fourth ventricle) Myelencephalon gives rise to the medulla and lower part of fourth ventricle A way to remember this is that there are five vesicles and they all occur in the fifth week. telencephalon is at the top and the rest are alphabetical.

pt is in his 40s and has fatigue, dizziness, palpitations and fevers without a source of infection-complete bld count shows normochromic, normocytic anemia with leukopenia, neurtropenia and thrombocytopenia. bone marrow biopsy would most likely reveal?

Myeloblasts with pink rod-like granules=auer rods seen in pts who has promyelocytic leukemia, the M3 subtype of acute myelogenous leukemia which will be seen on bone marrow biopsy.

how do you perform myofacial release at the plantar fascia?

Myofascial release plantar fascia technique is done by making a closed fist with one hand and applying pressure moving distal to proximal towards the calcaneus.

Structural exam reveals c3-c5 FLEXED, rotated left, and side bent left, T2-T7 warm and boggy on the left, and in the right, anterior fifth intercostal space he has a tender area that when pressed radiates down his right arm. the associated finding in the right fifth intercostal space is a?

Myofascial trigger point Myofascial trigger points are hypertonic bands of muscle tissue that refer pain in a predictable pattern when palpated. A myofacial trigger point is described in the right fifth intercostal space. A myofacial trigger point is defined as a taut band of muscle that has a local twitch response when palpated. A trigger point will radiate pain in a predictable pattern when palpated. This patient has a myofascial trigger point in the right pectoralis major muscle, which radiatd down the ipsilateral arm and can mimic the pain pattern of a myocardial infarction.

sideeffects of simvastatin

Myopathy, muscle weakness, and hepatoxicity with elevation of AST and ALT.

pts clothing is stained with vomit, earlier today she has complained of vague abdominal pain and nausea. since then she began complaining of feelings of shakiness, nervousness, anxiety, sweating, chills, and clamminess, confusion, rapid heartbeat, lightheadedness/dizziness, nausea, easy bleeding, lethargy and personality changes. her sister is concerned that she attempted an overdose on medication. the agent which is most appropriate to reverse the suspected overdose is?

N-acetylcysteine. N-acetylcysteine can replenish glutathione stores preventing severe liver damage. If a pt takes an overdose of acetaminophen, symptoms may occur within the first 24 hours. these symptoms may include NAUSEA OR VOMITING, SWEATING, LOSS OF APPETITE, AND DIARRHEA, AS SEEN IN THIS PT. IN THE NEXT 24-48 HOURS THE EARLY SYMPTOMS START TO DISAPPEAR BUT LIVER DAMAGE STARTS TO OCCUR. BECAUSE OF THESE potentially serious effects, early treatment after an overdose is important. Damaage to the liver, or hepatoxicity, results not from acetaminophen itself, but from one of its metabolites, N-acetyl-p-benzoquineimine (NAPQI) also known as N-acetyl iminoquinone). NAPQI depletes the liver's natural antioxidant glutathione and directly damages cells in the liver, leading to liver failure. risk factors for toxicity include excessive chronic alcohol intakes, fasting or anorexia nervosa, and use of certain drugs such as isoniazid. tREATMENT is aimed at removing the acetaminophen from the body and replacing glutathione. Activated charcoal can be used to decrease absorption of acetaminophen if the pt presents for treatment soon after the overdose; the antidote ACETYLCYSTEINE acts as a precursor for glutathione, helping the body regenerate enough to prevent damage to the liver. N-acetlycysteine can neutralize NAPQI by itself as well. A liver transplant is often required if damage to the liver becomes severe. patients treated early have a good prognosis, whereas patients that develop major liver abnormalities typically have a poor outcome. Acetaminophen is a reversible cyclooxygenase inhibitor used to relieve fever and pain. OVERDOSE OF ACETAMINOPHEN IS most likely to result in CENTRILOBULAR NECROSIS OF THE LIVER. (ACETAMINOPHEN OVERDOSE CAUSES THE GENERATION of free radicals which damage hepatic tissue causing hepatic necrosis and subsequent liver failure and death. treatment with N-acetylcysteine can assist in regeneration of glutathione). THE ACTION OF ACETAMINOPHEN INCLUDES THE CAPACITY TO REDUCE FEVER BY REDUCING PROSTAGLANDIN LEVELS ACETAMINOPHEN IS A PAIN RELIEVER AND FEVER REDUCER. IT DOES NOT HAVE ANTI-INFLAMMATORY OR ANTI-PLATELET EFFECTS. Reversibly inhibits cyclooxygenase, mostly in CNS. Inactivated peripherally. Antipyretic, analgesic, but not anti-inflammatory. Used instead of aspirin to avoid Reye syndrome in children with viral infection.

Superior mesenteric artery syndrome presentation may include

NAUSEA, VOMItING, Sharp postprandial abdominal pain, weight loss, and fear of eating. symptom is relieved when the body is in a position that elevate the root of the SMA (KNEES to chest, prone position or left lateral decubitus)

where do you insert the needle and the chest tube to relieve a pneumothorax

NEEDLE insertion is to the SECOND intercostal space in the midclavicular line Chest tube insertion is at the 4th/5th intercostal space at the midaxillary line the incision for chest tube placement is always one level below the site of chest tube placement such that one of the tube travels subcutaneously upward towards the site of insertion; this is to prevent air trapping resulting from the procedure.

What is the NNT?

NNT is (1/absolute risk reduction) and is a useful statistical calculation to determine the efficacy of a new medication. the lower the NNT, the better the medication is for society. A pharmaceutical company wants to begin marketing a new drug which treats strokes in the elderly with multiple co-morbid conditions. Their research shows that this has a relative risk reduction of 10% and the rate of death in the experimental group was 9%. what is the number needed to treat for this new medication? the number needed to treat is the number of patients that you need to treat in order to prevent one bad outcome. It is desirable to have an NNT of 1, meaning that every patient treated with the drug did not have the adverse outcome being measured. the NNT is equal to 1/absolute risk reduction. In order to choose the correct answer, you need to know how to calculate the absolute risk reduction which is the rate of death in the control group (10%) minus the rate of death in the experimental group (9%). therefore, the NNT becomes 1/(0.1-0.09)=100. this medication may not be considered very effective because it requires you to treat 100 patients in order to benefit one of them. an independent group reproduces a similar experiment to evaluate the same drug's role in the prevention of recurrent stroke. Similarly, the drug was only used in the elderly population with a history of stroke who had multiple co-morbid conditions. The rate of recurrent strokes in the control group was 8% and recurrent strokes in the experimental group was 10%. the equation 1/(0.10-0.08) best represents the? ANS=number needed to harm Number needed to harm (NNH) is similar to the number needed to treat (NNT), where NNT usually refers to a therapeutic intervention and NNH to a detrimental effect or risk factor. When the outcome is a harm rather than a benefit, a number needed to harm (NNH) can be calculated similarly. the number needed to harm is the measure of the number of people that need to be treated to have one bad outcome, in this case a recurrent stroke. The number needed to harm is found by the equation (1/attributable risk). The attributable risj is found by subtracting the percentage of people with recurrent stroke in the experimental group from the percentage of people with recurrent stroke in the control group, in this example (0.10-0.08). the NNH then is (1/0.02) or 50. number needed to TREAT (number of letters is bigger)=1/ARR number needed to HARM (number of letters is smaller)=1/AR

would you do HVLA on a person with tender muscle spasm?

NO

first line pharmaceutical agents for people suffering from dysmenorrhea?

NSAIDS. PRIMARY DYSMenorrhea is due to prostagalndin F2A (pgf2a), A POTENT myometrial stimulant and vasoconstrictor, in the secretory endometrium. evidence also shows that dysmenorrhea is related to prolonged uterine contractions and decreased blood flow to the myometrium. NSAIDS-inhibit COX-especially if given early enough in sufficient quantities.

4 yo boy complaining of a sore throat is brought to the Emergency department by his mother. physical exam reveals a temperature of 38.4 (101.1F), blood pressure of 96/58 mmhg, and multiple eroded vesicles surrounded by an erythematous halo scattered throughout his buccal mucosa, posterior pharynx, palate and lips. additionally, he has maculopapular lesions on both his hands and groin. which of the following best describes the capsid structure and genomic content of the etiologic agent?

Naked icosahedral capsid, single stranded RNA Coxsackie A virus type 16 is a non-enveloped, single-stranded rna enterovirus that is responsible for causing hand-foot-and mouth disease, which presents with vesicular lesions on the hands, feet, mouth and tongue. hands and fingers are affected more often than feet and most commonly, it is the dorsal surfaces that demonstrate the leions of the disorder. Infection is moderatley contagious and requires exclusion from group settings until it has resolved. the illness is self limited and usually subsides in 2-3 days without complications.

excessive daytime sleepiness, when excited or nervous feels as though cannot move legs and might even fall down-appropriate pharmacological treatment?

Narcolepsy can be treated with stimulants, such as modafinil or amphetamines. Narcolepsy is due to disordered regulation of sleep wake cycles and is characerized by excessive daytime sleepiness, pt's may also suffer from sleep associated hallucinations, sleep paralysis (partial or total paralysis that occurs at the beginning or end of sleep), and/or cataplexy.

what happens with REM sleep in people with narcolepsy?

Narcolepsy is due to a disorder of the sleep-wake cycle. symptoms of this disorder are the result of REM sleep components invading wakeful states. patients with narcolepsy commonly have a significant decrease in REM latency and enter REM sleep almost immediately after falling asleep,

According to DSM-5 males btwn the ages of 18 and 29 years has the highest rate of alcholism?

Native american and alaskan native.

flat effect, avolition and mutitism are these positive or negative symptoms?

Negative symptoms

triad of migratory polyarthritis, tenosynovitis, and dermatisis. what infection are you thinking about in this case?

Neisseria gonorrhea=gram-negative diplococcus. be careful here neisseria gonorrhea is gram negative

most common cause of bacterial meninigitis in a child greater than 10 years of age?

Neisseria meningitidis

LEVEL 1 routinely focuses on certain immunodeficiencies 10 yo girls present to the local emergency department with a one day hx of fever, myalgia, arthralgia, nausea, vomiting and a painful rash. Pt attends an after school daycare and lives at home with her parents and siblings. purpuric rash extends from her ankle up to her thighs and is now beginning to appear on her forearms and neck. pt blood pressure is 94/60 mmhg, pulse is 134/min, respirations are 24, and temperature is 39 degree celsius (102.3 degree fahrenheit). she is diaphoretic and in mild distress. what do you suspect this patient has? THe physician starts ceftriaxone with pending blood cultures. which if the following is most likely involved in causing this patients condition? Patients with a deficiency in C5-C9 complement proteins are at an increased risk for recurrent infections with what organisms?

Neisseria meningitidis which can lead to Sepsis and DIC. so this patient description indicates that she is likely suffering from sepsis. nEISSERIA meningitidis and gonorrhea is formed by the C5b-C9 complement proteins. Therefore, a deficiency in these molecules will lead to an increased risk for the development of meningococcal septicemia. prophylaxis with rifampin fot this condition should be given to all individuals who have come in contact with this patient, including health care workers and family.

what is the most definitive test for diagnosing carpal tunnel syndrome?

Nerve conduction studies are highly sensitive and specific. NCS would demonstrate slowed conduction velocity of the median nerve across the carpal tunnel.

Is pectoralis major utilized to treat somatic dysfunctions?

No

note when a person comes in with a sudden onset dyspnea, he flew from new york to california. physical exam reveals tachycardiac, tachypneic and his work of breathing is increasing. it is most likely that the pathology he is experiencing originated where?

Pathology seen in this patient commonly originates from the femoral vein. Majority of pulmonary emboli originate fom the femoral vein. -----This patient has a pulmonary embolism. Symptoms of this include sudden-onset dyspnea, tachycardia, and tachypnea. This may occur as the result of a long airplane or car ride, which contributes to venous stasis, a predisposing factor to pulmonary embolism. A large proportion of pulmonary embolisms originate from the femoral vein.

how do you differentiate Idiopathic thrombocytopenic purpura is a relatively common cause of thrombocytopenia. IgG antibodies form against the GPIIb/IIIa platelet membrane glycoprotein signaling platelte destruction by the spleen. This leads to increased platelet production in the bone marrow and increased megakaryocytes. --you should suspect ITP when you see multiple bruises, petechiae, and bleeding gums. giant platelets seen on blood smear and the WBC count and hemoglobin are normal. also on further eval-you will see increased megakaryocytes on bone marrow biopsy. Would you expect to see schistocytes on peripheral blood smear?

No, Schistocytes is an asymmetrical, fragmented part of a red blood cell. Multiple schistocytes are seen in patients with DIC/TTP/HUS and traumatic hemolysis. they are not present in patients with idiopathic thrombocytopenic purpura.

Does the cervical spine follow fryettes principle?

No, cervical spine does not follow fryette's principle. segments C2-C7 are similar to fryette type II mechanics meaning rotation and sidebending generally occur to the same direction, but dysfunctions can be in flexion, extension, or neutral (this is why it is not Fryettes mechanisc).

Do we expect to see an increase in REM LATENCY in depressed patients?

No, we expect to see a decrease in REM latency in depressed patients. and why is that? that is because what depressed patients want to do is sleep all day.

Subconjunctival hemorrhages are characterized by painless redness of the conjunctiva that is caused by rupturing of blood vessels. does this need to be treated?

No.

If you suspect a woman may be pregnant, should you still give her warfarin?

No. warfarin is absolutely contraindicated in pregnancy. it is listed under category x. Chadwick sign is often the first sign of pregnancy-bluish hue on vaginal mucosa

Is omm contraindicated n pregnancy?

No; only gentle treatment. only if there is pathologic bleeding

what is the first line prophylaxis for traveler's diarrhea?

Norfloxacin, ciprofloxacin, ofloxacin, levofloxacin, azithromycin and rifaximin. Although prophylaxis for traveler's diarrhea is not routinely recommended, antibiotics are prescribed in certain cases where the consequence of dehydration is greater than the risk of antibiotic use. A common example of where the benefots outweigh the risks would be inflammatory bowel disease (as in this pt), cardiac or renal disease, or an immunocompromised host. antibiotics should be taken if unformed stools occur more than four times per day or if the traveler develops feverm or blood, mucous, or pus in the stools.

what is a normal Q angle?

Normal Q angle is 14 degrees in male and 17 degrees in females

what is the diagnosis- positive standing flexion test on the left, left ASIS that is superior to the right and right PSIS that is superior to the left?

Note that your diagnosis is coming fromt h side of the dysfunction. so the diagnosis will be left posteriorly rotated innominate.

THE vagal nucleus located in the medulla of the brainstem are Nucleus solitarius Nucleus ambiguus dorsal motor nucleus what do each of these provide?

Nucleus solitarius-visceral sensory innervation Nucleus ambiguus-motor innervation dorsal motor nucleus-autonomic innervation

Commit to memory typical presentations of neuromuscular complaints that can be seen in the family practice setting or osteopathic manipulative medicine clinics. Common scenarios include: plantar fasciitis, low back pain, shoulder dislocation, clavicular fracture, lateral epicondylitis, medial epicondylitis, olecranon fracture, radial head dislocation, Achilles tendonitis, and ankle sprains. A young child who is suddenly pulled up too hard by the hand or wrist is a typical presentation for a diagnosis of subluxation of the radial head. what is the other name for this? how would the patient's hand present as? how do you reduce the patient's dysfunction?

Nursemaid's elbow radial head subluxation at the elbow joint is a partial or incomplete dislocation. in children, the annular ligament is not fully formed to hold the radial head in place. the radial head is prone to subluxation out of the annular ligament with an upward jerk on the hand. slight flexion at the elbow, adduction, and pronation. flexing the elbow to 160 degrees while also supinating the forearm. --aka suppination-flexion technique the child's arm is placed in extension/pronation before reduction, the physician places pressure over the radial head. then, the physician suddenly flexes the patient's elbow while supinating the forearm. A palpable or audible click is indicative of a successful reduction. The patient's initial position prior to the treatment would be extension/pronation. the classic treatment for reducing a nursemaid's elbow is applying pressure to the radial head while suddenly supinating the forearm and flexing the elbow.

a child has poor suckling what type of compression is likely responsible?

OA compression (or condylar compression) of cranial nerve XII can result in poor suckling of the newborn.

Orthotics(arch support), ice and NSAID, omt, stretching. are used in the primary treatment of plantar fasciitis. . the appropriate management of plantar fascitis is?

OMT and stretching.

The pelvic diameter that must be adequate to ensure a safe delivery is the?

Obstetric conjugate Obstetric conjugate represents the space available for the fetus as it descends through the pelvis during the delivery process. pearl-bilateral sacral flexion is an incredibly common diagnosis in pregnancy though multiple sacral dysfunctions can be seen, particularly with the additional laxity of ligaments that pregnancy will create.

name the shock--Low preload, low cardiac output, increased vascular resistance

Obstructive shock

what distinguishes obstructive shock from cardiogenic shock?

Obstructive shock secondary to a tension pneumothorax present with hypotension, tachycardia, and low cardiac output. although it can be classified as a subset of cardiogenic shock, cardiogenic shock typically is reserved for patients who have direct left ventricular failure. true or false?

People with otitis media what temporal bone are they likely to suffer?

Patients diagnosed with acute otitis media are likely to suffer from dysfunction of the temporal bone on the affected side.

A 16 yo male presents with knee pain. osteopathic examination reveals smooth and stable patellar tracking with quadriceps contraction and palpation does not reveal crepitus or effusion. A plain film radiograph of the knee is obtained and reveals an irregular ossification of the proximal tibial tuberosity and calcification within the patellar tendon. the most likely diagnosis is?

Osgood-schlatter disease common cause of knee pain in active teenager. pain is usually gradual in onset. and often abates with rest. it is generally self-limited, as long as the pt limits activities. treatment consists of ice, NSAIDS and limitation of activities especially jumping or squatting. -long-term, pts may have ossicles on the tibial tuberosity that may be painful with prolonged kneeling. OSGOOD-SCHLATTER affects the tibial tuberosity and shows calcification on plain film radiographs. osteopathic examination was not suggestive of patellofemoral dysfunction, in which the vastus medialis is weakened and the vastus lateralis pulls the patella laterally with quadriceps contraction.

Rheumatoid factor positive, osteophytes, bone spurs that may cause nerve impingement. pathophysiology?

Osteoarthritis inflammatory changes resulting in the biochemical breakdown of articular cartilage, synovium, and subchondral bone in joints.

Why does osteogenesis imperfecta cause hearing loss?

Osteogenesis imperfecta presents with easily fractured bones, easy brusing and blue sclerae. child abuse is high on the differential diagnosis.

Side effects associated with the long term use of phenytoin include hirsutism, gingival hyperplasia, peripheral neuropathies and?

Osteoporosis

1. pain in the mid-foot 2. Inability to bear weight right after the injury and in the emergency room (4 steps) 3. Bone pain at the posterior edge or tip of either medial malleoli.

Ottawa rules

overflow urinary incontinence is primarily caused by some form of neurological insult. this leads to deceased detrusor stimulation and should initially be treated with medication that can increase smooth muscle contraction. describe overflow incontinence

Overflow incontinence occurs from incomplete bladder emptying. this leads to elevated post-void residual urine volume. this occurs secondary to impaired detrusor contractility or outlet obstruction. the most common cause of impaired detrusor contractility or outlet obstruction. the most common cause of impaired detrusor contractility is neurogenic dysfunction from a lower motor neuron (LMN) lesion. causes include peripheral neuropathy such as neuropathy from diabetes, tumor compression or peripheral nerve fibers, nerve root compressions from disc herniations or spinal stenosis, lumbosacral plexopathies, and cauda equina syndrome. outlet obstruction includes benign prostatic hypertrophy, urethral strictures, or even from surgery from a baldder sling used to treat stress incontinence. Bethanechol is a cholinergic agonist that can increase detrusor muscle tone. it is a parasympathomimetic that activates muscarinic receptors within the bladder wall to increase bladder muscle tone, which in turn causes contraction and stimulates micturition.

right middle lobe pneumonia

PA and lateral views will reveal increased opacity obscuring the right heart border (which is in the same plane as the middle lobe) on the PA view and anteriorly over the heart on the lateral view. right middle lobe is the part of the lung adjacent to the right heart border. A silhouette sign involving the right heart border indicates a process in the right middle lobe. A silhouette sign is the obliteration or obscuremnt of an anatomic soft-tissue border by either a normal anatomic structure or a pathologic state, such as airlessness of adjacent lung or accumulation of fluid in the contiguous pleural space (like pneumonia). look for rib dysfunction overlying the consolidation, T4-6 correlating with middle lobe in this patient. Right middle lobe opacities wil obscure the cardiac shadow or silhouette. right lower lobe opacities wil not obscure the diaphragm silhouette.

where is the PC3 point located?

PC3 is a point located on the inferior lateral aspect of the C2 spinous process.

The length of the patellar tendon and the Q angle affect the function of the patellofemoral joint. Patellofemoral syndrome occurs more commonly in what kind of people?

PHYSICALLY active young adult women due to the tendency for women to have weak vastus medialis oblique muscles. The patella will be more laterally displaced on the femoral trochlea, and this patella maltracking will cause the patella to be unable to smoothly track within the femoral trochlear groove this will cause the patella and femoral condyles to grind together with flexion activities and produce articular cartilage damage, which leads to knee pain. The common presentation is Theater sign=Anterior knee pain in association with climbing stairs or sitting for a prolonged period of time.

type of hypersensitivity of post-streptococcla glomerulonephritis versus rheumatic fever?

POST STREP IS TYPE III hypersensitivity rxn and can follow wither pharyngitis or skin infection. Rheumatic fever can only follow pharyngitis and it is a type II hypersensitivity rxn. In post streptococcal glomerulonephritis, antibodies are made that target the M protein on S. pyogenes. high concentration of M protein and antibody form immune complexes that affect the kidney, resulting in a type III hypersensitivity rxn-this deposits disrupts the membrane and results in nephritis.

13 YO FEMALE, INTERMITTENT right knee pain, causes her to limp off and on. does poorly in school, husband has similar birth marks, physical exam has scoliosis, 25th percentile for height, she has skin lesions on her torso. what is a likely additional finding?

PROLIFERATION OF PERIPHERAL NERVES. chromosome 17 Neurofibromatosis type 1 is an autosomal disease characterized by cutaneous and neurological manifestations. neurofibromatosis, common in this disease are the proliferation of peripheral nerve components, which are schwann cells, fibroblasts, perineural cells, and mast cells. PROLIFERATION OF PERIPHERAL NERVES.

NON-LACTOSE FERMENTING GRAM-NEGATIVE ROD-SO HIGH PH

PROTEUS MIRABILIS

Osteopathic structural exam reveals a tenderpoint just medial to her posterior superior iliac spine at the same level as her sacral base. How do you treat posterior sacral tenderpoint?

PS1 is located over the ipsilateral sacral base, treatment, is to apply anterior pressure to the contralateral ILA. there are five posterior sacral tenderpoints, this patient has tenderness at PS1, which is located just medial to each sacral sulci at the level of the sacral base. In her case it is her left and treatment for PSI is to apply pressure anteriorly on the contralateral ILA of the sacrum in order to rotate the sacrum around an oblique axis. Extension of the sacrum (counternutation) would treat the PS2 and possibly PS3. PS2-PS4 are located midline on the sacrum at the corresponding spinal level.

Osteopathic structural exam reveals a counterstrain tenderpoint on the spine of the sacrum. Which of the following answers is correct regarding sacral counterstrain ps3?

PS3 is located midline on the sacrum on the spinous process of S3. there are 5 tenderpoints located on the sacrum itself. PSI and PS5 have two points associated with each, whereas PS2-PS4 are associated with corresponding segment of the sacrum. there are 5 tenderpoints located on the sacrum itself. the first and last tenderpoints are bilateral and therefore consists of two points. PS1 has two points located bilaterally on each sacral sulcus and PS5 has two points located midline on the sacrum at the corresponding spinous process of the sacrum.

what is PT/PTT a measure of?

PT/PTT are affected by defects in the coagulation pathways, while bleeding time is affected by platelet function and/or amount. DIC is a consumptive coagulpathy in which thrombi consume coagulation factors I, II, V, VIII, and trap platetlets. Since factors from both extrinsic and intrinsic pathways are involved, the PT,and PTT are both prolonged. sINCE Platelets are not available, bleeding time is also prolonged.

Timeframe for psychiatric illnesses brief psychotic disorder <1 month Symptoms of schizophreniform disorder lasts 1-6 months symptoms of schizophrenia lasts >6 months when does the disturbance of post-traumatic stress disorder have to be present? What about generalized anxiety disorder?

PTSD>1 MONTH GAD>6 MONTHS Know this timeframes

Wide QRS WHICH ARE not preceded by a P-wave and which are followed by a compensatory pause?q

PVC's (Premature ventricular contractions) This boy has premature ventricular contractions (PVCs) that are described as early, wide QRS complexes which are nit preceded by a P-wave. PVCs are common in adolescent males. they are usually asymptomatic but may present with palpitations. if symptomatic, they can be treated with beta-blockers, but usually PVCs are benign and require no treatment.

Crusting ulcers around the nipple is associated with what?

Paget disease of the breast presents as cruting ulcers around the nipple and is associated with an underlying invasive ductal carcinoma. those with a palpable mass are more likely to have DCIS as opposed to an invasive carcinoma. those with a palpable mass are more likely to have invasive cancer with lymph node metastasis.

which one carries the C vs A fiber? Paleospinothalamic tract Neospinothalamic tract

Paleospinothalamic tract carries A fibers Neospinothalamic tract carries A fibers

What are the steps of general myofascial release"

Palpation of restriction; application of compression (indirect) or traction (direct); addition of twisting/transverse forces; use of enhancements; awaiting release

due to peripheral location of the tumor, pulmonary symptoms such as cough, hemoptysis, and dyspnea are uncommon. lower brachial plexus is affected, babinski reflex is normal. What does this person have?

Pancoast tumor-superior sulcus tumor.

list affected enzymes involved in pancreatic insufficiency?

Pancreatic amylase, lipase, colipase and phospholipases. pancreatic insufficiency may occur with cystic fibrosis due to decreased bicarbonate secretion. pancreatic enzymes have a pH optimum of 7-8 and are active upon proteolysis in the small intestine. affected enzymes include: pancreatic amylase, lipase, colipase and phospholipases. Phospholipase A2 Is the enzyme that breaks down phospholipids to arachidonic acid in inflammatroy cells, but it also secreted by the pancreas to break down phospholipids in the GI lumen.

thyroid cancer strongly linked to a history of radiation exposure?

Papillary carcinoma. Papillary carcinoma is the most common form of thyroid cancer. remember that Papillary is Popular and has Psammoma bodies

Histology shows cells having empty looking nuclei and Psammoma bodies. the most likely diagnosis is?

Papillary thyroid carcinoma. histology shows papillary fronds, orphan Annie's eye nuclei, and Psammoma bodies.

what does paralysis of the primary motor cortex result in?

Paralysis of the primary motor cortex can result in dysarthria (damage to nerves innervating muscles of speech) as well as paralysis of the contralateral side of the body. damage to the primary motor cortex will involve stroke to the middle cerebral artery

55 yo male presents to the emergency after losing his footing on the rung of his back hyper-extended. the patient reports developing symptoms in his legs immediately after the fall, and a computerized tomography scan shows vertebral bony fragments compressing the anterior spinal artery. The most likely physical finding associated with this patient's condition is?

Paraplegia of the legs, loss of temperature and pain sensation bilaterally, with intact pinpoint and vibratory discrimination. Anterior spinal cord syndrome (ACS) is a medical emergency wherein the anterior spinal artery supplying the spinal cord is compromised leading to ischemia and infarction of the region it supplies, which is roughly 2/3rd of the cord. knowledge of this syndrome relies on the neuroanatomic knowledge of the cross-sectional spinal cord tract pathways, including 1-PAIN/temperature via spinothalamic tract 2-motor via corticospinal tracts 3-tactile/vibration via dorsal column-medial lemniscus (DCML) tracts. ACS leads to ischemia of the former two anteriorly located tracts with preservation of the DCML tract, as supplied by the posterior spinal artery.

extremely elevated hCG in the first trimester and an enlarged uterine size is associated with a complete mole which has the genetic composition of 46XX.How do you tell this apart from partial mole based on presentation?

Partial mole are less frequently associated with enlarged uterine size.

physical examination reveals rib four moves symmetrically into exhalation, but right rib four does not move into inhalation. which of the following muscles will be utilized using muscle energy in order to treat this dysfunction?

Pectoralis minor The pectoralis minor attaches to ribs 3 through 5 and is used to treat exhalation restrictions of those ribs. Rib stuck in exhalation=exhalation dysfunction Because it is stuck in exhalation, it will not move correctly during inhalation and is and is said to have an inhalation restriction. Rib dysfunctions are names by where they are stuck Rib restrictions are named by which motion they do not experience. make sure you do not get this mixed up. using muscle energy utilizes the muscles that this ribs are attached to.

where does the pelvis receive sympathetic innervation from?

Pelvis received sympathetic innervation from T9-T12

smooth flat erosions filled with exudate seen on endoscopy is characteritic of?

Peptic ulcer caused by H. pylori.

Where is perfusion and ventilation the highest in terms of apex and base of lung?

Perfusion and ventilation is highest at the apex compared to the lung base. but the V/Q ratio at the apex is higher. this is because perfusion drops more rapidly when moving from the base to the apex. In a person standingupright, the apex of the lung shows higher V/Q ratio, while at the base of the lung, the ratio is lower and nearer to 1.0. The main reason for lower V/Q ratios at the base is that, while both ventilation and perfusion increases when going from the apex to the base, Q does it more strongly thus lowering the V/Q ratio. The principle factor involved in all of this is gravity, and as a result of this V/Q ratios change as the position of the body changes. As the V/Q ratio is higher at the apex compared to the base of the lungs, a high ratio is consistent with a pulmonary embolism, and a low ratio is consistent with airway obstruction.

A 50 yo man presents to his primary care physician with multiple complaints. he reports frequent headaches, abdominal pain, diarrhea, muscle pain, and weight loss of 9kg (20 Ibs). over the past four months. he denies any upper respiratory involvement. physical exam is notable for palpable purpura on the skin and serum studies reveal an elevated erythrocyte sedimentary rate. a skin biopsy id performed, which showed the absence of granuloma formation. In this case, the most useful marker for monitoring his condition is?

Perinuclear antineutrophilic cytoplasmic antibodies is used for monitoring disease activity in microscopic polyangitis. This patient is presenting with symptoms of microscopic polyangitiis, an autoimmune necrotizing vasculitis of small-sized arteries. Males are affected more frequently than females with an average age of onset at approximately 50 years old. it frequently affects the vessels supplying vascular organs, including the liver, kidneys, and weight loss. a DERMATO-PULMONARY-RENAL SYNDROME IS THE FEATURE of the disease. presentations of microscopic polyangitis include the constitutional symptoms of fever, malaise, flu-like symptoms, myalgias, arthralgias and weight loss. Skin manifestations of palpable purpura along with mild renal insufficiency are common. pulmonary manifestations are lower respiratory tract symptoms of hemoptysis, dyspnea, or a cough. P-ANCA (perinuclear antineutrophilic cytoplasmic antibodies) is used to monitor disease activity directly measuring related antibodies. microscopic polyangitis differs from granulomatosis with polyangitis in that the upper respiratory tract is spared, and there is a pathological absence of granulomatous inflammation.

What kind of Q angle do you see in a person with flat foot?

Pes planus, or flat foot, leads to excessive pronation of the foot which results in lateral subluxation force on the patella. overtime this can lead to an increased Q angle as well

A 35 yo male presents to a community healthy health clinic as a new patient with complaints of pain in his knees, generalized muscle aches, and fatigue. history reveals that he was diagnosed with a chronic seizure disorder for which he is prescribed an anti-convulsant medication. the most likely pharmacologic agent he is taking is>

Phenytoin. Phenytoin is an anti-seizure medication known to cause drug-induced lupus which produce symptoms such as myalgias, arthralgias, lymphadenopathy, purpura, erythema nodosum, fatigue, and fever. laboratory testing is positive for anti-histone antibodies in approximately 95% of patients. it is important to know that drug-induced lupus generally occurs after months to years of treatment with the offending medication, rather than within the first few weeks of treatment. many drugs have been found to cause lupus-like symptoms; those commonly tested include hyralazine, isoniazid, procainamide and phenytoin.

Explain Pindolol and if it can be used for migraine headaches?

Pindolol is a nonselective beta blocker with partial b2-agonist activity. the only b-blocker approved for prophylactic treatment of migraine headaches are propranolol and timolol. pindolol and acebutolol has been shown to be ineffective in this class. Other drugs that have been approved for migraine prevention include amitriptyline, propranolol, timololm divalproex, sodium valproate, and topiramate

you find a tenderpoint a half-inch medial to the inferior aspecrt of the PSIS on the right. how would you treat this tenderpoint?

Place the pt in prone position and apply posterior to anterior pressure on the left inferior lateral angle of the sacrum for 90 seconds. when treating the posterior S1 tenderpoint, the physician applies posterior to anterior pressure diagnonally opposite the location of the tenderpoint. As stated int e question stem, the physician finds a tenderpoint on the right half-inch medial to the inferir aspect of the PSIS. This is the posterior S1 tenderpoint. treatment would be targeted at the left inferior lateral angle of the sacrum and this position would be held for 90 second.

15 yo boy has hx of recurrent bacterial infections. At 18 months, he had pneumonia caused by streptococcus pneumonia. when he was 3 years old, he had meningococcal meningitis followed by bacterial sinusitis caused by Haemophilus influenza type b. a defect in which of the following cells is most likely?

Plasma cells Plasma cells are derived from B-cells and secrete antibodies, which are the most important defense against bacteria with polysaccharide capsules. pts without CD4+ CELLS AREA EXPECTED TO HAVE INFECTIONS CAUSED BY VIRUSES AND FUNGI.

Dequervain's tenosynovitis

Positive Finkelstein's test-checks for abductor pollicis longus

hallucinations, delusions, and disorganized behavior. Are this positive or negative symptoms of schizophrenia?

Positive.

what kind of treatment do you want to do for a postoperative pt with ileus?

Post-operative care, such as the treatment of ileus, can include paraspinal inhibition at the thoracolumbar junction.

78 yo male, hx of atrial fibrillation and htn. uncontrollable flailing movements of his right arm and leg. he states he woke up up this morning with a terrible headache. temp is 37.1, hr is 90/min, and bld pressure is 185/96. where is the most likley location of the lesion responsible for his flailing movement?

Posterior cerebral artery Hemiballismus presents as unilateral uncontrolled flailing movments, usually caused by an infarct affecting the subthalamic nucleus. Posterior cerebral artery infarct leads to the damage of the subthalamic nucleus causing hemiballismus on the contralateral side. these flailing movements almost always resolve spontaneously. the PCA, supplies the occipital lobe, most of the thalamus, inferomedial portions of the temporal lobe, and the superior portions of the brainstem, and midbrain. An infarct of the PCA CAN RESult in two different syndromes: P1 syndrome, involving the central territory, and P2 syndrome involving the peripheral territory. the p1 syndrome results in midbrain, thalamic and subthalamic signs, including thalamic pain and involuntary movements, such as hemiballismus. P2 syndrome, results in cortical temporal and occipital lobe signs, the most common of which is contralateral homonymous hemianopsis with macular sparing. subthalamic nucleus is also supplied by the middle cerebral artery (MCA), and infarct of the MCA could also cause hemiballismus.

pHYSICAL examination reveals no ecchymosis or edema, there is restriction in right forearm supination and a point of intense tenderness just lateral to the right radial head. The most likely somatic dysfunction diagnosis is? Structural exam will most likely reveal? The most appropriate initial setup for muscle energy involves?

Posterior radial head Remember your diagnosis is based on the position of ease. Restricted anterior glide of the radial head Placing the patient's forearm in supination ---A restriction in supination is a common finding in posterior radial head dysfunctions. Additionally, this patient has fallen forward on a protonated forearm, which may lock the radial head in pronation. muscle energy is a direct treatment, meaning that the operator must first engage the restrictive barrier; therefore, supinating the forearm will move the radial head anteriorly against the physiologic barrier. bottom line to set up muscle energy for a posterior radial head dysfunction, one must engage the restrictive barrier.

Overall, the most common cause of secondary amenorrhea is? if a pt is ovulating what is likely?

Pregnancy progesterone will hit peak levels about seven days prior to menses. peak levels of progesterone is indicative of an active corpus luteum. progesterone peaks about day 21-23 of a 28 day cycle during the luteal (secretory phase). peak level of progesterone is an indicator of ovulation. The corpus luteum is a specialized structure that suppirts the released ovum. The corpus luteum secretes progesterone stimulating the endometrial tissue for implantation of the ovum, which is known as endometrial decidualization. Ovulation occurs approximately 36 hours after the onset of the luteinizing hormone (LH) surge and 24-36 hours after peak estradiol levels. peak estrogen levels occur during the follicular (proliferative ) phase. the rise in progesterone enhances proteolytic enzymatic activity, which breaks down the collagenous follicular wall of the developing follicular wall. After the ovum is released the granulosa cells increase in size and take on a yellowish pigmentation characteristic of the corpus luteum. the corpus luteum then takes over the majority of hormone secretions producing estrogen, progesterone, and androgens. progesterone can be detected by different methods, besides using serum progesterone as an indicator, a rise in basal body temperature of approximately 0.5 degree celsius in the initial stages of the luteal phase, and a measurement of the LH surge detected IN THE urine are commonly used. note -an increase of 0.5 degree celsius in temp occurs in the luteal phase. basal body temp is increased secondary to the release of progesterone by the corpus luteum. progesterone has potent effects on the hypothalamus, one of which increases the body temperature.

Person stabbed. Physical examination reveals a stab wound in the right lower back, decreased motor function in the right lower extremity, and a positive babinski sign on the right foot. He has intact strength in the left lower extremity and a negative babinski reflex in the left foot. the most likely additional finding would include?

Preserved crude touch below the level of the right side. SUMMARY IF YOU DON'T HAVE TIME TO READ IT ALL just read this one: Brown-sequard syndrome involves a hemi-spinal cord injury that results in ipsilateral hemiplegia, ipsilateral loss of vibration, fine touch, and proprioception, and contralateral loss of pain and temperature sensation. Brown-sequard syndrome involved loss of sensation and motor function that is caused by lateral hemisection of the spinal cord, usually due to trauma. This syndrome is characterized by hemiparaplegia, loss od vibration sense and fine touch, los of proprioception (position sense), loss of two-point discrimination, and weakness on the ipsilateral side of the spinal injury. these signs are evident due to injury of the dorsal column and medial leminiscus tract and corticospinal tract. the dorsal column lead into the medial leminiscus tract, which carries fine touch fibers and controls conscious proprioception, vibration, pressure and and two point discrimination. The corticospinal tracts carry motor fibers and are involved in innervating skeletal muscle. On the contralateral side of the lesion, there will be loss of pain and temperature sensation due to the decussation of the fibers involved in the spinothalamic tract at the level of the spinal cord. The corticospinal tract fibers decussate early on at the level of the medulla. therefore, ipsilateral signs of weakness will be evident. crude touch, pain, and temperature are carried within the spinothalamic tract. A hemisection of the spinal cord at that level will still demonstrate the ability to feel crude touch on the ipsilateral side of the lesion. The patient will be unable to localize exactly where they have been touched, only that they were touched. this is due to the injury of the fine touch fibers located in the dorsal column medial lemniscal pathway. therefore, a hemisection lesion of the spinal cord will demonstrate loss of fine touch sensation on the ipsilateral side but preserved crude touch on the ipsilateral side.

what is primary hypothyroidism characterized by?

Primary hypothyroidism is charactrized by low T3 and T4 and elevated TSH, TRH and an increased T3/T4. In hypothyroidism the enzyme deiodinase is upregulated thats why there is an increased in T3 compared to T4. IF PATIENTS HAVE PRIMARY ANTERIOR PITUITARY DISEASE THEN tsh MAY BE DECREASED.

G2P1 female delivers a neonate with jaundice. physical exam at 24 hours post-delivery reveals scleral icterus and a yellow pigmentation of the skin. lab studies reveal anemia and severely elevated indirect bilirubin. Reticulocytosis is noted on peripheral blood smear. bLOOD TYPE and screen reveal the mother's blood group to be A and the infants blood group to be B. The most likely additional finding, which contributed to this infant's jaundice is?

Prior Rh+ delivered infant. Hemolytic disease of the newborn is a condition in which the red cells of the newborn are destroyed by maternally derived antibodies. these antibodies arise in an Rh (D) negative mother who first pregnancy resulted in an Rh (D) positive fetus. this newborn is likely presenting with Rh hereditary disease of the newborn. note that the mother is G2P1, indicating that she has delivered a baby in the past. this occurs when an Rh(D) negative mother becomes initially sensitized to Rh (D) positive cells from fetal red cells that have gained entrance into the maternal circulation. This most commonly happens with prior fetal-maternal hemorrhage and initiates the formation of anti-Rh(D) IgG antibodies in the mother, which then cross the placenta and causing hemolysis during the neonatal period. This occurs during the second pregnancy in which the fetus of the first pregnancy was Rh (D) positive. The fetus develops anemia, congestive heart failure, and possibly hydrops fetalis. this is prevented by given the mother Rh immune globulin that provides passive immunization. almost half of affected infants do not require treatment, have mild anemia and hyperbilirubinemia at birth, and survive and develop normally. One quarter will become severly jaundiced with up to 90% dying or becoming significantly affected by kernicterus. the remaining quarter are affected in utero and develop hydrops fetalis, which is an accumulation of edema leading to ascites, pleural effusions, and pericardial effusions. Note ABO incompatibility occurs almost exclusively with mothers who have blood type O and neonates who have blood type A or B. this results in hemolysis, jaundice, and a positive direct Coombs test, positive direct antiglobulin test (DAT), and elevated reticulocyte counts. It is a typE II hypersensitivity reaction

WHAT ROLE DOES PROGESTERONE PLAY IN causing a miscarriag?

Progesterone is responsibel for enabling the endometrium to become receptive to the implantation of a blastocyst. it also promotes stromal vascularity to help increase the blood supply to the growing fetus. without enough progesterone on board in the transition from corpus luteum production to placental production of progesterone after 8-10 weeks, a miscarriage may result. This women are often supplemented with progesterone to aid the transition from corpus luteum to placental progesterone production to prevent a miscarriage.

atypical pneumonia caused by chlamydia psittacosis, an obligate intracellular gram-negative bacterium carried by domestic birds. it presents with fever, cough and headache, and may also demonstrate splenomegaly, facial rash, pharyngitis and pulse-temperature dissociation?

Psittacosis. working at a pet score is a soc hx possibly.

36 yo female complaints of painful urination, blood in urine and pain radiating from her right flank to her groin. hx reveals she has pain patterns of kidney stones, which have caused similar pain patterns in the past. which of the following muscles is being most affected as part of a viscerosomatic response?

Psoas major psoas syndrome can be defined as a muscular imbalance, strain, spasm, tendonitis or flexion contracture of the iliopasoas muscle. ureteral stones can cause a viscerosomatic reflex causing psoas syndrome. pt is presenting with symptoms consistent with a renal stone. specifically ureteral stone given the radiating pain from the flank to the groin. the ureters lie directly on the psoas muscles as they course from the kidney on theri way to penetrate the base of the bladder. inflammation of the ureter due to a ureteral stone can also irritate the psoas muscle due to its close anatomic relationship. ureteral stones has been implicated int he cause of psoas syndrome as have abdominal aortic aneurysm, intra-abdominal abscess, appendicitis, diverticulitis, inguinal hernia, prostate or sigmoid colon cancer, prostatitis, salpingitis, and crohn's disease. the kidney and ureters viscerosomatic reflex is found from T10-L1 (kidney near T10 and ureter near L1). the psoas is also innervated via the L1-L3 spinal nerves. therefore the anatomical relationship of the ureters to the psoas and the shared L1 spinal nerve make this the best answer. The iliacus is not the best answer because the ureter does not travel on top of it.

what is psychogenic polydipsia.

Psychogenic polydipsia is self-induced water intoxication. it can be differentiated from diabestes insipidus with a water deprivation test. This patient has polydipsia and polyuria with diluted urine that becomes more concentrated with water deprivation. This is most consistent with psychogenic polydipsia, a self-induced water intoxication found in up to 20% of patients with psychosis. the etiology is unknown but may be related to psychosis or certain medications (i.e. lithium and antipsychotics). this under-diagnosed condition can potentially cause seizures and/or delirium and should be suspected in patients with chronic hyponatremia.

treatment for pt's with conversion disorder?

Psychotherapy Patients suffering from conversion disorder are commonly unaware that their physical symptoms are due to some inner psychological conflict. psychotherapy is the treatment of choice for these patients because it guides them in making this connection. Once the patient is aware of this, the psychological currency of the symptom loses value, and the symptoms may be allowed to improve.

In what location would an anterior L5 tenderpoint be appreciated?

Pubic ramus, 1cm lateral to the pubic symphysis

differentiate pygmalian from hawthorne effect

Pygmalian effect refers to the fact that when we know the expected outcome, we tend to modify our behavior in ways that make such an outcome more likely. Hawthorne effect refers to the results of a study being altered by the fact that the subjects are aware that they are being studied. An example would be that people work harder and perform better when they know they are participating in a study.

what glycolysis enzyme is associated with neurologic defects and lactic acidosis?

Pyruvate dehydrogenase pyruvate dehydrogenase deficiency is characterized by failure to convert oyruvate to acetyl coenzyme A. The excess pyruvte becomes lactate that results in a lactic (metabolic) acidosis. patients generally present with poor feeding, developmental delay, seizures, abnirmal eye movement, ataxia, and mental delays. this condition is also known as leigh syndrome.

what muscle is utilized when treating rib 12? structural exam reveal T12 is flexed, rotated left, and sidebent left with an exhalation rib dysfunction on the left of rib 12. which of the following muscles should be targeted during muscle energy for the rib dysfunction?

Quadratus lumborum since this patient has an exhalation dysfunction, rib 12 id stuck in the anterior and superior position-the quadratus lumborum will pull this rib inferiorly. During inhalation, ribs 11-12 move posterior and inferior (visualize the diaphrgam pushing down and the ribs moving out of the way). During exhalation, ribs 11-12 move anterior and superior. The most appropriate muscle to activate for muscel energy in this case is the quadratus lumborum because it has attachments to the transverse process of lumbar vertebrae L1-l4, ILLIOLUMBAR LIGAMENT, ILIAC crest, transverse process of lumbar vertebrae L1-L4, and the inferiorborder of rib 12. Since this patient has an exhalation dysfunction, rib 12 is stuck in the anterior and superioe position-the quadratus lumborum will pull this rib inferiorly.

where does quadratus lumborum attach?

Quadratus lumborum is attached to rib 12, and the transverse processes of L1-L4

What will a recent myocardial infarction result in?

RECENT MYOCARDIAL INFARCTION CAN CAUSE A LEFT BUNDLE BRANCH BLOCK, WHICH PRESENTS WITH PARADOXICALLY SPLIT S2. LBBB and aortic stenosis can result in splitting of the S2 heart sound--this splitting can be seen in conditions that delay or slow the left ventricle from emptying. therefore, on inspiration the pulmonic component of the s2 moves closer to the aortic component of s2, functionally eliminating the split.

a RISE IN ACETYLCHOLINE AND A DECLINE IN NOREPINEPHRINE TRIGGERS THE ONSET OF?

REM sleep

74 yo male is transferred to the ICU following a motor vehicle accident in which he suffered a massive intracranial hemorrhage. the pt is ventilator-dependent and has no pupillary response to light. he is unresponsive to commands, and cold caloric testing fails to elicit nystagmus. An eeg is obtained and reveals no electrical activity. the pt has no advanced directive or power of attorney and he has lived alone in nursing home for several years. the most appropriate course of action is to?

REMOVE the patient from the ventilator clinical criteria for brain death include loss of brainstem reflexes, (i.e., loss of pupilary light reflex, lack of nystagmus with caloric testing, absent oculocephalic reflex, absent corneal reflex, and the failure to take spontaneous respirations). brain death is a clinical diagnosis, and although EEG is helpful in confirming brain death, it is not required for diagnosis (state dependent). The combination of cessation of spontaneous breathing with absent brainstem reflexes is more important than EEG findings. Physicians and family should realize that brain death equals the death of the patient. Clinical criteria for brain death include the absence of spontaneous ventilations coupled with loss of brain stem activity; meeting these criteria warrant removal of life-sustaining interventions such as mechanical ventilation if no documentation exists suggesting the pt (or family) wished otherwise. if family or caregivers are present, always discuss options with them before making drastic decisions (such as pulling the endotracheal tube). answer choices using words such as discuss and communicate are usually correct in these circumstances. also make sure there is no reason to suspect other etiologies that may mimic brain death, such as barbiturate toxicity or encephalitis, which may be reversible; in these patients, err on the side of caution.

between ribs 4-6 which one has more primary motion about a transverse axis?

RIB 4 riBS 1-5 use a pump handle motion to move around a transverse axis. ribs 1-2 have this as a primary motion and ribs 305 have it as a combination with bucket handle motion.

Rib fractures were rules out on chest radiograph. Physical examination reveals ribs 3-7 move symmetrically into exhalation but on the right side do not move with inhalation. How would muscle energy be performed to correct this somatic dysfunction?

RIb 3 would be treated by asking the patient to isometrically contract their pectoralis muscle while breathing. Remember BITE-Bottom Inhaled, Top Exhaled. Treatment of an exhalation somatic dysfunction is directed at the top rib in the group. treatment of an inhalation somatic dysfunction is directed toward the bottom rib in the group. When using muscle energy exhalation somatic dysfunction are treated using isometric contraction of muscle involved in respiration and inhalation somatic dysfunctions are treated by flexing the torso and the physician physically resisting inhalation and encouraging exhalation. This patient has an exhalation rib dysfunctio (inhalation restriction) in ribs 3-5. when treating a group rib dsfunction it is important to treat the key rib first. in the case of an exhalation dysfunction the top rib in the group is the key rib. The bottom rib is the key rib in group inhalation dysfunctions. the key rib must be treated first because it prevents the other ribs in the group from moving appropriately. In group exhalation dysfunctions the top rib prevents the lower ribs from moving superiorly with inhalation. to treat this using muscle energy you would engage the pectoralis muscle for ribs 3-5. this would be done by having the patient isometrically contract the pectoralis minor muscle against the physicians resistance while inhaling. the physician would simultaneously apply inferior traction to the posterior aspect of the rib.

what is a common cause of cough, fever, and rspiratory distress due to bronchiolitis in infants during the winter months?

RSV TYPICAL case-6 month old baby in winter month comes in. respiratory distress, cxr=hyperinflated lungs, child breathing 66/min and wheezing is appreciated. which diagnosis is likely=rsv. RSV usually causes cold like symptoms in adults and older ciblings while causing more severe symptoms in infants. this is due to the smaller bronchiole size in infants where inflammation can lead to the respiratory distress.

Damage to the radial nerve is the most likely complication. This nerve traverses through the spiral groove on the posterior side of the humerus and is commonly injured in distal spiral shaft (Holstein-Lewis ) fractures. what muscles does the radial nerve innervate?

Radial nerve innervates all extensor muscles as well as the lateral portion of the brachialis muscle. It is also most commonly injured by midshaft humerus fractures.

The apley scratch test is utilized to evaluate for shoulder range of motion. if this test does is abnormal what does it indicate?

Reduced rotator cuff function apley test does not check for flexion and extension to perform the apley test, a person is asked to reach behind their head and touch the superior medial tip of the opposite scapula. A person with normal function can reach approximately the level of the T4 spinous process. this part of the physcial examination maneuver is evaluating external rotation and abduction of the shoulder. to assess internal rotation and adduction of the shoulder, the person is asked to reach behind their back and touch the inferior tip of the opposite scapula. a PERSON WITH NORMAL function can reach approximately the level of the T8 spinous process. Both shoulders are always tested to compare the range of motion between them. A positive test is marked by a decreased active range of motion, and indicates that the muscles of the rotator cuff are either weakened or inflexible.

An infant diagnosed with their first UTI-what will be the test of choice to use to evaluate them?

Renal ultrasound- is the test of choice in infants with their first UTI to look for anatomic abnormalities that may suggest vesicoureteral reflux. it is preferred becasue it is noninvasive and does not require radiation. Vesicoureteral reflux is diagnosed with a voiding cystourethrogram. the clinical presentation of urinary tract infection (UTI) in infants can often be nonspecific such as fever, irritability, and poor oral intake. infants can also appear septic. uncircumcised males are at higher risk for developing urinary tract infections. other risk factors include female gender, constipation, and the presence of indwelling catheters. In nontoxic appearing infants, an oral course of antibiotics is sufficient with close follow-up. e.coli is still the msot common presenting organism, so antibiotic covergae can be tailored to its coverage. the american academy of pediatrics no longer recommends obtaining a voiding cystourethrogram (VCUG) in all neonates with their first uncomplicated febrile UTI if they are older than two months of age. instead, a renal ultrasound can be done to identify the anatomic abnormalities like hydronephrosis or bladder wall dysfunction.

25 yo female presents to her gynecologist for a normal checkup. she has a history of recurrent venous thrombosis, and has a fam hx of deep vein thrombosis and stroke. she had a miscarriage at 9 weeks' gestation at the age of 21. A characteristic of her most likely diagnosis is?

Resistance to activated protein c Recurrent DVT with a positive family history is most likely V leiden mutation, the most common cause of inherited hypercoagulability in the US particularly in caucasians. Hypercoagulability can present with venous thrombombolism (either deep venous thrombosis or pulmonary embolism) or with early miscarrige, less than 12 weeks gestation, due to placental thrombosis. A mutation in factor V prevents its inactivation by activated protein C. factor V is a pro-coagulant clotting factor that amplifies the production of thrombin, the enzyme that converts fibrinogen to fibrin. Thrombin slows its own production by creating a separate negative feedback loop converting protein C to activated protein C. Activated protein C is a protease that degrades activated factor Va and activated factor VIIIa, ultimately reducing thrombin production. Those with factor V leiden have a point mutation in factor V that lacks a cleavage site. Factor V Leiden is the most common hereditary hypercoagulable disease. it is caused by a mutation in factor V rendering it unable to be inactivated by protein C.

with a restricted internal rotation-would you expect to see an outflare or an inflare?

Restricted internal rotation is seen with an outflare of the innominate. Recall that somatic dysfunctions are named for the position of ease. In patients with an innominate outflare, there is a restriction of motion in internal rotation and ease of motion in external rotation.

What is the restrictive barrier?

Restrictive barrier is the maximum distance a patient can actively move a joint. it can be pathologic but is not always. The restrictive barrier is the maximum distance a patient is able to actively move a joint. the restrictive barrier lies before the physiologic barrier and prevents full range of motion.

immunology of rheumatic heart disease?

Result of immune-mediated damage to the cardiac valves (usually mitral) secondary to streptococcal derived M-protein activation of T and B lymphocytes.

Niacin deficiency results in what?

Results in 4 D's= DERMATITIS, DIARRHEA, DEMENTIA AND DEATH

what does Reye syndrome cause in the lIVER?

Reye syndrome is a common cause of hepatic encephalopathy in young children taking medication, which can result in acute liver damage and reversible fatty changes to the liver.

what is rhipicephalus sanguineus a vector for

Rhipicephalus sanguineus, or the brown dog tick, is the vector for boutonneuse fever caused by rickettsiae conorii. this disease is characterized by tache noir, an ulcerated eschar surrounded by an erythematous halo at the site of the tick bite.

WHERE IS RIB 2 TENDERPOINT LOCATED AND How would you treat it?

Rib 2 tenderpoint is located at the midclavicular point at the level of the second rib. to treat, you would flex the patient and sidebend and rotate towards the tender point so this would be FSTRT USing the head as a lever.

What type of fracture are common in patients with sickle cell disease?

Rib fractures.

what axis does ribs 6-10 move in?

Ribs 6-10 moves in bucket handle motion around an AP axis,

Neurological exam reveals dysarthria, loss of pain and temperature sensation on the right side of her face, and loss of pain and temperature sensation on the left side of her body. which of the following arteries is most likley implicated in the development of this patient's symptoms?

Right posterior inferior cerebellar artery Brainstem lesions present with mixed deficits, meaning deficits in the face are on the opposite side of the body. In addition, facial deficits appear ipsilateral to the side of the lesion.

physical exam reveals a positive standing flexion test on the right, a right ASIS that is superior COMPARED to the left, a left PSIS that is superior compared to the right and a deep right sacral sulcus. seated flexion test is normal bilaterally. the msot likely diagnosis is?

Right posteriorly rotated innominate utilize the standing flexion tests to indicate the side of the dysfunction. when dsciphering further landmark locations place all in relation to the positive standing flexion tests since that is the side of the dysfunction.

right sided buttock pain. pain began after arising from a crouching position. structural exam reveals a positive seated flexion test on the right, and posterior inferiorlateral angle on the left. good spring is found at the sacral base. Using an indirect technique with the pt prone, gentle anterior force should be applied to the?

Right sacral base. Indirect methods are directed away from the physiologic barrier and exaggerate the existing somatic dysfunction, placing the dysfunction in its position of ease. The above patient has a left-on left sacral torsion. therefore, the most anterior point of the sacrum is located at the right base and adding gentle anterior force at the right sacral base ill be placing it in ease. keep an eye out on INDIRECT OR DIRECT.

left sided back pain, rib 6 on the left is tender to palpation and does not elevate with the rest of the rib cage with inhalation. T4-T6 ARE NEUTRAL sidebent right and rotated left. Initial set up for HVLA for this dysfunction involves?

Right thenar eminence under the patient's left transverse process. the diagnosis of T6 neutral, sidebent right, rotated left. hIGH VELOCITY , LOW-AMPLITUDE TREATMENTs of the mid-thoracic vertebrae and ribs are accomplished using variations of the kirksville crunch, wherein the operator uses the thenar eminence of the opposite hand to localize and apply a HVLA thrust to the desired treatment area. The physician sidebends the pt left, down to the T6 to T7 junction, and the sidebeinding and rotation forces are localized at the T6 fulcrum by adjusting accordingly.

Ropiness is this chronic or acute? what happens to muscle tone in acute versus chronic conditons?

Ropiness is chronic. chronic somatic dysfunctiins present with cool, dry, thickened skin, decreased muscle tone, asymmetry with compensation in other parts of the body, decreased or no pain, dull or achy tenderness, prolonged blanching of the skin, and lack of edema and erythema. muscle tone is increased in acute dysfunctions as the body responds to the injury by spasming about the injured area in an effort to protect it. some characteristics of chronic descriptions are: cool, dry, thickened skin, decreased muscle tone asymmetry with compensation in other parts of th ebody, decreased or no pain, dull or achy tenderness, prolonged blanching of the skin and the lack of edema and erythema.

On exam, you diagnose C3 extended rotated right and sidebent right. how would you best treat this using a BLT technique?

Rotate the head right while bringing the right ear to the shoulder and have her take deep breaths. When performing a balanced ligamentous tension technique you want to exaggerate the injury pattern by moving towards the position of ease. hold the joint at the point of balanced ligamentous tension until a still point or a release occurs. iNDIRECT techniques that can be used with this patient besides BLT are counterstrain, FPR, and myofascial release. the relative contraindications of HVLA are recent whiplash injury, acute herniated disc, acute radiculopathy, osteoporosis, spondylolisthesis, hypermobility syndrome, and metabolic bone disease.

what are some diagnosis likely to be found in the lower quadrant and right pelvis?

SALPINGITIS, APPENDICITIS, URETERAL CALCULI, DYSFUNCTIONS INVOLVING THE URETER, AND PROSTATE CANCER IN MEN. ---ALL OF THIS ARE ORGANIC CAUSES OF PAIN IN THE LOWER RIGHT QUADRANT THAT CAN BE REFERRED AS PAIN IN THE HIP AND THUS SHOULD BE RULED OUT BEFORE NEUROMUSCULAR DIAGNOSIS.

middle aged african american woman, have dyspnea, non-caseating granulomas, bilateral hilar lymphadenopathy, blurry vision, and erythema nodosum, uveitis, painless swelling of the salivary and lacrimal glands. 50% of cases are asymptomatic with only incidental radiographic evidence, which is a chest radiograph with enlarged hilar and mediastinal lymph nodes. what is the diagnosis?

SARCOIDOSIS

What is SCID commonly caused by?

SCID is commonly caused by a deficiency in adenosine deaminase, and patients commonly present with recurrent viral, bacterial, fungal, and/or protozoal infections along with chronic diarrhea and failure to thrive.

Patient with lung cancer has free water retention relative to sodium causing a hyponatremia. what is causing this?

SIADH hormone is been produced by the lung cancer.

there are many diseases that most be reported the most commonly tested ones are:

STD: HIV, gonorrhea, syphilis Hepatitis: hEPATITIS, A, B, and C Childhood-mumps, measles, rubella, chicken pox Diarrhea: Salmonella, shigella Tuberculosis because salmonella can cause outbreaks it most be reported.

posterior cerebral artery

SUPPLIES most of the occipital lobes and the medial aspect of the temporal lobes. strokes involving the PCA are most commonly characterized by visual field cuts. Unilateral strokes involving the hemispheral branches cause a contralateral homonymous hemianopsia other more complex, visual deficits can occur and include formed or unformed visual hallucinations, visual and color agnosias or prosopagnosia

don't get confused with picky words hormone responsible for contraction is SYNTHESIZED WHERE?

SYNTHESIZED INT HE HYPOTHALAMUS AND released from the posterior pituitary

In female patients what should be ruled out prior to the diagnosis of psoas syndrome

Salpingitis

MULTISYSTEM GRANULOMATOUS DISORDER OF unknown etiology that is characterized by non-caseating granulomas in involved organs. often manifests with restrictive lung disease characterized by the formation of granulomas containing multinucleated giant cells.

Sarcoidosid

46 yo male itchy skin,particularly on his hands, been a regular volunteer at the homeless shelter and spends a lot of time in his green house. physical exam reveals the rash pictured. what is the suspected diagnosis and what is associated with it?

Scabies the reaction is caused by of eggs and feces. Scabies is a mite infection and commonly presents in the creases and webs of the fingers and toes. Scabies is a mite infestation of the skin. the arthropods (sarcoptes scabiei) burrow under the epidermis and lay eggs and leave their feces. the eggs and feces cause the irritated rash that appears. most likelya form of type IV hypersensitivity. Raised bumps in the webs of the fingers and toes are the classic finding. Areas of low hygiene and overcrowding, such as homeless shelters, facilitate the spread of scabies. Treatment must involve ridding the environment of the infestation to prevent re-acquisition. Permethrin is the treatment of choice, with other options also available.

explain the schilling's test?

Schilling's test is used to diagnose the cause of vitamin B12 deficiency. First, radioactive vitamin B12 is given orally, followed by intramuscular injection of non-radioactive B12. A normal person excretes the radioactive B12. if the excreted levels are low, there is defective GI absorption of vitamin B12. the test is then repeated administering intrinsic factor orally. iF THE LEvels of excreted radioactive vitamin b12 increase to normal range, it is confirmatory for a diagnosis of perncious anemia. in the schilling test radioactive B12 is given orally, then a non-radioactive B12 is given intramuscularly. the intramuscular B12 is givent to saurate B12 receptors. The oral radioactive B12 will be absorbed via the GI tract with the aid of intrinsic factor. since B12 RECEPTORS ARE already saturated, the radioactive B12 will be excreted through the urine. A normal person will excrete 7-22% of the radioactive B12. If GI absorption of B12 is defective, there will be below normal urinary levels of radioactive B12 . When the test is repeated administering oral radioactive B12 with intrinsic factor followed by intramuscular B12, the urinary levels of radioactive B12 will be normal if the patient has pernicious anemia since pernicious anemia is due to a lack of intrinsic factor. summary=oral RADIOACTIVE cyanocobalamin with intrinsic factor followed by intramuscular cyanocobalamin resulted in normal urinary excretion of cyanocobalamin.

What did Schloendorff V society of New York hospital establish?

Schloendorff v. society of new york hospital established a legal precedent for informed consent. the Plantiff Mary Schloendorrff consented to an exam under ether in 1908 but not to any surgical intervention. While she was under surgical intervention, the surgeon removed her uterus. Justice Cardozo wrote the following, which established a basis for informed consent: "every human being of adult years and sound mind has a right to determine what shall be done with his own body; and a surgeon who performs an operation without his patient's consent commits an assault, for which he is liable in damages.

44 yo caucasian female presents with small painful ulcerations on four of her fingertips and shiny, hypopigmented skin on the majority of her fingers. what is the most likely diagnosis?

Scleroderma is a multisystem disorder characterized by inflammatory, vascular, and sclerotic changes of skin and visceral organs. The cause of scleroderma is unknown, but the course of the disease involves initial edema, followed by fibrosis of the skin and organs due to overproduction of collagen. this causes a fibrotic or "shiny" appearance of the skin. patients often experience digital pain due to skin tightening of the digits leading to Raynaud's phenomenon and eventually insufficiency ulcers at the finger tips. FIngers may appear hypopigmented due to lack of blood which gives the fingers a noticeable white appearance.

A rash on the face and scalp consisting of greasy scales over an erythematous patch?

Seborrheic dermatitis has bimodal age distribution, occuring commonly in neonates or adults. number location of occurence in neonates is the scalp in which case it is referred to as cradle cap. it will commonly be seen on hair -bearing regions such as the scalp, eyelashes, and regions such as the scalp, eyelashes, and eyebrows. It can also be found in intertriginous regions such as the posterior auricle, neck, axilla, and groin.

45 yo woman presents for her routine annual physical examination. she has no significant past medical hx. her father has type II diabetes mellitus. As part of her examination, blood is drawn for a lipid profile. this testing is an example of which of the following?

Secondary prevention because of her age. screening tests in healthy individuals are a form of secondary prevention. screening tests, such as fasting glucose levels and lipid profiles, are performed on healthy individuals t detect early disease and provide early intervention if necessary this is considered secondary prevention because these procedures do not by themselves prevent these incidence but instead allow for preventative methods when necessary. primary prevention is an intervention that decreases disease incidence in healthy individuals. examples include vaccination, healthy lifestyle, aspirin, and antibiotic prophylaxis.

What are the hamstring muscles composed of? What is the action of the biceps femoris? What is the biceps femoris innervated by?

Semitendinosus, semimembranosus and biceps femoris knee flexion and hip extension Innervated by both the common peroneal and tibial branch of the sciatic nerve

39 YO caucasian male with Type II diabetes mellitus comes to the ER complaining of sudden vision loss in his left eye that was preceded by intermittent blurry vision and floaters of two days duration. fundoscopic exam reveals a wrinkled appearance of the retina. upon palpation of the spine you note paravertebral muscle spasm from T1-4. the most likely cause of these findings is?

Separation of the retina from underlying retinal pigment epithelium. rETINAL detachment involves separation of the retinal layer from the epithelial layer or choroid and presents with unilateral sudden loss of vision best described by patients as "window shade coming down on the eye--treatment of choice is laser photocoagulation or surgery. the choroid is a vascular membrane containing large branched pigment cells in between the retina and sclera. This vision loss is usually painless and is best described as "curtain or window shade pulled over the eye". some pts may experience seeing floaters in their peripheral vision filed that precede or coincide with blurry vision. these are caused by pigmented fragments floating in a vitreous humor. however, this is not a specific symptom of retinal detachemnt but can increase suspicion for the condition. A wrinkled appearance of the retina is commonly seen on fundoscopic examination, which is a result of retinal tear usually located in the periphery. Risk factors for retina detachment include trauma, cataract surgery, myopia, and a fm x. treatment is laser procoagulation or cryotherapy to prevent the further progression or tearing and reattachment of the retina . they may not fully restore the loss of vision, however.

drug of choice for panic attacks?

Short acting benzodiazepines are the drug of choice for panic attacks. An overdose of benzodiazepines can be managed with flumazenil, which is a direct benzodiazepine antagonist. chronic use can cause tolerance and dependence.

Name phsysiological strains?

Sidebending/rotation and torsions

Physical examination reveals a well-appearing male with an obvious group curve in the mid thoracic and upper lumbar regions as shown in the exhibit. so it appears as though he was leaning to the right-so his body was shifted to the right. WHich of the following dysfunctions are most likely present? A person with scoliosis what compromise is likely to occur first?

Sidebengt left, dextroscoliosis Dextroscoliosis refers to a curved spine that is convex on the right while sidebending occurs on the left. Levoscoliosis refers to a curved spine that is convex on the left while sidebending occurs on the right. so convex (think of where the spine is going) is where the bone is shifted and sidebent will be on the opposite side. Respiratory compromise generally occurs before cardiovascular compromise. Respiratory compromise generally occurs with less severe curvature than cardiovascular compromise in patients with scoliosis because a more severe angle of spine curvature is needed to compress the thoracic visceral structures enough to impair the function of the heart and great visceral structures enough to impair function of the heart and great vesssels. A thoracic curvature of more than 50 degress results in compromise of respiratory function and an angle of more than 75 degrees will seriously compromise cardiovascular function. patients with curves greater than 45 degrees should be evaluated by a spine surgeon for possible surgical correction of the curve. goal of OMM is to improve flexibility

what is the most likely abnormality to be seen in patients with pulmonary embolism?

Sinus tachycardia is the most common ECG abnormality seen in pts with acute pulmonary embolisms. note S1 Q3 t3 ECG pattern is associated with and specific for pulmonary embolisms, it is not the most likely abnormality seen on ECG. Non-specific ST segment and T wave changes are more likely to be seen in pts with pulmonary embolisms. S1 Q3 T3 pattern i smost seen in pts with massive pulmonary embolisms and is not often seen in those patients with smaller embolisms. this pattern is described as a large S-wave in lead I, large Q-wave in lead III and T-wave inversion seen in lead III.

Anti-Ro

Sjogren syndrome

physical examination reveals a bulging, erythematous tympanic membrane with decreased cone of lihgt--which of the following somatic dysfunctions will most likely be found?

T1ERSL t1-t4 SUPPLY THE SYMPATHETIC INNERVATION TO THE HEAD AND NECK. tHIS PT MOST LIKLEY HAS ACUTE OTITIS MEDIA, THE HEAD AND NECK RECEIVE SYMPATHETIC INNERVATION FROM t1-T4. viscerosomatics reflexes are created when visceral afferents transmit signals along the same segment of somatic efferent fibers. oVE-ABUNDANT STIMULI FROM THE MIDDLE EAR MAY RESULT IN FACILITATION F THE t1-4 THORACIC SEGMENTS AND HYPERTONICITY OF THE PARASPINAL MUSCLES IN THIS AREAA.

pt comes with foul smelling diarrhea, intermittent upper quadrant pain worse with fatty foods. lab shows hyperglycemia, gallbladder ultrasound performed shows biliary sludge. what are you suspecting?

Somatostatinoma tHE PANCREAS CONTAIN ALPHA, beta and delta endocrine glands. The alpha cells secrete glucagon, the beta cells secrete insulin, and the delta cells secrete somatostatin. Somatostatinomas ARE PANCREATIC ISLET DELTA CELL tumors that release an excess amount of somatostatin. Somatostatin normally acts to inhibit secretion of insulin, glucagon, gastrin, and growth hormone. This results in the development of diabetes mellitus, cholelithiasis, steatorrhea, and hypochlorhydria. Diabetes is the result of physiological action of somatostatin to inhibit the release of the insulin. Cholelithiasis is due to decreased cholecystokinin release, which reduces gallbladder contractility. Many patients with somatostatinomas also have gastric hypochlorhydria due to decreased gastrin secretion.

What is a somatovisceral reflexes?

Somatovisceral reflexes are produced when localized somatic stimulation produce patterns of reflex response in a segmentally related visceral structure for example: tissue texture abnormalities in T1-T4 leading to tachycardia.

DNA is cut into fragments by restricted endonucleases. these are then electrophoresed on a gel and then transferred to a filter. this filter is then soaked in a denaturant and exposed to a DNA probe that bind to its complementary strand. which of the following lab techniques does this describe?

Southern blot. do not be quick to pick an answer. read all the options well. southern blot is a technique useful in identifying a particular DNA sequence from a sample of DNA. PCR is a technique used to amplify a specific segment of DNA.

name the midline bones of the cranium?

Sphenoid, occiput, ethmoid, vomer

5 yo male presents with the complaint of "feeling sick" asociated with aches, pains, and a headache. history reveals the patient was recently visiting his grandmother in pennsylvania (northeast US) ONE WEEK ago. A rash (erythema migrans) is observed on his arm. what is the shape of the causative agent? what is the appropriate treatment?

Spiral shaped bacterium=borrelia burgdorferi =spirochete-transmitted by ixodes tick Amoxicillin due to bone and teeth malformations caused by doxycycline in children.

what does degenerative (type III) spondylolisthesis most frequently involve?

Spondylolistehsis most frequently invoLVES l4-l5 LEVEL-related to a number of factors including aging. ---patients with degenerative spondylolisthesis demonstrate symptoms of spinal stenosis with dysesthesias or leg pain. the spinal stenosis pattern of pain when walking beyond a well-defined distance (neurological claudication) is often present, relieved only by sitting down or bending over. There are different types of spondylolisthesis that can occur, each primarily affecting a different level of lumbar vertebrae. Type 1, the dysplastic form os spondylolisthesis, is a congenital deficiency of the superior sacral facet, the inferior fifth lumbar facet, or both. Type II is the isthmic form, is caused by a defect in the pars interarticularis but can also be seen with an elongated pars. tYPES I AND II are most COMMONLY seen in younger patients (less than fifteen years old) and most likley to occur at he L5-S1 level. Type III, the degenerative form of spondylolisthesis, is seen in older patients and most frequently involves the L4-L5 level. try not to confuse grade with type.

spondylosis, spondylolisthesis and spondylolysis

Spondylosis=degenerative cognition often involving the disks, casusing lipping of vertebral bodies. Spondylolisthesis is an anterior slipping of one vertebrae on the vertebrae beneath it. it is seen using a lateral x-ray. often will be felt in the patient's back. grade 1: 0-25%, grade 2: 26-50%, grae 3: 51-75%. grade 4: 76-100% Spondylolysis is casues by a fracture or defect of the pars interarticularis of the vertebrae. this may occur without a displacement of the vertebrae. it is seen using an oblique view of the spine. look for "scotty dog sign"

jehovah's witness refuses blood transfusion and loses consciousness. the next step in management would be to?

Stabilize with IV fluids. persons aged 18 years old and above are able to make their own decisions regarding health care treatment, especially with religious grounds. The autonomy of these decisions should be respected, no matter the expected outcome.

Name the four stages of the Schilling test?

Stage 1-used to measure cobalamin absorption by assessing increased urine radioactivity after an oral dose of radioactive cobalamin. If abnormal low values are obtained in stage I, a stage II schilling test is performed. stage II-60mg of active porcine IF is administered with the oral test dose to determine if this enhances the absorption if vitamin B12. if poor absorption of vitamin B12 IS NORMALIZED, the patient likely has classic pernicious anemia, which is a deficiency in IF. if poor absorption is again observed durng stage II, then other causes of vitamin B12 deficiency need to be sought after. Stage III-is repeat of stage 1 but after 5 days of tetracycline therapy--this is to rule out blind loop syndrome as a cause of the deficiency. In stage IV, the administration of trypsin or pancreatic enzyme with the radiolabeled test dose is given to the patient to see whether absorption is normalized. if it does then pancreatic disease should be considered. Pancreatic disease can produce cobalamin deficiency. Nonspecific R binders chelate cobalamin in the stomach, making it unavailable for binding to IF. Pancreatic proteases degrade the R binders and release the cobalamin so that it can bind IF. If this pancreatic proteases are not being made by the pancreas then cobalamin cannot be available to bind IF.

which of the following stages of sleep stages have the largest percentage of sleep time on a normal electroencephalography (EEG)?

Stage 2 sleep Stage 2 sleep comprises the largest percentage of sleep each evening ranging from 45-50% of sleep each evening.

which stage of sleep is the most replenishing"?

Stage III INFANTS REQUIRE LARGE AMOUNTS OF SLEEP, ESPECIALLY STAGE 3 BECASUE THE LARGEST AMOUNT OF GROWTH HORMONE IS RELEASED AT THIS TIME.

what is the deepest non-REM sleep?

Stage N3

Nocturnal enuresis, along with night terrors and sleep walking, occurs during what stage of sleep?

Stage N3 of sleep

IV drug abusers are at risk for infective endocarditis of the tricupsid valves. what is the most common organism for the acute form?

Staph aureus followed by pseudomonas aeruginosa

In a pat with an exacerbation of chronic bronchitis who develop penumonia that equires hospitalization, M. catarrharrhalis is the most common infection pathogen. what of an alcoholic who develops penumonia with a gram positive organism waht is the most common cause of the pneumonia?

Staphlococcus aureus staph aureus is one of the most common organism that can casue pneumonia in alcoholics. The sputum is often noted as salmon colored. S. aureus often resides on the skin, nasal passages, and sometimes the pharynx of the human body. it is also an opportunistic pathogen, which means it can casue disease if it sgets carried to an area where it can become virulent. cONDITIONS Associated with altered or reduced consciousness, such as alcoholism, increases the risk of aspiration pneumonia.

What is the most likely side effect of ethosuximide?

Steven Johnson syndrome-presents with malaise and fever accompanied by a rapid onset of erythematous macules that lead to epidermal necrosis and sloughing of the skin. other drugs that will cause steven johnson syndrome include lamotrigine, phenobarbital, phenytoin, sulfa drugs, and allopurinol.

what is Still's murmur?

Still's murmur is a common, benign heart murmur heard in healthy children bertween the ages of two and eight. Syill's murmur would be a mid-systolic murmur on the mid precordium--between the apex of the heart and the left sternal border. there is no congenital heart defect associated with this murmur. the cause of the murmur is thought to be due to either aortic blood flow or vibrations from pulmonic valve leaflets. the murmur may be more pronounce in children with fever, anemia, or who are lying supine.

what does stimulating the M3 receptor in the ciliary muscle of the eye cause to happen?

Stimulating the M3 receptors in the ciliary muscle will cause it to contract, relaxing the zonules attached to the lens. the lens will then round out and allow you to see up close (improve your near vision) If you block the M3 receptor, the ciliary muscle will relax (calld paralysis of accomodation), the zonules will tighten up, and then lens becomes less round (improvs your far vision but your near vision becomes worse).

what does stimulation of the alpha 1 receptor in the eye produce?

Stimulation of the alpha 1 receptor in the eye produces mydriasis and blocking of the alpha1 receptor produces miosis

What is the straight leg raise testing for?

Straight leg raise (also known as Lasegue's test) is used to evaluate sciatic nerve compression, resulting from a disc herniation in the lumbar spine a majority of the time (about 90% of cases). normally the leg can be lifted 70-80 degrees of hip flexion without symptoms. kEEP IN MIND THAT TRUE sciatic pain is typically felt down the posterior aspect of the leg and below the level of the knee. this test is not the gold standard for diagnosis, however, it can aid in further decision making.

what are the most common cause of bacterial meningitis in children between different three months and ten years of age?

Streptococcus pneumoniae followed by neisseria meninigitidis treat bacterial meningitis empirically with ceftriaxone plus vancomycin.

Most common causes of acute otitis media are:

Streptococcus pneumoniae, Haemophilus influenzae, and Moraxella. physical exam finding will show a red, bulging and non mobile tympanic membrane. The development of point tenderness over the posterior aspect of the temporal bone points to the progression of AOM to mastoiditis. Mastoiditis is an infection of the mastoid air cells which is part of the temporal bone of the skull. it is usually caused by delayed treatment of otitis media and can progress further to cause meningitis. Antibiotic treatment must be initiated early in the treatment of otitis media.

what class of drug does this medications below belong?-tolbutamide, chlorpropamide, glyburide, glimepiride, and glipize?

Sulfonylureas. tHIS DRUGS IS ASSOCIATED with hypoglycemia.

what oMM techniqure can you use to treat IBS?

Superior mesenteric ganglion release can be used to treat IBS. The celiac ganglion (T5-T9 via the greater splanchnic nerve) supplies the upper GI tract (lower 2/3 of the esophagus to the ligament of Treitz). the superior mesenteric ganglion (T10-11) via the lesser splanchic nerve) supplies the middle GI tract (ligament of Treitz to the mid-transverse colon). The lower GI tract is supplied by T12-L2 via the inferior mesenteric ganglion and least splanchnic nerve.

What is superior quadranopsia caused by?

Superior quadranopsia is the product of disruption at Meyer's loop or temporal radiation fibers from the later geniculate nucleus. Homonymous superior quadrantanopia is when the upper quadrant of the visual field ceases to function. Homonymy refers to the fact that the same quadrant (meaning both the left or right) is nonfunctional in each eye. it can occur as a left superior quadrantanopia, in which case the upper quadrant of the temporal field in the left eye and the upper nasal quadrant of the right eye lack vision. Left superior quadrantanopia can result from damage to the right hemisphere's lateral geniculate nucleus (LGN), which carries visual information to the striate cortex. Right superior quadrantanopia is opposite to left superior quadrantanopia (i.e visual field loss in the upper temporal field of the right eye and the upper nasal field of the left eye, with damage occuring at the LGN). Damage to the LGN can be caused by a partial lesion, which could be due to ischemia ( a deficiency in blood supply) of the LGN. Lesions to the Optic radiation of the temporal lobe in a region called meyer's loop (the anterior region of the optic radiation) may also be involved in causing superior homonymous superior quadrantanopia. Meyer's loop travels from the LGN to the striate cortex carrying visual information. Lesions in the right temporal lobe that affect meyer's loop wil cause visual field loss in the upper temporal quadrant of the left eye and upper nasal quadrant of the right eye, and vice versa when the lesion is located on the left temporal lobe.

An EKG was obtained and reveals ST-segment elevation in leads II, III, and aVF. Basedupon the information provided, treatment to which of the following spinal segments may calm the sympathetic influence on the affected system?

T2 PATHOLOGY of the CARDIAC SYSTEM CAN MANIFEST AS SOMATIC changes at the corrresponding spinal levels of T1-T5, with changes at T2 on the left being relatively specific for a myocardial infarction.

what is the first arterial branch of the external carotid artery?

Superior thyroid artery. it branches from the external carotid immediately suprior to the division of the common carotid artery. it sends branches to adjacent muscles, anastomoses with its counterpart of the contralateral side, and perfuses the thyroid gland, usually with two branches. it has four other named braches: hyoid artery, sternocleidomastoid artery, the superior laryngeal, and the cricothyroid artery. facial artery is a branch of the external carotid but it branches higher in the body. lingual artery is another branch of the external carotid. this artery arises above the superior thyroid facial arteries The maxillary artery is a continuation of the external carotid. this and the superficial temporal artery are the terminal branches of the EC.

you choose to palpate her cranium using vault hold and notice the greater wings of the sphenoid moving in a caudad direction, while the base of the sphenoid deviates cephalad in relation to the occiput. the most likely diagnosis is

Superior vertical strain. dental procedure likely caused the trauma resulting in the superior vertical strain as well as TMJ pain.

what does SADPUCKER MEAN

Suprarenal (adrenal) glands, Aortic/IVC, duodenum (second and third +/- fourth), pancreas(head, neck and body), Ureters, colon (ascending and descending and blood supplies), kidneys, esophagus, and rectum Middle colic artery is the only intraperitoneal structure. the middle colic artery is a branch of the superior mesenteric artery, and travels insude the transverse mesocolon (or transverse mesentery) to supply the transverse colon.

INABILITY TO ADDUCTO THE ARM SLOWLY from an abducted position at 90 degree is testing for

Supraspinatus tendon tear

When should you suspect type 1 complex regional pain syndrome?

Suspect type 1 complex regional pain syndrome in patients who suffer from continued pain (out of proportion to injury), allodynia, joint stiffness, localized edema, increased hair growth, and vasospasm following injury with no evidence of nerve damage. based on the above information, the patient is most likely suffering from complex regional pain syndrome type 1 (CRPS-1), previously known as reflex sympathetic dystrophy. complex regional pain syndrome is a chronic progressive disease broken down into two types based on the presence of nerve lesion following an injury. There is No evidence of nerve damage in type 1 injuries, whereas there is evidence of nerve damage in CRPS type 2. CRPS-1 is characterized by severe, burining pain at the site of injury. Muscle spasm, joint stiffness, restricted mobility, rapid hair and nail growth, and vasospasm causing edema and skin changes can also occur. Direct osteopathic manipulation to the region of the body affected in complex regional pain syndrome (CRPS) should be avoided due to worsening of the patients symptoms. Treatment should be directed peripheral to the body part that is affected.

What locations would you expect to find viscerosomatics of the heart in terms of sympathetic and parasympathetic?

Sympathetic would be T1-T4 Parasympathetic would be at the OA where the vagus nerve. someone comes with chedt pain going on for an hour this is acute and radiating this is acute

21 yo altered behavior, alternating laughing and crying uncontrollably, and without apparent reason. describing objects and people not in the room, symptom occuring in the past three to four hours and has not been present in the pt before. PE anxious male with involuntary tremor. PE is difficult to perform because he starts to panic, pushes you away and says that you are trying to hurt him. he is unable to give correct date or time of day. he admits to taking a liquid substance that was placed onto a sweet tart but cannot tell you when this happened. which of the following symptoms or signs is commonly associated with the drug he ingested?

Synesthesia He injected Lysergic acid Diethylamide (LSD). LSD is a hallucinogen that results in the alteration of sensory perception, mood m and thought patterns. Synthesia is a blending of the senses wherein users report hearing colors or seeing colors. users commonly place LSD onto candies or liquid droppers before ingestion. patients often experience alterations in mood, abrupt changes in emotion, illusions, and hallucinations. In extreme cases, they can become aggressive, suicidal, homicidal or severly depressed, patients may also experience heightened perception of sensation, a distorted sense of time, euphoria, and synesthesia. for those presenting to the emergency department, benzodiazepines are helpful for acute agitation and dysphoria, and haloperidol is helpful for any psychotic features. gastrointestinal decontamination is not necessary in the majority of cases.

steatorrhea with skin hyperpigmentation of the face is an indication of a bacterial infection with?

T. Whipplei and is DIagnosed by PAS-positive staining macrophages on biopsy.

physician doubles the dose of a drug so the pt can receive a good effect from it. which of the following is likely to remain constant?

T1/2 (HALF-LIFE ) OF THE DRUG WILL REMAIN CONSTANT METHOTREXATE FOLLOWS FITST ORDER kinetics like most drugs. half life=0.7*vd/cl, where vd is defined as the volume of distribution and cl is the clearance of the drug. this indicates that the half-life is constant regardless of the dose because neither the volume of distribution (Vd) nor the clearance (CL) changes with dose.

what is the viscerosomatic points for the appendix?

T10-T12

Where is the viscerosomatic reflex point of the sigmoid colon?

T12-L2

physical examination reveals tenderness to palpation in the left lower quadrant. the most likely location of these tissue texture change is?

T12FRSL. T12-L2

upon examination, the dysfunctional T9 segment is found to have restriction in the transverse plane and around a transverse axis. This vertebral segment is treated by applying prone pressure myofascial release technique which provides great relief to the patient. Which of the following dysfunctions best describes the position of T9 vertebral segment before treatment?

T9 extended, rotated left, sidebent left Vertebral motion around a transverse axis means that the vertebra is either in flexion or extension. Vertebral motion in a transverse plane means the vertebrae is rotated. therefore in this scenario, T9 must also be flexed or extended and rotated. According to Fryette's principles, type one somatic dysfunctions involve a group of vertebrae in neutral position with rotation and sidebending in opposite directions. type II somatic dysfunctions typically involve a single vertebral segment that is either flexed or extended with rotation and sidebending in the same direction. when treating a pt using prone pressure myofascial release, the patient is prone and the physician's elbows are locked in extension, pushing downward and laterally on the patient's paravertebral muscles.

The sympathetic viscerosomatic reflex of the kidney is what?

T9-L1.

what counterstrain tenderpoint is located on the anterolateral aspect of the scapula

THE SUBSCAPULARIS. IT IS IMPORTANT TO UNDERSTAND THE TENDERPOINTS FOR THE ROTATOR CUFF MUSCLES.

On physical exam, you find a positive standing flexion test on the right, a negative seated flexion test, ASIS are equal bilaterally and left pubic rami is superior comparEd to the right. Hamstring test is equal on both sides. What is this person's diagnosis? what is the correct initial set up for muscle energy from a supine position?

THIS PATIENT HAS A RIGHT INFERIOR PUBIC SHEAR. There is a right iliosacral dysfunction because the standing flexion test is positive on the right but the seated flexion test is negative. FOr the initial set up for muscle energy for treatment of a right inferior pubic shear the patient's right leg is flexed at the hip and knee until resistance is felt. so texts say you can add abduction. Remember that patient's presenting with knee pain can have pelvic somatic dysfuncton.

what does the sibson's fascia have to do with lymphatics technique?

TO IMPROVE symptoms of lymphatic congestion--the treatment first is thoracic inlet release which is initiated by contacting the sibson's fascia. After the thoracic inlet and diaphragm is treated the order of treatment is proximal to distal.

what do you treat before pumping technique?

TREAT the diaphrams first before pumping lymphatic fluid. Begin with the most restricted diaphragm or the one that does not fir the compensatory pattern of alternating pattern. . Zink's common compensatory pattern is as follows: OA PREFERS rotation LEFT cervicothoracic junction (cervicothoracic diaphragm) prefers rotation right the thoracolumbar junction (abdominal diaphragm) prefers rotation left the lumbosacral junction prefers rotation right

image has red string like things scattered everywhere dyspnea, productive cough of 2 months duration. high temp, cd4 count is 43, acid fast bacteria is present. what would be the appropriate medication?

TRIMETHOPRIM-SULFAMETHOXAZOLE. THIS PT is suffering from bronchopulmonary disease caused by norcardia asteroides. gram positive rod found in soil strict aerobic rods branched filamentous form in tissues and culture. norcardia cause pulmonary diseases (bronchitis, pneumonia, and lung abscesses) in immunocompromised especially those with T cell deficiencies. The median CD4 count in patients who develop nocardiosis is approximately 50 cells/mm^3. localized infection can be treated with trimethoprim-sulfamethoxazole. systemic can be treated with amikacin with carbapenem or broad spectrum cephalosporin.

Arenaviruses are an impt casue of viral hemorrhagic fever. they have bipartite ambisence ssRNA genomes and have distinctive sand-like particles on electron microscopy?

TRUE

De Quervain disease, or subacute thyroiditis is self-limited hypothyroidism that is typically seen following a viral, flu-like illness. a symptpm unique to this is jaw pain

TRUE

lateral strain pattern occurs when the sphenoid and occiput rotate in the same direction around two vertical aces. palpation of the cranium with this dysfunction feels as if the head is shaped like a parallelogram

TRUE

pts with polymyalgia rheumatica complain of morning stiffness but may have no associated muscle weakness. lab analysis will elevated ESR.

TRUE

treatment for absence seizure with either ethosuximide or valproic acid. true or false?

TRUE

true or false, chapman reflex points are a type of viscerosomatic reflex?

TRUE

True or false. Chapman reflex points correlate with increased sympathetic tone to a certain area of the body?

TRUE chapman point are small points of tenderness and sensitivity found on the deep fascial layers that correlate with increased sympathetic tone to a particular area of the body. so they are a type of viscerosomatic reflex. physiologically increased sympathetic tone as well as increased lymphatic blockages lead to the myofascial nodules. upon palpation, chapman reflex points are small (2-3mm), amooth, and firm. these points are mainly used as diagnostic indicators for dysfunction of a particular organ.

A closed head injury can result in damage to the afferent limb of the pupillary reflex, commonly known as the Marcus Gun pupil. this defect may be quickly deduced with a simple swinging flashlight.

TRUE,

UCL stabilizes valgus stress on the elbow and attaches between the medial epicondyle and the coronoid process of the ulna

TRUE.

septic shock is characterized by hypotension, high cardiac output that becomes depressed as the symptoms progress, low systemic vascular resistance, and low cardiac filing pressures. gram negative rod septic shock is caused by the lipid A portion of lipopolysaccharide. true or false? VIRULENCE FACTOR FOR GRAM NEGATIVE ROD? in septic shock will one see high systemic vascular resistance?

TRUE. Lipopolysaccharide No, one will see low vascular systemic resistance. septic shock is characterized by hypotension, high cardiac output that becomes depressed as the symptoms progress, low systemic vascular resistance and low cardiac filling pressures. key to treatment of septic shock is antibiotics and vasopressors, such as norpinephrine. vasopressors increase the patient's low systemic vascular resistance and help maintain peripheral perfusion pressures.

6 month old ashkenazi jewish descent presents with multiple developmental delays. the child's mother notes h no longer makes eye contact, has an increased startle response, and no longer reaches for toys. physical examination reveals cherry red spot on the macula. what is the most likely diagnosis?

Tay-sachs disease autosomal recessive enzyme deficiency=hexosaminidase A the deficiency leads to accumulation of gangliosides in the lysosomes of the central nervous system. there are multiple subclasses of Tay-sachs with variable age of onset and severity. Infantile onset is characterized by normal development until about 5 months of age where relentless neurodegeneration results in death around the age of 5 years. characterics include a decline in gross motor skills, paralysis, mental retardation, and blindness. The child will have delays in motor skills such as playing with toys and walking. hyperacusis, a heightened startle response, is noted secondary to blindness as well as decreased eye contact. A key fundoscopic physical exam finding is cherry red spots on the macula of the pt. Niemann pick-sphingomyelinase deficiency. accumulation of sphingomyelin in the central nervous system resulting in a rapid neurodegenerative course causing severe mental retardation and death by 2-3 years. there is hepatomegaly. Way to differentiate symptoms. tay sachs has motor skills issues.

Finding of a cherry red spot and a ground glass retina are the classic findings in retinal artery occlusion. what are some differential diagnosis for a cherry red macula?

Tay-sachs, niemann-pick disease, and central retinal artery occlusion.

know the most common symptoms associated with each of cranial bone dysfunction instead of memorizing every possible symptom. it is helpful to remember which cranial nerves pass through each foramen, and the corresponding bones. symptoms of cranial nerves are generally due to dysfunction of the bone which they traverse through. if a person presents with ringing in their ear, this symptoms are most likely associated with?

Temporal bone dysfunction. plus he had a blow to the side of his head as well.

where is the tenderpoint for the anterior cruciate ligament located?

Tenderpoint for anterior cruciate ligament is located in the posterior leg at the widest point of the popliteal space. in the case of ACL sprain, pts classically relay a story of sudden knee hyperextension in low velocity setting or quick deceleration or significant rotation. this injury is common in dynamic sports requiring quick changes in direction. A pt with a lateral knee pain, with such a mechanism of injury is at risk of ACL sprain and thus requires imaging to assess the degree of injury.

what vaccines contain gelatin and why should patients be cautious about this?

The CDC recommends extreme caution before administering the MMRor varicella vaccines to any person with a history of anaphylactic reaction to gelatin or gelatin-containing products. skin testing for sensitivity to gelatin should be considered. some gelatin containing goods include marshmallows, gummi candy, patient should be evaluated by an allergist prior to vaccine adminstration. gelatin containing vaccines include MMR, influenza, rabies, varicella and yellow fever.

Intention tremors are seen in patients with multiple sclerosis. what brain region is directly involved?

The Cerebellum

To treat a right anterior sacrum with HVLA, the physician stands to the right of a prone patient and places his right thenae eminence of his treating hand over the right ILA with the fingers of the same hand monitoring the right PSIS. With his left hand the physician extends and internally rotates the patient's right leg. the leg is then abducted until motion is felt at the PSIS. AT THE END OF EXHALATION THE PHYSICIAN exerts a downward thrust with his thenar eminence over the right ILA. Ananterior sacral dysfunction is treated with HVLA by the physician using his thenar eminence to directly thrust the ILAs anteriorly in a prone pt, thus bringing the sacral base posteriorly. How do you treat a RIGHT posterior sacrum?

The Patient is supune. the physician stands on the patients left side and sidebedns the pts legs and torso away, puttng the pt in a backward c shpae. then the pt clasps their hands behind their neck. the physisicn places his caudad hand on the right ASIS and the physician uses the cephalad hand to rotate the pt's torso toward the left and uses a rotational thrust at the end of the pt's exhalation.

what is the Q angle? When do you see an increase in Q angle?

The Q angle is the angle formed from the ASIS to patella And Patella to tibial tubercle Q angle is seen to be increased in genu valgum, increased femoral anteversion, external tibial torsion, laterally positioned tibial tuberosity, and tight lateral retinaculum

With dorsiflexion of the ankle, the tibia and fibula externally rotate while the talus glides in what direction?

The Talus glides posteriorly The Talus articulates superiorly with the ankle mortise (tibia and fibula), and inferiorly with the calcaneus. The superior surface of the talus is convex from front to back. the dome of the talus is trapezoidal in shape, and its anterior surface is wider than the posterior surface. Therefore with dorsiflexion of the ankle, the talus would glide posteriorly while the fibula externally rotates. Dorsiflexion with posterior glide is a more stable joint position because the wedge-shaped talus becomes engaged.

the alpha subunit of what hormones are similar?

The alpha subunit of TSH-FSH-LH AND B-HCG Some conditions that carry elevated HcG levels (i.e testicular cancer) can produce symptoms of other hormones. reported cases include hyperthyroidism through TSH, AND GYNECOMASTIA THROUGH LH. Patients with nonseminomatous testicular cancers can have markedly elevated levels of hCG. Due to hormone subunit homology, there is some cross-reactivity as mentioned above.

when you say you note a therapeutic pulse -it means that it has a frequency at a rate-consistent with patient's heart rate

true

Von gierke disease is a glycogen storage disease resulting in abnormal glycogen metabolism and an accumulation of glycogen within cells. Von Gierke disease that results in severe fasting hypoglycemia, increased glycogen in liver =, increased fasting hypoglycemia, increased blood lactate and hepatomegaly. glycogen-6-phosphatase deficiency. reverse with frequent oral glucose/cornstarch. true or false?

True

When a patient refuses treatments, or make decisions that are not in their best interest, physicians should ensure that the patient is capable of understanding the risks, benefits, and consequences of this decision. This is called assessing a patient's capacity true or false?

True

deficiency of the final complements of the complement patjhways, c5-c9 increases the risk of neisseria infections.

True

Local anesthetics are weak bases that are easily protonated in an acidic environment. While it is the cationic (protonated) species that block sodium channels, only the non-protonated or neutral species can cross cell membranes to gain access to the site of their activity. Carbon dioxide has a direct depressant effect on neurons and, in addition, as carbon dioxide diffuses across the membrane the intracellular pH will be decreased, protonating and trapping the anesthetics within the cell. what is the effect of this last step?

This increases the concentration of the cationic species inside the cell, increasing binding rate.

drug of choice for urinary tract infection during pregnancy is amoxicillin true or false?

True

grand canyon is in southwestern United States true or false?

True

The shafts of ribs 6-8 DO NOT MOVE upward with inhalation. EKG doen in office shows normal sinus rhythm with no T wave changes appreciated. which of the following is correct regarding this patient's diagnosis?

Treatment with muscle energy would include encouraging inhalation of rib 6 using accessory muscles of respiration. it does not move upward with inhalation so you want to encourage inhalation.

what is one of the most common adverse effect of lithium?

Tremor-mild tremor of the hand. thyroid problems are frequently associated with thoracic lesions, the most common of which is a flexion lesion of T2. lithium toxicity manifests as hypothyroidism, edema, and acne.

la crosse virus, a member of the bunyavirus family, causes california encephalitis characterized by mental status changes, neurological defects and fever.

True

telephone consent is just as valid as written consent and should be obtained when necessary true or false?

True

the blood that passes through the foramen ovale to be taken through the brain and coronary circulation is about 65% saturated with oxygen.

True

nerve distribution of the carpal tunnel?

The hallmark of carpal tunnel syndrome is pain or paresthesias in a distribution that includes the median nerve territory, with involvement of the first three digits and the radial half of the fourth digit. note the thenar eminence is innervated by the palmar cutanenous sensory branch of the median nerve. it is impt to note that this nerve comes off the median nerve just proximal to the carpal tunnel. since the palmar branch of the median nerve passes over, rather than through, the carpal tunnel, sensation change (pain or numbness) involving the thenar eminence is typically not observed in carpal tunnel syndrome.

Patients in respiratory distress should be intubatedm and placed on mechanical ventilation unless otherwise stated in an advance directive or confirmed by the medical power of attorney. true or false

True

Patients with chronic rheumatic heart disease have an increased risk of developing atrial fibrillation due to left atrial enlargement from longstanding mitral stenosis T/F?

True

The barrier the patient meets with active range of motion is?

The barrier the patient meets with active range of motion is the physiologic barrier. In a joint with somatic dysfunction, another barrier can be met. This barrier is referred to as a pathologic or restrictive barrier.

where is the chapman point for the larynx located?

The chapman point for the larynx is located on the second rib near the costochondral junction. larngitis signs include a hoarse voice and erythema of the posterior pharynx. laryngitis is most often of viral origin with bilateral cervical lymphadenopathy indicating an infectious process.

the portal triad housed within the hepatoduodenal ligament, consists of?

The common bile duct, the hepatic artery and the portal vein. The hepatic features of acute hepatitis includes enlarged reddened liver (green if cholestatic), hepatocyte swelling/necrosis, lobular disarray with regenerative changes, accumulation of debris in Kupfer cells, and an influx of mononuclear cells into the sinusoids causing inflammation of the portal tract. the main histologic feature of chronic hepatitis includes inflammation and fibrosis of the portal tracts with spillover into the adjacent parenchyma. chronic hepatitis B virus (HBV) infection will display ground-glass hepatocytes due to the accumulation of hepatitis B surface antigen. chronic hepatitis C virus (HCV) infection displays bile duct epithelial cell proliferation and lymphoid aggregate formation.

The most important superantigen of S. aureus is the toxic shock syndrome toxin-1 (TSST-1), which is the exotoxin responsible for toxic shock syndrome (TSS). rather than binding to a specific T-cell receptor, superantigens activate T-cells non specifically by binding outside of the CD4 T cell MHC II antigen groove. This causes elevated levels of interferon-y (IFN-Y) and interleukin-2 (IL-2). Typical manifestations include fever, rash, shock, desquamation, and end organ failure. this condition can occur in females using super absorbent tampons during menses or patients using nasal packing for a nosebleed. How is this toxin different from exfoliative toxin produced by staphlococcus aureus? what is the other disease caused by staph exfoliative toxin? how is this different from toxic epidermal necrolysis?

The exfoliative toxin produced by staph aureus in staphlococcal scalded skin syndrome does not act like a superantigen. this exfoliative toxin-causes epidermal desquamation of superficial intraepidermal layers of skin. it acts on the zona granulosa of the epidermis causing cleavage of desmoglein 1 complex,a protein in desmosomes that anchor keratinocytes together. Exfoliative toxin is produced in a focal lesion, spreads hematogenously, and thus the bullae are sterile. Staph exfoliative toxin also causes bullous impetigo. (not to be confused with impetigo with honey-crusted lesion, caused by strep species). bullous impetigo is a localized forma of SSSS where the patient has fluid-filled blisters that contain many bacteria. bullous impetigo does no spread hematogenously in bullous impetigo. Toxic epidermal necrolysis which is toxin-mediated destruction of epidermal-dermal junction.

30 yo G1P0 woman gives birth to a healthy baby boy. Based on the neonate's blood type, her obstetrician administered Rh(D) immunoglobulins (RhoGAM) at 28 weeks gestation and after delivery. which of the following is the most accurate statement concerning the treatment of the mother wITH RhoGAM?

The father is Rh (D) positve. RhoGAM is given to Rh (D) negative mothers when they give birth to Rh (D) positive babies. this mother was G1P0 indicating that she is presently pregnant without prior term, preterm, abortion, or live deliveries. Therefore, the mother must be Rh (D) negative because there is not an indication to provide RhoGAM to an Rh+ mother. this baby must have inherited the gene for the Rh-antigen from his fathehr, thus the father should be Rh (D) positive. The Rhesus factor is inherited in a typical mendelian fashion. fathers who are heterozygous have a 50% chance of passing the gene and fathers who are homozygotes have a 100% chance of passing the gene. A hemolytic disease is caused by presensitized antibodies in the Rh- mother attacking the Rh+ RBCs of the fetus. RhoGAM is given to Rh(D) negative mothers when they give birth to Rh (D) positive babies. it protects against hemolytic disease int he next Rh(D) positive baby. it is given at 28 weeks and postpartum.

HOW Do you differentiate bioterrorism from bacteria is it is caused by Yersinia pestis verus Francicella tularemia?

The high morbidity, mortality and transmissibility of the pneumonic bubonic plaque make it an excellent bio-weapon of choice. Yersinia pestis is highly virulent and is easily transmissible by aerolized droplets. The onset of severe myalgias, high fevers, headaches, and pronounced painful lymphadenopathy (in the form of buboes), disseminated intravascular coagulation, and necrosis of the fingertips and the tip of the nose (in its later septicemic stages).. the outcome of the respiratory manifestation of the plaque is often fatal. are the characteristic signs of illness. The incubation period of Yersinia pestis is 1-6 days and a highly virulent pneumonia or meningitis may develop as a result. the high respiratory virulence of this organism makes it desirable to use in bioterrorism attacks. Y. pestis organism are quite hardy. they can remain viable for one hour when outside the body and are easily dispersed for distances up to 10 kilometers. Francisella tularensis is similar in many ways to the plaque, and has been studied on its own merits for bio-weaponization; however, its manifestations are not quite as lethal and pronounced as the plaque. The patients in the question stem were presenting with very serious signs of septicemia and necrosis while F. tularensis manifests more with pharyngitis, conjunctivitis, pleuritis, fever, fatigue, chills, headache, and malaise. iT ALSO HAS A SLIGHTLY longer incubation period than the plaque. if the bomb did contain F. tularensis, the influx of patients would have started in 1-14 days.

structural exam reveals a positive standing flexion test on the left as well as a positive seated flexion test on the left. The patient's right ASIS is more superior compared to the right. which of the following findings would you expect on this patient? in terms of the relation of the left pubic ramus compared to the right?

The left pubic ramus is inferior relative to the right. If there is a positive left standing flexion test, eliminate all answers that have right in them and vice versa.

what structure separates the upper GI tract from the lower GI tract?

The ligament of Treitz is the demarcation between upper GI bleeds and lower GI bleeds. upper GI includes everything from the mouth to the duodenum. lower GI includes everything from the jejunum to the anus. hematemesis=vomiting of blood melena=dark sticky feces containing blood what nerve viscerosomatic level is associates with upper GI? t5-t9

Physical exam reveals the right transverse process of T12 is posterior. the right transverse process of T12 becomes more posterior in the sphinx position. the most likely diagnosis is ?

The posterior transverse process on the right indicates that T12 is rotated to the right. iN THIS CASE, T12 is rotated right in both the neutral and extended positions so it can be inferred that the ease of motion is flexed. Remember that when performing flexion/extension motion testing the position that creates more neutral mecahnic would be the ease position and thus the correct diagnosis. According to Freyette's second principle, rotation and sidebending occur in the same direction when the dysfunction is in the non-neutral plane. Since this dysfunction is flexed, sidebending must also be to the right. somatic dysfunction diagnosis is named for the way a segment is statically found, likes to go or becomes more symmetric.

what does pregnancy category D which phenytoin falls under mean? what are the other categories list them in order?

There is evidence of fetal risk, but potential benefits from use of drug in pregnancy may be acceptable despite its potential risks. PREGNANCY CATEGORIES Category A-studied in pregnant women and have been shown to pose minimal risk to first trimester fetuses with no evidence concerning later trimesters. Category B-medications have not been shown to pose significant risk to fetuses, but there are no adequate human studies Category C-Medications have been studied in animals and have shown adverse effects without adequate studies in humans. Category D-There is evidence of fetal risk, but potential benefits from use of drug in pregnancy may be acceptable despite its potential risks. Category X-Have been shown in either animal or human studies to cause adverse fetal effects. risks outweigh benefits of the medication, and it should not be prescribed.

Patients with prinzmetal angina are often younger with few cardiovascular risk factors, except cigarette smoking?

True

Physicians are responsible for both protecting patients from an impares physician and reporting physician to the appropriate authority.

True

What is the form of inheritance of Duchenne muscular dystrophy, G6PD, fanconi anemia, fragile X syndrome, huntington disease and sickle cell disease. An african american person comes in with fatigue and jaundice, the symptoms has occurred in the past after taking NSAIDs or trimethoprim/sulfamethoxazole. urinalysis reveals dark urine, and significantly increased bilirubin. physical exam reveals splenomegaly. what does this person have and the inheritance of what above is similar?

This person has G6PD deficiency. The inheritance of G6PD deficiency is similar to the inheritance of Duchenne muscular dystrophy both of which are X-linked. Fanconi is autosomal recessive. HUNTINGTON is AUTOSOMAL DOMINANT FRAGILE X syndrome is X-linked dominant Sickle cell is autosomal recessive. G6PD dehydrogenase is an enzyme in the pentose phosphate pathway that together with co-enzyme nicotinamide adenine dinucleotide phosphate (NADPH) is responsible for protecting erythrocytes from oxidizing free radicals that causes hemolysis to red blood cells. Although many individuals with G6PD deficiency are asymptomatic, those with this disease can have episodic intravascular hemolysis secondary to factors that induce oxidative stress such as infections, consumption of fava beans, diabetic ketoacidosis, and various medications including aspirin, NSAIDs, many antibiotics like sulfa drugs. G6PD deficiency manifests clinically with jaundice, fatigue, dark urine, and splenomegaly. These episodes of hemolysis can last anywhere from 8-14 days and self-resolve with the production of new erythrocytes that contain high levels of G6PD. G6PD deficiency is an x-linked recessive disease that is common in African Americans, aisans and those of mediterranean descent. autosomal DOMINANT is due to structural problems Autosomal recessive due to enzyme problems. that did not apply here though. so just a general rule with some exceptions.

What thoracic outlet syndrome will be positive for the tests below 1- between anterior and middle scalenes 2-between the clavicle and first rib 3-between the pectoralis minor and upper rib

Thoracic outlet syndrome is a compression of the neurovascular bundle as it exits the thoracic outlet. the compression can occur in three separate location: name where this locations are for the tests below 1-Positive adson's test 2-positive military posture test 3-positive wright's test

what is threatened abortion?

Threatened abortion presents as uterine painless bleeding the first 20 weeks gestation with a closed cervix and ultrasound will show a viable fetus or embryo. a pt with a threatened abortion should be advised to limit her activity or should be put on bed rest, especially in this patient with previous abortions. if a person with threatened abortion goes on bed rest and comes back a week after with continued bleeding, and now she complains of pelvic pain. ultrasound reveals an intrauterine fetus and pelvic exa, shows dilated cervix this is the 12th week of gestation. this pt now has?=Inevitable abortion=present with uterine bleeding and pain, detection of fetus on ultrasound, and an open cervix. sINCE abortion is now inevitable, she should be treated with misoprostol or dilation and curettage to remove all uterine contents.

explain thyroid storm

Thyroid storm is a rare, lifethreatening condition presenting with thermoregulatory dysfunction (usually fever over 101F), central nervous system effects (ranging from agitation to coma), gastrointestinal dysfunction (ranging from diarrhea to jaundice), cardiovascular dysfunction (tachycardia up to 140/min), and heart failure (ranging from lower extremity edema to severe pulmonary edema). how pt will present heart palpitations and chest pain. weight loss, diarrhea, increased sweating over the past few weeks. fever, tachycardia, high bp, PE- diaphoresis, proptosis, a wide pulse pressure and hyperreflexia. thyroid storm is a stste of excess release of hyroid hormone and it commonly associatd with graves disease. thyroid strom causes hyperension and tachycardia, and it may lead to heart failure, arrhythmia, and death if it is allowed to progress. graves disease=anti-tsh receptor antibodies other causes of thyroid storm include increased intake of thyroid hormone and poor management during thyroid surgery thyrotoxicosis may also present with fever, diarrhea, fatigue, diaphoresis, and anxiety. elevated alkaline phosphatase may also be seen due to increased bone turnover. this is similar to pheochromocytoma but pheochromocytoma is not associated with proptosis.

What is REM Latency?

Time between falling asleep and the start of REM SLEEP people with narcolepsy have a significantly decreased REM latency and commonly enter REM sleep within a few minutes of falling asleep. Some narcoleptic patients experience sudden and uncontrollable onset of REM sleep from active, waking state.

how do you treat anterior fibular head?

To treat anterior fibular head, the patient's foot is inverted and plantar flexed by the physician to the restrictive barrier. The patient then attempts to dorsiflex the foot against the physicians counterforce. true or false?-True.

Tourette's disorder is characterized by the presence of both motor tics and one or more vocal tic. the disorder is treated with second generation dopamine antagonist which act on the D2 Receptor-MEDICATIONS LIKE RISPERIDONE OR IN SEVERE CASES first generation antispsychotic medications such as fluphenazine or haloperidol. alpha agonist like clonidine and guanfacine for patients with tic and impulse control problem. fluoxetine for patients with tic and obsessive compulsive symptoms, and habits reversal training for milder cases of tics.

Tourette's disorder is characterized by the presence of both multiple motor tics and one or more vocal tic.

what do you treat gram negative anaerobic rods with?

Treat gram-negative anaerobic rods with either metronidazole or clindamycin. note ceftriaxone does not cover gram negative rods. if you do not see clindamycin but you see metronidazole for a anaerobic treatment-go ahead and pick it.

A young male with a long hx of chronic diarrhea, a new episode of rectal bleeding, and lack of other risk factors. And a positive family hx of colon cancer likely has inflammatory bowel disease. If the disease is confined to the rectum and involved only the mucosa, the most likely diagnosis is ulcerative colitis. what do you treat with?

Treat with 5-AMINOSALICYLATE (5-asa)

Calcium channel blockers-to reduce coronary vascular smooth muscle tone.

Treatment for Prinzmetal angina

Pityriasis Rosea is a self-limited exanthematous reaction that with initial herald patch that spreads into a diffuse rash with a "christmas tree" pattern.

True

What is the triangular space bound by? what is contained within the triangular space? What is the quadrangular space bound by? What is contained within the quadrangular space? what it is the triangular interval bound by and what does it contain?

Triangular space is bound by the teres minor, teres major and long head of the triceps Within it i the circumflex scapular artery Quadrangular space is bound by tests minor and major and the lateral/long head of the triceps muscle quadrangular space contains axillary nerve and the posterior humeral circumflex artery Triangular interval is bound by the teres major and the lateral/long head of the tricep muscle. It is bound by the profunda brachii (deep brachial artery and the radial nerve.

what should you suspect in an african american child who presents with patchy hair loss leaving balck dots on the scalp?

Trichophyton tonsurans Black dot tinea capitis, most commonly seen in the african american-population, presents usually in childhood but can occur in adulthood as well. The fungus preferentially colonizes and invades hair shafts, causing eventual weakness and breakage at the scalp. The black dots seen in this type of tinea capitis are casued by debris remaining in the hair follicle. Interestingly, unlike other causes of tinea elsewhere on the body, Trichophyton tonsurans does not fluorescence under ultraviolet light (wood lamp). DIAGNOSIS IS USUALLY MADE BY CLINICAL presentation in addition to KOH examination of affected hair shafts which should reveal hyphae and fungal spores. if confirmation of the specific organism is needed, fungal culture on saboraud dextrose agar can be performed. Treatment is with oral griseofulvin or terbinafine.

cARCINOID SYNDROME IS ASSOCIATED WITH RIGHT-SIDED VALVULAR DEFECTS LIKE?

Tricupsid regurgitation Reason is because the tricupsid valve is stenosed by serotonin action.

History reveals a grade 3 holosystolic murmur heard best at the left sternal border. the auscultatory findings are most consistent with ?

Tricupsid regurgitation so this is based on the description of where the murmur is located. TRICUPSIS regurgitation causes a murmur heard best at the left lower sternal border that is holosystolic. It causes peripheral edema, jugular venous distention, ascites, and right atrial enlargement. rheumatic fever is the most common cause of tricupsid regurgitation from deformation of the valve leaflets. Tricupsid regurgitation casues a murmur that is heard best at the tricupsid listening area, at the lower left sternal border. it radiated to the right sternal border. it is a holosystolic murmur described as a high-pitched blowing murmur. Tricupsid regurgitation increases in intensity with inspiration because of increased blood flow through the right side of the heart.

diabetes mellitus, swellign of his ring finger. history of locking and catching of the finger for a long period of time. physical exam reveals swelling over the distal palmar crease in line with the head of the 4th metacarpal. physician will intend to treat with myofascial release, muscle energy and counterstrain. the patient's most likely diagnosis is ?

Trigger finger. trigger finger is idiopathic but more common in diabetics. oMM TECHNIQUE is geared to reduce pain and release myotendinous fascial release, muscle energy and counterstrain. Trigger finger is a stenosing flexor tenosynovitis of the flexor tendons that are located over the distal palmar crease at the level of the metacarpal heads, it causes a catching or locking of the finger when it is actively flexed and extended.

Acute tubular necrosis is commonly caused by aminoglycosides or other antibiotics. other offending substances include ethylene glycol, heavy metals, and urate. pts will have a BUN:Cr<15, hyperkalemia, inability to concentrate urine, and brown tubular casts in the urine. true or false?

True

Acyclovir is a purine analog used in the treatment of HSV-2. It functions by inhibiting viral DNA polymerase true or false?

True

After 34 weeks, treatment for severe preclampsia involves IV magnesium sulfate as prophylaxis for eclamptic seizures and immediate delivery of the fetus true or false>

True

Autosomal dominant polycystic kidney disease (ADPKD) is caused by mutations in PKD1 and PKD2, which result in abnormal levels of ploycystin-1 and polycystin-1. the most common cause of death in patients with ADPKD is cardiac disease, however, th emost serious extrarenal complication are intracranial berry aneurysm.

True

C5-C6 is the most common disc herniation in the neck, affecting C6 nerve root, which provides sensory innervation to the thumb and first finger and motor innervation to wrist extensors and elbow flexors. true or false?

True

Iliotibial band syndrome can be a common problem in long distance tunners and can be treated using myofascial release with the patient in lateral recumbent position and the physician engaging and applying pressure to the patient's iliotibial band. true or false?

True

In forward sacral torsion, the rotation is toward the SAME as the axis

True

Leuprolide is a GnRH analog used to shrink fibroid tumors true or false?

True

Low intracellular pH and high extracellular pH increase the effectiveness of loca anesthetics-True or false?

True

Meniere's disease is due to increased pressure within the inner ear endolymphatic system pts present with fluctuating hearing loss, episodic tinnitus and the sense of aural fullness-true or false?

True

NF1 is on chromosome 17 and it is a tumor suppressor gene

True

NSAIDS and RICE (restm ice, compression, and elevation) are the initial management to be taken for acute meniscal injuries?

True

visual field defects are always described in terms of the filed defect from the patient's view. (nasal, temporal, superior, and inferior). the retinal image of an object in the visual field is inverted and reversed from right to left, like the image on the film of a camera. Thus the left visual field of each retina is represented in the opposite half of each retina, with the upper part of the field represented in the lower part of the retina. visual field defect resulting from occlusion of the PCA includes a contralateral hemianopsia, which can have macular sparing true or false?

True

while ectopic pregnancy must be considered in a woman presenting with history of IUD placement AND A POSITIVE pregnancy Test, an IUD will actually lower the overall incidence of ectopic pregnancy T/F?

True

Aspirin toxicity presents as GI distress, tinnitus, and hyperthermia. it also causes elevated leukotrienes, which could lead to bronchospasms true or false?

True A variety of acid-base disturbances can occur. most adults will have a primary respiratory alkalosis or a mixed primary respiratoty alkalosis and primary metabolic acidsosis. pts will present with an abnormally high anion gap [(Na)-(cl)+(hco3). treatment includes intravenous sodium bicarb, hemodilaysis, and volume resuscitation.

True or false -granuloma annulare is a skin condition that mimics other skin conditions with the exception that granuloma annulare is described as papules or plaques arranged in a circle or ring, not vesicles. remember that granuloma annulare is usually asymptomatic and can persist for many years before regression.

True Granuloma annulare-the papules and/or plaques are usually asymptomatic, but may be described as pruritic or burning. the cause of granuloma annulare is clustering of T cells below the skin, which eventually rise close enough to the skin's surface to be observed as small papules. although granuloma annulare is seen in healthy people, it is also associated with diabetes, thyroid disease, lupus, and other autoimmune diseases. the most common sites of papules eruption are the backs of the forearms, hands and feet.

Onchocercerciasis, also known as river blindness, presents 9-12 months after being infected with the parasite by the bite of a black fly and can lead to irreversible blindness true or false?

True I did not know what Iridocyclitis meant. iridocyclitis means basically irritation of the eye that can lead to redness The patient is most likely suffering from onchocerciasis, which is an infection caused by Onchocerca volvus. THis parasite is transmitted by the black fly, which is found in Africa, Yemen, and parts of south america, the majority of cases are found in the continent of africa and it is the second leading infectious cause of blindness worldwide (after trachoma caused by chlamydia trachomatis serotypes A-C). when a human is bitten by the balck fly, the fly transmits the parasitic larvae into the bloodstream of the human. After infection, the larvae grow to adult worms within the bloodstream and lymphatics. The adults embed into the skin forming subcutaneous nodules and bear microfilarial offspring which invade the eye and its inner chambers. overall, these offspring cause eye pain, redness, iritis, and anterior uveitis in the patient. These microfilarial worms disseminate throughout the body, causing an allergic-type reaction and a skin condition known as Onchodermatitis. This disease is also known as river blindness because the black flies live near the rivers and the native tribes realized that the individuals who went down to the rivers were the only people in the village who would eventually go blind. The treatment of choice for this disease is anantiparasitic ivermectin. Do not confuse this with Loiasis which is an infection with the parasite Loa loa. it can cause swelling of the skin and presence of subcutaneous worms. It infects the conjunctiva of the eye, in which worms are visible. While it does specifically affect the eye, loa Loa rarely leads to blindness. it is transmitted by chrypsops flies ( including the deer fly, horse fly, and mango fly). The treatment of choice is diethylcarbamazine.

when performing BLT: disengages the segment and take it into the position of ease. use the pt's breath or position as an activating force and hold it until a release is felt. true or false?

True complain of shoulder pain. A pitcher, glenohumeral joint is internally rotated. so you stand on his right side facing him and grasp his upper arm under the axilla with both hands. you want to disengage the humeral head by applying lateral and slightly inferior traction. in a balanced ligamentous tension technique what position would you put the patinet in for treatment? ---Right hand reaching across the chest toward the left clavicle (hand on heart flag salute position). reaching the right hand across toward the left clavicle causes internal rotation f the glenohumeral joint.

Sodium bicarbonate is the antidote in TCA overdose with cardiac involvement

True 25 yo female recovering from a seizure. history reveals major depresssive disorders treated with an unknown medication- she has dilated pupils, tachycardia, and dry mucous membranes. you note hypertonicity and tenderness in the paravertebral musculature from T1-4. electrocardiography reveals tachycardia with 122 msec QRS complex. the most appropriate pharmacological therapy for her overdose is===sodium bicarb TCA has a lot of various effects-inhibition of presynaptic neurotransmitter reuptake ofnorepinephrine and serotonin, antagonism of histamine1 receptors, antagonism of GABAA receptors, antagonism of peripheral alpha-1 adrenergic receptors, antagonism of central and peripheral muscarinic acetylcholine receptors and the blockade of cardiac fast sodium channels.

picture of a tubular adenoma,. tubular adenoma has a characteristic stalk attached to the colon. and they are the most common type of polyp discovered on a colonoscopy. true or false?

True Adenomas are associated with an increased risk of colorectal cancer since they can undergo a transition to malignant polyps. villious polyps have the most malignant potential and are relatively broad in their attachement to the colonic mucosa when compared to tubular adenoma-they also do not attach through a stalk.

Gemfibrozil and other fibrates enhance the activity of lipoprotein lipase, leading to decreased serum triglyceride levels true or false?

True Gemfibrozil and other fibrates, are effective in treatment of hypertriglyceridemia. It enhances the activity of lipoprotein lipase, which hydrolyzes fatty acids from VERY LOW DENSITY LIPOPROTEIN (vldl). it promotes that uptake of the fatty acids by adipose tissue and increases their conversion into triglycerides. These triglycerides stay in the adipose tissue and do not circulate in the bloodstream.

Patellofermoral pain syndrome (aka) RUNNER'S knee typically presents with anterior knee pain that worsens with prolonged sitting or with ambulation up/down stairs. look for the J sign when suspecting this diagnosis.

True J -sign is named for the course of the patella tracking in the joint with contraction of the quadriceps.

Lithium is the first line drug for bipolar disorder, concomitant use of NSAIDs could lead to toxic levels of lithium to accumulate. true or false?

True Lithium is the first line mood stabilizing drug for bipolar disorder. it is distributed to the CNS and inhibits norepinephrine release while increasing serotonin synthesis. NSAIDS ARE NOT SELECTIVE IN THIER INHIBITORY PATHWAY, and other organ systems may be affected, int he kidneys, prostaglandins cause arteriolar dilation and increased blood flow to the glomerulus. A decrease in prostaglandins from NSAIDS usage would cause a decrease in blood flow to the glomerulus and a fall in the glomerular filtration rate (GFR). the kidneys in reaction to a decreased in blood flow, would increase absorption of sodium and other electrolytes in an attempt to increase intracellular volume and maintain glomerular perfusion pressure. in the proximal tubule, lithium is handled similarly to sodium and would be reabsorbed just as avidly if the kidneys were attempting to retain fluid. Such a situation could lead to lithium toxicity, and it is advised that NSAIDS be used cautiously if at all with patients on lithium therapy.

Constipation, miosis, NAUSEA, and decreased respiratory rate are common side effects of opioids true or false?

True NAUSEA can hinder the initiation of an opioid regimen and can usually be given with an antiemetic such as ondansetron until the pt builds tolerance.

risk factors of preclampsia include Nulliprity, age>40 years, chronic pre-existing htn, or renal disease, pregestational or gestational diabetes mellitus, obesity, antiphospholipid syndrome, or a twin gestation. True or false is smoking a risk factor for preeclampsia?

True No smoking is not a risk factor it actually lowers the risk.

most posterior cervical spine midline points are treated in an extension, sidebend away rotate away (E Sa Ra) Position. PC1 INION HOWEVER IS A MAVERICK (treated differently) TENDER POINT. true or false? how is PC3 treated?

True PC3 midline is treated differently also =(F Sa Ra)

medial ankle sprains of the deltoid ligament group, though much less common than lateral ankle injuries, are extremely painful. many times the individual will hear a pop and significant swellign will occur with an inability to walk.

True ankle roll, edema and tenderness anterior and inferior to the medial malleolus. plantar flexion of the ankle exacerbated the pain and there is great difficulty in walking-the structure most likely to be injured is-tibiocalcalneal ligament.

Stokes involving the anterior cerebral artery cause weakness in the contralaeral lower extremity more than the contralateral upper extremity?

True anterior cerebral artery supplies the inferior frontal and parasagittal regions of the frontal and parasagittal regions of the frontal and anterior parietal lobes. impt in movement of the lower limb and bowel/bladder control bladder control traverses from the frontal lobes through the pons and the entire length of the sponal cord (any major cns injury is likely to affect bladder control). --urinary involvement is due to the involvement of the frontal micturition center.

CHOLELITHIASIS, or the presence of stones in the gallbladder, may lead to complications such as cholecystitis (gallbladder inflammation) and choledocholithiasis (gallstone in the common bile duct). choledocholithiasis may then lead to acute pancreatitis, which will present with elevated pancreatic enzymes and pain radiating to the back. true or false?

True be able to keep in what what the most common diagnosis is and the differentiate it from similar presentations of lesser commons on the comlex. this requires paying attn to the details of the pt presentation, carefully considering what is mentioned, in the stem and being careful to avoid assuming, and filling in the blanks when you're thinking about choosing a less common disease process which may sound or present in a similar manner. there will be many potential choices present, make sure you pick the best one for the given presentation.

Ependymomas are CNS tumors that arise within the roof of the fourth ventricle in children. A histological examination will reveal the presence of perivascular pseudorosettes and blepharoplasts. true or false?

True ependymoma can cause an obsructing hydrocephalus that impedes CSF flow through the foramina of luscka and Magendie. physica exam will show papilledema a histological exam will reveal perivascular pseudorosettes, which are cells that have peripherally located nuclei and form halo around an empty eosinophilic lumen. Belpharoplasts, which are rod-shaped intracytoplasmic eosinophilic inclusions, can also be found near the nucleus.

the treatment for pre-hypertension is lifestyle modification, which can include exercise, dietary changes, moderate alcohol intake, and qutting smoking, medications are not indicated. true or false?

True htn stages systolic /diastolic 120-139 /80-89=pre htn 140-159/90-99=stage 1 htn >160/>100=stage 2 htn Appropriate medication for someone with prehtn include exercise, a low-fat, low cholesterol, low salt diet, or quitting smoking. treatment for htn varies depending on the stage and other co-morbidities. for example first-line treatment for htn is generally hydrochlorothiazide, however, if a pt also has diabetes, an ACE-inhibitor is better choice due to its concurrent benefit of preventing diabetic nephropathy.

benign prostatic hyperplasia involves proliferation of all tissue elements of the prostate gland. growth is more pronounced in the region closest to the urethra true or false?

True hyperplasia occurs in both the glandular and stromal elements.

oNE OF THE most common causes of secondary thrombocytosis is infection. patient presentation is not unique and the course of this condition is often benign. lab results will demonstrate an isolated elevation of platelets

True infection (influenza or other viral infection) is the most common cause of reactive thrombocytosis.

In cases of hypokalemia refractory to oral potassium, check serum magnesium levels. 42% of hypokalemic pts also have an underlying hypomagnesemia that leads to potassium wasting in the urine. Hypomagnesemia can also lead to hypocalcemia

True magnesium is a cofactor in potassium channel function within the kidney, acting to close these channels in the absence of aldosterone. iN THE CASES OF low serum magnesium, these channels remain open in the collecting duct, and potasssium is wasted in the urine. hypokalemia often presents with weakness, rhabdomyolysis, cardiac arrhythmias, renal abnormalitiesm and glucose intolerance. most common causess are urinary losses, diarhea, diuretic therapy may also result from entry of potassiun into cells=redistributive hypokalemia.

True or false muscarinic agonist can decrease intraocular pressure? what does treatment for glaucoma involve?

True muscarinic activation and beta blocking. Glaucoma, most commonly open-angle is an optic neuropathy characterized by progressive peripheral vision loss followed by central vision loss. glaucoma is usually, but not always associated with elevated intraocular pressure (IOP) either due to decreased outflow of aqueous humor or increased production. risk factors include diabetes, age, african americans, famhx, and elevated IOP. An ophthalmic exam will reveal cupping of the optic disc (hollowed-out appearance with a thin or notched disc rim and peripheral visual field defects. Muscarinic agonist such as pilocarpine, are helpful because they contract the ciliary muscle and help increase the outflow of aqueous humor at the canal of schlemm here cranial nerve V1 (ophthalmic division of the trigeminal nerve) is targeted for treatment with the history of blurry vision. this nerve passes through the cavernous sinus, then enters the orbital fissure. treatment is directed toward the superior orbital foramina.

myotonic dystrophy is often recognized by its sustained muscular contractions. it exhibits anticipation through a ctg TRI-NUCLEOTIDE REPEAT expansion that is linked to instability during maternal meiosis true or false>

True myoTonic Dystrophy =CTG repeat, it is autosomal dominant. both men and women will demonstrate varied effects: weakness of distal muscles, cardiac and cataract complications, indifferent personality and myotonia (prolonged contraction, i.e. sustained grip). other tri-nucleotide repeats exhibit anticipation, but they differ in their source of the repeat expansion.

Endometriosis is characterized by cyclic pelvic pain around the time of menses and dyspareunia. it most commonly affects the ovaries, rectal pouch of douglas and fallopian tube.

True note examination of the uterus would be necessary to rule out endometritis.

Parkinson disease-parkinson usually starts with an asymmetric tremor, meaning it is found on only one side of the body, and is most frequently described as resting tremor of low frequency. other common features of parkinson disease are gait abnormalitty, particularly shuffling gait and impaired arm swing, resting tremor, and blank facial expression (blank facies)

True note when taking the test and you have an urge to change your answer to something else-then follow that and change your answer. or go with your first gut it is always not going to let you down.

Prior to placing an automatic implantable cardioverter defibrilator (AICD) is recommended that the physician discuss under what conditios the patient might want his device deactivated true or false?

True pURPOSE of AICD is to monitor the heart for an arrhythmia, and to provide an electrical shock should one occur to make the heart beat normally again. AICD are life-sustaining therapies, however, and prior to placement it is recommended that physicians discuss goals of care with the patient and document under what circumstances (stroke, dementia, incapacity, etc) the patient would like the AICD to be deactivated. this helps to serve as an advance directive and guide future end-of-life decision making.

PCL prevents posteior translatio0n of the tibia relative to the femur. the most common mecahnism for an isolated PCL injury is from a hyperflexion mechanism.

True pt heard a popping noise after sliding to third base really pay attn to the physical finding-when pt is asked to lie supine with hips flexed at 45 degrees and knees flexed at 90 degrres, the tibial tubercles seem to lie more posterior on the right than the left. when the pt then contracts the right quadriceps muscle against resistance, the tibia moves back into normal position. which of the following structures is most likely damaged?=posterior cruciate ligament Quadriceps activation test -checks for posterior cruciate ligament injury. PCL injuries occur most commonly in basebalk players

polyartheritis nodosa is a vasculitis characterized by inflammation affecting small to medium-sized, particularly affecting small to medium sized arteries particularly renal, cardiac, and gastrointestinal tract vessels. approximatley 30% of patients have been reported to have a histpry of a prior hepatitis B infection. true or false

True pulmonary vasculature is typically spared. cutaneous lesion may be a livedo reticularis (a lacy, mottled rash-an erythematous rash, purpura, gangrene, nodules, cutaneous infarcts, or raynaud phenomenion)

Ranson criteria can be used at admission and again at 48 hours to determine disease severity and predict mortality in acute pancreatitis. A blood glucose greater than 200 mg/dl is prognostic of increased mortality in this disease. true or false

True ranson criteria At admission -Age greater than 55 years -WBC COUNT greater than 16,000/m^3 -Blood glucose greater then 200mg/dl -serum LDH greater than 350 IU/l -Serum AST greater than 250 At 48 hours -decrease in hematocrit greater than 10% -Increase in BUN of greater than 8mg/l -Serum calcium less than 8mg/dl -PaO2 less than 60mm hg -Base deficit greater than 4 mEg/L -Estimated fluid sequestration greater than 600mL one point is assigned for the presence of each criterion. A totoal score of three or higher indicates a more severe case of acute pancreatitis and a higher mortality rate.

The discovery of an incidental mass during surgery presents ethical challenges; if there is a surrogate, then this individual should be approached for consent to minimize the risks engendered by multiple procedures true or false?

True so obtaon consent from the pt's husband and perform the biopsy.

Coagulase-positive, Beta-hemolytic staphlococcus aureus is a common cause of prosthetic valve acute endocarditis within two months of heart valve surgery.

True staph aureus is the earlies cause of it (almost exclusovely is also S. epidermis)

Patients with long-standing sickle cell disease commonly present with spleen dysfunction, putting them at high risk for streptococcus pneumoniae infections. true or false?

True strep pneumoniae is termed pneumococcal pneumoniae.

ACUTE PYELONEPHRITIS IS A SERIOUS INFECTION DURING PREGNANCY AND MUST BE TREATED AGGRESSIVELY. IT INCREASES the risk of pre-term labor and intrauterine infection. true or false?

True the diagnosis of acute pyelonephritis=wbc CASTS AND PYURIA are diagnostic findings. this is a serious condition in pregnancy. it can lead to pre-mature labor because toxins can cause early contractions. additionally there is a risk for intrauterine infection. pyelonephritis must be treated immediately and aggressively in pregnancy. the patient should be hospitalized and given intravenous antibiotics. antibiotic to use would be ceftriaxone or cefepime OR aztreonam OR ampicillin and gentamicin Aminoglycosides like gentamicin bind to the 30S ribosome and inhibits protein synthesis. (read question and answer choices well)

Theophyline has a narrow therapeutic level and can easily become toxic due to a medication interaction. some of its many toxic effects involving the gastrointestinal tract and include nausea, vomiting and diarrhea. true or false?

True theophylline works through an increase in catecholamines and blockade of adenosine. Adenosine blockade can theoretically reduce histamine release and indirectly reverse bronchospasm. -High level of theophylline inhibit phosphodiesterase, resulting in elevation of cyclic adenosine monophosphate (cAMP) and consequent adrenergic stimulation and subsequent bronchodilation. medications, diets, and underlying diseases can alter theophylline's already narrow therapeutic window. In particular, any medication (cimetidine, erythromycin, or ketoconazole) that inhibits the cytochrome p-450 system (CYP1A2) can prolong the half-life of theophylline since it is metabolized via cytochrome p-450 SYSTEM. This pt was likely prescribed ketoconazole for his tinea pedis infection. the ketoconazole inhibited the cytochrome p-450 system, resulting in theophylline toxicity. Acute theophylline toxicity usually causes predominately gastrointestinal disturbances. Individuals will complain of nausea, severe and protracted vomiting, diarrhea and abdominal pain. Due to diarrhea, individuals can also present with electrolyte disturbances including hypokalemia, hypophosphatemia and hypomagnesemia.

Zolpidem, sedative/hypnotic that targets GABA receptors, exhibits minimal daytime drowsiness and has a low tolerance when compared to benzodiazepines. True or false?

True use in someone having trouble falling asleep. --read question well-it says modulates GABA receptors with minimal daytime drowsiness and a low tolerance profile.

what manuever will intensity mitral valve prolapse?

True valsalva maneuver The midsystolivc click of mitral valve moves more toward S1 (and therefore becomes more prominent) with manuevers that decrease left ventricular volume, such as standing or the valsalva maneuver. On the other hand, the click moves more toward S2 (and therefore becomes less prominent) with maneuvers that increase left ventricular volume such as squatting and lying recumbent. In stark contrast to most other murmurs, MVP is enhanced by valsalva maneuvers and decreased by squatting. this is because those manuvers which decrease the volume of the left ventricle (valsalva, standing) will cause the prolapse to occur sooner and more severely, while those that increase venous return and diastolic filling (squatting) and thereby enhance the ventricular volume, help to maintain tension along the chordae and to keep the valve shut. tHE ONLY OTHER MURMUR which for similar reasons responds in this paradoxical ways to these common maneuvers is hypertrophic cardiomyopathy, also known as idiopathic hypertrophic subaortic stenosis.

true or false, one of the top eleven patient practices is to ask the patient to teach back?

True when you are not sure what an answer is flag it and come back to it because there is a chance of easier questions ahead you do not want to miss. .

True or false for inhaled anesthetis-high blood solubility will lead to an rapid uptake that can be altred with changes in respiratory rate?

True, specifically hypoventilation would slow the rate of uptake. volatile anesthetics are delivered as inhaled gases. their speed of uptake depends on three main factors: 1) blood-gas solubility 2) respiratory rate and 3) concentration of anesthetic in the lungs. it is impt to distinguish the difference btw uptake and onset of action. uptake implies systemic absorption and onset implies cerebral penetration and action. high blood gas solubility implies an anesthetic gas is very soluble in the blood and low blood-gas solubility implies it is very insoluble in the blood. iF A VOLATILE anesthetic is highly soluble, it will be rapidly taken up into the pulmonary arterial capillaries. the faster a volatile anesthetic is absorbed, the more dead space it creates in the lungs. in order to compensate for the dead space and increase the amount of volatile anesthetic in the lungs, an anesthesiologist can either increase the concentration of the volatile gas or increase the respiratory rate of the pt. Assuming a pt has no other changes in physiology other than a decrease in respiratory rate, hypoventilation would slow the rate of uptake of a highly soluble volatile anesthetic; hyperventilation would infact increase the rate of uptake of a highly soluble volatile anesthetic. insoluble anesthetic gases are already poorly absorbed into the pulmonary capillaries and uptake therefore not as affected by respiratory rate changes.

ESSENTIAL THROMBOCYTOPENIA IS A CHRONIC myeloproliferative disorder that is specific for megakaryocytes. patients will have an increased platelt count with normal red blood cell and white blood cell values. pts may complain of a headache, GI bleeds and/or extremity pain. However, 25%-33% of patients with essential thrombocytosis are asymptomatic and the only abnormal finding on physical exam may be mild splenomegaly. true or false?

True-do not be too lazy to pull open the lab values and compare them.

Elder neglect or a failure to provide what is needed to maintaon health or to protect one from harm is considered to be a form of elder abuse (also referred to as elder mistreatment). in this case, the son is controlling the patient's fiancies, her presentation should raise concern that he is not providing her with medications and food. the physician should suspect neglect. true or false?

True.

Mifepristone is both a glucocorticoid and progesterone antagnoist and is used with misoprostol (PGE1) to terminate a pregnancy. true or false?

True.

the gait associated with an upper motor neuron lesion is referred to as a spastic, scissoring gait. the patient's leg cross over each other with each step. this gait is also associated with tight hip abductors.

True.

Projection is a defense mechanim in which a person attributes to others unwanted feelings they are internally experiencing. tRUE or false?

True. a woman accusing her husband of having an affair is attracted to another man.

A unilateral lesion to the facial motor nucleus in the caudal pons display ipsilateral facial droop, including the forehead?

True. although not discussed, the pt may experience some upper motr nuron signs such as babinski reflexes, spasticity, hypertonicity, and increased deep tendon reflexes signs of lower motor neuron lesions includes fasiculations, decreased deep tendon reflexes and hypotonicity.

Subarachnoid hemorrahage is an emergency and requires prompt surgical intervention. medical management is directed toward liberal blood pressure control and the prevention of rebleeding and vasospasm. pts on warfarin with SAH require immediate reversal of their anticoagulation with fresh frozen plasma. true or false?

True. note when you see severe headache think subarachnoid headache,

Volvulus is most common in elderly patients who have a history of constipation and are debilitated or bedridden. a plain radiograph can reveal the diagnosis in about 60% of cases.

True. volvulus occurs when part of the bowel twists on its mesentery often resulting in bowel obstruction. in elderly sigmoid colon is the most common location for volvulus. eight decade is average age of volvulus. abdomen will be tympanic the image will show dilated sigmoid filling up most of the abdomen it has a characteristic coffee bean appearance due to the loop formed with torsion of the sigmoid colon. --treatment is aimed at detorsion of the volvulus to restore blood supply and relieve bowel obstruction. sigmoidoscopy is one method of detorsing the bowel. since recurrence is common, definitive surgical treatment is recommended.

pts with severe symptomatic aortic stenosis should undergo valve replacement to decrease mortality true or false?

True. name =percutaneous valve replacement

what should you do when you see really long question stems?

Try to go straight to reading the last sentence.

1 yo comes in with a tumor, abdominal pain and constipation. an abdominal mass is palpated in the right flank area. iN ADDITION TO recommended biopsy to rule out the underlyining pathology you wil also do OMM. on biopsy you find groups of cells arranged around central pink neural fibrils. this is most likely due to a ?

Tumor of the adrenal medulla =neuroblastoma. Neuroblastoma is a neural crest tumor of the adrenal medulla that ocurs in childhood. A urinalysis will reveal elevated catecholamines and homovanilic acid, which is a breakdown product of dopamine.

There is an increased risk of gastric cancer among people with what type of blood type?

Type A. --gastric cancer think gAstric cAncer -gastric ulcers are found more around the lesser curvature

A recent study published stating a newly developed drug has no benefit on cognitive recall in patients with alzheimer disease. the drug developed seeks to disprove the study results. it would be most beneficial for the drug develper to show that the study contained?

Type II error type II error in a data set is the probability of stating that there is not a significant relationship between two factors, when a relationship does exist. stated another way, it is the failure to accept the alternative hypothesis (HA) when in fact it is true. This drug developer would like to prove that there actually is a relationship btwn the drug and cognitive recall. Thus a high type II error could help the drug developer's argument that there was a significant effect of the new drug, and that the authors of the study failed to show statistical significance when in fact the results were significant. Power=1-type II error. A high value for the type II error would decrease the study's power, and help disprove the study's validity, which would be the goal of the drug manufactureur in this case.

Hemolytic anemia caused by mycoplasma pnemonia is what type of hypersensitivity?

Type II hypersensitivity

sudden influx of antigen (rattlesnake bite ) and present with a delayed systemic reaction known as a serum sickness, with symptoms including: fever, lymphadenopathy, joint pain, proteinuria and urticarial rash what type of hypersensitivity is this?

Type III hypersensitivity. example-suffering a rattlesnake bite, no pmhx, no medications, fever, hr 98/min; bp125/66;wound is cleaned and antitoxin adminstered. five days later the pt experiences fever, rash, and polyarthritis. which os the following most likley occured in this pt=type III hypersensitivity. requires 5 days because this is the time required for class switching from IgM to IgG.

what medication do you use to treat schistosoma?

USE Praziquantal-affects the membrane permeability of the parasite which causes vacuolization of the tegument. it paralyzes the worm and exposes it to attack by the host immune system.

how do you differentiate ulnar nerve entrapment at the cubital tunnel and at guyon's canal?

Ulnar nerve entrapment at the wrist (Guyon's canal) results in hypothenar atrophy and difficulty with finger adduction/abducton IF THE ENTRAPMENT occurs at the level of the elbow (cubital tunnel), there will be impaired flexion of the 4th and 5th digits.

method of choice to detect Intussuception

Ultrasonography or contrast enema (which also can reduce the intussusception in 75-90% of children)

hyperactive detrusor muscle that causes incontinence regardless of bladder urine volume. patients presnt with increased urgency, frequency and nocturia. most common causes are idiopathic. other common causes include infection, cancer, and stones. what dysunction is this>

Urge incontinence many women complain of not being able to reach the bathroom in time or of dribbling or leakage triggered by just seeing the bathroom.

pts shopped at local grocery store, where they purchased fresh vegetables stored under the mist machines. there is difficulty obtaining sputum samples-which of the following samples should then be obtained in order to test for suspected causative organism?

Urine sample this is legionella. legionella is commonly found in contaminated water sources, such as mist machines and water parks, and grocery stores, air conditioning units and other aerosols. pts should be screened with both a urine antigen test as well as attempted sputum culture pn buffered charcoal yeast extraact.

what technique do you use for treating isolated somatic dysfunction at any cranial suture?

V-spread

Prinzmetal angina

Vasospastic coronary artery disorder characterized by spontaneous episodes of angina in association with ST-segment elevation on ECG. The cause is a transient reduction in the luminal diameter of a coronary artery due to spasm, leading to transient myocardial ischemia.

Patients with syncope will have a negative Romberg testing=true

Vasovagal syncope is a transient loss of consciousness with spontaneous recovery. there is a loss of postural control and possible fall if the patient is standing. this occurs in absence of organic causes, such as stroke, seizures, myocardial infarction or cardiac arrest. some patients with syncope may report prior episodes of lightheadedness and feeling unsteady without actual loss of consciousness. Unlike the other causes, pts with syncope will not have a positive romberg examination, as the vestibular, cerebellar and prioprioceptive systems are not affected in this condition. negative romberg test is no sway with eyes closed.

Due to its negative inotropic effects, verapamil is contraindicated in patients with heart failure and a low ejection fraction. so in other words what does it do to patients with congestive heart failure?

Verapamil increases mortality in patients with congestive heart failure. verapamil is a non-dihydropyridine calcium channel blocker (CCB). CCBs inhibit the L-type calcium channel. By inhibiting this channel, CCBs decrease the flow of calcium into the cells of the cardiac conduction pathway, which leads to an inhibition of the phase 0 in cardiac pacemaler cells and slows the phase 2 plateau in purkinje cells, cardiac myocytes , and vascular smooth muscle cells. the overall affect that calcium channel blockers have in an individual would be decreased heart rate, (decreased inotropy) and vasodilation of the peripheral arterioles. Verapamil, being a non-dihydropyridine CCB, will have more of an effect on the heart than it would on the peripheral vasculature. It has been demonstrated that patients with heart failure and a low ejection fraction have an increased mortality when given calcium channel blockers with a negative inotropic effect as verapamil. do not give calcium channel blockers with a negative niotropic effects to patients with heart failure and a reduced ejection fraction

Common cause of decreased height in ELDERLY are?

Vertebral compression fracture is the most common cause of reduced stature in post-menopausal women with osteoporosis. these fractures may be asymptomatic. significant loss of height is not from normal aging.

Cellulitis on the hands of salt-water shell-fish handlers should make you think of what organisms? It lives in warm, high salt waters. it is a gram-negative, motile curved rod that can infect via puncture wound, eating contaminated shellfish, or exposure through swimming or wading. The cellulitis and bullae are aggressive and rapidly expanding. it can quickly cause septicemia.

Vibrio Vulnificus Do not confuse with mycobacterium marinum-mycobacterium marinum is an acid-fast rod that can cause fish tank granuloma in patients who handle salt-water fish. While M. marinum can enter via puncture wounds, symptoms usually consist of granuloma formation and not cellulitis and bullae as is present for Vibrio vulnificus

what do you expect on video clips for the exam?

Video clips will show postive physical exam findings.

Viscerosomatic dysfunctions are type II or type 1?

Visceral somati dysfunctions are type 2 somatic dysfunctions, not type 1 somatic dysfunction. In general, eliminate the type 1 dysfunctions in your answer choices for viscerosomatic questions. viscerosomatic dysfunction for heart is T1-4(T5-6)

what pathway is this: lateral geniculate nucleus of the thalamus received information about ? and transmits it to the calcarine sulcus of the occipital cortex.

Vision.

What is vital capacity?

Vital capacity is the maximum amount of air a person can expel from the lungs after a maximum inhalation. it is equal to the inspiratory reserve volume plus the tidal volume plus the expiratory reserve volume. Calculate using the equation VC=IRV+VT+ERV

Classic triad of intussusception

Vomiting, abdominal pain and blood per rectum (currant jelly stools) diagnosis and treatment can be achieved with contrast enema

median nerve can result in

Weakness of flexion of the metacarpophalangeal (MCP) joint of digit 1. flexor pollicis brevis allows for the flexion of the thumb at the MCP joint, which can be weak in pts with long-standing carpal tunnel syndrome. pts will have decreased sensation over digit one though

pupillary light reflex can be used to detect lesions of CN II or CN III.

When the light is shown p the right eye, both pupils dilate, indicate that there is an abnormality of the afferent pathay controlled by the right cranial nerve II (optic nerve)

A positive Homan sign is present when there is pain in the calf on forceful and abrupt dorsiflexion of the foot and the ankle while the knee is extended. when there is a positive Homan sign what is the next step?

When there is a positive Homan sign. the next step is to do compression ultrasonography. this is the imaging modality of choice for diagnosis of symptomatic people with a first episode of suspected DVT. A positive noninvasive study in patients with a first episode of DVT usually establishes the diagnosis.

Tropheryma whippelii causes a systemic disease that includes abdominal pain, diarrhea, arthralgias, neurologic and cardiac problems, and lymphadenopathy. whipple's disease triggers intestinal malabsorption and is treated with antibiotics, such as doxycycline or macrolides. what mechanism does this bacteria use to cause disease>

While in the small intestine, macrophages that phagocytose the organism compress the lacteals, preventing absorption of chylomicrons. whippelii occurs most commonly in older caucasian males. once macrophages detects T. Whipellii in the lamina propria of the small bowel, they attempt to destroy the bacteria by phagocytosis. on microscopic exam, we can observe foamy macrophages that stain positive with the periodic acid schiff (PAS) STAIN. The large amount of foamy macrophages compresses the lacteals int he villi of the small intestine, which prevents chylomicrons from entering into lymphatic system. since more chylomicrons cannot form due to decreased space, fat absorption is inhibited, leading to fat malabsorption and steatorrhea. treatment involves antibiotics, such as doxycycline or macrolides.

65 yo male presents with diarrhea and steatorrhea of two weeks duration. he reports that he has also experienced joint pain and confusion int he same period. which of the following is the most likely?

Whipple disease diarrhea and steatorrhea can be indicative of malabsorption. whipple disease is a malabsorption syndrome caused by infection with Tropheryma whipplil. it can be diagnosed by staining for PAS-positive macrophages within the digestive tract. Additionally, it is common in oler men and can present with arthralgias, cardiac, and neurologic symptoms

Is bone metastasis an absolute contraindication to HVLA?

YES. Absolue contraindication to HVLA include osteoporosis, osteomyelitis, fracture in the area of thrust, severe rheumatoid arthritis, achondroplastic dwarfism, septic joint history of stroke, ankylosing spondylitis, metabolic bone disease, vertigo/syncope with cervical rotation or extension, down's syndrome (weak atlantoaxial subluxation) and bone metastasis. Relative contraindications to HVLA WHIPLASH, Severe muscle spasm, radiculopathy, spondylolisthesis, pregnancy, post surgical patient, herniated nucleus pulposus, patient on anticoagulants, hemophiliacs, and vertebral artery ischemia.

what kind of murmur do you hear with hypertrophic cardiomyopathy?

YOU HEAR a systolic murmur. TYPICALLY in a crescendo-decrescendo fashion, which is best heard between the apex and left sternal border and radiates to the suprasternal notch but not to the carotid arteries or neck. the murmur and the gradient across the left ventricle outflow tract diminishes with any increase in preload or increase in afterload. The murmur and the gradient increases with any decrease in preload or with any decrease in afterload. Mueller and squating --increases preload Handgrip increases afterload Valsalva and standing decreases preload Difference between aortic stenosis and hypertrophic cardiomyopathy. Aortic stenois is a fixed lesion which when more blood is passing through the stenotic area, there will be a greater intensity in the murmur. Hypertrophic cardiomyopathy is a dynamic lesion. More blood passing through the stenotic area pushes open the obstructed portion making a larger space for it to pass through, lowering the intensity of the murmur.

Wernicke-Korsakof =f syndrome occurs in alcoholics or malnourished patients as a result of thiamine deficiency, which causes oxidative damage to the CNS. it results in ataxia, diplopia, confabulations and confusion. true or false?

YTrue free radical oxidative damage to CNS neurons.

Is tryptophan an essential amino acid?

Yes, hartnup disease is characterized by defective GI absorption and renal reabsorption of the essential amino acid tryptophan, RESULTING IN DEFICIENCY of vitamin b3 (Nicotinamide and nicotinate)-niacin=vit b3 tryptophan deficiency leads to a deficiency in niacin leading to pelagra-4 D's dermatitis, diarrhea, dementia, and death. dermatitis resembles eczema and occurs on the facem arms, and legs, sometimes in a stokcing -glove distribution due to photosensitivity.

does acidification of the urine help clear ammonia that occurs in hepatic encephalopathy?

Yes, hepatic encephalopathy can be treated with lactulose which acidifies the gut lumen and decreases the amount of ammonia int he body. Lactulose is nonabsorbable and is broken down by gut bacteria to lactic acid. this acidification of the gut lumen leads to the conversion of ammonia to ammonium ions and increases movement of ammonia from tissue into the gut lumen. Another treatment option for hepatic encephalopathy is antibiotics. decreasing the amount of gut bacteria will decrease the production of ammonia by those bacteria.

can a pregnant woman who potentially has HIV refuse care?

Yes, pregnant women has the same rights as a non-pregnant woman and can refuse care (which includes HIV testing and cesarean section), even if it can benefit or be potentially life-saving to the fetus. After a thorough discussion takes place between the physician and the patient regarding the importance of HIV screening, the patient may still wish to not have the test, as is the situation in the above case. At this point, it is necessary to document the decision and to abstain from performing testing without her consent.

When administering a patient magnesium sulfate, what do you want to make sure you check? What are signs of magnesium toxicity?

You want to make sure you check the patient's deep tendon reflexes. loss of deep tendon reflexes (DTRs), Respiratory paralysis and coma Increased magnesium decreases impulse transmission across the neuromuscular junction producing signs and symptoms of neuromuscular toxicity. Because the loss of DTR's always appears first before the respiratory arrest and coma, it is of utmost importance to continually test for them.

what would you do to shorten the quadriceps when doing counterstrain?

You will extend it

What kind of sound do you hear when you auscultate mediastinum of someone with Boerhaave syndrome?

You will hear crackles with each heart beat Patients with Boerhaave syndrome classically present with the Mackler triad of a history of vomiting, thoracic pain, and pneumomediastinum (subcutaneous emphysema). on physical exam, this presents with a hamman crunch, which is a crackling sound heard with each heartbeat. treatment is immediate surgery. other things to consider would be fluid resucitation, antibiotics, and prompt surgical intervention.

what class of Highly active antiretroviral therapy is Zidovudine part of?

Zidovudine is aslo known as Azidothymidine (AZT), is a nucleoside reverse transcriptase inhibitor (NRTI). it can be incorporate itself into the growing DNA chain, synthesized by viral reverse transcriptase, causing chain termination. stop rushing, when you know what the answer is-that is when you should slow down to make sure you get it correct

The most common histological lesion is lewy bodies. what is the triad of parkinson disease?

a typical pt case who has parkinson disease- -tremor that improves with movement, difficulty initiating movement, and a tendency to fall, admits to fatigue and depressed mood, seems to be forgetful, started 18 months ago but have gotten slowly and steadily more pronounced. so that case is parkinson's disease with lewy body dementia-which is the most common form of degenerative dementia after alzheimer disease. TRIAD=RESTING tremor, bradykinesia and rigidity. in parkinson disease, there is a loss of dopaminergic neuron activity in the substantia nigra and pars compacta, where histology amy show eosinophilic (pink with H & E stain) lesions, known as lewy bodies. these so called lewy bodies are intranuclear inclusions composed of alpha-synuclein. lewy body dementia is characterized by attn deficits, visuospatial deficits, fluctuating cognition, recurrent visual hallucinations, and spontaneous motor features of parkinsonism. other features may include autonomic dysfunction, delusions, sleep disorders, depression, and may more. Parkinson disease has been loosely linked to MPTP, which is a precursor to a neurotoxin often used as a filler in street drugs. treatment involves medications that increase cns levels of dopamine by targeting precursors for the CNS. Anti-depressants may also be used.

Kawasaki disease is an autoimmune vasculitis seen in children under the age of five. treatment is by intravenous immunoglobulin-what is the classic presentation?

abrupt onset of fever in a child for five days duration and an accompanying rash. Physical exam shows an irritable child wiht a maculopapular rash at the trunk and extremities, mild swelling of the dorsal hands and feet, strawberry tongue and lip fissures and non-purulent conjunctivities. so the treatment is --IMMUNOGLOBULIN. -treat kawasaki with IVIG and aspirin

Administer Rhogam to Rh Negative mothers with Rh positive (or unknown Rh status) fathers who meet the above criteria.

administer anti-d IMMUNE globulin following chorionic villus sampling at the 12th week of gestation situations that require Rhogam adminisration: Routine administration at 28 weeks -immediately following invasive procedure such as amioncentesis or chorionic villus sampling as well as elective abortions, stillbirths, ectopic pregnancies, and any other circumstance where transplacental bleeding is a possibility. --rHOGAM SHOULD be provided within 72 hours of delivery of an Rh positive infant.

what is beta thalassemia

adult Hemoglobin -usually is hbA= alpha and Beta fetal hemoglobin HBF= A2Y2 -IN NORMal persons a minority of adult hemoglobin is made up of Hb A2 which consists of 2 alpha and 2 delta globin chains. beta thalassemia is due to impaired production of betal globin chains, leading to a relative excess of alpha globin chains. --key to diagnose is to see Hb A2

what is the next step in management for an obese pt?

advise a balanced diet that decreases caloric intake by 500 calories daily. reduce caloric intake by 500-1000 calories per day. The medical benefits of weight loss include better control of blood sugars in diabetic and pre-diabetic states, blood pressure control in htn, and lower lipids in pts with cardiovascular risk factors. the most appropriate initial management is to order-lipid panel, tsh, AND BETA HCG. --ORGANIC CAUSES must be ruled out in pts presenting with weight gain. common causes of weight gain in a female are pregnancy, hypothyroidism, and hyperlipidemia.

Anterior spinal artery lesion

anterior spinal artery arises from branches from the vertebral arteries. it supplies the anterior 2/3rd of the spinal cord. Disruption of blood flow through the anterior spinal artery results in anterior spinal artery syndrome characterized by paralysis and sensory impairments below the level of the lesion, with impaired sensation of pain and temperature (spinothalamic tract), Fiber tracts for autonomic control are also typically compromised, resulting in bladder, bowel and sexual dysfuction. vibration sense and proprioception are preserved (dorsal column-medial lemniscus)

Weakness, severe dyspnea and wide pulse pressure, occasional angina; presence of high pitched decrescendo diastolic murmur is the hallmark. best heart on the right upper sternal border. presence of a wide pulse pressure.

aortic regurgitation

due to the insufficiency of the aorta, blood flows backward from the aorta the left ventricule after systole, decreasing the diastolic pressure and widening the pulse pressure.

aortic regurgitation

dyspnea, weakness, and hypotension

aortic regurgitation

what chapman point occurs at the transverse process of T11?

appendix Anterior chapman point for the appendix is located at the tip of the 12th rib.

why is aspirin relatively contraindicated in gout?

aspirin is relatively contraindicated because both salicylic acid and uric acid compete at renal transporters, thereby increasing the amount of uric acid in the body. however, patiens on aspirintherapy for more serious conditions like coronary artery disease of stroke prophylaxis should not stop aspirin therapy due to the benefits outweigh the risk.

a man present to emergency department with a shoulder deformity. fell six feet from a ladder and landed on the back of his shoulder. admits to feeling numbness and weakness in that shoulder. physical examination is negative for shoulder dislocation, but plain radiographs reveal a proximal humerus fracture. After full passive extension of the shoulder, he is unable to maintain the extended position. the most likely injured nerve is the

axillary nerve The axillary nerve is derived from the posterior cord of the brachial plexus. it innervates the deltoid (abduction of the shoulder) and teres minor (external rotation of the shoulder) muscles. The usual mechanisms of axillary nerve injury are trauma either by direct blow to the posterior aspect of the shoulder, following dislocation of the shoulder, or fracture of the proximal humerus. The deltoid extension lag sign is indicative of axillary nerve injury. To perform this test, the examiner elevates the arm into a position of near full extension, and then releases the arm while asking the patient to hold the arm in this position. if there is complete deltoid paralysis, the arm drops. note radial nerve injuries usually occur when the midshaft of the distal humerus is fractured.

what are the attachment of the dura that allows for craniosacral motion?

axis (C2), C3, foramen magnum, and S2.

exudative versus transudative of pleural fluid

based on light's criteria, this pt's pleural fluid meets two out of the three criteria, which is consistent with an exudative effusion. in order to classify an effusion as exuDative, at least one of the following must be present: RATIO of pleural fluid total protein to serum total protein >0.5, ratio of pleural fluid ldh to serum LDH >0.6 pleural fluid LDH >2/3 UPPER limit of normal of serum LDH. however the more criteria that are met, the higher the sensitivity and specificity indicating an exudative effusion. If none of the criteria are present, then the effusion is most likely transudative. the most common conditions that cause transudative effusions are congestive heart failure, cirrhosis and nephrotic syndrome. the differential for exudative effusion is wider. most common are lung infections, malignancy, pulmonry embolism, and collagen vascular disease.

what is battery?

battery is defined as the intentional and unauthorized touching of a pt. Assault-causing someone apprehension of harm or unwanted contact. to assult someone, you do not need to touch them. battery is the unauthorized act of touching someone without their express consent. this is not what occurs in an emergency because of implied consent. malpractice-to have malpractice , the physician must have a duty to the pt and have either neglected or breached the duty and caused the pt some form of harm. negligence-not providing the standard of care appropriate to the situation. you expect a diff standard of care from a family practice than you do from an emergency department.

what region of the brain is affected in Kluver -bucy syndrome- and presents with docility, hypersexuality and hyperorality.

bilateral amygdala lesions this condition can result from a diffuse axonal injury secondary to a traumatic brain injury. This patient will suffer from this condition for the rest of her life, as currently there is no cure for her syndrome. this pts will present with placidity with the absence of emotions, loss of facial animation, marked increased sexual dribe and libido as well as hyperorality (inappropriate objects in the mouth). t is also associated with herpes simplex virus 1 infections. other causes include infarction, neoplasm, infection, and inflammatory and degenerative disease of the temporal region of the brain in the location of the amygdala

-westnile virus is transmitted by the culex mosquito in the US. -POSITIVE KERNIG SIGN INDICATES EITHER A POSITIVE meningitis or subarachnoid headache. -what kind of cast is associated with chronic renal failure?

broas waxy casts.

pulmonary edema, high cardiac filling pressures, low cardiac index and hypoperfusion, inadequate ventricular function. what type of shock is this?

cardiogenic shock

what type of shock is this INCREASED CVP, PCWP, AND sVR and a decrease in cardiac output and venous oxygen saturation. clinically the pt will be cold, and clammy, hypotensive, and tachycardiac with evidence or a hx of heart failure?

cardiogenic shock The equation systemic vascular resistance =80 * (mean arterial pressure-central venous pressure)/cardiac output. this equation shows that cardiac output and systemic vascular resistance are inversely proportional. therefore if cardiac output is low as in the case of cardiogenic shock, the svr will be high. In cardiogenic shock, venous oxygen saturation will decrease for two reasons: 1) blood will not be oxygenated in the lungs because blood flow from the right side of the heart through the pulmonary tree will cease and a decrease in arterial oxygen saturation will lead to a decrease in venous saturation. 2). decreased delivery of arterial oxygen will lead to increased uptake by the tissues and thus a decrease in venous oxygen content. the tissues, since blood is backing up in the peripheral circulation, will then unload more carbon dioxide into the venous system. other changes would be an increase in central venous pressure, pulmonary vascular resistance, systemic vascular resistance and release of vasopressin. cardiac output=stroke volume * HR. if stroke volume decrease HR will increase to help compensate.

what do you expect to find in a joint aspirate for gout?

cloudy yellow fluid, with wbc, neutrophils, rbc and normal glucose. avoid treating a medication side effect with another medication if possible.

what is this called? measuring the intersection of parallel lines drawn from the superior endplate of the superior segment of the curve and the inferior endplate of the inferior segment of the curve.

cobb angle used to asses te severity of scoliosis. use a standard PA and lateral radiograph of the spine. pt with cobb angle of 12 degrees appropriate treatment woulf be to observe. ---A mild idiopathic scoliosis should be carefully watched with serial radiographs. these pts should all be evaluated for sacral base unleveling, a short leg and muscle imbalances before diagnosis of idiopathic scoliosis is made. mild scoliosis is cobb angle of 0 to 20 degress spinal bracing considerd 20-45 46 cobb angle-surgical consulation

where do you expect to find the chapman point for colon cancer?

colon cancer would lead to a chapman's point on the lateral thigh within the IT band from the greater trochanter to just above the knee. just a sidenote the normal hematocrit for an adult woman is 38% to 46%.What

what kind of fall leads to an anterior radial head? what about a posterior radial head? an anterior radial head will have restriction where?

common mechanism of injury for an anterior radial head is fall backwards on an outstretched hand. Common mechanism of injury for a posterior radial head is a fall forward on an outstretched hand. An anterior radial head will have restricted pronation of the forearm.

report symptoms that are difficult to reproduce, locate, or occasional change during the initial intake. thorough hx and examination combined with documentation of what was said and done in the interview is the of managing factitious or malingering pts. how do pts present?

compaining of fatigue, and abdominal pain of two days duration. denies nausea, vomiting, diarrhea, or constipation. states stools are unformed and unchanged from baseline and no hematochezia or melana. period was 7 days ago and normal. admitted in the past for symptoms of lethargy. extensive testing reveals nothing.

HYPERTENSION, HYPOKALEMIA, METABOLIC ALKALOSIS, AND LOW RENIN WHAT DO YOU SUSPECT?

conn syndrome=hyperaldosteronism aldosterone increases the absorption of sodium and bicarb, secretion of hydrogen and potassium.

21 yo, persistent lump in her left breast, tenderness in the lump a few weeks before menses. physical exam reveals a firm, rubbery, motile, well-demarcated mass. what is this mass?

fibroadenoma most common breast tumor in women under 25 years old.

Binding of CO to hemoglobin increases the affinity of remaining sites for oxygen, causing a shift of the dissociation curve to the left. Hemoglobin will bind carbon monoxide of the smoke and lead to decreased in oxyhemoglobin levels. Decreased oxygen delivery to the active tissues will then result in a lactic acidosis due to enhanced anaerobic respiration. The resulting lactic acidosis will be buffered and decrease the blood bicarbonate levels. so you will see in a carbon monoxide poisoning what?

decreased oxyhemoglobin saturation and decreased bicarbonate.

Linear bullous rash in a patient with a history of being in the woods is most likely a rhus dermatitis that is caused by what type of hypersensitivity?

delayed type 4 hypersensitivity

Marbug virus is a hemorrhagic fever, which begins as a flu-like illness for five days and then progresses to organ failure and hemorrhage. For those who recover, their immunity can be assessed via detection of what?

detection of IgG to the Marburg virus

pramipexole is added to the treatment plan. what is the mechanism of action of pramipexole?

dopamine agonist so is Ropinirole pramipexole is a synthetic dopamine agonist used as monotherapy in early parkinson's disease to delay levodopa initiation or used with levodopa in advanced disease to decrease the dose of levodopa. Pamipexole and ropinirole are non-ergot synthetic dopamine agonists. they can be used to alleviate the motor deficits seen in patients with parkinson's disease. Dopamine agonists are commonly used to delay the need for levodopa in early parkinson's disease. This is important because long-term use of levodopa leads to dyskinesia in most patients. therefore, most physicians attempt to delay the initiation of levodopa for as long as possible. Dopamine agonists are also commonly used in advanced parkinson's disease in order to decrease the dose of levodopa needed for its therapeutic effect. Non-ergot dopamine agonists are preferred over ergot derivatives, such as bromocriptine. Unlike the ergotamine derivatives, pramipexole and ropinirole do not exarcerbate vasospasm in patients with concomitant peripheral vascular disease, not do they cause pulmonary and/or retroperitoneal fibrosis. both of which are serious complications associated with bromocriptine.

When assessing the patient you discover a tenderpoint on the left lateral side of the spinous process of c5. wHAT TREATMENT POSITION would you put the patient in for strain/counterstrain?

extended, sidebent to the right, and rotated right strain counterstrain treatment for most posterior cervical tenderpoints is extended, sidebent away, and rotated away fro, the tenderpoint. the exceptions to this are the inion counterstrain point (flexion) and the pc3 COUNTERSTRAIN point (FSARA).

What is the pathology in cirrhosis that leads to esophageal varices?

fIBROSIS OF THE PORTAL VEIN.

be familiar with findings consistent with acute somatic dysfunction in the lumbar region

findings in acute somatic dysfunction=increased temperature, boddy texture, increased moisture, rigid.board-like muscular tension, significant tenderness, edema, venous congestion, and persistent erythema chronic somatic dysfunction=decreased temperature, smooth texture, drgging this, ropy/stringy muscular tension, dull tenderness, nonedematous, neovascularization, and fading/blanching erythema. not if stem says pt has a past medical history of chronic back pain be aware of that.

first line treatment for pyelonephritis?

fluoroquinolone antibiotics with trimthoprim-sulfamethoxazole and amoxicillin-clavulanic acid (as a second agent) used to treat UTIs and pyelonephritis in pregnant women and children

Ventral posterior medial nucleus receives sensory information from where?

from face and sends it to the primary sensory cortex.

two most common causes of pancreatitis in the U.S

gALLSTONES AND ALCOHOL Several infectious agent may cause pancreatitis in children. typically the infectious cases of pancreatitis tend to be milder than cases of acute biliary or alcohol-induced pancreatitis. iN AN OLDER PATIENT GALLSTONES ARE THE MOST LIKELY ETIOLOGY FOR HER ACUTE PANCREATITIS AND NOT AN INFECTIOUS CAUSE.

most common cause of meningitis in a neonate before three months of life?

group B streptococcus

In diffuse diabetic glomerulosclerosis, there is diffuse thickening of the glomerular basement membrane, which early in the disease process cannot be appreciated by light microscopy. As the disease progresses, the mesangial matrix continues to accumulate along with mesangial cells. this results in a characteristic nodular glomerulosclerosis pattern (kimmelstiel-wilson nodules). how to tell diabetes in a case presentation. it will be really hard but the key thing to keep in mind are

high glucose over 315. person is overweight flame-shaped hemorrhags urinating a lot kidney failure and lots of pink on the renal biopy think mesangial expansion caused by diffuse diabetic glomeruloscerosis.

sequence of how heart failure occurs? what increases digoxin toxicity? most likley adverse effect of catopril is? must know things about ACE inhibitors?

hypertension-->left ventricular hypertrophy--> left sided heart failure--> right sided heart failure. the most common cause of right sided failure is left sided failure. hypercalcemia. other electrolyte abnormalities that can cause digoxin toxicity include hyper or hypokalemia and hypomagnesemia. if there is hypokalemia present, then there will be a greater probability of binding of digoxin, which will enhance the possibility of toxicity. hyperkalemia-ace inhibitors block the production of angiotensin II, which leads to a significant decrease aldosterone secretion (renin-angiotensin-aldosterone system), increasing the risk for hyperkalemia. 1)cough and angioedema 2)hyperkalemia 3)increase survival in congestive heart failure 4)they are contraindicated in bilateral renal artery stenosis because they exacerbate renal failure in these patients 5) slow the progression of diabetic nephropathy 6)contraindicated during pregnancy.

name the shock--LOW PRELOAD, low cardiac output, decreased intravascular volume

hypovolemic shock

A tenderpoint medial to the ipsilateral ASIS, a positive pelvic shift test to the contralateral side, sacral dysfunction on an oblique asis and spasm of the contralateral piriformis muscle this are all indications of a positive?

iNDICATION OF A POSITIVE THOMAS TEST

5 yo male. has type 1 diabetes. receives treatment but soon comes to the ER with muscle weakness, fatigue, and chest palpitations what is the most likely cause?

increased cellular potassium uptake hypokalemia commonly presents with muscle weakness, cramps, and fatigue. pts may also experience polyuria, nausea, vomiting, paralytic ileus, and/or chest palpitations. decreased potassium levels can prolong the normal cardiac conduction and cause life-threatening arrhythmias. therefore it is critical to administer oral or IV potassium and monitor cardiac rhythm once hypokalemia is detected.

Caliculi in the uteropelvic junction can present with deep flank pain without radiation to the groin, due to distention of the renal capsule. Stones in the ureter cause abrupt , severe, colicky pain in the flank and ipsilateral lower abdomen with radiation to the testicular or vulvar area-intense nausea with or without vomiting is usually present. pain from upper ureteral stones tend to radiate to the flank and lumbar areas can be confused with cholecystitis or cholithiasis if present on the right

info on how ureteral stones present

patient has all the symptoms of hyperthyroidism and you are given that she also has a suprasellar mass. what is the likely origin of her hyperthyroidism?

it is likely of secondary origin fromt he adenoma. Pituitary adenoma secrete pituitary hormones such as TSH. Elevated TSH will stimulate the thyroid gland and lead to a subsequent over secretion of T3 and T4. the increase in T3 and T4 causes the symptoms of hyperthyroidism. the elevated levels of T3 and T4 induce a negative feedback mechanism on the hypothalamus-pituitary axis, preventing release of thyrotropin releasing hormone (TRH) in the hypothalamus. Overall, then TRH levels are decreased.

A 43 yo has slurred speech, difficulty swallowing, loss of gag reflex. an mri shows area of infarction in the brainstem. of the following foramina the one that contains the cranial nerve affected by the lesion would be?

jugular foramen A lesion of the vagus nerve (CN X) may manifest as difficulty with speech and swallowing and loss of the gag reflex. this nerve exits the skull via the jugular foramen, which also allows the passage of CN IX, XI and the jugular vein.

Bilirubin Total=5.2 Direct=0.2 what is the most likely cause of this finding? This patient has an indirect/unconjugated hyperbilirubinemia with a high total serum bilirubin but a normal direct bilirubin. In a healthy 15 yo individual, this is most likely due to a deficiency in functional UDP-glucuronosyltransferase (UGT), LEADING TO A decrease in bilirubin uptake known as gilbert disease. gilbert disease ia a benign, hereditary condition that causes increased bilirubin levels with fasting, dehydrartion or stress. Although commonly asymptomatic, some patients present with vague abdominal pain, fatigue and mild jaundice that is self limited.

key thing to realize is that you have a normal bilirubin and the total is elevated -so it is the unconjugated that is leading to this elevation thus why it will be because of unconjugation.

structural exam shows positive seated flexion test on the right, deep sacral sulcus on the right, and posterior inferior lateral angle on the left. good spring is found at the sacral nase. what is the likely diagnosis?

left on left sacral torsion. A negative spring test confirms that the sacral base moves well; a positive spring test indicates a backwards torsion (R on L or L on R). aDDITIONALLY, AN L-on L sacral torsion is associated with a convex right lumbar curvature and neutral L5 mechanics with L5 sidebent left and rotated right. sacral dysfunctions-the oblique axis is named for the side of the sacral base from which it originates! for the SPRING test P (for positive)=PATHOLOGY, so if the spring test is positive , the sacrum is backwards.

A pt does not know the names of his medications or why they are being prescribed, are more passive in the clinical encounter and rarely ask questions, have difficulty completing forms, and exhibit problems in navigating and completing diagnostic tests, procedures, medication refills, and/or consultation processes. what do you suspect?

low health literacy Health literacy is defined as the degree to which individuals have the capacity to obtain, process, and understand basic health information and services needed to make appropriate health decisions. people with low health literacy have poor health outcomes when compared to others with adequate literacy. this pt shows many of the red flags associated with low health literacy.

Struvite stones/crystals ("triple phosphate" stones, consist of magnesium, ammonium, and phosphate). in the renal pelvis and calyces of the kidneys. what do they consist of?

magnesium, ammonium, and phosphate (Mg-NH4-PO4)

what is the equation for maintenace dose that if you use can help you know the bioavailability of a drug?

maintenance dose of a drug is inversely proportional to bioavailability of the drug. MD=CP*CL/F MD =maintenance dose CP= desired plasma concentration CL= clearance. F=bioavailability bioavailabilty is the fraction of a drug that reaches systemic circulation

Acute myelogenous leukemia generally present with symptoms related to complications of pancytopenia. diagnosis of acute myelogenous leukemia is via bone marrow aspiration and biopsy displaying at least 20% myeloid blast (myelobalsts and monoblasts), myeloblasts have 2-4 nucleoli with larger cytoplasm than lymphoblasts, often containing Auer rOD (aZUROPHILIC, needle-shaped granules) as seen in the FAB subtypes M2-M3. MONOBLASTS HAVE FOLDED/LOBULATED NUCLEI, LACK auer rods, and are esterase positive. how is the bone marrow biopsy described as?

many cells with large, oval-shaoed nuclei and basophilic cytoplasm

who own's medical records?

medical records are owned by the provider who constructed them; however, patients have a right to see and get a copy of their records. The federal privacy rule as established to assure that individuals's health information is properly protected while allowing the flow of health information needed to provide and promote high-quality health cate. Although, medical records are considered property of healthcare professional who created them, patients have a right to review and make a copy of their healthcare information, as well as request amendments are made. An exception exists for mental health information records in cases where information contained within may cause harm to the patient.

When do you want to do a mesenteric lift?

mesenteric lift is a treatment for the GI system to reduce lymphatic congestion in the mesentery. this will likely be beneficial to reduce lymphatic congestion but could be painful due to recent incision and should be done after apinal treatments and autonomic interventions. therefore paraspinal inhibition would be a better choice as this would directly address the patients hypersympathetic tone. once hypersympathetic tone is reduced more efficient blood flow would be noted to an from the bowel and mesenteric lift technique would be more effective.

Structural exam reveals a left seated flexion test with the sacral base anterior on the left and ILA posterior on the right. there is a negative lumbosacral spring test. Based on the physical examination finding the most likely somatic dysfunction at L5 is?

neutral, sidebent right, rotated left general rule for L5 in relation to the sacrum is this: 1-L5 rotates opposite of sacral rotation 2-L5 sidebends toward the side of the oblique axis of the sacrum -This patient has a right on right sacral torsion. -Right oblique axis is engaged as demonstrated by the left positive seated flexion test (left PSIS would be cephalad compared to the right PSIS when the patient seated and bends forward). the sacrum is rotated to the right, demonstrated by the deep left sacral sulcus and right shallow right inferior lateral angle (ILA). A negative lumbosacral spring test (patient prone with operator push with palm of hand on lumbosacral junction and release, if resists or lack spring then test positive) indicates there is forward sacral torsion. ---so basically having a negative lumbosacral spring test means there is good spring. Diagnosis at L5 can be deduced by using the L5 rules. L5 would be rotated to the opposite of the sacral rotation which would make L5 rotated left and L5 would be sidebent towards the side of the oblique axis of the sacrum which would make L5 sidebent right. Since L5 is rotated left and sidebent right it would be a type one dysfunction and therefore be neutral. a type 1 dysfunction can also be deduced knowing the sacral diagnosis is a forward torsion which are associated with type 1 dysfunctions. remember forward torsions only have 1 letter so it is a type 1.

people with Marfan syndrome can have mitral valve prolapse but can it cause abdominal pain?

no. --aortic aneurysm would marfan syndrome results in cystic medial degeneration in the ascending aorta, which causes aortic aneurysms or aortic dissections. Ruptured aortic aneurysms presents with the triad of abdominal pain, hypotension, and a palpable pulsatile abdominal mass. when these symptoms are present, no further diagnostic testing is needed and emergent laparotomy is indicated.

Morphine can cause hypotension and is contraindicated in persons with a low bld pressure at presentation.

note morphine is an opiate. activation of opioid results in inhibition of synaptic neurotransmission in the central nervous system and peripheral nervous system. physiological effects of opioid is via the mu and kappa receptors. Mu receptors effects include-analgesia, euphoria, respiratory depression and miosis. kappa receptor effects include-analgesia, miosis, respiratory depression and sedation. morphine sulfate is an opioid analgesia that bind to receptors in the CNS, causing inhibition of ascending pain pathways, morphine has potent analgesic and anxiolytic effects as well as hemodynamic effects. that are potentially beneficial in persons with unstable angina. mORPJHINE CAUSES VENODILATION AND CAN PRODUCE MODEST REDUCTIONS IN HEART RATE (THROUGH INCREASED vagal tone) and systolic blood pressure to further reduce myocardial oxygen demand. The major adverse reaction to morphine is an exaggeration of its theraprutic effect, causing hypotension, especially in the presence of volume depletion and/or vasodilator therapy. --so if a person is alreay hypotensie-morphine is contraindicated.

What did the Schloendorff v society of new york hospital establish

obtaining informed consent

the most appropriate therapy for mcARDLE DISEASE IS?

oral ingestion of sucrose before exercise

Why is there symptom of pain in bowel obstruction?

pain signals in the bowel are generated in response to stretch. In bowel obstruction, intraluminal pressure builds proximally to the transition point, which stretches the bowel wall and generates pain. Additionally, bowel peristalsis will continue to attempt to advance luminal contents past the transition point, bringing increased tension to the bowel wall and a concomitant increase in symptoms. This process does not produce rebound or guarding, and can help distinguish bowel obstruction from other acute abdomen emergencies. Abdominal radiograph will show dilated loops of bowel and air-fluid levels. Next step in management is to insert a nasogastric tube and place it on low intermittent suction

person fell down catching themselve with their right wrist, bending wrist=feels tight, there is decreased flexion and extension of the right wrist, there is a tenderpoint located on the dorsum of the right wrist. what is the correct way to treat this patient's counterstrain point?

palpate the tenderpoint, extend the wrist until 70% of the tenderpoint is reduced, hold for 90 seconds, slowly return to neutral, and recheck the tenderpoint. because the patients tenderpoint is on the dorsum of the wrist. the position of treatment is in extension, in order to shorten the affected muscle.

pain radiates to the back? pain radiates to the shoulder what should you be thinking of? what is the imaging of choice for pancreatic cancer

pancreatic cancer-think both start with b and p sounding things Gallbladder--it has an l on it. A contrasr-enhanced helical compouted Tomography (CT) scan with 3D reconstruction is the study of choice to view the pancreas since 2/3 of the pancreas is retroperitoneal (head and body)

name the physiologic strains and how they are named name the nonphysiologic strains and how they are names as well

physiologic strains (torsions anD SBR) ARE NAMES FOR THE side of the more superior greater wing of the sphenoid and the side of the convexity respectively. Non-physiologic strains (compression, lateral and vertical) are names for what occurs at the SBS.

Plummer vinson syndrome manifests with spoon-shaped nails of koilonychia, glossitis, iron deficiency anemia, and dysphagia and is considered a premalignant process with an increased risk for what carcinomas?

plummer vinson syndrome is commonly seen in women in their fifth decade of life, dysphagia is due to the presence of esophageal webs, and is limited to solids. this can result in choking spells and aspiration because of the proximal location of the web. weakness, fatigue, and dyspnea occur secondary to iron deficiency anemia. physical examination will reveal atrophy of the lingual papillae, cheilitis, atrophic glossitis, stomatitis, koilonychia, and pallor. blood tests will reveal a hypochromic microcytic anemia that is consistent with an iron-deficiency anemia. A barium esophogram may reveal the presence of the esophageal webs. plummer-vinson syndrome is considered a pre-malignant process b/c increased risk for hypopharyngeal and esophageal squamous cell carcinoma

structural examination reveals L5 flexed, sidebent right, rotated right. given these findings at L5, which of the following findings is the most likey?

positive seated flexion test on the left L5 rotates opposite of sacral rotation L5 sidebends to toward the oblique axis of the sacrum.---Forward torsions are associated with type 1 DYSFUNCTIONs at L5 and backward torsions are associated with type II dysfunctions at L5. In this case the L5 somatic dysfunction has fryette type II (non-neutral) mechanics which would be associated with a backward sacral torsion. since L5 rotates right, the sacrum would rotate left and since L5 sidebends right, the sacrum would have a right axis therefore the patient has a L on R sacral torsion. This specific backwards torsion would also present with positive seated flexion test on the left (left PSIS would be cephalad compared to right PSIS when patient seated and bends forward), positive lumbosacral spring test (patient prone with operator push with palm of hand on lumbosacral junction and release, if resists or lack of spring then test positive), landmarks would show shallow left sacral sulcus and left ILA, and deep right sacral sulcus with right ILA. remember in all sacral torsions the seated flexion test is positive on the side opposite to the oblique.

A 53 yo male presents to your office 24 hours after falling backwards in the shower and is complaining of mid back pain. upon palpation you find T7 to be ESLRL. what palpatory findings are consistent with this segmental diagnosis?

prominence of the left transverse process with flexion. In somatic dysfunctions of the spine that are held in extension, the affected transverse process would be expected to become prominent when the spine is flexed. -this makes sense because the extension part of the diagnosis is indicating in what direction the patient likes to go so in a sense what feels better. This patient has been diagnosed with a type II somatic dysfunction in which T7 vertebrae is extended, sidebent left, and rotated left. Since somatic dysfunction is diagnosed in the direction of ease this positions would be in the ease position. By placing the vertebrae in extension, the direction of ease, the left transverse process of T7 would become less prominent (closer to neutral) and more prominent when the vertebrae is placed in flexion (the direction of restriction).

how do you distinguish subacute granulomatous thyroiditis from other hyperthyroid or hypothyroid etiologies?

sUBACUTE granulomatous thyroiditis is a self-limiting hyper-or-hypothyroidism that often follows a flulike illness. it can be distinguished from other hyperthyroid etiologies in that it is the only type of thyroiditis that presents with a painful, enlarged goiter. comlex likes to give you hints to help narrow what the answer possibly could be. this hints can go something like-acute conditions such as boggy, tender, warm, and moist. --this are associated with acute processes. chronic conditions would be medication induced if there is an acute changes in medication; or graves disease which has a more insidious, longer onset; chronic pathology is suggested when lesions are cool, dry, ropy, or achy. don't neglect the osteopathic hiunts--they may narrow the answer choices faster on test day.

17 yo female-complaint of lower abdominal pain and cramping occuring at the onset of her menstrual cycle-the appropriate osteopathic technique to initiate at the onset of her menstrual cycle would be?

sacral inhibition. pt is presenting with symptoms associated with dysmenorrhea including cramping and lower abdominal pain associated with her menstrual cycle but without irregular menses or heavy bleeding. sacral inhibition has been shown to be helpful in patients suffering from dyemenorrhea. sacral inhibition is performed in the prone position while deep pressure is applied tot he sacrum following the motion with respiration for two mins. this technique influences parasympathetic activity to the pelvic organs. S2-S4 via the pelvic splanchnic nerved is the corresponding parasympathetic innerveation to the lower GI TRACT, Pelvic organs except for the testes and ovaries. sacral inhibition will directly affect the glands of the endometrium as well as the muscular layer of the myometrium and thus aid in reduction of the patient's dysmenorrhea. key point-Sacral inhibition is indicated for the treatment of dysmenorrhea.

what kind of toxin does salmonella produce?

salmonella does not produce an enterotoxin. salmonella elicits a direct immune reponse that leads to increased intracellular cAMP levels. this accounts for the diarrhea associated with salmonellosis. causes watery to bloody diarrhea. has rose spots.

Chromoblastomycosis is a skin infection that causes verrucous and plaque-like lesions after traumatic inoculation, shows thick-walled cigar-colored sclerotic cells. how does it present

scaly, verrucous skin lesion covering right foot and ascending to the mid-calf. acid traveler and she often participates in long journeys hiking through the south american terrain. she denies contact with noxious plants but reports a deep laceration to the right foot that healed by secondary intention. Chromoblastomycosis is a chronic fungal skin infection usually caused by traumatic inoculation by a specific group of fungi including fonsecaea spp, phialophora verrucosa, or cladosporium carrinoii. this group of fungi is more prevalent in tropial and subtropical areas, located in certain areas of africa and south america but is not commonly found endemically in the united states. the lesion develops insidiously at the site of inoculation, beginning as a warty nodule. overtime, the lesion spreads over the skin and subcutaneous tissue forming plaques and nodular tissue resembling cauliflower-like lesions. examination of the skin using KOH reveals thick-walled, cigar-colored sclerotic cells know as medlar bodies which are pathognomic f chromoblastomycosis.

secondary biliary cirrhosis

secondary biliary cirrhosis is caused by an obstruction of the bile duct outside of the liver. carcinoma of the head of the pancreas can cause this resulting in increased pressure in the intra-hepatic bile ducts.

The humerus and elbow are not completely ossified at birth. the order in which the humeral ossification appear is as follows: shaft, capitulum, medial epicondyle, trochlea, and lateral epicondyle

shaft capitulum medial epicondyle, trochlea, and lateral epicondyle SCMTL-way to remember 5 things the correct order of ossification in a pediatric humerus and radius is completed by endochondral ossification from primary and secondary ossification centers.

what is shaken baby syndrome characterized by?

shaken baby syndrome is characterized by retinal hemorrhages, bruised ribs, subdural hematomas. shaken baby syndrome destorys a child's brain cells and prevents his or her brain from getting enough oxygen. shaken baby syndrome is a form of child abuse that can result in permanent damage or death. the case is suspicious of a shaken baby syndrome, since the child was misbehaving who is 6 months old and there are retinal hemorrhages. pupillary dilation and vomiting can be a sign of increased intracranial pressure. Head trauma can also result in a decreased cri. the normal is 10-14 cycles/minute. subdural hematomas are characteristic of shaken baby syndrome.

Regarding the ability of the airways to overcome collapsing pressure in order to open and bring air into the alveoli, it is appropriate to state?

smaller alveoli experience a larger collapsing pressure. By the law of laPlace, if the surface tension (T) is constant, collapsing pressure of alveoli (P) is inversely related to the radius (r) such that P=2T/r. Thus smaller alveoli (those with a decreased radius) will experience a larger collapsing pressure and greater tendency to collapse. iNFANTS WITH RESPIRATORY DISTRESS syndrome (RDS) lack the surfactant needed to decrease surface tension, so the increased surface tension, along with small radius, makes all alveoli more likely to collpase, and not be able to reopen.

ribs 3-6 move easily during exhalation but do not move easily during inhalation. which of the following matches the rib dysfunction and the rib to be treated in this case? the key rib in this case has what characteristic?

so things are names based on the position of ease- so this is an exhalation somatic dysfunction exhaled ribs are restricted during inhalation. the key rib in exhalation dysfunctions is the top rib of the group remember the mnemonic BITE for ket ribs: bottom inhaled, tOP Exhaled. Somatic dysfunction are named for the freedom of motion. This patient is suffering from an exhalation rib dysfunction because the ribs are restricted in inhalation motion and prefer exhalation. in an exhalation dysfunction, the key rib is the uppermost rib of the dysfunctional group and should be treated first. treating the top rib first in a group exhalation dysfunction allows the lowr ribs to be easily treated because the top rib is no longer holding the lower ribs down in an exhalation position. In an inhalation dysfunction, the key rib is the lowest rib in the dysfunctional group. The key rib in this case has a pump handle rib motion mechanics. the top rib is the key rib in a group exhalation dysfunction. ribs 1-3 have primarily pump handle. you name is based on the position of ease but the mnemonic of BEKI is treating dysfunction.

what does sphinx position mean?

sphinx position means flexion.

urease enzyme that proteus mirabilis posses does what reaction?

splits urea into NH3 and CO2.

L2 FRLeftSleft-you want to perform an articulatory technique-how would you perform it-which structure will you stabilize?

stabilize spinous process of L3. In this question, the dysfunctional vertebrae is L2 so L3 should be stabilized at the spinous process. this is treating it in the seated position. If you are treating it in the lateral recumbent position-you want to stabilize L2 -USING LEG as a lever. remember when diagnosing L2-you are really diagnosing dysfunction in how well L2 articulates on L3.

stable versus unstable angina treatment regimen

stable angina is angina that occurs with exertion unstable angina is angina that occurs at rest angina give vasodilation and beta blocker stable angina refers to chest discomfort with activity and minimal discomfort at rest or after the administration of nitroglycerin unstable angina is classified as new onset of severe chest discomfort, occuring in a crescendo pattern, occuring at rest, or sustaining throughout nitroglycerin therapy. stable angina should be managed with a short-term and long-term medicine.-A resting eletrocardiogram is often normal in the absence of a previous myocardial infarction (MI) or other cardiovascular disease-however during pain episodes the EKG may reveal ST-segment depression or T wave inversions. NITRATES AND B-BLOCKERS ARE THE FIRST CHOICES. NITROGLYCERIN FORMS FREE RADICAL NITRIC OXIDE. IN SMOOTH MUSCLE, NITRIC OXIDE ACTIVATESS GUANYLATE CYCLASE WHICH INCREASES cGMP leading to dephosphorylation of myosin light chains and smooth muscle relaxation. this produces a vasodilator effect on the peripheral veins and arteries with more prominent effects on the veins. --It primarily reduces cardiac ocygen demand by decreasing perload (left ventricular end-diastolic pressure), may modestly reduce afterload, and dilates coronary arteries and improves collateral flow to ischemic regions. THE ADDITION of beta blocker for long-term control is also first line therapy. B-blockers relieve anginal symptoms by reducing heart rate and contractility. they are the only antianginal drugs proven to prevent re-infarction and to improve survival in those who have a previous MI. B-blockers should NOT BE USED IN THOSE WITH PRINZMETAL (VARIANT OR VASOSPASTIC) ANGINA BECAUSE THEY INCREASE THE TENDENCY FOR CORONARY VASOSPASM WITH UNOPPOSED A-RECEPTOR ACTIVITY

Proteus mirabilis has urease, which causes an increase in urinary pH due to increased ammonia levels. -what kind of stone does this lead to the formation of?

struvite stones/crystals ("triple phosphate" stones, consist of magnesium, ammonium, and phosphate). in the renal pelvis and calyces of the kidneys

complaint of double vision when look down. MRI shows lesion adjacent to the inferior colliculus. the most most likely paralyzed as a result of the trauma is?

superior oblique--lesion of the trochlear nerve (CN IV) result in dysfunction of the superior oblique muscle and an inability to move the globe inferomedially. the inferior colliculus is a structure located on the posterior aspect of the midbrain. the trochlear nerve (CN IV) emerges from the midbrain just inferior to the inferior colliculus before coursing anteriorly through the wall of the cavernous sinus and into the orbit. trochlear nerve is the only nerve that emerges posteriorly from the brainstem.

A tenderpoint is located laterally to the anterior inferior iliac sPINE AIIS on the left and decide to treat with counterstrain technique-how would you position him to treat his somatic dysfunction with counterstrain?

supine with lumbar flexion and side bending to the left. The classic position to treat AL3 and AL4 using counterstrain is, with the patient supine and the physician standing on the contralateral side of the dysfunction (lEFT al3, PHYSICIAN STANDS ON RIGHT), TO PASSIVELY FLEX THE LUMBAR SPINE USING flexion of the hips and knees to 90 degrees. you then do induce sidebending of the lumbar spine away from the dysfunctional side. you do this by leaving the knees at midline while you bring the ankles away from the dysfunctional side. bringing the ankles to the right while leaving the knees at midline causes the left side of the pelvis to move inferior which induces right sidebending through the lumbar spine.

baseball pitcher with shoulder pain. you notice dysfunction with the left clavicle, with the sternal end held in a relative caudad position with decreased motion on shoulder shrug What is the position and initial force you would apply to treat the patient with balanced ligamentous tension at the sternoclavicular joint?

supine with the arm in slight abduction, applying a force to the medial clavicle in the caudad direction. blt-deb(disengage, exaggerate and balance) three phases of BLT=disengagement, exaggeration and balance. disengage the affected joint, exaggerate the dysfunction, and balance the tension through either direct or indirect forces as necessary. In this example, the pt has an inferior medial clavicular head and decreased motion with shrugging the shoulders which indicates restricted cephalad motion of the clavicle. balanced ligamentous tension (BLT) focuses on treating articular dysfunctions by restoring normal ligementous tension. there are three phases of BLT technique: disengagement, exaggeration, and balance. The joint must first be disengaged, in this instance this can be done with the pt supine, seated, or standing. joints may be disengaged wither through compression or decompression. in order to disengage the sternoclavicular joint in this pt the joint must be decompressed as it is already compressed against the sternum. Thus the practitioner would place one hand on the sternal head of the clavicle and apply a caudad force to it. The practitioner places their other hand on the lateral end of the clavicle with the patients arm draped over theirs. this will create abduction of the arm and will help to disengage the joint caudally. The subsequent steps would then be to exaggerate the direction of relative freedom which is generally in the direction of the original injury, and finally to resolve the dysfunction through finding a balance point. the balance point is found by awaiting a softening or release of the tension in the tissues, and then accentuating the motion through the ligaments as you follow the tissues unwind either in a direct or indirect manner. following a cephalad and somewhat lateral release would treat directly, alternatively further engaging the caudad ease would treat indirectly. additionally you can position the arm and use respiratory assist as needed to better balance the tissue across the sternoclavicular (sc) JOINT. a common indication for clavicular BLT is shoulder pain with decreased range of motion at the SC joint.

middle cerebral artery

supplies the lateral aspect of the motor-sensory cortex which contols the face and arm. on the dominant side, speech centers are also supplied (Broca's area (expression) in the posterior frontal region and Wernicke's area (reception) on the superior aspect of the temporal lobe. middle cerebral artery stroke presents as contralateral hemiplegia, hemianesthesia, conjugate eye devviation toward the side of the infarct, and homonymous hemianopia. global aphasia can present in stroke that occurs in the dominant hemisphere, while hemineglect is more likely in non-dominant hemisphere strokes.

Ulceroglandular tularemia, caused by francisella tularensis, most commonly presents as an acute febrile illness and an ulcer with regional lymphadenopathy at the site of entry after contact with ticks or rabbits.

suspect tularemia in someone who went hunting, has fever, malaise, myalgias. was bite by a tick, has anorexia, nausea vomiting and abdominal pain. elevated WBC, AST, ALT. species is determined to be of the species Dermacentor andersoni and is found to contain pleomorphic bacteria that grew on buffered charcoal yeast extract agar. So on physical exam-pt is likely to have- ulcer with swollen regional glands

why does OA not FOLLOW FRYETTE'S PRINCIPLE?

the cervical spine does not follow fryette's laws of intersegmental motion. The occipitoatlantal (OA) joint between the occiput and C1 (atlas) exhibits rotation and sidebending in opposite directions, but unlike type-1 dysfunctions in the thoracic and lumbar spines, dysfunctions can occur in flexion or extension. The major motions of the OA joint are flexion and extension. Rotation and sidebending are minor but can be major restrictions. The atlantoaxial joint exhibits primarily rotation and C2-c7 exhibit type II like motion.

what is the physiologic barrier?

the physiologic barrier is the maximum point at which a patient can actively move a joint in the absence of somatic dysfunction.

Facial nerve provides taste sensation to the anterior two-thirds of the tongue via the chorda tympani branch. the glossorpharyngeal nerve provides taste sensation to the posterior one third of the tongue. what about the vagus nerve?

the vagus nerve provides taste sensation to the epiglottis.

where do the cervical nerve roots emerge from? C7 nerve root problem will likely show?

there are eight cervical nerve roots. the first seven exit above their corresponding vertebrae. the c8 (the last cervical nerve root) exits above T1. the remaining nerve roots below this exit their corresponding vertebrae paresthesia of middle finger with paresis of elbow extension knowing how to identify cervical vertebrae on plain radiograph and know where cervical nerve roots exit. Be aware that comlex may show an abnormal cervical spine radiograph and ask you to identify which nerve root is affected. dermatomes and myotomes are high yield information for comlex. be very familiar with the dermatomal map. they like hypersensitivity too

scm is at the level of AC7 so this is the counterstrain tenderpoint for sternocleidomastoid. how do you treat this tenderpoint?

this is FSTRA=FLEX SIDEBEND TOWARD AND ROTATE AWAY. THIS IS DIFFERENT FROM THE OTHER TENDERPOINTS WHICH ARE FSARA=flex, sidebend away, rotate away

burning substernal chest pain that radiates to his back. assymetric upper extremity pulses and high blood pressure. what is pathophysiology of this condition?

this is aortic dissection An aortic intima tear that connects the aortic media with the aortic lumen. blood enters the false lumen that is created by the intimal tear. blood and the high prressure of the aorta continue to dissect through layers of the vessel wall creating a burning or tearing sensation that the patient classically experiences.

left elbow pain. elbow and wrist were very painful, particularly when fully extended. you note tenderness around the olecranon process of the left elbow. when the pt faces her palms forward and lets her arms down by her side you notice a decreased carrying angle int he left arm. to treat this this dysfunction using high velocity low amplitude, the physician grasps the patient's elbow with fingers of the monitoring hand on either side of the olecranon. what is the next step to treat this dysfunction using the high velocity low amplitude technique?

this patient has an abduction restriction (adduction somatic dysfunction) of the ulnohumeral joint. diagnosed by her hx and physical exam indicating a decreased carrying angle. this is treated by holding the arm in extension and supination and applying a medially directed thrust to the lateral elbow. carrying angle less than 3 degrees is considered cubitus varus and can be caused by abduction restriction (adduction somatic dysfunction) of the ulnohumeral joint whereas a carrying angle greater than 15 degrees, cubitus vlagus, can be caused by an adduction restriction (abduction somatic dysfunction). Treatment of an adduction dysfunction in the ulnohumeral joint, decreased carrying angle, involves extending and supinating the pt's arm and then applying thrust to the patient's lateral elbow.

15 yo male present to the office with the compaliant of back pain. history reveals the pain has been worsening over the last 2 years since he started growing rapidly. physical examination reveals a rib humb on the right and forward flexion. T12-L4 ARE SIDEBENT RIGHT with an apex at L2. SIDEBENDING to both directions does not improve the curves while int he standing flexion test position, the most likely diagnosis is?

thoracic dextroscoliosis and lumbaer levoscoliosis due to the patient's age, history of recent rapid growth, and physical examination, the most likely cause of this patient's pain is scoliosis. this pt has a double major scoliotic curve, with a convex right thoracic curve and a convex left lumbar curve. the convex right or dextroscoliosis, thoracic curve can be identified with right sided humb in the thoracic region. the levoscoliotic curve in the lumbar spine does not present with a rib hump becasue of the lack of ribs in the lumbar spine. The group curve that is inflexible during side-bending indicates a convex left curve in the lumbar spine with an apex at L2. the fact that the curve do not resolve with sidebending in the forward flexion position indicates that both curves are structural, rigid curves and are not functional flexible curves. A standing AP radiograph of the spine should be obtained in order to determine the cobb angle. scoliosis is the opposite on how we name somatic dysfunctions. structural scoliotic curves do not improve with side-bending

when we say lesions are sepated in time and in space what do we mean?

time=intermittent periods of recovery space=affecting different antomical locations this is multiple sclerosis- symptoms include transient sensory deficitys (involving extremities), fatigue, upper motor neuron signs (weakness, spasticity, hyperreflexia, positive babinski's sign), optic neuritis (pain and vision loss), and/or neuropathic pain.

The patient has resorption atelectasis on the right as a result of a mucus plug from surgery. with this diagnosis, one can expect the diaphragm to be elevated on the right and the trachea to deviate to the right as well. what are some findings?

trouble breathing after surgery, not using incentive spirometer,, absent tactile fremitus on the right side and an inspiratory lag. so the pts diaphragm is likely elevated on the right with the trachea deviated to the right.

Acute ethanol ingestion and increased phenytoin bioavailability increased ROS, the interaction with acetaminophen and the possibility of generating NAPQI

true

Aortic dissection is a life-threatening emergency. predisposing factors include longstanding hypertension, trauma, or connective tissue disorders that make the blood vessels prone to mechanical damage. Dissection occurs when blood penetrates the tunica intima of the aorta and creates a false lumen within the tunica media

true

Autosomal recessive disorders present earlier and with more severe symptoms, and are often found in smaller ethnic groups. autosomal dominant disorders will either have less severe symptoms or a later age of onset, allowing the disease to be passed on to subsequent generations. X-linked disorders are predominantly found in males, and are generally closer to recessive disorder in terms of their severity and age of onset.

true

Ehlers danlos syndrome affects type III and V collagen and presents with hypermobile joints, elastic skin, and frequent bruising. it also causes weakness of vessels, leading to aortic aneurysam formation and rupture.

true

In the event of a suspected bioterrorism event with inhalation anthrax, local or state health department officials should be immediately notified first

true

Isotretinoin is used to decrease sebum secretion from sebaceous glands. Acute vitamin A deficiency can result in seizures, stroke, increased intracranial pressure, psychosis, liver damage, depression, and gastrointestinal upset. note headache and papilledema reflect increased intracranial pressure.

true

Memantine is a NMDA receptor antagonist that is also used for alzheimr dementia-often in combination with an acetylcholinesterase inhibitor

true

Patients with Guillain -Barre syndrome present with ascending muscle weakness, typically after infection. treatmet is with plasmapheresis and IVIG. PTS SHOULD BE MONITORED closely for respiratory failure due to weakness of the diaphragm and other muscles needed for respiration. supportive care with mechanical ventilation should be utilized when needed.

true

Pertussis is a highly contagious infection, whose transmission occurs via direct contact with respiratory secretions. the difference bten droplet spread (particles greater than 5 microns) and airborne spread (particles less than 5 microns) is a subtle but impt distinguishing factor.

true

Preeclampsia is hypertension that first presents after 20 weeks of gestation with associated proteinuria.

true

Pts should be informed about common complications as well as ones that pose significant risk for morbidity and/or mortality, even if they are rare. true or false?

true

Several medications are capable of decreasing the constrictive tone of the lower esophageal sphincter, and consequently increasing the risk for GERD. These medications include anticholinergics, a-adrenergic receptor blockers, B-adrenergic agonists, and calcium channel blockers. true or false?

true

Splenomegaly is a very common manifestation of infectious mononucleosis. the spleen is an intra-abdominal organ of mesodermal origin, which enlarges due to expected immube response.

true

The most common bacterial cause of prostatitis is E.coli, which can be treated with doxycycline, ciprofloxacin or trimethoprim -sulfamethoxazole.

true

The navicular is the bone most commonlu fractured of the midfoot. it is treated with a walking cast for 3-4 weeks. true or false?

true

The patellar reflex is created by stimulation of the L4 nerve root, which contracts the quadriceps muscle. The achilles reflex is created by stimulation of the S1 nerve root, which contracts the gastrocnemius muscle causing foot push off. true or false?

true

Treatment of posterior counterstrain tenderpoints for the ribs involves a seated pt with the physician standing behind the pt. A tenderness scale is then established, and the pt's torso is rotated and sidebent away from the tenderpoint. occasionally slight flexion is also incorporated. correct positioning should aim to decrease the tenderness by about 70%. the treatment position should then be maintained for at least 120 seconds before returning to a neutral position and retesting.

true

True or false common conditions that are mandated to be screened for include-congenital hypothyroidism, phenylketonuria (PKU), medium-chain acyl-CoA dehydrogenase deficiency, neonatal hyperbilirubinemia, biotinidase deficiency, sickle cell anemia, congenital adrenal hyperplasia and many others.?

true

True or false-gestational diabetes taxes the baby's metabolism and changes its growth pattern and its ability to adapt outside of the mother's body. infants of diabetic mothers are often macrosomic and are at risk for hypoglycemic episodes soon after delivery.

true

A spinal cord hemisection (Brown-sequard syndrome) causes contralateral loss of pain and temperature sensation, ipsilateral loss of vibratory and position sense, and ipsilateral spastic paralysis below the level of the injury.

true A spinal cord hemisection, or brown-sequard will cause an upper motor lesion on the same side below the level of the injury due to an injured corticospinal tract. Hyperactive reflexes indicate an upper motor neuron lesion. bROWN-SEQUARD is characterized by ipsilaterla loss of motor function, vibration, and proprioception below the level of the lesion with contralateral loss of pain and temperature approximately two levels below the lesion (spinothalamic tract decussation)

true or false, tetracyclines are a component of bismuth quadruple therapy for h, pylori infections and are frequently associated with photosensitivity true or false?

true Clarithromycon triple therapy --a proton pump inhibitor, clarithromycin, amoxicillin or metronidazole Bismuth quadruple therapy includes ---bismuth subsalicylate, an H2 blocker such as ranitidine, metronidazole, tetracycline.

Cirrhosis is one of the major causes of portal hypertension. Late-stage liver disease leads to decreased protein synthesis which will decrease oncotic pressure, leading to worsening ascites. true or false>

true 75 yo alcoholic male complains of swelling in both of his legs. physical examination of the abdomen reveals jaundice, shifting dullness to percussion, splenomeagly, and dilation of the periumbilical vessels. Laboratory studies reveal a serum-ascites albumin gradient of 0.94 g/dL. the most likely mechanism that explains his symptoms is?=decreased plasma colloid oncotic pressure. this patient is experiencing signs and symptoms related to alcoholic cirrhosis, which is a consequence of chronic liver disease characterized by the replacement of normal liver tissue by fibrosis and regenerative nodules giving the liver a bumpy appearance on gross inspection. Cirrhosis is most commonly caused by alcoholism, hepatitis B and C, and fatty liver disease. Ascites is the most common complication associated with cirrhosis, which is the accumulation of fluid in the peritoneal cavity. As the liver becomes dysfunctional, its ability to filter toxins and drain blood back to the heart diminshes, leading to a build up of blood through the portal circulation. As blood builds up, there is an increase in hydrostatic pressure which leads fluid to leave the vascular space and into the interstitial space, leading to swelling. However, in cases of chronic liver damage leading to cirrhosis, the liver's ability to produce albumin, the body's most abundant serum protein decreases. As a result, there is less albumin circulating within the vasculature, leading to a decrease in plasma colloid oncotic pressure. As a result, the fluid leaves the vascular space and enters the interstitial space, lading to edema, ascitis, and generalized swelling. Malnutrition can also lead to decreased serum protein as well. An important lab value to focus on the etiology of ascties is the serum ascites albumin gradient (SAAG). If the SAAG is >1.1g/dl, this indicates the ascites is due to portal htn. This is due to increased hydrostatic pressure within the blood vessels. Other causes include heart failure, budd-Chiari syndrome. A low (<1.1 g/l) SAAG points toward other causes than portal htn. in this case, it is most likley the decrease in serum albumin production due to cirrhosis of the liver. This can also be seen in patients with nephrotic syndrome as patients lose albumin through the urine, which leads to decreased serum albumin.

A person with ASHERMAN SYNDROME, WHEN GIVEN A PROGESTERONE withdrawal test, this patient would not undergo withdrawal bleeding because there is no basalis that would have proliferated in the first place true or false?

true FSH, LH, and estrogen levels are normal with Asherman syndrome. typically, TSH levels, prolactin levels, and a progesterone challenge is the algorithm for secondary ameonorrhea following a negative pregnancy test.

Moonlighting hours do count towards the 80 hour maximum weekly limit for resident

true What the clinical presentation was- a resident is hoping to moonlight to cover additional expenses and to gain additional clinical experience. according to the accreditation council for graduate medical education, which of the following is true about external moonlighting? ---2003 ACGME-residents can works up to 80 hours a week averaged over four weeks. however,, in late 2016 the policy in regards to PGY1 (interns) duty hours have changed. --first year residents are not allowed to work longer than a 24 hour shift. this policy change is expected to be implement during the 2017-2018 academic year. previously it was restricted to 16 hours. moonlighting hours do count towards the 80 hour maximum weekly limit for resident; however, no other duty hour requirements apply to moonlighting.

AVIDIN IN egg whites bind avidly to biotin so those who consume large amount of raw egg will have biotin deficiency true or false?

true biotin deficiency can lead to depression, alopecia, enteritis, and dermatitis, and erythematous perioral macular rash

rotavirus is the most common casue of a profuse watery diarrhea and dehydration among children under five years of age. rOTAVIRUS is a double stranded, linear, segmented rna VIRUS.

true brlongs to reo family

pneumonic plaque caused by yersinia pestis is a potential bioterrorism threat (category A). these potential bioterrorism AGENTS are classified into three categories: A, B and C depending on their ease of dissemination, mortality rate and overall public healthy impact.

true category A=highest priority-person to person and have high mortality rate. category b-second order of priority-easily transmissible person to person category C-third highest order of. emerging-new rare ones.

Melioidosis is a tropical disease caused by bukholderia pseudomallei, a motile gram-negative rod with bipolar "safety pin" appearance. Meloidosis frequently progresses to sepsis, causing disseminated microabscesses in the skin, liver and spleen. true or false?

true melioidosis, is an infection that is rare in the US. but which carries a high mortality rate even with adequate treatment. Melioidosis (sometimes called whitmore disease) is a systemic infection caused by bukholderia PSEUDOMALLEI. which is a motile gram-negative rod that exhibits safety pin appearance on microscopy. the bacteria is found in the soil and water of tropical regions, particularly in southeast aSIA and australia. and is endemic in these areas. travelers and immigrants account for 0-5 cases per year in the US. PTS WITH chronic comorbidities particularly diabetes, alcoholism, chronic kidney disease, are at a higher risk of contracting and dying from the disease. -because of the high mortality associated with this condition (95% if untreated ,or 50% with treatment), as well as its natural abundance and lack of familarity among clinicians , it is considered by the CDC to be a potential agent of bioterrorism. after inoculation through inhalation, ingestion or skin contact, incubation occurs for approximately 1-5 days (for local infection) 1-2 weeks (for pulmonary or systemic infection). the initial infection presents with nonspecific symptoms, including fevers, chills, rigors, myalgia and headache; urticaria may also be present. cutaneous disease presents as a localized ulcerating lesion, with associated reactive lymphadenopahty. pulmonary manifestations include pneumonia, pulmonary abscesses and pleural effusions. The infection has a strong to produce bacteremia, and pts who become septic develop even more severe symptoms. this include a localized generalized cellulitis, with an associated pustular eruption. pts can develop confusion, dyspnea, rigors, abdominal pain, jaundice, diarrhea and other symptoms. Hematogenous seeding can cause microabscesses to form not only in the skin but also in the liver and spleen (most common) as well as the kidneys, parotid gland and central nervous system; the presence of such lesions on CT scan should raise suspicion for melioidosis. pts can rapidly progress to septic shock and death. pts that have survived will require prolonged antibiotic therapy (usually 150 days), and 23% of survivors experience a reactivation of the disease years later, which is what this patient is currently experiencing. pts may also progress to a chronic form of the infection which is not clinically resolved.

Spongiform encephalopathy of CJD involves the entire cerebrum not just a particular lobe

true rapidly progressive over the course of months

Xeroderma pigmentosum is a result of a defect in nucleotide excision repair that prevents repair of thymine dimers. patients are at increased risk of developing skin cancers at young ages.

true treat with voiding sunlight and the use of retinoids to help prevent skin neoplasms.

BASAL CELL carcinoma is characterized by palisade nuclear arrangements at the periphery of tumor cell clusters it is usually cured by surgical resection and rarely metastasizes.

true you will see an image. dark purple with white things around it. ---he is a farmer-it is slow growing, will happen on sun exposed regions.

The JC virus is widespread, but only causes fatla diseases (progressive multifocal leukodystrophy) in immunocompromised patients, most commonly patients infected with AIDS.

true JC virus is a DNA virus.

Anabolic androgenic steroids are derived from testosterone and used in the treatment of severe burns. testicular atrophy is a common side effect due to negative feedback on LH, leading to decreasd intratesticular testosteron production.

true-example is Oxandrolone.

Chronic kidney disease/chronic renal failure (CDF) is defined as irreversible, progressive reduction in GFR and other renal functions that occurs over a period of months to years. it is commonly associated with uncontrolled diabetes or hypertension. electrolyte abnormalities seen in crf INCLUDE VOLIME overload, hyperkalemia, hypermagnesemia, hyperphosphatemia, uemia, and metabolic acidosis. treatment includes a low protein, low salt diet, blood pressure control (ACE inhibitors are preferred early on to reduce the risk of progression to (ESRD), glycemic control, electrolyte correction, and possibly dialysis. kidney transplantation is the only cure.

true.

somatic symptom disorder is a disorder that causes psychological distress and impairment in function due to one or more somatic symptoms that are not explained by another medical condition

true.

A bilateral sacral flexion will hav a preserved lumbosacral spring, bilateral deep sacral sulci and a symmetrical rise in PSISs on seated flexion.

true. read question well.

where does the uterus receive sympathetic innervation from?

uterus received sympathetic innervation from T10-L2 and the cervix has parasympathetic innervation from s2-s4. KNOW THESE COMMONLY TESTED ORGANS AND THEIR INNERVATIONS.

iS TOPIRAMATE an inducer of p450?

yES. SO PTS ON birth control, the efficacy may be reduced.

Patient's with narcolepsy may also experience frightening hallucinations either at the start of sleep (hypnagogic) or just before they wake up (hypnopompic). is there a strong genetic component associated with narcolepsy?

yes

a PT WHO you suspect may be abused-when she is leaving, the most appropriate thing to d would be to assess the pt's capacity to participate in discharge planning.

yes a person who shows signs of elder neglect. it would be ethically impermissible to discharge a patient who lack capacity home to an unsafe environment. On the other hand, if she has capacity she can choose how to proceed.

Short leg syndrome is associated with anterior innominate rotations, characterized by an inferior ASIS and superior PSIS on the affected side.

yes cepahlad medial malleoli will be present with both type of short leg such as anatomical or functional. anatomical short leg usually present with compensatory changes including sacral base unleveling, lumbar sidebending away from the lesion, and anterior innominate rotations. have chronic low back pain.


Kaugnay na mga set ng pag-aaral

Chapter 27: Antipsychotics and Anxiolytics

View Set

ARM 402: Full Practice Exam: Frequently Missed Questions & Terminology

View Set

chapter 3 race and ethnicity exam review

View Set

Inequality Midterm Discussion Questions

View Set

Ch 4 Davis Advantage Maternal Nursing

View Set